You are on page 1of 312

lOOlsoLvEo PROBLEMs

- -
IN
ENGINEERING MATHEMATICS
_____ .....
, .._.....,.,.

Second Edition

JAIME R. TIONG
•1 BSCE,.UV 1985 (Summa Cum Laude)
.,..·. UY College of Engineering First Summa Cum Laude
, 1st Placer, PICE National Students' Quiz, 1985
.. Awardee, Outstanding Alumni, UV
. ·' Prestaent, Excel First Review and Training Center, Inc.
·· · Past President, Rotary Club of Cebu North
Pr~sident, UV Engineering & Arch. Alumi Association
··Del~ate, Rotary International Convention, Chicago, USA
r(~mt(l~,. Ramon Aboitiz Foundation Inc. Triennial Awards
' . . ' Former Plant Engineer, University of the Visayas
·· Fo~mE!r Reviewer, Besaviiia Engineering Review Center
f,ormer Reviewer, Salazar Institute of Technology
·· Former Faculty, UV College of .Engineering
Author, Various Engineering Reviewers

·~ ROMEO A. ROJAS Jl-i.


"·'BSEE, CIT 1991 (Cum Laude), BSECE, OT 1996
1st Placer, RME Licensure Examinations, Oc;:tober 1997
8th Placer, REE Licensure Examinations, April 1999
Former Faculty, Cebu Institute of Technology
Former Technical Assistant, CIT Automation Center
Reviewer, Excel Review Center
Author, Various Electrical Engineering Rexiewers

IMPORTANT: Any copy of this book not bearing the signature ofany one
ofthe authors or ofthe publisher on this page shall be considered as
comingfrom an illegal source.

AUTHOR I PUBLISHER
TABLE OF CONTENTS

Preface to the First Edition


Preface to the Second Edition
Dedication •ndamenta~~~ 2
1
Algebra
THEORY:
Properties of Integers 23
DAY Properties of Addition 23
of Numbers Properties of Multiplication 23
Conversion Additive Identity 23
Additive inverse 23
THEORY: Multiplicative Identity 24
Number Multiplicative Inverse 24
1
Types of Numbers 1 Properties of Equality 24
Numerals Properties of Zero 24
1
Digits Exponents 24
2
Real Numbers 2 Radical 25
Imaginary number 2 Surd 25
Complex number Types of Surds 25
3
System of Numbers Special Products 26
3
Fractions Proportion 26
3
Types of Fractions 3 Properties of Proportion 26
Composite numbers Least Common Denominator 26
4
Prime numbers Least Common Multiple 27
4
Fundamental Theorem of Greatest Common Factor 27
Arithmetic 4 Remainder Theorem 27
Types of Prime Numbers Factor Theorem 27
5
Perfect Number Trivia 27
5
Abundant Number Quote 27
6
Deficient Number 6
Perfect Number TEST (50 Problems for 3.75 hours) 28
6
Amicable Number SOLUTIONS 34
6
Friendly Number Notes 43
6

3
Factorial 6

~@tJc
Significant Figures & Digits 7 If DAY


Forms of Approximation 7
Conversion 7 Equation,
Celsius Scale 7 mfal Theorem &
Fahrenheit Scale 7 Logarithms
Kelvin Scale 8 THEORY:
Rankine Scale 8
Quadratic Equation 45
Degrees, Radians, Grads & Mils 9
Nature of Roots 45
Trivia 9 5
Properties of Roots 46
Quote 9 Binomial Theorem 46
Properties of Expansion 46
T"EST (50 Problems for 2 hours) 10
Pascal's Triangle 46
SOLUTIONS 15
Coefficient of any term 47
Noles 20
Formula for rth term 47
Stjm of Coefficients 47 Infinite Geometric Progression 89 Odd For 122
DAY
8
Sum of Exponents 47 Harmonic Progression 89 Odd Against 122
Degree of Polynomial/Equation 47 Other related sequences 90 Mathematical Expectation 122
logarithm 47 Fibonacci Numbers 90 Card Games 122 Geometry
Common & Natural Logarithms 48 Lucas Numbers 90 Probability with Dice 123
·Euler's Number 48 Figurate Numbers 90 Trivia 124
Binary Logarithm 48 Triangular numbers 90 Quote 124 THEORY:
Properties of Logarithms 48 Square numbers 90 Polyhedrons 179
Trivia 48 Gnomons 90 TEST (50 Problems for 4 hours) 125 Five Regular Polyhedrons 179
Quote 48 Oblong numbers 90 SOLUTIONS 131 Platonic Solids 179
Pentagonal numbers 90 Notes 141 Prisms 181
TEST (40 Problems for 3 hours) 49 Cubic numbers 90 Cylinders 182
SOLUTIONS 54 Tetrahedral numbers 90 Pyramids 182
DAY
1
Notes 60 Cubic numl;>ers 90 Cones 182
Square pyramidal numbers 90 Frustum of Pyramid 183
Supertetrahedral numbers 90
Geometry Frustum of Cone 183
Trivia 9t Prismatoid 183

Mixture,
DAY4 Quote 91 THEORY:
143
Prismoidal Formula
Sphere
183
184
Definition of Geometry
Digit, Motion TEST (65 Problems for4.5 hours) 92 Branches of Geometry 143 Zone 184
SOLUTIONS 100 Basic Postulates of Euclid 144 Spherical Segment 184
Problems Notes 114 Basic Geometry Elements and Spherical Sector 184
THEORY:
Figures 144 Spherical Pyramid 184
Age Problems 63 Types of Angles 144 Spherical Wedge 185
Work Problems
DAY
6
63 Bisector 145 Torus 185
Mixture Problems 64
Digit Problems Units of Angles 146 Ellipsoid 185
64 n Diagram, ·Polygons 146 Spheroid 185
Motion Problems 64
Coin Problems 64
trmutation, Triangles 147 Trivia 186
Combination & Other Types of Triangles 147 Quote 186
Trivia 65 Quadrilaterals 148
Quote 65 Probability Types of Quadrilaterals 148 TEST (30 Problems for 2.5 hours) 187
Bramaguptha's Theorem 150 SOLUTIONS 191
TEST (40 Problems for 4 hours) 66 THEORY: 200
Ptolemy's Theorem 150 Notes
SOLUTIONS 73
Notes Venn Diagram 117 Areas and Perimeters of
84 150

9
Combinatorics Regular Polygons
'~'f
Fundamental Principle of
118
Perimeter 151 DAY
Counting 118 Circles 151
DAY
5
Permutation 118 Useful Theorems involving
Inversion 118 . Circles . 152
Variation, Cyclic Permutation 119 Ellipses 153 THEORY:
II Problems &
Progression
Permutation with Identical
Elements
Assortment
119 '
119
Trivia
Quote
154
154
Definition of Trigonometry
Branches of Trigonometry
Classification of Triangles
203
203
203
Combination TEST (50 Problems for 3.75 hours) 155 Solution to Right Triangles 204

~
THEORY: 119
Relation between Permutation SOLUTIONS 161 Pythagorean Theorem 204
Clock Problems 87 And Combination 119 Notes 176 Solution to Oblique Triangl(!s 204
,,'
' Variation Problems 88 Probability 120 Law of Sines 204
l'i.!'' ·' Diophantine Equations 88 Principles of Probability 120 Law of Cosines 205
Sequence 88
Series Mutually Exclusive Events 120 Law of Tangents 205
I. 88 Independent Events 121 Trigonometric Identities 205
Progression 89 Binomial Distribution 121
Arithmetic Progression 89 Odd 121
Geometric Progression 89
Exponential Form
Fundamental of the
Tngonometnc . =EST (15 Problems
~OLUTION for 1.5 hours) 240
242 .j,·'·:·• .r' . DAY
. 13 .~ :·~ . ·· . . DAY 15
Function
Amb1guous Case
Other Parts of Triangle
Radius of Inscribed Circle and
Circumscribing Circle
206
206
207

207
Notes
.
~~,: ·
'fi:;; ..·
Geometry
/'''i,X;AJtalytic
DAY 11
247

THEORY:
':

D · f
enva IVeS
Dlff~re_ntlal
ulus .(l1m1ts &
)
THEORY:
u.ral Calculus
0':

Definition of Integral Calculus 361


Plane Areas (Triangles) 208 ~;~~ \~0 ' Points . Lines & Definition of Calculus 309 Definite and Indefinite Integrals 361
Plane Areas (Quadrilaterals) 209 ' c· I Limits 309 Fundamental Theorem of
Ptolemy's Theorem 209 Ire es Theorems of Limits 309 Calculus 361
Important Properties of Triangles 209 THEORY One-Sided Limits 310 Basic Integrals 362
Important Points in Triangles 209 Rectangular Coordinates System 249 Continuity 311 Exponential & Logarithmic
Conditions for Congruency 210 Distance Formula 250 Special Limits 311 Functions 362
Conditions for Similarity 210 Distance Between Two Points Derivatives 311 Trigonometric Functions 362
Tnv1a 210 in space 250 Algebraic Functions 311 Inverse Trigonometric Functions 362
Quote 210 Slope of a Line 250 Exponential Functions 312 Hyperbolic Functions 362
Angle Between Two Lines 251 Logarithmic Functions 312 Trigonometric Substitution 363
TEST (50 Problems for 4 hours) 211 Distance Between a Point Trigonometric Functions 312 Integration by Parts 363
SOLUTIONS 217 and a line 251 Inverse Trigonometric Functions 312 Plane Areas 363
Notes 230 Distance Between Two Lines 251 Hyperbolic Functions 31.2 Centroid 364

DAY
·o Division of Line Segment
Area by coordinates
252
252
Inverse Hyperbolic Functions
Trivia
312
313
Length of Arc
First Proposition of Pappus
364
364

1
Lines 252 Quote 313 Volume 365
• Conic sections 253 Second Proposition of Pappus 365
Sphencal General Equation of Conics 253 TEST (40 Problems for 4 hours) 314 Work 366
onometry Circles 254 SOLUTIONS 319 Hooke's Law 366

THEORY:
Definition of Spherical
Tngonometry
Great Circle
233
233
Trivia
Quote

TEST (50 Problems for 4 .hours)


SOLUTIONS
255
255

256
261
Notes

.~;;
:.*.
1t<~..
DAY 14
329 Moment of inertia
Ml:lltiple Integrals
Trivia
Quote
366
366
366
366

Small Circle 234 Notes 275 0~,.al Calculus TEST (50 Problems for 4 hours) 367

12
Pole 234 rna/Minima & SOLUTIONS 373
Polar Distance 234 Time Rates) Notes 391

·. •
1~
Spherical Wedge
Spherical Triangle

Tnangle of Spherical
Propositions
Solutions to Right Triangle
Nap1er's Rules
234
234

235
235
235
~···
~:'{t:
1\a.lytic
'" '"• .
DAY
Geometry
.
Parabola, Ellipse
& Hyperbola
THEORY:

Max~
Max1mamum
Time :Rates
a~~
Minimum Values 331
I M1mma
.
Relat1on between the vanables &
332
332
.

• . • .{•.•.·.· ..·.· . . ·. . DAY


1.£)ifferential .
>iEouations
16
'"' · Quadrantal Spherical Triangle 236 maxima I minima values 332
Solution to Oblique Triangles 236 THEORY: Trivia 337
Area of Spherical Triangle 237 Parabola 277 Quote 337 THEORY:
Terrestrial Sphere 237 Ellipse 279 Types of DE 393
Pnme Meridian 237 Hyperbola 281 TEST (35 Problems for 3 hours) 338 Order of DE 394
International Date Line 237 Polar coordinates 284 SOLUTIONS 343 Degree of DE 394
Greenwich Mean Time 238 Trivia 285 Notes 359 Types of Solutions of DE 394
Coordinated Universal Time 238 Quote 285 Applications of DE 395
Latitude i':lnd Longitude 238 Trivia 396
Terrestrial Sphere Constants 239 TEST (55 Problems for 4 hours) 286 Quote 396
Trivia 239 SOLUTIONS 292
Quote 239 Notes :l(fT
TEST (30 Problems for 2.5 hours)
SOLUTIONS
Notes
397
401
410
Centripetal Force
Law of Universal Gravitation
Work
Energy
Law of Conservation of Energy
Power
443
443
444
444
445
445
;,.:. 5 '~ ',.,
~'3',,

., "::<'•···
-"

~~~t~,£" E~~~~=~ii~~
DAY
20 ~~~>
~~~>l: 'tf;~·.·:·~: :.
;~r1: < >«;
,,;~;;:r:>,;: ..
Engineering
mfi!fiebhomy (Simple &
DAY 22
DAY
11 Momentum 445
(Dynamics) Compound Interest
Law of Conservation of THEORY:
Momentum 445 Types of Rectilinear Translation 487 THEORY:
Impulse 445 Horizontal Translation 487 Definition of Terms 531
Types of Collisions 445 Vertical Translation 488 Consumers & Producers
THEORY: Coefficient of Restitution 445 Free Falling Body 488 Goods and Serv1ces 531
Complex Numbers 413 Gas laws 446 Curvilinear Translation 489 Necessity and Luxury 531
Different Forms of Complex Properties of Fluids 446 Projectile or Trajectory 489 Market Situations .532
Numbers 413 Archimedes Principle 446 Rotation 490 Demand 533
Mathematical Operation of Trivia 447 D'Aiembert's Principle 490 Supply 534
Complex Numbers 414 Quote 447 Centrifugal force 491 Law of Supply and Demand 534
Matrices 415 Banking of Highway Curve 491 Interest 535
Sum 6( two matrices 416 TEST (40 Problems for 3 hours) 448 Trivia 492 Simple Interest 535
Difference of two matrices 416 SOLUTIONS 453 Quote 492 Discount 536
Product of two matrices 416 Notes 462 Compound Interest 536'
Division of matrices 417 TEST (45 Problems for 4 hours) 493 Continuous Compounding 537
Transpose matrix 417 SOLUTIONS 499 Nominal & effective rates of
Cofactor of an entry of a matrix 417 Notes 511 interest 538
DAY
Cofactor matrix .
Inverse matrix
Determinants
Properties of Determinants
Laplace transform
Laplace transforms of elementary
• 417
417
418
418
419
THEORY:
·'"t'W
·~-1-B~%\,

·!~lJtingineering
. h~rdcs (Statics) 19 DAY
Strength of
Materials 21
Trivia
Quote

TEST (40 Problems for 3 hours)


SOLUTIONS
Notes
538
538
539
545
551
functions 419 Definition of Terms 465
Trivia 420 Branches of Mechanics 465 THEORY:
'iJ'''l[''''''
Conditions for Equilibrium 465 Definition of Terms 513 ·,#z~!;< DAY
23
TEST (50 Problems for 4 hours) 421 Friction 466 Simple St~ess 513 . ·~-il;;\
' 1Z. ·.' • •
r"'f"''t"' ~

'
SOLUTIONS 428 Parabolic Cable 466 Types of Normal Stress 514 ::~'~l:t Engmeermg
Notes 438 Catenary 467 Simple Strain 514

-\{•
Hooke's Law 514 "foilomy (Annuity,
Moment of inertia 467
Mass moment of inertia 468 Stress-Strain Diagram 515 Depreciation, Bonds,
DAY
18
Trivia 469 Thermal Stress 515 Breakeven analysis,
'II '!:"''',
~it.'. Physics
Quote

TEST (35 Problems for 3.5 hours)


469

470
Thin-Walled Cylinder
Torsion
Helical springs
516
516
517 THEORY:
etc.

SOLUTIONS 475 Trivia 517 Annuity 553


THEORY: Notes 484 Quote 517 Capitalized Cost 555
Vector & Scalar Quantities 441 Annual Cost 555
Classifications of Vectors 441 TEST (30 Problems for 2.5 hours) 518 Bonds. 555
Speed and Velocity 442 SOLUTIONS 522 Depreciation 557
Distance and Displacement 442 Notes 528 Break Even Analysis 558
Acceleration 442 Legal Forms of Business
Laws of Motion 442 Organizations 558
Force 442 Trivia 559
Frictional Force 443 Quote 559
TEST (51 Problems for 4 hours) 560
SOLUTIONS 568
Notes 580

I I :RJ·;~i;t~t£
-
1ces
A. GLOSSARY 583
B. UNITS & CONVERSION 625
c. PHYSICAL CONSTANTS 633
D. NUMERATION 634
E. MATH NOTATION 634
F. GREEK ALPHABETS 635
G. DIVISIBILITY RULES 636

Pl:R~ON~F\L-
)",,.._
•;, ef(opt:: \<..11' Cf .
k.
~b~
~ ~ -Ft' f'ffl'ff.c:5
)
' ..v
rr
.)

f&t ir. ~~fl..lc.tl- Efolt;; '<;; _.l>WCL


Topics

~
Mon
Cardinal and Ordinal Numbers
Numerals and Digits
System of Numbers
D
Tue
-Natural numbers, Integers,
Rational numbers, lrrationC!I
numbers & imaginary numbers

~ D Complex numbers

D ,--
Theory Wed Types of Fractions
Composite Numbers
_j Prime Numbers
Problems Thu Defective and Abundant Numbers

Cl LJ
Solut1ons Fri
Amicable Numbers
Significant Figures and Digits
Forms of Approximation

D [_]
Notes Sat
Conversion

The most widely used numerals are the


What is a number?
Arabic numerals and the Roman
numerals.
A number is an item that describes a
magnitude or a position.
Arabic numerals were simply the
~-'
modification of the Hindu-Arabic number
What are the types of numbers?
signs and are written in Arabic digits.
Taken singly, 0, 1, 2, 3, 4, 5, 6, 7, 8, 9 and
Numbers are classified into two types,
in combination 20, 21, 22, ... 1999, ...
namely cardinal numbers and ordinal

il numbers.

Cardinal numbers are numbers which


The Roman numerals are numbers which
are written in Latin alphabet. Example
MCMXCIV.
~' ,,./ allow us to count the objects or ideas in a
given collection. Example, 1 ,2,3 ... , 1000,
The following are Roman numerals and
100000 while ordinal numbers state the
their equivalent Arabic numbers:
position of the individual objects in a
sequence. Example, First, second, third ..
I = 1 C = 100
v =5 0=500
X = 10 M "' 1000
IIYJ:l.!lt are numerals? L :: 50
Numerals CJre symbols, or combination of
The Romuns used the following to indicate
·;ymb<)ls wt1ich describe i1 number.
large nurnbers:
2 l 00 l s21v~d Problems ·in Engineering Mathematics (2"d Edition) by Tiong & Rojas
Day l - Systems of Numbers and Conversion 3
1. Bracket - to (11ultiply it by 100 Examples: - 4, -1, 0, 3, 8
Imaginary number and its equivalent:. What is an absolute value?
times.
3. Rational numbers - are numbers i =~ '
i2 = -1 The absolute value of a real number is
lVI = 500 which can be expressed as
a quotient (ratio) of two i =-i =- ~
3 the numerical value of the number
neglecting the sign.
integers. The term "rational" i4 = 1
2. Vinculum (bar above the comes from the word
humber) - to multiply the number For example, the absolute value of- 5 is 5
"ratio".
1000 times.
Examples: 0.5,
2
, -3, 0.333 ...
What is a complex number?
while of -x is x. The absolute value Ia Iis
v= 5,000
3 A complex number is an expression of either positive or zero but can never be
In the above example, 0.5 can be both real and imaginary number combined. negative.
1 It takes the form of a + bi, where "a" and
3. Doorframe- to multiply the expressed as - and -3 can be "b" are real numbers. What are fractions?
2
number by 1000000 times
I -6 If a = 0, then pure imaginary number is Fractions are numbers which are in the
expressed as -,.hence the two
2
fVl = 5, 000,000 examples are rational numbers.
produced while real number is obtained
form of ~or a/b, where a is called!he
when b = 0.
numerator which may be any integer while
The number 0.333 ... can also be What is a system of numbers?
What i2J! digit? b is called the denominator which may be
express·ed as __!_ and therefore a any integer greater than zero. Fraction is
A digit is a specific symbol or symbols 3 A system of numbers is a diagram or chart also defined as a part of a whole.
used alone or in combination to denote a rational number. which shows the two sub-classifications of
number. the two basic classifications of numbers, What are the types of fractions?
The number 0.333 ... is a repeating and namely real numbers and imaginary
For example, the number 21 has two non-terminating decimal. As a rule, a numbers. 1. Simple fraction - a fraction in which
digits, namely 2 and 1. In Roman non-terminating but repeating (or the numerator and
numerals, the number 9 is denoted as IX. periodic) decimal is always a rational System of denominator are both
So the digits I and X were used together to number. Also, ajl integers are rational Numbers integers. This is also known
denote one number and that is the number numbers. as a common fraction.
9.
4. Irrational numbers - are numbers 2 6
In· mathematical computations or in some which cannot be expressed Examples: .
engineering applications, a system of as a quotient of two /Real Imaginary
3 7
numbers using cardinal numbers was integers. Numbers Number 2. Proper fraction - is one where the
established and widely used.
numerator is smaller thai!
What ~re real numbers? Examples: ..J2, n, e, ... the denominator.

The number system is divided into two The numbers in the examples above 5 2
categories namely, real numbers and can never be expressed exactly as a Examples: 7' 3
imaginary number. quotient of two integers. They are in
fact, a non-terminating number with 3. Improper fraction - is one where the
Real numbers are classified as follows: non-terminating decimal. numerator is greater than
1. Natural numbers - numbers which the denominator.
What is an imaginary number?
are considered as the
"counting numbers". 5 12
An imaginary number is denoted as "i" Examples: 2' 7
which is equal to the square root of
Examples: 1 ,2, 3 ...
negative one. In some other areas in 4. Unit fraction - is a fraction with unity
2. Integers- are all the natural number, mathematical computation, especially in for its numerator and
the negative of the natural electronics and electrical engineering it is positive integer for its
numbers and the number denoted as "j". denominator.
Pi~l<J~::jtn showing the riumbe~ system
zero.
Day 1 - Systems of Numbers and Conversion 5
!_lt:'O 1 Solved Problems in Engineering Mathematics (2"ct Edition) by Tiong & Rojas
2,3, 5, 7, 11, 13, 17, 19,23,29,31, 37, 9 7,11; 5,13
1 11. Undefined fraction- a fraction with a 41,43,47,53, 59,61,67, 71, 73, 79,83, 10 1. 1a a 11
Examples: 4· 25 denom1nator of zero. The 89, 97,101, 103,107,109, 113,127,131, 11 5, 17; 3, 19
example below means that 137. 139, 149, 151, 157, 163, 167, 173, 12 11, 13; 7, 15; 5,19
8 is divided by 0, which is an 179, 181, 191, 193, 197, 199,211,223, 13 7, 19; 3, 23
5. Simplified fraction - a fraction whose 14 11,17.5 ;;_· ..
numerator and denominator
impossibility because 227,229,233,239,241,251,257,263,
nothing can be divided by 269,271,277,281, 283, 293,307, 311, 15 13, 17, ''i' 9; 7, 23
are 1ntegers and their 16 15, 17; '!,., 19; 3, 29
greatest common factor is 1.
zero. 3.13, 317,331, 337, 347, 349,353, 359,
367,373,379,383,389,397,401,409, 17 11,23;6.2~ 3,31
8 419,421,431,433,439,443,449,457, 18 17, 19; 1.l, 23; 7, 29; 5, 31
1 8 Examples: - 19 9,29; 7, 31
Examples: 2' -11 0 461,463,467,479,487,491,499,503,
509,521,523,541,547,557,563,569, 20 17, 23; 11, 29; 3, 37
· 12. Indeterminate fraction - a fraction 571,577,587,593,599,601,607,613, 21 19, 23; 13, 29; 11,31; 5, 37
6. An lnteJer Represented as fraction which has no quantitative 617,619,631,641,643,647,653,659, 22 13, 31; 7, 37; 3,41
" - a fraction in which the
meaning. 661,673,677,683,691,701,709,719, 23 17,29; 13, 33; 5,41; 3,43
denominator is 1. 727,733, 739, 743, 751, 757, 761, 769, 24 19, 29; 17, 31; 11, 37;
2 3
0 773,787,797,809,811,821,823,827, 7,41; 5, 43
E xamples: -1 , --
1 Examples: 25 19, 31; 13, 37, 7,43; 3,47
829,839,853,857,859,863,877,881,
0 883,887,907,911,919,929,937,941,
7. Reciprocal- a fraction that results 947,953,967,971,977,983,991,997, Emirp (prime spelled backwards} is a
13. Mixed number- a number that is a prime number that remains a prime when
from interchanging the
combination of an integer its digits are reversed.
numerator and der.1ominator. The number 2 is the only prime number
and a proper fraction. The following are emirps less than 500:
which is an even number.
Examples: 4 is the
1 8 What are the types of prime numbers? 11, 13, 17, 31, 71, 73, 79, 97, 101, 107,
Examples: 5-. 9-
reciprocal of -~ 2 11 113,131, 149,151, 157, 167, 179,181,
4 191,199,311,313,337,347,353,359,
Na.tural prime numbers are those that have
8. Complex fraction -a fraction in which 373,383,389
What is a composite number? only two factors; 1 and the number. Twin
the numerator or
primes are a set of two consecutive odd
denominator, or both are Relatively prime numbers are numbers
Composite numbers are positive integers primes, which differ by two. The following
fractions. whose greatest common factor is 1.
that have more than two positive whole are twin primes less than 100.
number factors. It can be written as
3 5 Unique product of power of primes is a
- product of two or more integers, each 3, 5 29, 31
Examples: __4_
7'
-~1' -13 greater than 1. It is observed that most 5, 7 41,43 number whose factors are prime numbers
- - 2 integers are composite numbers. The 11' 13 59, 61 raised to a certain power.
•r""''"''
8 4 number 6 is a composite number because 17, 19 71, 73
its factors are 1, 2, 3 and 6. The number 1 Example of unique product of pow~r of
9. Similar fractions -two or more simple is the only natural nu"'lber ti1at is neither Symmetric primes are a pair of prime primes:
fraction that have the same composite nor prime. numbers that are the same distance from 360 =23 . 32 . 51
denominator. a given number in a number line.

II Examples:
1
g-· ·g-·
4 !_
9
What is a prime nu111ber?

A prime number is an integer qreater than


Symmetric primes are also called Euler
primes. The following are symmetric
primes for the number 1 through 25.
What is a perfect number?

A perfect number is an integer that is ~


1 that is divisible only by 1 and it~elf. equal to the sL m of all its possible divisors,
Number Symmetric prime except the number itself.
10. Zero fraction - a fraction in which the
According to the fundamental theorem of 1 None
numerator is zero. A zero
arithmetic, " Every positive integer greater 2 None Example: 6, 28, 496 ...
fraction is equal to zero.
!h.an 1 is a prime or can be expressed as a 3 None
In the case of 6, the factors or diviscrs .-e
unique product of primes and powers of 4 3,5 1, 2 and 3. When the factors are added the
0 0
Examples: 2 11 primes". 5 3, 7
sum is ~qual to the number itself and
6 5, 7
shown in the following equation.
The following 1s a list of the prime numbers 7 3, 11
less than 1,000. 8 5, 11; "3, 13
1+2+3=6
6 1001 Solved Problems in Engineering Mathematics (2"d Edition) by Tidng & Rojas Day I - Systems of Numbers and Conversion_ 7
What are an abundant numbers and
deficient numbers?
2P-1 (2P -1) Example: n! = n(n- 1) ... 3, 2, 1 integer number, fewer value-carrying (non"
zero) digits.
If n = 0, by definition:
If the sum of the possible divisors is where:
Examples:
greater than the number, it is referred to as
abundant number.
p and ( 2P -1) are prime numbers (n!)(n+1) = (n+1)! ~ 3.14159 shall be rounded up to 3.1416
(0 !)(0 + 1) = (0 + 1)! 3.12354 shall be rounded down to 3.1235
Formula Perfect number
A defective number is an integer'with the i (2 -1)
2
6 0!(1)=1! Truncation refers to the dropping of the
sum of all its possible divisor is less than 0! =1 next digits in order to obtain the degree of
the number itself. It is also called deficient 2 3
2 (2 -1) 28_ accuracy beyond the need of practical
number. The factorial symbol ( ! ) was introduced calculations.This is just the same as
2 (2 -1)
4 5
496 by Christian Kramp in 1808. rounding down and truncated values will
The following is a list of the first 25 always have values lower than the exact
7
numbers with its corresponding type, D for 26 (2 -1) 8128 What are significant figures or digits? values.
deficient and A for abundant.
What are amicable numbers? Significant figures or digits are· digits Example:
Factors Excluding that define the numerical value of a 3.1415 is truncated to 3 decimal as 3. 141
Number Itself Sum Tvoe number.
Amicable numbers or friendly numbers
1 0 D What is a conversion?
refer to two integers where each is the
2 1 1 D A digit is considered significant unless it is
sum of all the possible divisors of the
3 1 1 D used to place a decimal point. Conversion is the process of getting the
other.
4 1, 2 3 D equivalent value in another unit of
5 1 1 D The smallest known amicable numbers are The significant digit of a number begins measure of a certain value with a different
6 1, 2, 3 6 Perfect 220 and 284. with the first non-zero digit and ends with given unit of measure.
7 1 1 D the final digit, whether zerc, or non-zero.
8 1, 2, 4 7 D The number 220 has the following Most conversions can be done
9 1, 3 4 D factors/divisors: 1, 2, 4, 5, 10, 11, 20, 22, Examples: conveniently by using a prepared
10 1,2, 5 8 D 44, 55, & 110 which when added sums up conversion table while other conversions
11 1 1 D to 284, while the number 284 has the 16.72 . 4 significant figures can be done through mathematical
3
12 1,2,3,4,6 16 A following divisors 1, 2, 4, 71, and 142 1.672 X 10 4 significant figures computations using formulas.
13 1 1 D which adds up to 220. 0.0016 2 significant figures
1, 2, 7 10 D The authors suggest to the user of this
14
There a.re more than 1000 pairs of Example 2 is expressed in scientific book to familiarize the values in the
15 1,3,5 9 D
amicable numbers have been found. Th~ notation and figures considered significant conversion table which is found at the last
16 1, 2, 4, 8 15 D
following are the ten smallest pairs of are 1, 6, 7 and 2 excluding 103 Example 3 part of this book labeled as "Appendix A".
17 1 1 D
amicable numbers. has 2 significant figures only because the
18
19
1,2,3,6,9
1
21
1
A
D
3 zeros are used only to place a deciMal. How toconvert a temperature in degree
220 and 284 Celsius to degree Fahrenheit and vice
20 1,2,4,5, 10 22 A The number of significant digit is versa?
1184 and 1210
21 1, 3, 7 11 D considered the place of accuracy. Hence,
2620 and 2924
22 1' 2, 11 14 D a number with 3 significant digits is said to The unit "Celsius" was named after the
5020 and 5564
23 1 1 D have a three place accuracy and a Swedish astronomer, Anders Celsius
6232 and 6368
24 1,2,3,4,6,8, 12 36 A 10744 and 10856 number with 4 significant figures is said to (1701 -1744). In this unit of temperature,
25 1, 5 6 D 12285 and 14595 have a four place accuracy. the boiling point and freezing point are 100
17296 and 18416 degrees and 0 degree, respectively.
What is a perfect number? 63020 and 76084 What are the forms of approximations?
66928 and 66992 The unit "Fahrenheit" was named after the
Perfect number is a number that is equal There are two forms of approximations, German physicist, Gabriel Daniel
to the sum of its factors excluding itself. What is a factorial? namely rounding and truncation. Fahrenheit (1686- 1736). In this unit of
They are mathematical rarities that have temperature, the boiling point and freezing
no practical use. The formula to find a Hounding of a number means replacing point are 212 degrees and 32 degrees,
A factorial denoted as n!, represents the
perfect number is a follows: IIi" n11rnber with diH >flier JHirnber having respectively.
product of all positive integers from· t to n,
I• ·w• ., •;1qnlfle<HII tl< ,, II I I'll tl1qils, m for
inclusive.
8 10.01 Solved Problems in Engineering Mathematics (2nd Edition) by Tiong & Rojas Day .1 - Systems of Numbers and Conversion · 9

Solution: How to convert one unit of an angle to Problema


For same reading, F =C another unit? There are how many grads in 1200 mils?
9
F=-C+32 There are four units commonly used to
5 Solution:
9 measure an angle. They are degrees, x 1200 mils
'iling point F=-F+32 radians, grads and mils.
5 400 grads 6400 mils
90
80 O.BF = -32 The degree is the mostpommonly used x=75 grads.
70 F = -40° measure of an angle. The radian is an
60 angle subtended by an arc equal to the Study Appendix B - Prefixes which is
How to convert temperature in degree length of the radius of the circle. found in the last part of this book.
100- 0~
1:30
_ _
212-32
Celsius or degree Fahrenheit to
oc- 0
The following is a tabulation of the unit of Proceed to the next page for your first test.
~--li'oF- 32
absolute temperature?
angle measurement and the Detach and use the answer sheet provided
· 20
Absolute temperature may be expressed corresponding value in one revolution: at the last part of this book. Use pencil

tj·: ''· __ )::"'


-10
in Kelvin or in Rankine.

Kelvin was named after British physicist,


Unit
Degree
1 Revolution
360
number 2 in shading your answer.

GOOD LUCK!
Centigrade Fahrenheit William Thompson (1824-1902) the First Radian 21t
Scale Scale Baron, Kelvin. Grad 400
Mil 6400
By ratio and proportion: Rankine was named after Scottish
engineer and physicist, William John Problem&
'Ol:ribia:
C·-0 F-32 Macquom Rankine (1820-1872). Did you know that... the symbol 1t (pi),
--=-- How many mils are there in 90 degrees?
100-0 212-32 which is the ratio of the circumference of a
The formulas for conversion to absolute circle to its diameter was introduced by
c
-=--
F-32
temperature are as follows:
Solution:
William Jones in 1706 after the initial
100 180 x 90° letter of the Greek word meaning
100 0 6400 mils 360° "periphery".
C=- -(F-32) °K= °C+273 R=°F+490
180 x =1600 mils
5
C =~(F-32) Problem:
Problema ~uote:
9 Express the temperature of 60°C to "I could hardly ever known a
How many radians is equivalent to 320
absolute temperature. mathematician who was capable of
grads? ·
9 reasoning."
or F=-C+32 Solution:
5 Solution: -Plato
°K = °C+273
°K=60+273 _x__ = 320 grads
Problem:
°K =333° 2Tt radians 400 grads ·

II
Convert 45°C to °F.
x = 5.03 radians
Solution: Problem:
9 Express the temperature of 150°F to Problema
F=-C+32 absolute temperature. 145 degrees is equivalent to how many
5 grads?
9 Solution:
F=-(45)+32
5 0
' R=°F+490 Solution:
F = 113°F 0 X 145°
R = 150+490
0
400 grads = 360"
Problema R = 640°
At what temperature will the Centigrade x = 161.11 grads
scale and the Fahrenheit scale will have
the same reading?
Dayl -Systems of Numbers and Conversion 11
8
B. 2.37 X 10"
13. EE Board April1993
C. 0.2371 X 10"9
Express decimally: Seven hundred twenty-
D. o.oo2371 x 1o· 11 five hundred thousandths

7. EE Board October 1994 A. 0.000725


7 + Oi is B.
Topics 0.00725
c. 0.0725

~
A. irrational number D. 0.725
Cardinal and Ordinal Numbers
B. real number
Mon Numerals and Digits C. imaginary number 14. EE Board April1993
System of Numbers D. a variable

D
Tue
-Natural numbers, Integers,
Rational numbers, Irrational 8. ECE Board Marc:h 1996
The number 0.123123123123 ........ is
Express decimally: Four and two tenth

A.
B.
0.042
4.02
numbers & imaginary numbers

D D
Theory Wed
Complex numbers
Types of Fractions
A.
B.
irrational
surd
C.
D.
4.2
0.42

C. rational .A$~/ECE Board November 1995


~
Problems
D
Thu
Composite Numbers
Prime Numbers
Defective and Abundant Numbers
D. transcendental

/«"ECE Board April1991


Express 45" in mils

A. 80 mils

D D
Solutions Fri
Amicable Numbers
Significant Figures and Digits
Forms of Approximation
Round off 6785768.342 to the nearest
one-tenth.
B.
C.
D.
800 mils
8000mils
80000 mils
A. 6785768

D D
Notes Sat
Conversion B.
C.
D.
6785768.4
6785768.3
None of these
~.fi: ME Board April1997
What is the value in degrees of 1 radian?

A. 90
10. EE Board April1993 B. 57.3
•• ME Board October 1996 D. 34.0 Express decimally: Fourteen Ten C. 100
"ifow many significant digits do 10.097
thousandths D. 45
have? 4: ME Board April 199&
·'which number has three significant A. 0.0014
A. 2 figures? Jl'/• CE Board May 1993
B. 0.00014 3200 mils is equal to how many degrees?
B. 3 C. 0.014
c. 4 A. 0.0014 D. 0.14
D. 5 A. 45"
B. 1.4141
B. 90"
C. 0.01414 u. ECE Board Marc:h 1996
,£icE Board April1991 C. 180"
D. 0.0141

il Round off 0.003086 to three significant


figures. ~1!:CE Board April1991
'Round off 149.691 to the nP.arest integer
MCM~CIV is equivalent to what number?

A: 1964
D. 270"

18. ECE ~rd November 1995


B. 1994 An angular unit equivalent to 1/400 of the
A. 0.003
C. 1984 circumference of a circle is called
B. 0.00309 A. 149.69 D. 1974
C. 0.0031 B. 149.7
D. 0.00308 A. mil
C. 150 12. EE Board April 1993 B. degree
D. 149
Express decimally: Fourty-Sevenmillionth C. radian
Y,ECE Board April1991 __.x.
D. grad
Round off 34.2814 to four significant &~CECE Board April1991 A. 0.00000047
figures. Round off 2.371 x 10"8 to two significant B. 0.0000047
A. 34.2814
figures. c 0 000047
D 0. 00000004 7
B. 34.281 A. 2.4 X 10·8
C. 34.28
12 lOOi Solved Problems in Engineering Mathematics (2nd Edition) by Tiong & Rojas
Day 1 - Systems of Numbers and Conversion 13

.19. EC:E Board April1999 zs. ME Board


1
October 1996 ,:Ji:· If a foot has 12 inches, then how C. 441 statute mile
4800 mils is equivalent to __ degrees. 10 to the 12 h power is the value of the
many hands are there in one foot? D. 414 nautical mile
prefix
A. 135 A. 3 57· EE Board October 1994
B. 270 A. micro B. 4 ' Carry out the following multiplication and
C. 235 B. femto c. 6 express your answer in cubic meter:
D. 142 C. tera D. 8 8 em x 5 mm x 2in.
D. atto
,.a( ME Board April 199ft :JZ~ How many feet difference is 1 nautical A. 8x 10"2
How many degrees Celsius is 100 deg~::ees :&Cn EE Board October 1994 ' mile and 1 statute mile? B. 8x 102
Fahrenheit? The micro or ~ means C. 8X 10"3
A. 100 feet D. 8x 10_.
A. 2.667° C A. 10"2
B. 1.334°C B. 10-e B. 200 feet :JS(vVhich of the following is equivalent to
c. 13.34° c C. 10"3 C. 400 feet ·'1 hectare?
12
D. 37.8° C D. 10" D. 800 feet
A. 100 ares
u. EE Board October 1990 :&7: RME Board October 1994 :J~ In a hydrographic survey, a certain a. 2 acres
What is the absolute temperature of the The prefix pico means point below the surface of the water C. 1000 square meters
freezing point of water in degree Rankine? measures 12 fathoms. It is equivalent to a D. 50000 square feet
A.· 10"12 of a unit deep of how many feet?
A. 492 B. 10-e of a unit :J9. Ten square statute miles is equivalent
B. 0 C. 10"15 of a unit A. 72 · to sections.
c. 460 D. 10·9 of a unit B. 60
D. 273 C. 48 A. 100
:&8. ME Bo8rd April1999 D. 36 B. 5
-~ "8 Board October 1994 The prefix nano is opposite to C. 10
·What is the Fahrenheit equivalent of 100 w; The legendary ship, Titanic that sunk D. 20
degrees Celsius? A. mega in 1912 was estimated to be at the sea
B. giga bottom at a deep of 18 cables. How deep it 40. The land area of the province of Cebu
A.. 200 C. tera is in feet? 'is 5088.39 sq. km. This is equivalent to
B. 180 D. hexa
c. 212 A. 12,000 A. 5088.39 hectares
D. 100 :&9•1 foot is to 12 inches as 1 yard is to B. 12,343 B. 1964.64 sq. miles
_·_spans. c. 12,633 C. 2257907.2 acres
Z31EE Board Aprii199:J D. 12,960 D. 5.08839 acres
The temperature 45° C is equal to A. 4
B. 6 . :~s: ME Board October 1994 .u~"'u Board October 1994
A. 45° F c. 9 How many square feet is 100 square ,/How many cubic feet is 100 gallons of
D. 24
II B.
c. 5rF
D.
113° F

81° F / :J{). EE Board .Juae 1990


meters?

A 328.1
liquid?

A. 74.80
A one-inch diameter conduit is equivalent B. 929 B. 1.337
Z4;ME Board October 1994 to C. 32.81 C. 13.37

I) · How many degrees Celsius is 80 degrees D. 1076 D. 133.7


Fahrenheit? A. 254mm /

B. 25.4 mm :56. A certain luxury ship cruises Cebu to ,.u;·io ~rd Octo.,...1994
A. 13.34 C. 100mm Manila at 21 knots. If it will take 21 hours ' ME Board April1998
B. 1.334 D. 2.54mm to reach Manila from Cebu, the distance How many cubic meters is 100 gallons of
c. 26.67 traveled by the ship is nearly liquid?
D. 2.667
A. 847.5km A. 1.638
B 507 15 statute mile B. 3.785
14 .1001 Solved Problems in Engineering- Mathematics (2nd Edition) by Tiong & Rojas
5
C. 0.164 C. 10
D. 0.378 D. 104

4~.-ME Board October 1994 49_.ME Board April ~:99s


.·How many cubic meters is 100 cubic feet c5'ne horsepower is equivalent to
of liquid?
A. 746 watts Topics

~
A. 3.785 B. 7460 watts
B. 28.31 C. 74.6 watts Cardinal and Ordinal Numbers
C. 37.85 D. 7.46 watts Mon Numerals and Digits
D. 2.831 System of Numbers

!l'>
4.4•-'f~n (10) cubic meters is equivalent to
r'llow many stere?
$0(ME Board October 1994
"'1iow many horsepower is 746 kilowatts? 0
Tue
- Natural numbers, Integers,
Rational numbers, Irrational
numbers & imaginary numbers
A.
B.
C.
5
10
20
A.
B.
c.
D.
500
74.6
100
1000
0
Theory
0
VVed
Complex numbers
Types of Fractions
D. 100

45. ME Board Aprii199S


0
Problems
0
Thu
Composite Numbers
Prime Numbers
Defective and Abundant Numbers
The standard acceleration due to· gravity is

A.
B.
32.2 ft/s 2
980 fUs 2
·:;..:.

Solutions
0
Fri
I
Amicable Numbers
Significant Figures and Digits
Forms of Approximation
C.
D.
58.3 fUs 2
35.3 fUs 2 0Notes
D
Sat
Conversion

46. ME Board October 1996


A ?kg mass is suspended in a rope. What
is the tension in the rope in Sl?
ANSWER KEY RATING
A. 68:67 N 1. D 14. c 27. A 40. B
B.
C.
70 N
71 N
2. B
3. c
15. B
16. B
28. B
29. A
41. c
42. D
c:J 43-50 Topnotcher
D. 72 N 4. D
5. c
17. c
18. D
30. B 43. D ' c:J 30-42 Passer

47. A 10-liter pail is full of water. 6.A 19. 8


31. A
32. D
44. B
45. A c:J 25-29 Conditional
Neglecting the weight of the pail, how
heavy is its water content?
7. B
8. c
9. c
20. D
21. A
22. c
33. A
34. D
35. D
46.A
47. c
48. B
0 0-24 Failed

A. 5kg . 10. A 23. 8 36. 8 49.A IfF AILED, repeat the test.
B. 6.67 kg 11. 8 24. c 37. D 50. D
c. 10 kg 12. c 25. c 38. A
D. 12.5 kg 13. 8 26. 8 39. c
,.,._ l<
~ '-~
48;;'The unit of work in the mks system is
.<known as joule (J) and the unit of work in
the cgs system is erg. How many ergs are
there in one joule?
6
A. 10
7
B. 10
16 .1061 Solved Problems in Engineering Mathematics (2nd Edition) by Tiong & Rojas ________________D_a...y_l_-_S_y._s_t..,..e_ms
__o_f_N_u_m
__,bers and Conversion 17

a Ill
MCMXCIV = M CM . XC IV
II
Using the formula,
m
10·6 means micro
The number 10.097 has 5 significant
·c=~(F-32)
figures.
= 1000 + 900 + 90 + 4
= 1994 9 El
II El ·c=%(100-32) The prefix pico is equivalent to 10'12 of a
unit
The number 0.003086 when rounded off Fourty-seven millionth = -~
to three significant digitsbecomes 1000000
Fourty-seven millionth = 0.000047
°C=37.8°C
El
0.00309.
Ell The prefix nano is equivalent to o·
1 9 of a
II lEI The freezing point of water is equal to
unit while the prefix giga is equivalent to
9
10 of a unit . ·
:t·
The number 34.2814when rounded off to
four significant digitsbecomes 34.28.

II
Seven hundred twenty-five hundred
thousandths =
725
100000
= 0.00725
32•F oro•c.
0

0
R= "F+460
R=32°+460
m1 span is equivalent to 9 inches
0.0014 has two significant figures
1.4141 has five significant figures
Ill 0
R =492°R • 1 yard = 3 ft = 36 inches, thus
. 1 span


Four and two tenth =4.2 36 mches x - - - - :::: 4 spans
0.01414 has four significant figures
El 9inches
0.0141 has three significant figures m-Ans
Using the formula, El
II By ratio and proportion:
X 45°
•F =~(•c)+32
.
1 em
10mm
1 mch = 2.54 em x--- =·25.4 mm
The number 149.691 when rounded off to
the nearest integer becomes 150.
6400 mils= 360°
.
°F=~(100)+32
5 II
x =800 mils

II lrl
°F = 212°F 1 hand is equivalent to 4 inches, thus

m
8 . hand
The number 2.371 x 10' when rounded 1 foot= 12 mches x - .-h-
off to two significant digitsbecomes 2.4 x By ratio and proportion:
4 1nc es
10'8 .
1 foot = 3 hands
x 1 rad

m
Using the formula,

II 360° .= 2Jt rad


X= 57.3°
°F=*(•c)+32
7 + Oi = 7 thus, the answer is, "real 1 statute miie = 5280 ft.
number". °F=~(45)+32 1 nautical mile = 6080 ft

II Ill °F=113°F
Let: x = the difference between a nautical

m
By ratio and proportion:
Repeating decimal number is a "rational x 3200 mils mile and a statute mile
X ::6080 - 5280
number". 360° = 6400 mils
x = 800 feet
II X= 180° Using the formula,

·c=~(F-32) El
The number 6785768.342 when rounded
off to the nearest one-tenth becomes
6785768.3.
m oc
9
=~(a0-32)
9
1 fathom is equivalent to 6 feet, thus
12 fathoms= 6(12) = 72 feet
Grad

II 14
Ill oc = 26.67•c
Ell
1 cable is squivalent to 120 fathoms,
Fourteen ten thousandths=
10000
Fourteen ten thousandths= 0.0014
By ratio and proportion:
x 4800 mils
360° = 6400 mils
m thus:
18 cables= 120(18) = 2160 fathoms
I hu prefix tera is equivalent to 1012 of a 6feet
18 cables= 2160 fathoms x---
X =270° unit fathom
18 100 1'Solved Problems in Engineering Mathematics (2"d Edition) by Tiong & Rojas
Day 1 -Systems of Numbers and Conversion 19
ft3 3

-
18 cables= 12 960 feet 100 gallons x _ = 13.37 ft
' 7.48 gallons
II
1 meter is equivalent to 3.281 ft, thus
1 m 2 = (3.281 )2 ft
2
m
1 gallon = 3.79 liters
1 horsepower = 746 watts

1 m 2 = 10.76 ft
2 1000 liters = 1 cubic meters
100m2 = 100(10.76) ft
100m2 = 1076 ft
2
2
3.79 li m
100 gallons x - - x - - -
gallon 1000 li
3
II
1 hp = 746 watts
3
1 hp = 0.746 kilowatts
El 100 gallons= 0.379 m

Solving for distance,


D = Vt
V = 21 knots = 21 nautical miles
m
Given volume is 100 cu. ft.
746 kW x. hp =1000 hp

3
D=21(21)
hour
V = 100 ft 3x(-m-)
3.281 ft
D = nm x 1.15 statute miles V = 2.831 m
3
441
nautical mile

Iii
=
D 507.15 statute mile
m
1 cubic meter = .1 stere, thus,
10m 3 = 10 steres
1m
8 em

5 mm x
x-~-=
100 em
1
m
1000 mm
0.8 m

= 0.005 m
m
The following are the standard
0.08(0.005)(2) = 8 x 1
4
m
3
o· gravitational acceleration:
2

m 32.2 ft/s
981 cm/s 2
9.81. m/s 2
1 hectare = 100 ares
&Core: 1 are= 100 sq. meters
1
1 hectare = 100 ares x - 0.:..0.::_::_sq.:.o.·-.-m.:..
m
The unit of force (tension) in the Sl
1 are
system is newtons (N).
1 hectare= 10,000 sq. meters
9
x( -~~
m Tension= 7 kg m) = 68.67 N

1 square statute mile = 1 section


10 square statute mile = 10 sections
El
Density of water ( p ) = 1000 _k~
m
1 square km = 0.386102 sq. miles
,,~ kg
Density of water ( p ) = 1 -.-
liter
mo

A= 5088 _39 km 2 x 0.3861 02" miles


2 W= p·V
km W=1 ~ x 10 liters = 10 kg

-
A = 1964.64 sq. miles liter

m 7
1 cubic ft. = 7.48 gallons 1 joule= 10 ergs
22 100 1 Solved Problems in Engineering Mathematics ~2~d Editionl by Tiong &: Roiaa

Topics

0
Mon
Properties of Addition of 1nteger
Properties of Multiplication of
Integers
Properties uf Equality
Tue Properties of Zero
I" Properties of Exponents

Theory
0
Wed
Properties of Radicals
Surds

0
Problems
0
Thu
Special Products
Properties of Proportion
Least Common Denominator

0
Solutions
0
Fri
Least Common Multiple
Greatest Common Factor
Remainder Theorem

0
Notes
LJ
Sat
Factor Theorem

What are properties of integers? 4. Identity property

Integers have special properties. a+O=a


Computations of integers will become
easier by understandir;g these special The num 2r 0 is called the
properti~s. The commutative property, for
additive identiy
instance, allows you to change the order of
adding or multiplying while the associative 5. Inverse property
property allows you to change grouping.
a +(-a)= 0
The properties of adpitiori of integers:
The number -a is called the
Closure property additive inverse
\.

a + b = integer 6. Distributive property

2. Commutative property a(b+c) = ab:t-ac

a+b=b+a
The properties of multiplication of
integers:
:\ Associative proper1y
Closure properly
(a 1 ll) 1 < " 1 II' 1 ' )

o~h illl••<i<'l
24 ~ 00 I· Solved Problems in Engineering Mathematics (2nd Edition) by Tiong & Rojas Day 2 -Fundamentals in Algebra 25
2. Commutative· property 4. Substitution property would say that 3 is the power and that Property Example
"power" and "exponent" mean the same
If a =b, then a can be replaced
ab=ba
by b in any expression inl'olving .
thing. The misuse has probably come from
a misunderstanding of statements such 1.
0
vam
r,;;
=(
m
cya) ~ =(Wf =22
3. Associative property a "nine is the second power of three". ~=4
(ab)c=a(bc) 5. Addition I Subtraction property The exponential notation states that if a is 2. cya ·cyb = 'fab ¥5 ·~675
a real number, variable or algebraic
If a = b, then a + c = b + c expression and n is a positive number, = ~(5)(675)
4. Identity property
If a = b, then a - c = b - c then:
a+1=a a" = a · a · a · a · · · · = ~3375 = 15
6. Multiplication I Division property "---y---J
The number 1 is called the n factors
~r If a = b, then ac = be
multiplicative identiy
If a = b, then ! = ~ with c ~0 The properties of exponents with 3. cya~
- = n - b;t:O
% b'
we= 3rso =¥5
5. Inverse property c c corresponding examples: V10 \/10
7. Cancellation property Property Example 4. 'ifiFa = mzya ~~=1tfl5
a(;)=1 1. am +a"= am+n
(~f =2x
x2 + xa = x2+3 = xs
lfa+c=b+c,thena=b 5. (cya)" =a
The number ~- *
If ac = be and c 0 , then a =b am m-n 8
a is called the . 2. -=a X a-3 s
~(-12) 4 =l-121
-=X =X
multiplicative inverse The properties of zero: a" x3 ?t !fa"= lal
Consider a, b and c as integers or real 2
3. (am)" =amn (y6) = y12 :::12
numbers or variables of an algebraic
6. Distributive property expression. (For n =even no.)
4
4. (abt =ambm (2x) =2 4 x 4 =16x 4
a(b+c)=ab+ac 1. ·. a+O=a and a-O=a !fa"= lal ~(-12)3 = -12

7. Multiplication property of zero


2. a(O)=O
5.
(~r = : : (~r=24=~
4 4
(For n =odd no.)
3. ~ == 0 , with a ;t 0
x x x
a m What is a surd?
a(O) =0
a"=~ 5
(4x)3 = 3 (4x)
((44
4. ~ is undefined
6.
Surd is a radical expressing an irrational
The properties ofequality of integers:
Consider a, b and c as integers or real
5. If ab = 0, then a = 0 or b =0. This is 7. a
-m
=-
1
I(
-5 =1- number. The surd is described after the
numbers or variables of an algebraic
known as Zero-Factor property am x5 index of the radical. For example, .J3 is a
expression. quadratic surd, ~ is a cubic surd, ~ is a

·1. Reflexive property


What Is an exponent?
8. a0 = 1 (a;;, 0) (x
2
+2t =1 quartic surd and so on.

Exponent is a number that gives the Different types of surds:


a=a What is a radical?
power to which a base is raised. For
example, in 32 , the base is 3 and the Pure surd, sometimes called an entire
2. Symmetric property Radical refers to the symbol that indicates surd contairls no rational number and all
exponent is 2.
a root, F.
It was first used in 1525 by its terms are surds.
If a = b, then b =a
Exponent should not be misunderstood as Christoff Rudolff in his Die Coss. Example: .J3 + J2 .
"power" Power is a word that is almost
3. Transitive property
never used in its correct, original sense In the expression, cya ,
n is called the Mixed surd is a surd that contains at least
any more. Strictly speaking, if we write
If a = b and b = c, then a = c
32 = 9, then 3 is the base, 2 is the
Index, a (the expression inside the one rational number. s.J3 is a mixed surd
symbol) is called the radicand while the
exponent and 9 is the power. But almost because 5 is a rational number while .J3 is
everyone, including most mathematicians,
symbol J is called radical a surd.
26. 1001 Solved Problems in Engineering Mathematics (2nd Edition) by Tiong & Rojas Day 2 - Fundamentals in Algebra 27

Binomial surd is an expression of two What is a proportion? Solution: GCF = 2(7)


ter111s with at least one term a surd.
GCF = 14
Example: 5 + F2 Proportion is a statement that two ratios 8 =2 3
are equal.
9 = 32 What is a Remainder Theorem?
Trinomial surd is an expression of three 2
Properties of proportion 12=3·2
terms with at least two or. them are surds Remainder Theorem states that if a
and cannot be expressed as a single surd, 15=3·5 polynomial in an unknown quantity x is
otherwise it will become a binomial surd. a x
1. If - = - . then a : x = y : d divided by a first degree expression in the
y d 23 (32 )(5)
Example: 5 + F2 + J3 LCD= same variable, (x- k), where k may be
any real number or complex number, the
a c a b LCD= 360 remainder to be expected will be equal to
2. If - =- then - =-
b d. c d the sum obtained when the numerical .
What is a special product?
What is a least common multiple value. of k is substituted for x in the
a c b d (LCMI? polynomial. Thus,
Special products are the expressions 3. If - =- , then - =-
b d a c
where the values can be obtained without A common multiple is a number that two
execution of long multiplication. 4. If~=~ then a- b = c- d other numbers will divide into evenly. The remainder = f(X)
b d. . b d least common multiple (LCM) is the x->k
With x, y and z as real numbers or lowest multiple of two numbers.
variables or algebraic expression, the 5. If~ = ~ then a + b = c + d What is a Factor Theorem?
following are the special products. b d. b d Problema
What is the least common multiple of 15 Factor theorem states that if a polynomial
1. Sum and difference of same terms or 6 If ~ = ~ then a + b = c + d and 18? is divided by (x - k) will result to a
Difference of two squares · b d' a-b c-'d
remainder of zero, then the value (x- k) is
(X + y){X - y) = x2 - y2 In number ( 1), quantities a and d are Solution: a factor of the polynomial.
called extremes while x and y are called
2. Square of a binomial means. If x = y, then its value is known as 15=3·5 Both remainder theorem and factor
mean proportional. In the ratio xly, the theorem were suggested by a French
2
( x + y) = x + 2xy + y
2 2 18 = 3 .2
2
mathematician, Etienne Bezout (1730-
first term x is called the antecedent while
the second term y is called the 1783).
(x -
2
y) = x 2 - 2xy + l consequent. LCM = 32 (5}(2)
antecedent LCM = 90

r:
3. Cube of a binomial Proceed to the next page for your second
extremes
3 test. Detach and use the answer sheet
3 2 2 3 What is a greatest common factor
~a:x=y:dl
(x + y) = x + 3x y + 3xy + y provided at the last part of this book. Use
(GCFI?

·o
pencil number 2 in shading your answer.
3 2 3
(x- yf =x - 3x y + 3xy2- y

u
A factor is a number that divides into a
GOOD LUCK I
4. Difference of two cubes larger number evenly. The greatest
common factor (GCF) is the largest
means
3 3
x -y =(x-y)(x
2
+xy+l) consequent number that divides into two or more
numbers evenly. 'Orribia:
5. Sum of two cubes What is a least common denominator Did you know that. .. the two long parallel
(LCD)? Problema lin;)S (=)as a symbol for equality was
3
x +y
3
=(x- y)(x 2
- xy + l) What Is the greatest common factor of 70
and 112?
introduced by Robert Recorde in 1557!
Least common denominator (LCD)
6, Square of a trinomial refers .to the product of several prime
numbers occurring in the denominators, Solution: ~note:
2 2 2 2
( x + y + z ) = x + y + z + 2xy :t 2xz + 2yz each taken with its greatest multiplicity. "Among the great things which are found
70 =2. 5. 7 among us, the existence of Nothing is the
greatest,"
Problem: 112=2 4 -7
What is the least common denominator of - Leonardo da Vinci
8,9,12and15? • ' Common factors aro 2 and 7.
Day 2 - Fundamentals in Algebra 29
,.,,}

SS'~CME Board October :1995 D. x=-2,y=-3,z=-1


Solve for the value of x and y.
4x + 2y = 5 c.o. EE Board Apri11997
13x -3y = 2 Multiply the following: (2x + 5y)(5x- 2y)

A. y = 1/2, X = 3/2 2
A. 10x - 21xy + 10/
Topics B·:
C.
y =3/2, x =112
y = 2, X= 1
B.
C.
2
-10x + 21xy +
10x2 + 21xy- 10y
10t
0Mon
Properties of Addition of Integer
Properties of Multiplication of
D. y = 3, X = 1

Sb. ME Board October :199&


D. -10~- 21xy -10y2

61. EE Board March 1998


Integers
~
Solve the simultaneous equations: Determine the sum of the positive valued
Properties of Equality 2x 2 -3y2 = 6 solution to the simultaneous equations: xy
:t- Properties of Zero = 15, yz = 35,.zx = 21.
Tue 3x 2 +2y 2 = 35

u
Theory
0Wed
Properties of Exponents
Properties of Radicals
Surds
A.
B.
C.
x = 3 or -3;
x = 3 or -3,
x = 3 or -3,
y = 2 or -2
y = -2 or 1
y = -2 or -1
A.
B.
c:
D.
15
13
17
19

~
Problems
0Thu
Special Products
Properties of Proportion
Least Common Denominator
D. =
x 3 or -3,

57. ECE Board May :1997


y = 2 or -3
c.z. ECE Board April1991
2 3 2
t (x-3 yz3 f~
0
Solutions
0 Fri
Least Common Multiple
Greatest Common Factor
Remainder Theorem
Find the value of w in the following
equations:
3x- 2y +w = 11
x + 5y -2w = -9
Simplify: (x y z-

( xyz-3 )-~

0
Notes
0 Sat
Factor Theorem
A.
2x+y-3w = -6

3
A.
x2y1zs
B. 2
C. 4 B.
<,sj~<ECE Board April :1999 D. -2 2Y
Xy
If 16 is 4 more than 4x, find 5x- 1.
10[~+~ ]=A ss. EE Board October 199~ c. 1
~
Xy
2[3~-4~]=A
A. 14 Solve for the value of x.
B. 3 2x-y+z=6 1
D.
C. 12 x-3y- 2z = 13 ~
Xy
D. 5 2x- 3y- 3z = 16
A. 50/9
6~. ECE Board November 199~
si~ EE Board October :199:l B. 80/9 A. 4 Simplify the following equation
C.· 70/9
x
F .md t h e va Iue o f x .m -+-1 + -2x = 4 7 - 2 x . D. 60/9
B. 3
3 4 C .. 2 5x x +3 2x+1
D. 1 -=---- - + -:::---
2
54. EE Board October :1997 2x2 + 7x + 3 2x 2 - 3x - 2 x +x- 6
A. 16.47
' Find the values of x and y from the
B. 12.87 S91 ME Board October 199&
equations: 4
C. 18.27 Solve the simultaneous equations: A
X - 4y + 2 = 0
D. 20.17 x+y=-4 X+3
2x + y -4 = 0
x+z-1=0 2
n: EE Board October :199:1 A. 11/7, -5/7
y+z+1=0 B. -
x-3
'"Find the value of x in the equations:
B. 11/9, 8/9 A X "- -1, y = -5, Z = 3 C. _j_
C. 4/9, 8/9
II X ::: 1, y =2, Z =-3 x-3
D. 3/2, 5/3 t; X ~ ·1, y =: -3, Z = 2 '
30 1001 Solved Problems in Engineering Mathematics (2"d Edition) by Tiong & Rojas Day 2 - Fundamentals in Algebra 31

D. a- b C. 2 78. Find the value of k so that 4~ + 6x + k


2 D. 2.5
D. is a perfect square.
X+3
&8. ME Board October :199&
. 7~d:ECE Board April :199~ A. 36
Reso Ive X+2 'Ifract1on.
.mto part1a .
4( 52n+1) _ 10( 52n-1) B. 2.5

rr
&4, ECE Board April1.99:l · x2 - 7x + 12 ,Evaluate: y = C. 9
2(5 2n)

~,l [,-!,-l (,,,-Tl


D. 2.25
6 2
s;mplify A. -----
x-4 x-3 A. y = 5" 79· ME Board Aprill.995
B. y =9
B. -3
- - -5-
x-4 x-3
C. Y = 52n Factor the expression 3x 3 - 3x 2 -18x
5 D. y = 18
A.
y2 C. -6- - -5- A.
B.
3x ( X - 3 )( X + 2 )
3x ( X + 3 )( X + 2 )
X x-4 x-3 74. ECE Board April :1990
7 C. 3x ( X + 3 )( X - 2 )
3
y2
D. - - - -5-
x-4 x-3
Given: (an)( am)= 100,000 D. 3x (X- 3 )(X- 2)
B.
X ~=10
5 &9. CE Board May :199& am so. If p - q = 5 and pq = ~ , then p2 + q2
y2 Find the value of A in the equation: Find a:
c. equals
~ x 2 + 4x + 10 A B(2x + 2)
--=--~- = - + -::-'------"-
3 x3 + 2x 2 + 5x x x 2 + 2x + 5 A. 12
A. k
y2 B. 9
D. c B. 25k
~ + x 2 +2x+5
C. 11
G. 25 + k
D. 10
A ~
D.
k
&s. ECE Board April :199:1 a 1~
75. ECE Board November 1.991. 25
Simplify: 7 8
+
2
- 8(7) "" 1 + 5(7)8 + 49(7)8 - 2
8
c. -1~
~ 2 Give the factors of a 2 - x2 .
81.. ME Board Apri11.995
A. -5a Simplify bm/n
B. -3a 11o. ME Board October 199&
A. 2a -2x
The value of (3 to 2.5 power) square is B. ( a + X )( a -X )
C. -7a
D. -4a equal to: C.
D.
( a + x )( a + x )
2x-2a A.
Jbffi
n
&&. Solve for x: A. 729
7&. ME Board April1.99& B. bm+n
2 2 B. 140
( b -4b+16) ( b -16)
X = ~-----:::-'--'-----<­
C. 243 Factor the expression x 2 + 6x + 8 as c. ifbiTi
b3 +64 D. 81 completely as possible. bm
D. -
"'!1/1. Evaluate: 64x . 4Y. A. { X+ 4 )( X + 2 ) n
A. b+4
b-4 B. ( X- 4 )( X + 2 )
B. A. 256"Y C. ( X- 4 )( X- 2 ) 82. ME Board Aprill998
b+2 Find the value of x which will satisfy the
2
B. 4x+3y 0 .. ( X + 6 )( X + 2 )
b -4 C. 64x+ 3y ,Jx- 2 = JX + 2.
c. - - D. 43x+y
following expression:
b+2 77. ECE Board November 1990
D. b-4 (a-b) 3 =? A. 3/2
/(2. ECE Board Aprlll99~ B. 9/4
&7. ECE Board April :199~ Solve for x in the following equations. C. 18/6
3
A. a - 3a 2 b + 3ab 2 + b 3
Solve for : _x_ = _Y_ = _z_ 27x = 9Y 3
D. None of these
Y (b-e) (a-c) (a-b) LJ. a -- 3a 2 b- 3ab 2 - b3
81Y 3-x = 243
2
<: a:I + 3a b + 3ab 2 - b3
A. x-z
A. 1 Il ;~·
1 2
3a b + 3ab 2
-· b3
B. x+z
B. 1.5
C. a+ b
3Z 1001 Solved Problems in Engine~ring Mathematics (2"d Edition) by Tiong & Rojas

B. (n -1)!
8~. Simplify V~ ~ C.
B. 11. 99· ECE Board April :1998
n! c. 15 The arithmetic mean of 80 numbers is 55.
D. (n- 1)" D. 13 If two numbers namely 250 and 850 are
removed, what is the arithmetic mean of
A. ~ 89. What is the least common multiple of
.c,!S;EcE Board April 1999 the remaining numbers? ·.
15 and 18? "' "'t>iven: f(x) = (x + 3)(x- 4) + 4. When f(x) is
B. Jab divided by (x - k), the remainder is k. Find
ab A. 42.31
A. 3 k..
c. 'Jab B. 5
B. 50
c. 38.62
ab C. 90 A. 2 D. 5'7.12
D. D. 270 B. 4
~
C. 6 :lOO. ECE Board Aprill.998
90. ECE Board April 1998 D. 8
,)14. ME Board April 1996 The arithmetic mean of6 numbers is 17. If
What is the lowest common factor of 10
I~" two numbers are added to the
,. If x to the 3/4 power.equals·8, x equals and 32?
96. ·The expression x4 + ax3 + 5~ + bx + 6 progression, the new set of numbers will
A. -9 when divided by (x - 2) leaves a have an arithmetic mean.of 19. What are
A. 320
B. 6 remainder of 16 and when divided by (x + the two numbers if their difference is 4?
B. 2
c. 1) leaves a remainder of 10. Find a and b.
9 c. 180
A. 21, 25
D. 16 D. 90 ·
A. a= 5, b = 7 B. 23,27
8S. 9:1. The numbers 12 and 16 has the
B. a= -5, b 7 = C. 8, 12
C. a=-5,b=-7 D. 16,20
Solve for x: .Jx + 2·J2x + 3 -3 = 0 greatest common divisor of
D. a= 5, b = -7
A. 2
A. 3 97. The mean of x and y is a, the mean of
B. 4
B. 23 y and z is b and the mean of x and z is c. ·
C. 6
C. 3 and 23 What is the mean of x, y and z?
D. 192
D. 20
/
92. EE Board April 1996 a+b+c
A.
.s'- CE Board November 1991 EE Board March 1998 3
Solve for X from the given equation:
The polynomial x 3 + 4x 2 - 3x + 8 is a+b+c
B.
~8 ~2.J8X =2 divided by x - 5, then the remainder is, 2

A. 4 A. 175 a+b+c
C.
B. 2 B. 140 abc
c. 3 C. 218 abc
D. 5 D. 200 D. --·
a+b+c
9~· Find the quotient of 3x5 - 4x + 2>t? +
3
r . 3 98. · ECE Board April 1999
..,..u Board October 1991 36x + 48 divided by x -2>t? + 6.
Find the mean proportional of 4 and 36.
Hf(x) = 2>t? + 2x + 4, what is f(2)?
A. 3>t? - 4x - 8
A. 72
A. 4x + 2 B. 3>t? + 4x + 8
B. 24
B. 16 C. 3>t? - 6x - 8
C. 12
C. ,(l +X+ 2 D. 3>t? + 6x + 8
D. 20
D. 8
94~<CE Board November 1991
88. EE Board Aprll199'7 Find the remainder if we divide ·
If ri is any positive integer, when (n-1)(n- 4y 3 + 18y 2 +By- 4 by (2y + 3).
2)(n-3) ... (3)(2)(1) =
A. 10
A. e<n·1l
Day 2 - Fundamentals in Algebra 35

II 14
X
18
10
16=4X+4
x=3 140
X=-
18
5x-1=5(3)-1 70
Topics X=-
9
5x -1= 14
froperties ~~Addition of Integer
D
Mon Properties of Multiplication of
Integers
II Bl
x-4y+2 =0

!"
.__ Tue
Properties of Equality
Properties of Zero [ x; 1 + 2: = 47- 2x} 2
x=4y-2
2x+y-4,;0
--+ Equation 1
--+ Equation 2
Properties of Exponents

D D
Theory Wed
Properties of Radicals
Surds
4x+4+6x = 564-24x
34x=560
Substitute equation 1 to equation 2:

2(4y-2)+y-4=0

D D
Special Products X= 16.47
8y-4+y-4=0
Properties of Proportion
Thu 9y=8
Least Common Denominator
Problems
Ill
~l D
Least Common Multiple y =% --+ Equation 3
Greatest Common Factor
10 [A + A]=A}-1 Substitute equation 3 to equation 1:
Solutions Fri Remainder Theorem { X y 10A

D D
. Notes Sat
Factor Theorem
1 1
-+-=-
X
1
Y 10
X=4(%)~2
32
X=--2
.!.--10-
Y
1 x1 --+ Equation 1 14
9
X=-
RATING 9
ANSWER KEY
1. D
2. 8
3. c
14. c
15. 8
16. 8
27.A
28. 8
29. A
40. 8
41. c
42. D
0
0
43-50 Topnotcher

30-42 Passer
{2[3~-4~]=A}1 m 4x+2y = 5
4.0 17. c 30. 8 43. D
5. c
6.A
18. D
19. 8
31. A
32. D
44. 8
45.A
D 25-29 Conditional ~-.!!. =1
y
--+ Equation 2 y= %- 2x --+ Equation 1
7.8
8. c
20. D
21. A
33.A
34. D
46.A
47.C
D 0-24 Failed
X
13x- 3y = 2 --+ Equation 2
Substitute equation 1 in equation 2:
9. c 22.C 35. D 48.8
36. 8 49.A Substitute equation 1 to equation 2:
10.A 23. 8 If FAILED, repeat the test.
11. 8 24. c 37. D 50. D ~-a(_!__.!.)= 1
12. c 25. c 38.A X 10 X
13x-3(%-2x) = 2
13. 8 26. 8 39.C 6 8 8
---+-=1
' ' ' A -~ " ' -•o "'"''"''
X 10 X 18x=2+~
.!i= 1+~ 2
X 10 18x=~
2
X=.!_
2
36 100 !··solved Problems in Engineering Mathematics (2nd Edition)' by Tiong & Rojas Day 2 - Fundamentals in Algebra 37

y =%-2(~)=%
Substitute equation 5 in equation 7:

9[7w1;38]-w=-12
Substitute equation 4 in equation 3:

y+(5+y)+1=0
m
(x2y3z·2t (x-3yzT! (x...sy-9zs){x~y--lz-!)
63w- 342 -17w = -204 2y =6 (xyz· 3 r~ x·~y-~zlf
~~ ( 2x
2
-
2
3y = 6 ) 3 w=3 y = -3
Solving for z:
6x 2 -9y 2

( 3x 2 + 2y = 35 ) 2
=18-7 Eq.

2
1
m 2x- y + z = 6
x- 3y- 2z = 13
-7 Eq. 1
-7 Eq. 2
z = 5 + (-3) =x -6+1+~
22y
-9-1+~ 6-1-U
22Z22

Z=2
2x- 3y- 3z = 16 -7 Eq. 3 1
t>x 2 2
+ 4y = 10 -7 Eq. 2 -=X
-2 y· 7 z -3 = 2-
X.Y
7 z3
Solving for x:
)fl Subtract equation1 from equation 2:

2
6x + 4y 2
- ( 6x 2 - 9y2 ) = 70-18
Subtract equation 2 from equation 3:
X= -4- (-3)
X= -1
m
( 2x - 3 y - 3z) - ( x - 3 y - 2z) = 16 -13 5x x +3 2x +1
13y2 =52 2. +-2-·-
2x 2 + 7x +3
y =±2
x-z=3
x=3+z ID 2x - 3x - 2 x + x - 6

Consider above equation as Eq. 4 5x x +3 2x+1


(2x + 5y){5x- 2y) = 10x2 - 4XY+ 25xy - +~~-
2 2 • (2x + 1)(x + 3) (2X+ 1)(x- 21 (x + 3)(x- 2)
6x - 9(2) = 18
-10y 2
6x 2 =54 Multiply equation 1 by equation 2:
6x - 3y + 3z = 18 -7 Eq. 5 (2x + 5y)(5x- 2y) = 10x2 +21xy -10y 2
X=±3 5x(x- 2)- (x + 3)(x + 3) + (2x + 1)(2x + 1)
Subtract equation 2 from equation 5: Ill (2x + 1)(x + 3)(x- 2)

Iii 3x - 2y + w = 11 -7 Eq. 1 (5x- 3y + 3z)- (x- 3y- 2z) =18-13 Multiply the three given equations: 5x 2 -10x-x 2 -6x-9+4x2 +4x+1
x + 5y - 2w = - 9 -7 E;q. 2 5x + 5z = 5 (xy)(yz)(zx) = 15(35}(21)
.\1~ - (2x + 1)(x + 3)(x- 2)
z = 1-x 2
2x + y- 3w = - 6 -7 Eq. 3 (xyz) . = 11025 8x 2 -12x-·8
Consider above equation as Eq. 6 xyz = 105 -:+ Eq. 4 (2x + 1)(x + 3)(x- 2)
Multiply equation 2 by equation 3:
3x+15y-6w=-27 · -7Eq.4 _ 4(2x 2
- 3x -.2)
Substitute equation 6 in equation 4: Substitute xy = 15, in equation 4: - (2x + 1)(x + 3)(x ~ 2)
X=3+Z
Subtract equation 4 by equatiqn 1: 15z =105 4(2x + 1)(x- 2)
x=3+(1-x)
(3x + 15y -6w) -(3x -2y + w) = -27-11 (2x + 1)(X+ 3)(x- 2)
2x =4 Z=7
17y -7w = -38 4
X=2 Substitute yz = 35, in equation4: X+3
7w-3
y=--
17 II X + y =-4 -7 Eq. 1 35x = 105
m
+ z - 1 = 0 -7 Eq. 2

Jr
X X=3
Value of y above is considered as Eq. 5

Multiply equation 2 by 2:
y + z + 1 = 0 -7 Eq. 3
Substitute zx = 21, in equation 4: { x~ [ x·sy·J (x y·2 r~
2

2x + 10y- 4w = -18 -7 Eq. 6


Subtract equation 1 from equation 2: 21y = 105
= x4 x· 3 y- 2 x- 3~ y3
1 1 • ]3
Subtract equation 6 by equation 3: Y=5 [

xtz-(x+y)=1-(-4)
(2x + 10y -4w) -(2x + y- 3wl= -18- (-6)

Consider the above equation as ;q. 7


9y-@= -12
z =5 + y -:+ Eq. 4 Thus, x + y + z = 3 + 5 + 7 = 15
=x
4
[x-~yi r= x4 [ x· 5 y~ J
38 100 i Solved Problems in Engineering Mathematics (2"d Edition) by Tiong & Rojas Day_g_- f'll.rldamentals in Algebra 39

~ A=1-B ~ Eq.1 6
(3) '(3t' ':' (3)
5
~~~ 2 2
a -· x =(a+ x)(a- x)
= x-1y~ = '£_ Equate constants:


6x- x = 5
X 2 = - 4A - 3B ~ Eq. 2
5x =5 IZI 2
x +6X+8 = (x+4)(x+2)
Substitute equation 1 in equation 2: X= 1
r· 2 - 8(7)8 +1 + 5(7)8 + 49(7)8 -
2 2 ::; - 4(1 -B) - 3B
2 =- 4 + 4B- 3B
Iii 3
(a-b) =a3 -3a 2b+3ab 2 -b 3

= r7 2 - (8)7.7 1 + (5)r +
49 7
< ">
72
B=6
A=1-6=-5
Ill 4( 52n+1) -1 o( 52n-1) a [ 2
4x + 6x + k = 0 l±
=r(49-56+5+1 >
X+2 6 5 y= 2(5 2") x 2 + 1.5x + 0.25k = 0
=-7" Thus, =-----

m
x2 - 7x + 12 x- 4 x- 3
y=
2(52n+1) _ 5( 5 2n-1}
52"
(X+ \n2 =0
i"
2
( b2 - 4b + 16 ) ( b -.16 ) m 2(5
12 2 1
-5 )-5(5 " ·5- )
"

r=0.~5k
Since it is a perfect square, then
x-- .
x2 +4x+10 A B(2x+2)
= ~ + -=-'"----'- y= 52"
b3 + 43
x3 + 2x 2 + 5x x x 2 + 2x + 5 1 1 5
2 y=2(5 )-5(5- ) [\1
( b - 4b + 16 ) ( b- 4 )( b +.4 )
x-~----~~~~~ + 2
c
y=9
- ( b + 4 )( b2 - 4b + 42 ) x +2x + 5 k = 2.25

X=b-4
A(x 2 +2x+5}+Bx(2x+2)+Cx Ill II
m (b-e) by-cy
x=y--=-.--
2
x('x + 2x + 5}

Ax 2 + 2Ax + 5A + 2Bx 2 + 2B)\. + Cx


(a"){am) = 100,000

a"
m = 10 ; a" = 10 am
~

~
Eq.1

Eq. 2
3x 3 - 3x 2 - 18x = 3x ( x 2 -- x - 6 }
3x 3 - 3x 2 -18x = 3x ( x-3 )( x + 2 )
a-c a-c x 3 +2x 2 + 5x a
(a-b) ay-by
z=y--=---
a-c a-c By equating constants: Substitute (2) in (1):
ED
p-q=5
by -cy ay -by By squaring both sides:
X+Z=--+-- 10=5A 1oamam = 100000
a-c a-c
ay-cy y(a-c) A =2
X+Z=---=--- (amt = 10000 (P- q)2 =52
a-C a-c
X+Z=Y 11 [ 25
(3) . J=243 am= 100 p 2
-
2
2pq + q = 25
p 2 + q2 = 25 + 2pq
II Substitute am =100, in equation 1:
ID 64x4y = (4)3x(4)Y = (4}3x+y
a" (100) = 100000
2
p +q
2
=25+2(~)
X+2 X+2
x 2 -7x+12- (x-3)(x-4) Iii {81)Y(3tx = 243 a" = 1000 p2 +q 2 = 25 + k
X+2 A B (3) 4Y(3tx = {3)5 ~ Eq. 1
=--. +--
bm/n = (bm)~ = ~
x2-7x+12 x-3 x-4
{27)x = {9)Y
(a")(am) =am"= 100000

(a")( am)= (a")"'= 100000


Ell
x+2 =A(x-4)+B(x-3)
(x- 3)(x- 4) (x- 3)(x- 4)
(3)3x = {3)2y
1000m = 100000 Ill Jx- 2 = JX + 2
By squaring both sides:
x+2=A(x-4)+B(x-3)
x + 2 = Ax - A4 + Bx - B3
Squaring both sides:
(3)ox = ( 3)4y ~ Eq.2
Substitute
m=2

m = 2, in equation 3:
( Jx- 2 r =( JX + 2 r
x-2=x+4JX+4
Equate coefficients of x: 2
Substitute equation 2 in equation 1: a = 100 4JX =-6
1=A+B
(3)4Y(3t' = (3)5 a= 10
40 __! 00 i~olved Problems in Engineering Mathematics (2nd Edition) by Tiong & Rojas Day 2 - Fundamentals in Algebra 41

JX =-~
2
By completing the square:
1!1 k -2k-8 =0
2
k =k2 -k-8

9
2
(x-13) =-69+{13t f(x) = x3 + 4x 2 - 3x + 8 ; divisor = x- 5
X=- (k-4)(k+2)=0
4 2
(x -13) = 100 Note: Using remainder theorem, k=4
x-13=±10 remainder= f (5).
Note: Since x = 9/4 will not satisfy to the k=-2
given general equation when substituted, x = 23 -+ Absurd
Remainder= (5)3 + 4(5) 2 - 3(5) + 8 = 218
this equation is classified as defective and
thus, the a~swer is "None of these".
X=3
B 4
f(x) = x + axl + 5Y! + bx + 6

a Etl ~8 ~2$x =2
Ell
3
3x 2 +6x+8 Note: Remainder = f(r).
x +2x 2 +6}3x5 - 4x 3 + 2x 2 + 36x +48
~ jab- lei) When divisor is (x- 2), r =2 & f(r) = 16
VVab- ~(ab)~ By raising both sides to exponent 4: (-) 3x 5 -6x 4 +18x~
4 3
8 ~2$x = (2)4 =16 6x 4 -4x 3 -16x 2 +36x f(r) = 2 + a(2} + 5(2t + b(2) + 6
lab r>
vif.ili ="(ab)' ~2../8X =2 (-) 6x 4 - 12x3 + 36x f(r) =Sa+ 2b + 42
16 =Sa +2b+42
8x 3 -16x 2 + 48
Vifab
lab [
= (ab)"']1' By raising both sides to exponent 3:
(-) 8x 3 -16x 2 +48
-26 =Sa +2b
2$x = (2)3 = 8 0 .
b=-13-4a
fab'
v7a--'
/ ab
b =(ab)' .J8X=4

By raising both sides to exponent 2:


m f(y) =4y 3 + 1Sy2 + Sy _ 4
When divisor is (x + 1), r

3
=-1 & f(r) = 10
v~=~ Sx =(4)2 =16 divisot= 2y + 3 = y- [ -%]
f(r)=(-1t +a(-1) +5(-1t +b(-1)+6

X=2 f(r)=-a-b+12
Note: Using remainder theorem,
10=-a-b+12
IIJ [x~ =8J EFI f(x) = 2Y! + 2x + 4
f(2) = 2(2)2 + 2(2).+ 4 = 16
Remainder= f [-%] 2=a+b

X= (8)3
• Substitute equation 1 in equation 2:
11!9!11
t[-%]=4[~%J +18[-%J +8[-~]-4
X= 16 liiilll
n! = n(n-1)(n-2)(n-3) ... (3)(2)(1) 2=a+(-13-4a)
=15
ID (n -1)! = (n-1)(n-2)(n-3) ... (3)(2)(1)
t[-%]=11
-3a
a =-5
.Jx + 2~2x + 3
By squaring both sides:
= 3
El 15 =5·3
ID b=-13-4(-5}
18 =6·3 f(k) =(x + 3)(x- 4) + 4
LCM = 5 · 6 · 3 = 90 b=7
f(x)=x 2 -4x+3x-12+4
x +2J2x +3 = 9
2../2X+ 3 = 9 - X
m 10 =5·2
32 = 2 . 2 . 2 . 2 . 2
f (X) = X2 - X- 8 Iii
Lowest common factor =2 X+Y=a· y+z=b· X+Z=C
By squaring both sides: Remainder = f(k) 2 ' 2 . ' 2
2
4(2x + 3) = (9- x)
8x + 12 =81-18x + x 2
II 12=4.3=4·3
16=4.4 =4·4
f(k) = k2 - k - 8 By adding a, b and c:
Greatest common divisor = 4 a+b+c= X+y +Y+Z+X+Z
x2 -· 26x = -69 Substitute the given remainder = k in ' 2 2 2
equation 1:
""' '
\
1
a+ b + c = -[2x + 2y + 2z]
2
.

'~
4~ -1001 Sb1ved Problems in Engineering Mathematics (2nd Edition) by Tiong & Rojas '.
~~

a+b+c=x+y+z

Mean = x + Y + z _ a + b + c

-
3 - 3

Topies
Let: x = the mean proportion of 4 and 36
t
Jx
= __>5_
36
D
Mon
Properties of Addition of Integer
Properties of Multiplication of
2
Integers
x = 144
Properties of Equality
X= 12 Properties of Zero
Tue
Properties of Exponents
m
Let: x = the arithmetic sum of 80
D D
Theory Wed
Properties of Radicals
Surds

D D
II Special Products
numbers,
Properties of Proportion
Problems Thu Least Common Denominator
Arithmetic mean = __>5_ = 55
80
=
X 80(55) 4400 = D D
Solutions Fri
Least Common Multiple
Greatest Common Factor
Remainder Theorem
y = new Arithmetic mean

y-
_ x-(250+850)
. 80-2
[I] D Factor Theorem

-- -
Notes Sat
y = 42.31

111 '~~ -~ '---~ ~-- ---~..,.-,- ~~---r.


Let: x = the first number
x + 4 = the second number
y = sum of the original 6 numbers. l .•

T
Arithmetic mean = 'j_ = 17
6
y = 17(6) = 102

y+x+(X+4) 19 =
6+2
102 + 2x + 4 =
8
19

106+2X =19(8) I ·.
[ 1

2x = 46 ;\.:;\; I
X= 23 ~--i-i·
il'l·'·r li
!~
\
1

X+ 4 = 27 'J
l
44

Topics
r--. -~-·-

Dj Quadratic Equation
Quadratic Formula
I
Mon
Properties of Roots

I" 'II I>JJ ·II


D
Tue
Discriminant and Nature of Roots
Binomial Theorem

~
Binomial Expansion
~
Theory Wed
Properties of Binomial Expansion
Pascal's Triangle

D D
Problems Thu
Degree of Polynomial or Equation
Logarithms
Properties of Logarithms

D D
Solutions Fri
Modulus of Logarithms

D D
Notes Sat

What is a Quadratic Equation? The following is the quadratic formula:


Quadratic is an expression or an equation
that contains the variable squared, but not -B±JB2 -4AC
X=-----
raised to any higher power. Quadratic 2A
equation in x contains x2 but not x3 .

' I he general quadratic equati'm is


~~xpressed as:
The quantity Js 2 - 4AC in the above
equation is known as the discriminant.
The discriminant will determine the nature
of the roots of the quadratic equation.
Ax 2 +Bx+C = 0
The table below shows the value of the
where, A, Band Care real numbers and discriminant and its corresponding nature
wrth A ±0. of roots.

When B = 0, quadratic equation is known JB 2


-4AC Nature of roots
o~s a pure quadratic equation. 0 Only one root .
. (Real and ~qual)
1\ quadratic equation in x is also known as >0 Real and unequal
.r ~u~cond-degree polynomial equation. <0 Imaginary and unequal
1111 • :;olution to a quadratic equation is
The sum and product of the roots of a
•·rill<~~ by factorin~J or by th(~ liSe of the
quadratic equation can be solved even
<Jlloldl.tliL IO!IIIItla
without using factoring or quadratic
4~ 100 i Solved Problems in Engineering Mathematics (2"d Edition) by Tiong & Rojas Day 3 ~Quadratic Equation, Binomial Theorem, Logarithms 47

The following are just a few examples of a In Italy, this triangular pattern is known as Sum of expon~nts of the expansion of
formula as long as the general equation is Tartaglia's triangle while in many parts of
given. binomial expansion: (x+y)":
Asia, it is referred to as Yang Hui's
0
triangle.
The following are the properties of the (X+Y) =1 Sum= n(n + 1)
roots of a quadratic equation: Binomial Pascal's Tiangle
(x+yf=x+y (x + y)o 1
Let r1 and r2 be the roots of a quadratic 2 2 (X+ y)1 1 1
( x + y ) = x 2 + 2xy + y What is a Degree of a Polynomial or
!3quation: {x+d 1 2 1
3 3 2 2 3 Equation?
(x + y) = x + 3x y + 3xy + y {x+d 1 3 3 1
Sum of the roots: {X+ y)4 1 4 6 4 1
When the two roots are added, the result Degree of a polynomial or equation with
(X+ y)s 15101051 only one variable refers to the exponent of
is: (X+ y)6 1 6 15 20 15 6 1
As observed in the binomial expansions the variable. For a polynomial or equation
B
r1+r2=-;;. above, some properties were established that contains two or more variables, the
" and are enumerated as follows:
Another way to determine the coefficient of
any term in the binomial expansion is to
degree is the maximum sum of the
exponents of the variables in a single term.
Product of the roots: Properties of Binomial Expansion use the following formula:
When the two roots are multiplied, the Example:
of (x + y)":
result is: C =(Coeff. ofPT)(Exponent of x of PT)
1. The number of terms in the resulting E;xponent ·of y of PT + 1 1. What is the degree of the monomial 7x5 ..
0 Answer: 5
ri·r2""A expansion is equal to n + 1.
where: C = coefficient of any term
2. The exponent of x decreases by 1 in 2. What is the degree of the
PT = preceding term
What is a Binomial Theorem?
succeeding terms, while that· exponent polynomial3x 4 y- 2x 3 z4 + 7yz 5 ? .
of y increases by 1 in succeeding The r1h term of the binomial expansion of Answer: 7.
terms. 7 is the sum of 3 and 4 in the second
Binomial is an expression containing two ( x + y )" may be calculated using the
terms joined by either+ or-. term.
3. The sum of the exponents of each term· following formulas:
is equal ton. What is a Logarithm?
Binomial theorem gives the result of
raising a binomial expression to a certain th n(n -1)(n- 2)- .. (n- r + 2) x(n-r+1) H
4. The first term is x" arid the last term is r = (r -1)! Y The logarithm of a number or variable x to
power. The expansion and the series it
y" and each of the terms has a base b, Iogb x , is the exponent of b
leads to are called the binomial
coefficient of 1
expansion and the binomial series, needed to give x.

il
respectively.
5. The coefficient increases and then
rtt\ = nCr-1X(n-r+1)yr-1

-J- - -..- ----n


1 r
decreases in a symmetric pattern.
The binomial theorem is expressed as
follows: A term involving a variable with a specific
~
The Pascal's Triangle:
exponent is obtained by using the
Logr6=4 maybewOttena• =16
(x'f:y)" =x~ +•nx"-!;y +·p(n-: )xn-~y~· + ·•·
1 The coefficients of a binomial expansion following formula:
. ·.·'f' . " . . •·· .. ·21 ·. can also be conveniently obtained by
... +nxy't!~1 + yn arranging them in a triangular array or
pattern. This is known as Pascal's Triangle
y' =-n(n-1)(n-2)···(n-r+1)
r!
_
x" 'y'
named after the famous French The term "logarithm" comes from Greek
Binomial coefficient is a coefficient of x words, "logus" meaning "ratio" and
mathematician Blaise Pascal (1623- Sum of the coefficients of 'the expansion of
in the expansion of ( x + y)", The binomial 1662). "arithmus" meaning "number". John
(x t- y)": Napier (1550 -1617) invented logarithm
coefficient ncm gives the number of ways Each number in the triangle is equal to the in 1614 using e = 2.718 ... for its base.
of picking m outcomes {not in any sum of the two numbers immediately Logarithm with base e {loge or In) is called
Sum = ( Coeff. of x + coeff. of y )"
particular order) from n possible outcomes. above it. the natural logarithm or the Napierean
logarithm. In 1616, through the
The binomial coefficient forms the rows· of suggestion of John Napier, Henry Briggs
the Pascal's triangle. (1561 0 1630), a professor of Geometry at
48 ·1001 Solved Problems in Engineering Mathematics (2nd Edition) by Tiong & Rojas

Gresham College in London, improved the Proceed to the next page for your third
logarithm using 10 as the base. The test. Detach and use the answer sheet
logarithm with base 10 is known as provided at the last part of this book. Use
common logarithm or the Brlggsian pencil number 2 in shading your answer.
logarithm.
GOOD LUCK!
The number ·e· which is the base of the
Topics
natural logarithm is known as Euler's
number, named after the Swiss ll 'o.,.,,...,._.--~
Quadratic Equation
Quadratic Formula
mathematician, Leonhard Euler (1707- ~ribia: Mon
Properties of Roots
D
1783) and is defined as, Did you know that ... Isaac Newton while
a student at age 22, invented differential Discriminant and Nature of Roots
and integral calculus, discovered the law Tue Binomial Theorem
e = um(1+!)n of universal gravitation, formulated the
·- n ~- Binomial Expansion

Binary logarithm (denoted as lb) is a


logarithm with a base value of 2.
three laws of motion, developed the new
theory o flight in just 18 months and set a
record of the most productive periods of
achievement by an individual in the history
D Wed
Properties of Binomial Expansion
Pascal's Triangle

Relation between natural logarithm and


common logarithm:
of science!
·' Problems
0 Thu
Degree of Polynomial or Equation
Logarithms
Properties of Logarithms
<!auote:
The natural logarithm can be converted
into a common logarithm and vice versa.
To obtain this, a factor known as the
"The art of asking the right questions in
mathematics is more important than the art
of solving them."
D [_]
Solutions Fri '
Modulus of Logarithms

modulus of logarithm is necessary, such


as:

lnx = 2.30261ogx
- Georg Cantor
D D
Notes Sat
logx = 0.43431nx
1011 ECE Board March 1996
The coefficients 0.4343 and 2.3026 are the 8. ±3
!"he equation of whose roots are the
referred, to as the modulus of logarithm.
reciprocal of the roots of 2x2 -- 3x - 5 = 0 c. ±4
I
'~o IS,
D. ±5

~-
What are the Properties of Logarithms? J11 1\
2
5x + 3x - 2 =0 J:04: ME Board October :1.996
~~~:. ll. 2x2 + 3x - 5 =0 Solve for x that satisfies the equation 6x2
1. log(xy) =logx +logy 1,,
C. 3Jt - 3x + 2 ::: 0 ...:7x-5 = 0.
.~ U. 2x-2 + 5x- 3 =0
log(~)= logx -logy
5 -1
2. A. -or-
tJ lOZI EE Board October 199~ 3 2
' In the equation Jt + x = 0, one root is x 3 3

l •·qual to B. -or-
3. logx" =nlogx 2 8
7 -7
logx 1 f\ 1 C. -or-
4'. logbx=- 5 5 15
Iogb
i 1!.
I:. 1/4
D.
3 3
-or-
5.
logbx
1og.x= logba
~ I> none of these 5 4

( O:Jt ECE Board Aprilt990 lOS: EE BoCJ~rd Oetober 1:997


.• nlw for the value of "a" in the equation Find the values of x in the equation 24~ +
6. log.a=1 ·•" I ,,,·• I 1G '~ 0. 5x- 1 "= 0

I\ t .'
SO 100 1 ·solved Problems in Engineering Mathematics (2"d Edition) by Tiong & Rojas Day 3 - Quadratic Equation, Binomial Theorem, Logarithm 51
2
1.1.1.: Given the equation 3x + Bx + 12 = ".n7: CE Board November 1.996
(~.1)
A. 1.86
A. 0. What is the value of B so that the roots · Find the 6th term of the expansion of B. 1.68

t~ ~)
of the equation are equal? C. 178
I 1 )t6
1--3 D. 1.98
B. A. 4 \_2a .
6'5
B. 8 1.23: CE Board November 1.997
c. (~ ~) C. 10
A.
-66939 Evaluate the log6 845 = x:
2'5 D. -12 256att

D. ( ~8' -~)
3
J...,;;
Find the term involving y5 in the B. -66339
(A.
B.
3.76
5.84
.;expansion of (2x 2 + y) 10 128att
C. 4.48
-33669 D. 2.98
1.06: EE Board October 1.990 10 C.
2 A. 8064 x l 256a1t
i" Determine k so that tl.e equation 4x + kx 5
·l
+ 1 = 0 will have just one real solution.
B.
C.
D.
8046
8046
4680
x
x
10
x5 y
t D. -39396
128a11
1.24: ME ~ard April 1.997
What is the value of log tci base 10 of
1000 33 ?·
A. 3
B. 4 .l·J:il Find the 5th term of the expansion of 1.181 What is the coefficient of the term A. 10.9
C. 5 'free of x of the expansion of (2x- 5y) 4 ? B. 99.9
D. 6 / ( x2 +-
1 )10
C. 9.9
x A. 256 D. 9.5
1.07: ME Board April 1.996 B. 526
2
Solve for x: 10x + 10x + 1 = 0 A. 260 x 8
C. 265
8 1.25: ECE Board April 1998
B. 5040 x D. 625 What is the value of (log 5 to the base 2)
A. - 0.113,- 0.887 8

d:;~·Find the 6th term of (3x- 4y~~)


C. 210 x +(log 5 to the base 3)?
B. - 0.331;- 0.788 8
D. 420 x
C. -0.113,-0.788
A. 7.39
D. -0.311,- 0.887 ulli'ECE Board April 1.998 A. -148,288 x3l B. 3.79
12 B. -548~l
'In the expression of ( x + 4y ) , the C. 3.97
J.08: If 1/3 and -3/2 are the roots of a numerical coefficient of the 5th term is, C. -154,288 xV D. 9.37
quadratic equation, then the equation is D. - 1,548,288 xV
A. 63,360 1.26: Find the value of log4 (log3 5).
A. 6~ + 7x - 3 = 0 B. 126,720 tzo:ECE Board November 1.995
B. 6x2 - 7x + 3 = 0 C. 506,880 What is the sum of the coefficients of the A. 1.460
C. 6~ - 7x- 3 = 0 D. 253,440 expansion of (2x -1 )20 ? B. 0.275
I D. 6~ - 7x + 1 = 0 C. 1.273
usf What is the fourth term of the A. 0 D. 0.165
1.09: Which of the following is a root of 2 100 B. 1
'expansion of (x + x ) ?
this quadratic equation, 30~ + 49x + 20 = C. 2
0? 103 D. 3 :l%7: Given: log4 7 = n. Find log4!
A. 1650 x
103 7
B. 161700 X
A. 0.6 C. 167100 x
103 1.21.: ECE Board April 1.995 A. 1/n
B. -0.6 100 What is the sum of the coefficients in the
D. 167100 x
c. -o.8 expansion of (x + y- z) 8 ? B. n
D. 0.75 .1.-1.6: What is the numerical coefficient of C. -1/n
4 8 A 0 D - n
/the term next to 495x y ?
1.1.0: What is the discriminant of the 13. 1
equation 4~ = 8x- 5? C. 2 1.28: CE Board November :199:2
A. 660
D. 3 CE Board May 1.994
B. 792
If log a 10 = 0.25, what is the value of
A. 8 c. 990
log 1oa?
B. -16 D. 1100 t:u:r CE Board November 1.99~
C. 16 ECE Board Nov. 199:J
D. -8 I 1nd the valut! of loq., 'Ill • A. 2
n 11
I
.::_f
,.~

52 l 00 l·.Solved Problems in Engineering Mathematics (2"d Edition) by Tiong & Rojas


Day 3 - Quadratic Equa!1e_n, Bin_o!'lial Theorem, Logarithm 53
C. 6 :134: ECE Board November :199:1
:1401 Solve for the value of x:
D. 8 Given: Iogb 1024 =~ Find b. 6
log2x 3 +log-= 6.278
2 X
:l:t9: ECE Board November :1995
Given: log b y = 2x + log b x. Which of the A. 2560
following is true? A. 379.65
B. 16 .~
·B. 365.97
c. 4 •'>'·~· .·
A. Y = b2x ·,'j~t C. 397.56
D. 2
B. y = 2xb i:l D. 356.79
2x :l:JS: Given: log3 (~- 8x) = 2. Find x.
c. y=-
. b
D. y = xb2x A. -1
B .. 9
:l:JO: ME Board October :1996 C. -1 and 9~
Which value is equal to log to tlte base e D. 1 and- 9
of e to the -7x power?
:13&: ECE Board April :1993
A. -7x Solve for the value of x in the following
B. 10 to the -7x power equation: x 3109 x = 1OOx .
C. 7
E. -7 log to the base 10 A. 12
it
B. 8 ,··~
:l:J:l: ME Board April :1996 c. 30
Log of the n1h root of x equals log of x to D. 10
1/n power and also equal to: 'l
'" --~
:137: EE Board October :l99:t
logx Given: log 6 + x log 4 = log 4 + log (32 +
A. 4•). Find x.
n
B. n log x
C. log (x to the 1/n power) A. 2
B. 3
n
c. 4
D. (n -1)1ogx D. 6
:l:J:tl ECE Board November :1990 :1:581 ECE November :1998
Log (MN) is equal to: If log of 2 to the base 2 plus log of x to the
base 2 is equal to 2, then the value of x
A. Log M- N is,
B. Log M + N
C. N log M A. 4
D. Log M + Log N B. -2
c. 2
:l:J:JI ME Board April :1997 D. -1
What expression is equivalent to log ( x )
-'log ( y + z )? :1391 ME Board October :1997

.~1
Find the value of x if log12 x = 2.
A. log x + log y + log z ·{
B. log [ XI ( y + z )] A .. 144 ,f-
C. log x -log y - log z B. 414
D. log y + log ( x + z ) c. 524
D. 425
~

Day 3- Quadratic Equation, Binomial Theorem, Logarithm 55

ml 2x 2 - 3x- 5 = 0 7 ± ~(7) -_
2
4(6)(-5)
X=--'--'-,.:__,_ __
(2x+2)(x-25)=0 2(6)
X= 2.5 7 ± 13
X=--
Topics X= -1 12
Thus,

D Quadratic Equation
Quadratic Formula
Thus. the roots of the second equation
are:
5
x1 =-
3
1
and x 2 = - -
2


tv! on 1
Properties of Roots X1 = - = 0.4

DTue
Discriminant and Nature gt;Roots
Binomial Theorem
2.5
1
x2 = - = -1
-1
24x 2 + 5x -1 = 0

Binomial Expansion Using the quadratic formula;

D [QJ Properties.of Binomial Expansion Solving for the second eqution:


X
-5±~(5) 2
= ----L'----'--,-::-c---
-4(24)(-1)
Theory Wed Pascal's Triangle 2(24)
Degree of Polynomial or Equation (x-0.4)(x+1)=0

D D
Problems Thu
Logarithms
Properties of Logarithms
2
x + x- 0.4x- 0.4 = 0
{ x 2 + 0.6x- 0.4 = 0 ). 5
X=--
48
-5 ± 11

1
Thus, x1 = B and x2 = -
1

D

Modulus of Logarithms 5x 2 + 3x -2 = 0 3
Solutions

D D
Notes
Fri

Sat
• x(x+1)=0
X=O
X= -1
Note: There is only one solution to the
equation (4x2 + kx + 1 = 0), if the
discriminant (8 2 - 4AC) is equal to zero.

where: A = 4; 8 = k & C = 1

101. A
102. D
103. A
ANSWER KEY

111. D
112. A
113. c
121. 8
122.A
123.A
131. A
132. D
133.8
c:J
c:J 26-33
o
RATING

34-40 Topnotcher

Passer
• a8 -17a 4 +16 = 0
2
Let: x = a4 and x = a
x2 -17,+16=0
8
8 2 -4AC = 0
k 2 -4(4)(1)=0
k
2
= 16
k = ±4


Using the quadratic formula;
104.A 114. B 124.C 134. 8 20-25- Conditional
105. D
106. 8
107. A
115. 8
116. 8
1·17. 8
125. 8
126. 8
12"1. D
135.c
136. D
137. 8
o 0-19 Failed 17±~(17)
X= _
2
-4(1)(16)
_.!....:__:__ __

2 10x2 + 10x + 1 = 0
108. A 118. D 128.8 138.c If FAILED, repeat the test. 17 ±15 Using the quadratic formula:
109.c 119. D 129. D 139. A X=--
110. 8 120.A 130. A 140.C 2 X= -10±~(10) 2 -4(10)(1)
Thus,
2(10)
x1 = 1 X2 = 16
-10 ± 7.746
a4 = 1 a4 = 16 X=----
20
a= ±1 a= ±2
Thus, x1 =- 0.887 and x2 =- 0.113
ml lml x1= i and x 2 =-~
6x 2 - 7x- 5 = 0
1J:;lll~J the quadratic formtila;
S6 ..100 I Solved Problems in Engineering Mathematics (2"d Edition) by Tiong & Rojas Day 3 - Quadratic Equation, Binomial Theorem, Logarithm 57

x1 +x2 = -;;;.
-B where: A = 4; 8 = - 8 & C = 5 a (X+X2)100 5th term = 4 C 4 (2x) 0 (-5y) 4
41
Substitute:
.c 4th term= ncr-1 (x2tr+ 1(~)'"1
(4-4) (1)(625 4 )
y
x1·x2 = - !4! .
A 82 where: n = 100; r = 4
- 4AC = 0 5th term = 625 l
i+(-%)=~~ 8 2
- 4{3){12) =0 4th term = 100 c3 (x)97(x2)3 ~~~ (3x-4d
8 = 144
2

~(-%)=~
10! 97 6
- x (x )
8 = ±12 (100-3)!3! 6th term= ncr-1 (3x)n-r+ \-4y)'" 1
4th term= 161,700 x103 where: n = 8; r = 6
B=~A liB
il'
6
C =_:!A
2
Note: The term involving
of the expansion (2~ + y)
10
is the 61h term i a
Coefficient of next term =
6th term= 8 C5

81
(3x) 3(-4y) 5

3 5
= ( ) (27x )(-1024y )
Substitute to the general quadratic 8-5 !5!
(Coefficient of PT)(Exponent of x}
equation: 61h term= ncr-1 (2x2 )n-r+ 1 (yf 1 6th term = - 1,548,288 x 3y 5
{Exponent of y) + 1
where: n ::: 10; r = 6
2
Ax + Bx + C =0 . 495 (8)
Coefficient of next term "" - - - El
2
A:x +(!_A
5 )
lx +(-=~At,,
2 )
0 6th term= 1oCs (2x2)s Ys 4 +1
= 792 Note: To solve the sum of the coefficients
0!- - ( 32x 10) y5
= - -1-
20
of (2x -1 ) , substitute one ( 1) to x,
' 7 1 "J'6 (10-5)!5! calculate, then subtract a value of (-1 )20
2
[ Ax + 6Ax- 2.A = 0 A )16
61h term= 8064 x 10y5 1&1 (-
1
2a
-3 from the result

1 )10 Sum of coefficients= [(2)(1) -1t- (-1)20


1
6x?. + 7x- 3 = 0

• ~~~ 6th term = C (__!__)n-r+ (-3)'-1


( X2+;: n r-1
2a =0
2
30x + 49x + 20 = 0
Using the quadratic fon'ru.l)a;
1h
5 term = n c r-1 (x2)n-r +1
( <, )r-1
-
X
where: n = 16; r = 6
IBI
1 )11 (-3) Note: To solve the sum of the coefficients
5 8
-49 ± J(49)2 -=4(30).(20) where: n = 10; r = 5 6th term = 1s C 5 - a
(2 of (x + y -z) • substitute one(1) to all the
X=-------·---- 4 variables and calculate.
2(30)
-49 ± 1
5th term =
10
C
4
(~}. 6 (lx..!.) - 16! ( 1 )( 243)
X=--- - (16-5)!5! 2048 a 11 - Sum of coefficients = ( 1 + 1 - 1 )8 = 1
60
I 10! 12 ( 1)
Thus, x,
-49±1
= - - = -0.8
60
and = (10-4)!4lx x4
4368(-243) 16
= 2048 a 11 + 16
lfD log8 48 = 10log
9to 48
6 = 1.86
10
-49-1 5th term= 210 x8
X2 = - - = -0.833
60

1111 6th term = 66339 lED logs 845 = log1o 845


log = 3. 76
11.1 2
4x - 8x + 5 = 0 (x + 4y)t2 128 a11 10 6
where: A = 4; B = - 8 & C = 5
Discriminant = B2 - 4AC
= (- 8) 2 - 4(4)(5)
5th term= nc,_, (x)n-r+ 1(4y)'" 1 a
Note: The term in the expansion (2x -
IBI log10 1000
33
= 3.3 log10 1000
= 9.9
= -16 where: n = 12; r = 5 sd which is free of x is the last term
or the 5th term.
IDI 5th term= 12 C 4 (x) 8(4y}
i2!
4

8 4 sth term= ncr-1 (2x)"·r+ 1(-5y)'" 1


ml log 2 5 + log 3 5 = ~91o ~ + log10 5
log1o 2' log1o 3
Noie: The roots of the equation (3x2 + Bx "" - - - - - - (x )(256y ) = 3.79
=
+ 12 0) are equal, if the discnminant (82 (12-4)!4! where: n =4; r = 5 ---.._....
- 4AC) is equal to zero. 5th term = 126,720 x8 l
.JII
1
!.,
58. 1001 Solved Problems in Engineering Mathematics (2"d Edition) by Tiong & Rojas . '~ Day 3 --Quadratic Equation, Binomial Theorem, Lo$J!rithm 59

• log (1og 5) = log1o(log3 5)


4 3
log1o4
IBI Iogb 1024 =

log1o 1024 5
2

log1ob = 2
~

log.10 b -_ ~..:.:!_--=.
.
log1o 1024
2.5
II
log6 + xlog4 = log4 + log(32 + 4")

log6 + log4" = log4 + log(32 +4")


• log2x 3 +log§.= 6.278
X

log(6){4"} = log(4)(32+4") ·iog2x3 + log6.: logx = 6.278


log1o b = 1.204 3
log2x -logx = 6.278 -log6
(6}(4"} = (4)(~2+4x)
1&1 log 4 ~=log 4 1-log 4 7
b = 16
(6)( 4"} = ( 4}(32) ~ (4)( 4•)
2x 3
log-= 5.49984
X
Note: Logarithm of 1 to any base is eq~Jal
, to zero.
1
IB1 le~g 3 ( x2 - 8x) =2
2(-4") = 128
4" = 64
log2x 2 = 5.49984
2x
2
=antilog5.4998
log 4 - = 0 - log 4 7 = - n 2 2
7 log10 (x -8x) = x ""158055.6425
2 Take logarithm on both sides:
ml log.10=0.25
'log 10 3
log10 (x 2 - 8x) = 2 log 10 3 = log 10 (3) 2 log4" =log64
X= ±397.56

log1o 10 = 0.25
log10 (x 2 - 8x) = log10 9 xlog4 = log64
log 10 a
log 10. x 2 - 8x = 9 log64.
Iog1oa = - 10
--= 4
X=--
log4
0.25 x2 - 8x -9 = 0

• Iogb y = 2x + Iogb x

Iogb y -·Iogb x = 2x
log1o Y- log1o x = 2x
log 10 b log10 b
ml
(x +1)(x -9) = 0

x31ogx =1OOx
X= -1
x=9

Take logarithm on both sides:


ml
X=3

log2 2 + log2 x = 2
log10 2 + log10 x =
log10 2 log10 2
2

log10 y -log 10 x =2x log10b x


log 3109 x = log 1OOx 1+ log1o x = 2
log10 2
log1o 'j_ = log1o b2x log1o x = 1
X
3(1ogx)(logx) = lcg1 00 + logx log10 2 ·
y = xb2x

• log. e-7• = (-7x)log. e


= (-'7X)(1)
=-7x
3(1ogx) 2 -logx- 2 = 0
(3(1ogx) + 2)(1ogx -1} = 0

Equating factors to zero:


El log12 x = 2
log10 x = log10 2
X=2


31ogx +2 = 0
nC 1
1 122
log vx =log(x)' =-log(x)
n . 31ogx = -2 X:::

-2 X= 144
logx=-
1!1 log MN = log M + log N 3
x = 0.215 (absurd)

Ill!. (x) -log (y + z) =log(, :z]


logx -1 = 0
logx = 1
62 100 1 Solved Problems in Engineering Mathematics (2nd Edition) by Tiong & Rojas

Topics

DMon
Age Problems
Work Problems
Mixture Problems
D Tue
Digit Problems
Motion Problems
Coin Problems
I~
~
Theory
DWed

D ~
Problems Thu

D D
Solutions Fri

D D
Notes Sat

Age Problem
1/5 -?This is what the person
One of the most common problems in ~ finished in 1 day
Algebra is the age problem. This type of
problems must be solved meticulously by
qiving more emphasis to the tenses (i.e.
past, present or future) of the statement.

Example:
fhe ages of a certain person in the past,
present and future in terms of x are as
lui lows:
This is the work
For a complete job,
II
rc;tte x time,= 1 .
Work Problem When there is a specific work and specific
time and manpower, the rate of doing the
· ;11ppuse that a person can do a certain work may be computed using the number
w< '' k in 5 days. This means that the said of man-hours.
1"'"·"'1 c.1n finish 1/5 of the work in one
""Y lllu~;. lli~; rah~ IS 1/~i of the work P• 'I
d.iy
64 100 I Solved Problems in Engineering Mathematics (2"d Edition) by Tiong & Rojas Day 4- Age, Work, Mixture, Digit, Motion Problems ~

Example: Digit Problems


.. ,.., If 20 bakers can bake 40 pizzas in 8 hoars,
how many bakers can bake 10 pizzas in 2 Let h, t and u be the hundreds, tens, and
hours? units digit, respectively. A three-digit
number must be represented in the
Solution: following manner.
Get the rate (in man-hour) of baking a Dime
Quarter
pizza. Number= (h ){100) + (t)(1 0) + (u) 10 cents
25 cents

(20bakers )(8hours) A two-digit number is represented by;


Rate=
40pizzas
Rate= 4baker- hour Number= (t)(1 0) + ( u) Half Dollar
, pizza
Motion Problems
50 cerits

This means that to bake a pizza, you need


In Algebra, the problems pertaining to
either 4 bakers to work in 1 hour or 1
motion deals only with a uniform velocity,
baker to work in 4 hours.
i.e., no acceleration or deceleration in the
process. The following is the relationship Proceed to .the next page for your 4th test.
No. of bakers= ( 4 ba~er-hourJ(10 pizzas) between the distance, time and velocity. Detach and use the answer sheet provided
p1zza · 2 hours at the 1ast part of this i:)ook. Use pencil
= 20 bakers Time= 0 Time= t number 2 in shading your answer.
v
• GOOD LUCK I
Mixture Problems

The easiest way to solve a mixture D


problem is to draw a rectangle or square 'ijtribia: ·
which will illustrate the content of the Did you know that... 161h century italian
mixture as shown in the following mathematician and physician Gerolamo
illustration.
D=Vt V=~ t=~ Carda no, was the first to introduce the
t v concepts of probability and define it as the
Consider a 5 cubic meter mixture number of favorable outcomes divided by
containing 65% alcohol and 35% gasoline. the number of possible outcomes.
I Coin Problems Because of this, he is regarded as the
"Father of the Theory of Probability".
Problems in Algebra about coins are more
focus on the dollar denomination than local
.65%
Philippine currency.
~uote:
Alcohol The entire "Where there is matter, there is geometry."
mixture
The following are the equivalent value for
each coin. - Johannes Kepler
35%

!~ Gasoline

V= 5m 3

The quantity of alcohol is (0.65)(5) = 3.25


cubic meters while that of gasoline is
(0.35)(5) = 1.75 cubic meters.
Penny Nickel
1 cent 5 cents
Day 4- Age, vvork, Mixture, Digit, Motion Problems 67

uS: A girl is one-third as old as her


,1"
"ISO: Debbie is now twice as old as Jerry.
/ brother and 8 years younger than her Four years ago, Debbie was three times
sister. The sum of their ages is 38 years. as old as Jerry then. How old is Debbie?
How old is the girl?
A. 14
A. 4 B. 16"

,------- Topics
...-----~·
B.
c. 6
5 C.
D.
18
24

D tvlon
! Age Problems
j Work Problems
1 Mixture Problems
D. 7
, ...,
J46: Paula is now 18 years old and his
lSi: ME Board April1998
··A pump can pump out water from a tank

D /colleague Monica is 14 years old. How in 11 hours. Another pump can pump out
I Digit Problems many years ago was Paula twice as ol·d water from the same tank in 20 hours.

Tue j Motion ~roblems as Monica? How long wiil it take both pumps to pump
out the water in the tank?
Coin Problems

D D
Theory Vl/ed
A.
B.
C.
D.
5
7
8
10
A.
B.
C.
7
6
7
hours
hours
1/2 hours

Problems.
IQJ
Thu
14'7: A father tells his son, "I was your
age now when you were born." If the
D. 6
,,~
1/2 hours

J.Si: CE Board November 1993


"A. 400-mm 0
D D father is now 38 years old, how old was pipe can fill the tank alone in
his son 2 years ago? 5 hours and another 600-mm 0 pipe can
Fri fill the tank alone in 4 hours. A drain pipe
Solutions
A. 15 300-mm 0 can empty the tank in 20

D D
Notes Sat
B.
C.
D.
17
19
21
hours. With all the three pipes open, how
long will it take to fill the tank?

A. 2.00 hours
1.!18i""'Six years ago, Nilda was five times B. 2.50 hours
.w:'ECE Board April1995 14-(:"GE Board February 1994 as old as Riza. In five years, Nilda will be C. 2.25 hours
ECE Board April1999 "Robert is j 5 years older than his brother three times as old as Riza. What is the D. 2.75 hours
Mary is 24 years old. Mary is twice as old Stan. However "y" years ago, Robert was present age of Riza?
as Ann was when Mary was as old as twice as old as Stan. If Stan is now "b" ,63: A tank is filled with an intake pipe in
Ann is now. How old is Ann now? years old and b>y, find the value of (b- A. 17 2 hours and emptied by an outlet pipe in 4
y). B. 16 hours. If both pipes are opened, how long
A. 16 C. 15 will it take to fill the empty tank?
B. 18 A. 15 D. 14
C. 12 B. 16 A. 3 hours
D. 15 C. 17 s49i At present, the sum of the parents' B. 4 hours
D. 18 ages is twice the sum of the children's C. 5 hours
.14i: EE Board April1997 ages. Five years ago, the sum of the D. 6 hours
The sum of Kim's and Kevin's ages is 18. .•144:JJ is three times as old as Jan-Jan. parents' ages was 4 times the sum of the ~":,.,.-4:"" .
In 3 years, Kim will be twice as old as Three years ago, JJ was four times as old children's ages. Fifteen years hence, the Js.:i: A tank can be filled in 9 hours by
1
·Kevin. What are their ages now? as Jan-Jan. The sum of their ages is sum of the parents' ages will be equal to one pipe, 12 hours by a second pipe and
the sum of the children's ages. How many can be drained when full by _aJ!l_jrd pipe in
A 4,14 A. 20 children are there? 15 hours. How long will it take to fill an
B. 5, 13 B. 24 empty tank with all pipes in operation?
C. 7, 11 C. 28 A 3
D. 6, 12 D. 36 B. 4 A. 7 hours and 12 minutes
c 5 B. 7 hours and 32 minutes
() (:) C. 7 hours and 42 minutes
68 100 LSolved Problems in Engineering Mathematics (2"d Edition) by Tiong & Rojas
,_
7 hours and 50 minutes -·
.<1.59: ME Board April :1.995
Day 4- Age, Work, Mixture, Digit, Motion Problems 69
3
cr· A and B working together can finish C. 12 A. 20 m of solution with 35% alcohol,
,iSS: ME Board April :1995 D. 13 3
painting a house in 6 days. A working 40 m of solution with 50% alcohol
If A can do the work in "x" days and Bin 3
alone can finish it in 5 days less than B. """'"'#.[
B. 50 m of solution with 35% alcohol,
"y" days, how long will they finish the job :1.64: ECE Board November :1.99:1. 3
How long will it take each of them to finish 20 m of solution with 50% alcohol
working together? Crew No. 1 can finish installation of an 3
the work alone? C. 20 m of solution with 35% alcohol,
antenna tower in 200 man~hour while 3
50 m of solution with 50% alcohol
Crew No. 2 can finish the same job in 300 3
x+y A. 8, 13 D. 40 m of solution with 35% alcohol,
A. man-hour. How long will it take both 3
xy B. 10, 15 20 m of solution with 50% alcohol
C. 6, 11 'I'
crews to finish the same job, working (.'''
~y ,Ja,
B. D. 7, 12 together? ~~ A goldsmith has two alloys of gold,
2 1
,.,..,,<"~ the first being 70% pure and the second
__!'!_ A:6o: EE Board April :1.99& A. 100 man-hour being 60% pure. How many ounces of the
C.
X+y '·A and B can do' a piece of work in 42 B. 120 man-hour 60% pure gold must be used to make 100
I' days, Band C in 31 days and C and A in c: 140 man-hour ounces of an alloy which will be 66%
D. FY 20 days. In how many days can all of D. 160 man-hour gold?
them do the work together? ..,_.<"'""

.s6; ECE Board November :1995 . u)s: ME Board October :1.994 A. 40


· Pedro can paint a fence 50% faster than A. 19 On one job, two power shovels excavate B. 35
Juan and 20% faster than Pilar, and B. 17 20,000 cubic meters of earth, the larger C. 45
together they can paint a given fence in 4 C. 21 ,'jl;h, shovel working 40 hours and the smaller D. 38
'i 'f~ f
hours. How long will it take Pedro to paint D. 15 ·w' for 35 hours. On another job, they /
/
the same fence if he had to work alone? removed 40,000 cubic meters with the ,d9: ME Board October :1.994
J.&"i; It takes Myline twice as long as larger shovel working 70 hours and the Two thousand (2000) kg of steel
A. 6 ·Jean a to do a certain piece of work. smaller working 90 hours, How much containing 8% nickel is to be made by
B. 8 Working together, they can finish the work earth can each remove in 1 hour working mixing a steel containing 14% nickel with
C. 10 in 6 hours. How long would it take Jeana alone? another containing 6% nickel. How much
D. 12 to do it alone? of each is needed?
A. 169.2, 287.3
,_s{:"'Glenn can paint a house in 9 hours A. 9 hours il B. 178.3, 294.1 A. 1500 kg of steel with 14% nickel, 500
"while Stewart can paint the same house B. 18 hours C. 173.9, 347.8 kg of steel with 6% nickel
in 16 hours. They work together for 4 C. 12 hours D: 2P0.1, 312.4 B. 750 kg of steel with 14% nickel, 1250
hours. After 4 hours, Stewart left and
Glenn finished the job alone. How many
D. 14 hours f :lflft: EE Board October :1.997 C.
k g of steel with 6% nickel
500 kg of steel with 14% nickel, 1500
more days did it take Glenn to finish the .:u;i; ECE Board April :1.999 Ten liters of 25 % salt solution and 15 k g of steel with 6% nickel
liters of 35 % salt solution are poured into D. 1250 kg of steel with 14% nickel, 750
; job? Mike, Loui and Joy can mow the lawn in
a drum originally containing 30 liters of k g of steel with 6% nickel
4, 6 and 7 hours respectively. What
A. 2.75 hours 10% salt solution. What is the per cent
fraction of the yard can they mow in 1
B. 2.50 hours hour if they work together? concentration of salt in the mixture? ,t!fo: How much water must be
C. 2.25 hours /evaporated from 10 kg solution which has
D. 3.00 hours A. 47/84 A. 19.55% 4% salt to make a solution of 10% salt?
B. 22.15%
/ B. 45/84
C. 27.05% A. 4kg
.J:S81 CE Board November :1993 C. 84/47
D. 25.72% B. 5 kg
It takes Butch twice as long as it takes D. 39/60
Dar'l to do a certain piece of work. C. 6kg
:1.67: ME Board October :1.99:Z
Working together they can do tile work in ~6:1: A farmer cari plow the field in 8 D. 7 kg

.~
A Chemist of a distillery experimented on
6 days. How long would it take Dan to do days. After working for 3 days, his son
·; two alcohol solutions of different strength, . :1.'f1: EE Bo!li'd October :1.994
it alone? joins him and together they plow the field
35% alcohol and 50% alcohol, If a two digit number has x for its unit's
in 3 more days. How many days will it
respectively. How many cubic meters of digit and y for its ten's digit, represent the
A. 9 days require for the son to plow the field alone?
each strength must he use in order to number.
B. 10 days
produce a mixture of 60 cubic meters that
C. 11 days A. 10
contain 40'Vo alcohol? A. 1ox+y
D. 12 days B 11
B 10y +X
·Day 4- Age_,_Wor}S_Mixture, Digit, Motion Problems 71
7 0 100 1 Solved Problems in EnQ'ineering Mathematics (2nd Edition) by Tiong & Rojas
C. 58 /1,S&: CE Board November 1.994
c. yx .,;771 GE Board February 199Z · An airplane flying with the wind, took 2
D. 69
D. xy The product of ·~ and i·of a number is
l8%: The second of the four numbers is
hours to travel 1000 km and 2.5 hours in
flying back. What was the wind velocity in
1?.1-:z: EE Board October 1994 500. What is the number? three less than the first. the third is four kph?
P:.Une number is 5 less than the other. If more than the first and the fourth is two
their sum is 135, what are the numbers? more than the third. Find the fourth A. 50
A. 50 number if their sum is 35. B. 60
A. 85, 50 B. 75 C. 70
B. 80, 55 c. 100 A. 10 D. 40
C. 70, 65 D. 125 B. 11 ..,..,.,,.,z,
D. 75,60 C. 12 ,;::187: CE Board May 1.998
.J78: If 3 is subtracted from the D. 13 A boat travels downstream in 2/3 of the
,1?3: ECE Board March 1996 numerator of a certain fraction, the value time as it goes going upstream. If the
Ten less than four times a certain number of the fraction becomes 3/5. If 1 is l83: EE Board April1997 velocity of the river's current is 8 kph,
is 14. Determine the number. subtracted from the denominator of the A jogger starts a course at a steady rate determine the velocity of the boat in still
same fraction, it becomes 2/3. Find the of 8 kph. Five minutes later, a second water.
A. 6 original fraction. jogger starts the same course at 10 kph.
B. 7 How long will it take the second jogger to A. 40 kph
C. 8 A. 35/55 catch the first? B. 50 kph
D. 9 B. 36/55 c. 30 kph
c. 3/7 A. 20 min D. 60 kph
~4: ECE Board March 1996 · D. 32/41 B. 21 min
·the sum of two numbers is 21 and one C. 22 min /188: Two planes leave Manila for a
number is twice the other. Find the ,,f:t79: ECE Board November 1997 D. 18 min southern city, a distance of 900 km. Plane
numbers. •' The denominator of a certain fraction is ,c''
A travels at a ground speed of 90 kph
three more than twice the numerator. If 7 .1114: EE Board April1997 faster than the plane B. Plane A arrives in
A. 6, 15 is added to both terms of the fraction, the A boat man rows to a place 4.8 miles with their destination 2 hours and 15 minutes
B. 7,1'4 resulting fraction is 3/5. Find the original . the stream and back in 14 hours, but finds ahead of Plane B. What is the ground
C. 8,13 fraction. that he can row 14 miles with the stream speed of plane A?
D. 9,12 in the same time as 3 miles against the
A. 8/5 stream. Find the rate of the stream. A. 205 kph
J'15: EE Board April1993 B. 13/5 B. 315 kph
lf eight is added to the product of nine c. 5/13 A. 1.5 miles per hour C. 240 kph
and the numerical number, the sum is D. 3/5 B. 1 mile per hour D. 287 kph
seventy-one. Find the unknown number. C. 0.8 mile per hour
180: Find the product of two numbers D. 0.6 mile per hour 1.89: EE Board April 1.997 #'
A. 5 ' such that twice the first added to the A train, an hour after starting, meets with
/"
B. 6 second equals 19 and three times the first _J.SS: ECE Board November 1998 an accident which detains it an hour, after
C. 7 is 21 rnore than the second. A man rows downstream at the rate of 5 which it proceeds at 3/5 ot its former rate
D. 8 mph and upstream at the rate of 2 mph. and arrives three hbur after time; but had
How far downstream should he go if he is the accident happened 50 miles farther
A. 24
1'76: Find the fraction such that if 2 is to return in 7/4 hours after leaving? on the line, it would have arrived one and
B. 32
one-half hour sooner. Find the length of
(ubtracted from its terms its becomes 1/4, c. 18
the journey.
but if 4 is added to its terms it becomes D. 20 A. 2.5 miles
1/2. B. 3.3 miles
j A. 910/9 miles
·'1:81: The tens' digit of a number is 3 less C. 3.1 miles
D. 2.7 miles B. 800/9 miles
A. 3/5 than the units' digit. If the number is
C. 920/9 miles
B. 5/12 divided by the sum of the digits, the
D. 850/9 miles
c. 5/14 quotient is 4 and the remainder is 3. What
D. 6/13 is the original number?

A. 36
B. 47
72 l 00 I· Solved Problems in Engineering Mathematics (2"d Edition) by Tiong & Rojas ·

190: On a certain trip, Edgar drive 231


km in exactly the same time as Erwin
drive 308 km. If Erwin's rate exceeded
·~
')~,
'L
that of Edgar by 13 kph, determine the •I'!
'\''
rate of Erwin. :'I:'
,·!

A. 39 kph Topics
.,,, ------·-"-'""'""-"".._..._u,_____,,__,
B~
c.
D.
44 kph
48 kph
52 kph
(

,:
(

0Mon
Age Problems
Work Problems
Mixture Problems
0
1,\\
Digit Problems
Tue Motion Problems
Coin Problems

·1v·
0
Theory
0Wed
nr.
:{f
i~
0 [QJ
Problems Thu

[
Solutions
0 Fri

0Notes
0Sat

ANSWER KEY RATING

141. B 154. 0 167. 0 180. A 0 43-5{] Topnotcher


142.8
143. A
155. c
156.C
168. A
169. c
181. B
182. 0 c::J 33-42 Passer
144.0
145.C
157.A
158.A
170. c
171. B
183. A
184. D
c:J 25-32 Conditional
146. D
147. B
159. B
160.A
172. c
173. A
185. A
186.A
D 0-25 Failed
148.A 161. B 174. B 187.A
149. c 162.A 175.C 188.C 1:11 lfFAILEO, repeat the test.
150. B 163. c 176.C 189. B
ill' I I 151. A 164. B 177. c 190. D
152. B 165.C 178. B
153. B 166.A 179. c
74 100'1 Solved Problems in Engineering Mathematics (2nd Edition) by Tiong & Rojas Day 4- Age, Work, Mixt~e, Digit, Motion Problems 75

Ill Present
24
X
x-12=3
X=9

3x = 3(9)
• Nilda
Riza
Past
5(x-6)
x-6
Present
?
X
Future
3(x+5)
x+5

Let: x = time needed to complete the work
1
-+-=-
1 1
20 11 X


3x= 27
24- x =x-12 x = 7,.096 hours
3(x + 5) -5(x -6) = x +5-(x -6)
2x =36 Sum of ages = 9 + 27 =36 3x + 15- 5x + 30 = x + 5- x + 6
X= 18 1 1 1 1
-2x+45=111
Ill Ill -+---=-


X =17 5 4 20 X
Let: x = age of the girl
x = 2.5 hours
=
y age of her brother


Future
z = age of her sister
,,. X +3
v+3 Past Present Future 1 1
Y= 3x ~ Eq.1 ---=-
Parents 2x-10 2x 2x+30
2 4 X
X+ y =18 z.= x + 8 ~ Eq. 2 Children x-5n X x+15n
x = 4 hours
y=18-x ~Eq.1 X + y + z = 38 ~ Eq. 3
2x-10=4(x-5n)
(y + 3) = 2(x + 3) ~ Eq. 2
Substitute equations 1 & 2 in equation 3:
·J 2x -10 = 4x- 20n liD 1 1 1 1

l
Substitute y in equation 2: x+3x+(x+8)=38 2x = 20n-10 -+---=-
9 12 15 X
X=6 x = 1On- 5 -+ Eq. 1
(18- x) + 3 = 2x + 6 x = 7.826 hours

lEI i x = 7 hours & 50 minutes


21-x=2x+6 2x + 30 = x + 15n
X=5 Present X = 15n- 30 -+ Eq. 2
18
y=18-5 14 Equate equations 1 and 2: Let: n = number of days needed to
complete the work
y =13
15n-30=10n-5 1 1 1
(18-x)=2(14-x) -+-=-
ml

n=5 X y n
18-x = 28-2x
_!=x+y
I Past Present X =10
n xy
Robert I b+ 15- v b + 15 Present
Stan I b-y b Ill 2x n=.2L


Present X X+Y
Past ~'

(b+15-y)=2(b-y) Father X 38 t
~ 2x- 3(x- 4) = x -(x -4)
b + 15- y = 2b- 2y Son 0 X
~ Let: A = number of hours, Pedro can
'!:
2x - 3x + 12 = x - x + 4
2b - b- 2y + y = 15 paint the house
38- X= X- 0 -x+12=4 B = number of hours, Juan can paint
b-y=15
X =19 X=8 the house
Ill Past Present
Two years ago, the son was
2x=16
C = number of hours, Pilar can paint
the house
(19- 2) = 17 yea'rs old 1 1 1 1
3x -3 3x -+-+-=- ~Eq.1
x-3 X Thus, Debbie is now 16 years old. A B C 4

3x- 3 =4(x- 3)
_!_ = 1.5(1) · _!_ = 0.666(2-) ~ E . 2
3x- 3 = 4x -12 A B'B A· q
Day 4- Age, Work, Mixture, Digit, Motion Problems 77
76 100 i Solved Problems in Engineering Mathematics (2nd Edition) by Tiong & Rojas

*=1.2(~} ~=0.833(l) ~ Eq. 3


28-5
~=2(~) ~ Eq.2 1
-+-=-
1
200 300 X
1

=e-


8 2 -58 x = 120 man-hours
Substitute equations 2 & 3 in equation 1: Substitute equation 2 in equation 1:
128 - 30 = 8 2 - 58 2 1 1
-+-=-
82 178 + 30 = 0 y y 6
1
A+0.666 A.
(1) +0.833 (1)
A = 1
4
-
Let: x = capacity of the larger shovel in
{B-15}(8-2)=0 3
A= 10 hours -=- m3/hr
B = 15 days y 6 y = capacity of the smaller shovel in
El B = 2 (absurd) y =18 hours m3/hr
Note: (rate)(time) = 1(complete job)
1 1l
( -+-14+-(x)
9 16;
1
9
=1
Substitute B = 15 in equation 2

A =15 - 5 =10 days •


Let: x = fraction of the lawn that can
40x + 35y = 20000
X= 500- 0.875y --+ Eq. 1 .

• mowed after one hour 70x + 90y ,;, 40000 --+ Eq. 2
0.6944 + 0.111x = 1
x = 2. 75 hours X =(..! + _! + .!) (1) Substitute equation 1 in equation 2:
ml Let: x = number of days needed by A, B
4 6 7
42+28+24 70( 500- 0.875y) + 90y =40000
Let: x = time for l!utch to finish a certain and C to finished the work X=_:_-~-=-..

job working alone working together. 168


35000-61.25y+90y = 40000
y = time for Dan to finish a certain 1 1 1 1 94
-+-+-=- X=- y = 173.9 m3 /hr
job working alone A B c X 168
1 1 1 47
-+-=- ~ Eq.t X=- X= 500-0.875{173.9)
1 1 1


-+-=- ~ Eq. 1 A B 42 84
X y 6 x = 347.8 m3 /hr

•c:]+I
1 1 1
-+-=- ~·~q. 2
X= 2y ·~ Eq.2 8 31 c
1 1 1 Let: x = time for the farmer to ~_!}he

I+ c:J c:J
Substitute equation 2 in equation 1: -+-=- ~ Eq. 3
A C 20 field
y = time for the son to flow the field, 35% =
1 1 1 Add the three equations:
-+-=- 10 15 30 55
2y y 6
1+ 2 1 ..!] +[..!B + _!_] +[_!_ + ..!] = ~ + __!_ + ~
1 [1 1]
-(3)+ -+- (3)=1
X X y
~ Eq.1
2Y=6 [ _!_
A B
+
C A C 42 31 20
,.
.~.·.· 0.25{10}+0.35{15) + 0.10(30) = x(55)
y = 9 days =0.106 ' Substitute x =8 in equation 1: X=19.55%

• 1
-+-=-
A B 6
A=B-5
1 1
~

~
Eq. 1

Eq.2
2
A B C
1 1 1
2 2
-+-+- = 0.106

-+-+- =0.053 =-
A B C
1
x
x = 18.87days
r~(3)+[i+~ J(3)= 1H
1 1 1 1
-+-+-=-
8 8 y 3
y = 12 days
•c=]+lso%1=14o%J

m

Substitute equation 2 in equation 1: x~19days x 60-x 60

1 1 1
i For the 35% solution:
-+-=- Let: x =number of man-hours needed by 0.35(x) + 0.50(60- x) =0.40{60)

'
B-5 B 6 Let: x = time for Myline to finish the jo crew number 1 and number 2
y = time for Jeana to finish the job to finish the job. 0.35x + 30- 0.5x = 24
B+(B-5} 1
B{B-5) =5
1 11 X=40 m3
-+-=- ~Eq.1
X y 6
18 1001 Solved Problems in Engineering Mathematics (2"d Edition) by Tiong & Rojas Day 4- Age, Work, Mixture, Digit, Motion Problems 79

For the 50% solution: Substitute equation 1 in equation 2: . . 36


.Let: x = the first number Thus, the fractlon 1s -
x - 5 = the second number
55
60-x=20~ m 3 2x +8 = (4x -6)+4

•c=:J c=J c::J


100-x
+
x
=
. 100
x+(x-5)=135
2x = 140
X= 70

x-5 =65
I~i1i
1
10 =2x
X=5

y=4(5)-6
y = 14 <q,
ml
Let: x = numerator of the faction
y = denominator of the fraction

Y = 2x + 3 ~ Eq. 1

0.70(1 00..., x) + 0.60(x) = 0.66(100) 5 X+7 3


Thus, the numbers are 70 and 65. Therefore, the fraction is - . y+ 7 5
70- 0.7x + 0.6x = 66 14
x =40 ounces Ill L:et: x = the number
1111
5x + 35 =3y + 21 ~ Eq. 2

•~+ x
c:J= c:J
2000-x 2000
DD
Ax-10=14
X=6

Let: x =-the first number


2x = the sec;ond number
·.~
Let: x = the number

[ l(x) ][ ~(x) J= 500


2
~=500
20
Substitute equation 1 in equation 2:

5x + 35 = 3(2x + 3) + 21
5x + 35 = 6x + 9 + 21
X=5
For 14% substance: x 2 = 10000
x + 2x = 21
x=7 X =100 y=2(5)+3
0.14(x) + 0.06(2000- x) = 0.08(2000)
0.14x + 120- 0.06x = 160

For 6% substance:
X= 500 kg

2000 - X = 1500 kg
2x =.14

Thus, the numbers are 7 and 14.

~~~~
• Let:

X -3
--=-
~ = the fraction
y

y
3
5 IB!I Let:
y = 13

Therefore, the original fraction is ~.

x = the first num.ber


13

•c::J- c:J= 6
Let x = the number ~~l 3y = 5x -15 y = the second number

· 9x+8=71 5
y=-x-5 ~Eq.1 2x+ y = 19
X=7 3
y = 19- 2x ~ Eq. 1

I Ill Let: ~ = the fraction


X 2
3x=y+21 ~ Eq. 2
10 X 10-x y-1 3
y
3x=2y-2 ~Eq.2
0.04{10)- O(x) = 0.10(10- x) x-2 Substitute equation 1 in equation 2:
-=-
0.4 = 1-0.1x y~2 4
Substitute equation 1 in equation 2:
3x=(19-2x)+21


X=6 kg 4x-i3 = y -2
=2[~x -5 ]-2
,-;,_"

y=4x-6 ~ Eq.1 3x 5x=40


x=8
10
3x=-x-10-2
Let: y = tens' digit of the number X+4 3
--=-
x = units' digit of the number y+4 2 0.333x = 12 y=19-2{8)
2x + 8 = y + 4 ~ Eq. 2 X =36 y=3
The two digit number is represented by:
1Qy +X. 2x+8=y+4Jar
:. prqduct of the numbers is 8(3) = 24:
5
y (36) 5 -55
3
''II
'li

80 IOO"l Solved Problems in Engineering Mathematics (2"d Edition) by Tiong & Rojas
--------'-----.:D=ay'-4 -Age, Work, Mixture, Digit, Motion Problems ~J. I
Ill w + (w- 3)+(w +4) +(w + 6) = 35
4w=28
., at,= 101,-50
t, =25 min. 14 3
Let: t = ten's.digit of the number '·.!··'..
---- ------·-· '::':.

u = unit's digit of the number W=7 ,I V1 +V2 V1 -V2


~~ t2 = 25-5 14V1 -14V2 '"" 3V1 + 3V2
Substitute w = 7 in equation 3: '.':.~
Number= 10t + u
~ t 2 = 20 min. 11V1 = 1'7V2
'~~ V1 = 1.545V2 -~ Eq. 2
Z=7+6
•=u-~ ~ Eq.1
z =13
·r.
,''.if
,:\r
.I
IIEII Let: v, = velocity of boatman
v2 =velocity of stream Substitute equation 2 in equation 1:
10t+U= 4 +~
t+U t+U Ill Total time= 14 hrs 49.6(1.545V2 ) = 14(1.545V2 ) -14V/
2
10t+u-3 Let: V 1 = velocity of the first jogger ·-.(···· .............................................................

-
4
t+U V2 = velocity of the second jogger i4.832V2 =19.418V/
10t+ u -3 = 4t + 4u
Direction of the
v2 "'0.76 mph
6t-3=3u ~· Eq. 2 Point where stream current
· jogger 2 · ~~~.%,'{W"''V' !IP.Jl!ll dis tan ce
. Substitute equation 1 in equation 2:
t1 starts running ll&iill Note: time= -v,.ei"OcitY
f.t----------------1
4.8miles
6(u-3)-3=3u
fhe two figures below that the same time.
6u-18-3=3u
3u=21

2__,__ ~----.r
u==7 s1 -v,+V2

t=7-3 t2=t1-5
t =4 / ................................. ····l·.................................. c...
14miles
Number= 10{4)+ 7 --=-- v2 t, + t2 = ttotol
Number= 47 s s 7


~~,

~
P twtttfl!.t!lklfW*iP!X!lf.. ?'MP.tEt.,. !IC,.i!IM. ~'\WI' -·+-=-
5 2 4
Let w = first number s2 .... - . v,_ .. v2 0.7S =d.75.
x = second number
y = third number
z =fourth number
Point where
jogger 2 catches
up jogger 1
--c_-
~· 3miles
S=;2.5 miles

=
V, velocity of airplane

,
-7 Eq. 1 • Let:
x=w-;3
V2 =velocity of wind
y =w+4 -7 Eq.2
V, = Skmx~=_!_km/min ~~+~=14 11=2
hr 60min 60 V1 +V2 v,- V2 .....................................................................

~
z = y + 2 = (w + 4) + 2 4 8(V1 +V2 }+4.8(V1 + V2 ) I
v2 = ~~ km/min -
(V1 + V2 }(V1 - V2 )
=14 V!+V2-+
z =w+6 7 Eq. 3 , !
..----------
w+xr35 ->Eq.4

Substitute eq1f3tions 1, 2 and 3 in


8
S1 =S 2
v,t, = V2t 2

-t, =-(t,-5)
10
,1 IIV1 ~: 4.8V2 + 4.8V1 + 4.8V2
V/ . . v,v2 + v,V2 - v/
96V1 14V/-14V/~Eq.1
=14 I
r
1000
s,=1ooo +I
!

60 60 v, + V2 =----·- = soo 7 Eq. 1


equation 4: / ' 2
I
1\
.
Direction of the wind -
I 2= 2.5
-<······· ...................................... .
- v.-v,
L.,
"-
...
82 ·100 I Solved Problems in Engineering Mathematics (2nd Edition) by Tibng & Rojas
RR
Let: v, =ground speed of plane A
V2 '= ground speed of plane B

Let: t = time needed to travel and reach
destination without any delay
Day 4- Age, Work, Mixture: Digit, Motion Problems 83

Mi£li
,...•. III:J. J/
Point where
the accident
happened

I· 82=1000 •
1 ~ ,
PlaneA.................................. . .............._.............
V = velocity of the train

General equation:
Time consumed by the train traveling, s, = v +50
.l
S2=S-V-SO

v, - v2 = 1ooo =400 ~ Eq.2


~-v ,,
,,'
before the accident + Time during
which the train was detained + Time t, = v +50 t2
S-V-50
2.5 :w needed to continue the course and v ~v
=
·_.:1-~·.
reach the destination Time needed 5
s = 900

1 to travel and reach the destination
Subtract equation 2 from equation 1:
.,
']' without any delay + Time of delay S =VI "
(v, + V2 ) - (v, - V2 ) "' 500 - 400 \:
Plane B . t-2.25 Condition 1: If the accident happened 1 50+V + 1+ S-(50+V) =I+~
2V2 = 100 hour after, substitute values to the v ~v 2
v2 =50 kph general equation: · 5
..,cz~ -V+90
\1 Point where 50+ V +~(S-50- Vh~+_!] V
Ill the accident
happened
[ V 3V . V 2

Let: V = velocity of the boat in still water ~


5 250 5 . v
50+V+-S----\i =S+-
s, = distance traveled upstream 3 3 3 2
=
s2 distance traveled downstream
S1 =S 2
Vt = (V + 90){1- 2.25) Substitute equation 1 in above equation:

-
Direction of
stream current
- V+8 Vt = Vt- 2.25V + 90t- 202.5

2.25V- 90t + 202.5


900
=0 ·
50 +§.+~S-
4 3
250 -~(§.)=s+.!(~)- 100
3- 3 4 2 4 3

=--S
3
8

F- :,== But t =

2.25V- 90
. (900)
V
v
+ 202.5 "" 0
. S-V
1+1+-3- = t+3
S = Vt

I9R Let:
s =

V = rated of Erwin
800
9
miles

V-8- Multiply both sides by V: -V lllii6ill V - 13 = rate of Edgar


5
DireCtion of 2.25V2 - 81000 + 202.5V = 0 S-V =t+1
11
-stream current 2.25V2 + 202.5V -· 81000 = 0
~v ~ -V·13
Divide all by 2.25:
b . s
5 -,~,.,, .
S- 231 km.
:I·
Su st1tute t = V :
V 2 +.90V- 36000 =0 12
(V -240}(V +150) = 0
5 s
-(S- V)=-+1
S 1 =S 2 3V V
v =2-:r:, )~ph
(V-8)1=( V ·E:.· !~ph (absurd) ~(~)=~ S=308km

~V+ 163
t, = h
s
V-
;A3=~ V=-
4 ~ Eq . 1 231 308
V-13 V
/ V=40 kph 231 v = 308 v -4004
< :ondition 2: If the accident happened 50
I 111lln'~ fnrthcr. suhstitutP. ve~luP.s to tiH' v =52 kph
I\ 'i<'IH!IOIII•qliOIIIOil
86 1001 S~lved Problems in Engineering Mathematics (2nd Edition) by Tiong & Rojas

~ ---- --

Topics

D tv1on
Clock Problems
Variation Problems
Diophantine Equation
D Tue
Sequence
Series
Arithmetic Progression
D
Theory Wed
Geometric Progression
Infinite Geometric Progression

D 0
Harmonic Progression
~I Figurate Numbers
Problems Thu

D ~
·splutions Fri

D D
Notes Sat

What is a clock problem? Most of the problems in clock problem


involve only the minute hand and the hour
A clock problem is a mathematical hand. The diagram shows the relation
problem which focus on the relationship of between movement of the minute and hour
tile movements of the hands (hour hand, hands.
minute hand, second hand) of the clock.
rhis type of problem is for mechanical
docks only and never for a digital clocks.

rhe longest hand is the second hand while


llle shortest hand is the hour hand. By
principle, the second hand ($H) always
rnoves faster than the minute hand (MH)
;md the minute hand always moves faster
lllan the (HH). The relation between the
thn~e hands of the clock <:~re as follows:

HH=~ MH..,.!t!
~
HH"'MH
12 720 12

where : SH is in number of seconds 12


MH is in numb('r of minutes The earliest known clock problem was
posed in 1694 by Jacques Ozanam in his

\
~~ 1. Solved P:ro~~ns in Engineering Mathematics {?nd Eqition) by Tiong & Rojas Day 5- Cl_ock, Variation, Miscellaneous Problems & Progr§lssion 89

"Recreations Mathematiques et Diophantine equation is named after a What is a progression? Also, there are only two formulas (i.e. last
Physiques". Greek mathematician, Diophantus of term and sum) to remember and used in
Alexandria (AD c200- c284) who A progression is simply another term for a solving a problem in geometric sequence.
sequence.
What is Variation Problem? " developed his own algebraic notation and
is sometimes called the "Father of What is are the types of progression? Last term (n 11' term):
Variation problems are problems in Algebra".
Algebra which show the relationship The most common types of progression " an == a1rn-1
between the variables in terms of are:
expressions such as "directly proportional What i~ a sequence?
or inversely proportion or simply 1" Arithmetic Progression (AP) Sum of all terms:
proportional". A sequence is a set or collection of 2. Geometric Progression (GP)
numbers arran9ed in an orderly manner 3, Harmonic Progression (HP)
The expression x varies directly as y is s"uch that the preceding and the following S = a.1(rn .:_1) or S = a1(1-rn)
expressed as follows: numbers are completely specified. What is an arithmetic progression? r-1 1-r
xocy What is the difference between an " A sequence is said to be in arithmetic
infinite and a finite sequence? progression if its succeeding terms have where: a, = first term
The symbol varies ( oc ) is replaced by an a common difference" an = last term (d" term)
equality symbol and a constant of An infinite sequence is a function whose n = number of terms
proportionality, k, hence: domain is the set of positive integer. If the The corresponding sum of all the terms in r= common
domain of the function consists of the first arithmetic progression is called as
n positive integers only, then it is said to arithmetic series. ratio= a2 = a 3. = ...
X =ky a1 a2
be a finite sequence.
The expression x varies inversely as the There are only two formulas (i.e. last term
1+3+5+7+9 - finite sequence and sum) to remember and used in solving What is an Infinite Geometric
square of y is expressed as follows: Progression?
a problem in arithmetic sequence.
1 + 3 + 5 + 7 + 9 + ··.-,..infinite sequence
X= k_!_ Last term (n1h term): This type of progression is a geometric
y2 What is an element? progression only that the number of terms
(n) is extremely large or infinity.
a0 =a1 +(n-1)d
Elements are the term used to describe
the numbers in a given sequence" An If r > 1, sum of all terms is infinite
Sum of all terms: If r < 1, the sum of all terms is
A Diophantine equation is an equation element is·sometimes called a term.
that has integer coefficients and for which
integer solutions are required. What .is a series? S=%(a 1 +a.) or S=%[2a 1 +(n-1)d]
S=-3_
The best examples of Diophantine Series is the sum of the terms in a 1-r
equations are those from Pythagorean sequence. where: a 1 = first term
Theorem, a 2 + b 2 = c 2 , where a, band c an= last term (n1h term)
What is an alternating series? n = number of terms where: a, = first term
all required to be whole numbers. · r = common ratio
d = common difference
Despite its simple appearance, An alternating series has positive and = =
d a2- a, a3- a2 = ...
What is Harmonic Progression?
Dir;>phantine equations can be fantastically negative terms arranged alternately.
difficult to solve. A notorious example
comes from Fermat's Last T~which What is the difference between a
,I What is a ge.ometric progression?
A sequence of numbers whose reciprocals

is a" = b" + c" , where


converging series and a diver..aent
series? I !\ sequence is said to be a geometric
progression if its succeeding terms have
.1common ratio.
form an arithmetic progression is known as
harmonic progression. In solving a
problem, it would be wise to convert all
Diophantine eqw;rtions may refer to a given terms into arithmetic sequence by
If an infinite series has a finite sum, it is
system of equations where the number of IIH~ corresponding sum of all the terms in getting its reciprocals. Use the formulas in
referred to as convergent sorlos and
equations is o)feless ihan the number of divergent series if it has no sum nl all .,, ·onwtric progression is called as arithmetic sequence and take the

number for it,


unknowns. Ttjese equations yield whole
a11swers"
quometric series" reciprocal of resulting value to obtain the
equivalent harmonic term for an answer.
1,1

\ l
~ 90 1001 Solved Problems in Engineering Mathematics (2"ct Edition) by Tiong & Rojas
Dav 5- Cloc:k, Variation, Miscellaneous Problems & Progression 91

D. ObloJOtg numbers: Numbers Proceed to the. next page for your 5th test.
What are the Fibonacci and Related
which can be drawn as dots and Detach and use the answer sheet provided
Sequences?
arranged in a rectangle shape. at the last part of this book. Use pencil
number 2 in shading your answer.
The following are the some of the famous
••••• ••••••
and related sequences:

1. Fibonacci Numbers - Named after ••


2
•••
•••
6
••••
••••
••••
12
•••••
• ••••
20
• •••••
••••••
••••••
• •••••
30
...... GOOD LUCK I

the Italian merchant and


mathematician, Leonardo di Pisa or
E. Pentagonal numbers:
\!l:ribia:
Fibonacci (Figlio dei Bonacci, "Son of Did you know that... the eminent German
the Bonnaccis"). mathematician, Carl Freidrich Gauss'
father was an accountant and· young
1, 1, 2, 3, 5, 8, •13, 21, 34, 55, 89,
144 ...

Each number is equal to the sum of


. (.: 5
' fit {jjJ
\~l -:-:11
12
ril . 22 35
Gauss corrected his father's spreadsheet
at the age of 3 !

@uote:
the two preceding numbers.
F. Cubic numbers: "Mathematics is the queen of sciences and
2. Lucas Sequence - Named after
Edouard Lucas (1841- 1891). Like
the Fibonacci numbers, every term of
the Lucas. sequence is the sum of the
two preceding numbers.
• riD
8
~re
t=-~
....
~:-·
.- If
....,__. -·
27
....
~

64
arithmetic is the queen of mathematics.
She often condescends to render service
to astronomy and other natural sciences,
but under all circumstances the first place
is her due."

1' 3, 4, 7, 11' 18, 29, 47, 76, 123 ... - Carl Freidrich Gauss
G. Tetrahedral numbers:
3. Figurate Numbers:

A. Triangular numbers: Numbers


which can be drawn as dots and
arranged in triangular shape .
• 4;.
4
4. ·~
. 10 20

• ••
. H. Square pyramidal numbers:

• •••
• ••

••
•• •••• •••
••
• •• •••••
• ••• ••••••
• •••• .·.··'·

••
1 3 6 10 15 21

B. Square numbers: Numbers •1 ~ 14 30


which can be· drawn as dots and
arranged in square shape.
I. Supertetrahedral numbers:
••••• ••••••
••••••
••• •••• •••••
•~·····
••••••
•••• ••••• By piling up tetrahedral numbers,
•• •••
•1 ••4 ••• ••••
• ••• •••••
••••• •••••
•••••• 1, 4, 10, 20, 35, 56, etc, we make
9 16 25 36 four-dimensional numbers:

C. Gnomons:y.«fmbers which can 1, 5, 15, 35, 70, ...


be drawr;Vas dots on equally long
leg7:sa right angle. • 1 =1
1+4 =5
•• ••• •• 1+4+10 =15

• •
• '• • ••• • • • •
• • • • • •
• • • • • e •
•• 1 + 4 + 10 + 20
1 + 4 + 10 + 20 + 3!1
:15
{()
1 3 5 7 9 11
Day 5 -Clock, Variation, Miscellaneous Problem!J & Progression 93
/

.1.11&: EE Board October 1990 ~oo: CE Board May 1993


A man left his home at past 3:00 o'clock ·· Given that "w" varies directly as the
PM as indicated in his wall clock, between product of "x" and "y" and inversely as the
2 to 3 hours after, he returns home and square of "z" and that w = 4 when x = 2, y
noticed the hands of the clock = 6 and z = 3. Find the value of "w" when
interchanged. At what time did the man x = 1, y = 4 and z = 2.
T~pics

Dr
leave his home?
A. 3
Clock Problems A. 33147 B. 4
Mon Variation Problems B. 321.45 C. 5
j Diophantine Fquation C. 346.10 D. 6

p ~
D Tue
j
j
Sequence
Series
D.
/
3 36.50

197: GE Board February 1994


""
~1: ECE Board November 1993
If x varies directly as y and inversely as z,
Arithmetic Progression From the time 6:15PM to the time 7:45 and x = 14 when y = 7 and z = 2, find the

LJ D
Theory VVed
I
j
Geometric Progression
Infinite Geometric Progression
PM of the same day, the minute hand of a
standard clock describe an arc of
A. 14
=
value of x when y 16 and z 4. =

D
Problems Thu
Harmonic Progression
Figurate Numbers
A.
B.
c. fso·
D.
60°
go•
540•
B.
C.
D.
4
16
8

lJ ~
Solutions Fri
198; EE Board Aprill990
A storage battery discharges at a rate
~: EE Board Marc:h 1998
The electric power which a transmission
line can transmit is proportional to the
which is proportional to the charge. If the product of its design voltage and current

D D
Notes Sat
charge is reduced by 50% of its original
value at the end of 2 days, how long will it
take to reduce the charge to 25% of its
capacity, and inversely to the
transmission distance. A 115-kilovolt line
rated at 100 amperes can transmit 150
original charge? megawatts over 150 km. How much
"' power, in megawatts can a 230 kilovolt
1gt·; CE Board May 1995 A. . 3:02.30 A. 3 line rated at 150 amperes transmit over
.fu how many minutes after 2 o'clock will B. 3:17.37 B. 4 100 km?
the hands of the clock extend in opposite C. 3:14.32
directions for the first time?
c 5
D 3:16.36 D. 6 A. 785
il A. 42A minutes
/
.:144: GE Board February 1997
B. 485
199: ECE Board April :1990 C. 675
B. 42.8 minutes "'At what time after 12:00 noon will the lhe resistance of a wire varies directly D. 595
C. 43.2 minutes hour hand and minute hand of the clock with its length and inversely with its area.
·D. 43.6 minutes first form an angle of 120°? If a certain piece of wire 10 m long and ~--3~ ME Board October 1992
19ii CE Board November 1995 A. 12:18.818
C.10 em in diameter has a resistance of 'ihe time required for an elevator to lift a
tOO ohms, what will its resistance be if it weight varies directly with the weight and
'1n how many minutes after 7 o'clock will B. 12:21.818 1s uniformly stretched so that its length the distance through which it is to be lifted
the hands l;le directly opposite each other c. 12:22.818 becomes 12 m? and inversely as the power of the motor. If
for the first time? D. 12:24.818 it takes 30 seconds for a 10 hp motor to
/\. 80 lift 100 lbs through 50 feet, what size of
A. 5.22 minutes ./1.95: At what time between 8 and 9 ll. 90 motor is required to lift 800 lbs in 40
B. 5.33 minutes o'clock will the minute hand coincide with
G. 5.46 minutes
c 144 seconds through 40 feet?
the hour hand? I) 120
D. 5.54 minut ~ A. 42
A. 8:42.5 B. 44
193:,/'~
CE oard.·
May 1997 R 8:43.2 C. 46
What ti e after 3 o'clock will the hands of c. 8:43.6
D. 48
the clo · k be together for the first time? D. 843.9
I
94 . 1001 Solved Problems in Engineering Mathematics (2"d Edition) by Tiong & Rojas Day 5 - Clock, Variation, Miscellane~us Problems & Progression 95
r
204: The selling price of a 1V set is 208: EE Board Marc:h 1998 B. 308.1 ft US: EE Board April1997
double that of its cost If the 1V set was In a certair.~ community of 1,200 people, C. 241.5ft ,.., A stack of bricks has 61 bricks in the
sold to a customer at a profit of 25% of 60% are literate. Of the males, 50% are D. 305.9 ft bottom layer, 58 bricks in the second
the net cost, how much discount was literate and of the females 70% are layer, 55 bricks in the third layer, and so
given to the customer? .literate~ What is the female po'pulation? 2~3r-if the first term of an arithmetic on until there are 10 bricks in the last
'progression is 25 and the fourth term is layer. How many bricks are there all
A. ~.7% A. 850 13, what is the third term? together?
a ~.7% B. 500
C. ~.5% C. 550 A. 17 A. 6~
D. ~.j% D. 600 B. 18 B. 6~
C. 19 C. 6~
205: A group of EE examinees decided D. 20 D. ~
209: ECE Board Marc:h 1996
to hire a mathematics tutor from Excel A merchant has three items on sale,
:t:t~ECE Board November J:998
. CE Board May 1998
)
~2·J:'9:
Review Center and planned to contribute namely a radio for P 50, a clock for P 30
equal amount for the tutor's fee. If there Find the 30th term of the arithmetic • Determine the sum of the progression if
and a flashlight for P 1. At the end of the
were 10 more examinees, each would ,l:i progression 4, 7, 10, .. there are 7 arithmetic mean between 3
day, he sold a total of 100 of the three
have paid P 2 less. However, if there items and has taken exactly P 1,000 on and 35.
'~
were 5 less examinees, each would have the total sales. How many radios did he
A. 75
paid P 2 more. How many examinees are sale? B. 88 A. 171
there in the group? C. 90 B. 182
A. 16 D. 91 C. 232
A. 14 B. 20 D. 216
B. 16 C. 18 :.t~·s: CE Board May 1993 ./'

C. 18 D. 24 CE Board May 1994, ·2ZO: ECE Board April J:995


D. 20 CE Board November J:994 A besiege fortress is held by 5700 men
,v:z.t-0:~ ME Board October J:996 How many terms of the progression 3, 5, who have provisions for 66 days. If the
206: EE Board Marc:h J:998 The arithmetic mean of a and b is 7, ... must be taken in order that their garisson losses 20 men each day, for
A bookstore purchased a best selling sum will be 2600? how many days can the provision hold
price book at P 200.00 per copy. At what out?
a+b
price should this book be sold so that, A. A. 48
2 B. 49 A. 72
giving a 20% discount, the profit is 30%?
B. Jab C. 50 B. 74
JO D. 51 C. 76
ab
B. P 500 C. - D. 78
C. P 357
2 U6~ME Board Apri11995
D. P 400 a-b In a pile of logs, each layer contains one .~z.rf'CE Board May 199J:
D.
2 more log than the layer above and the top In the recent "Gulf War" in the Middle
207: ECE Board November 1993 contains just one log. If there are 105 logs East, the allied forces captured 6400 of
Jojo bought a second hand Betamax VCR .zii: The sum of three arithmetic means in the pile, how many layers are there?. Saddam's soldiers and with provisions on
and then sold it to Rudy at a profit of 40%. between 34 and 42 is hand it will last for 216 meals while
Rudy then sold the VCR to Noel at a profit A 11 feeding 3 meals a day. The provision
of 20%. If Noel paid P 2,856 more than it A. 114 G. 12 lasted 9 more days because of daily
B. 124 (~. 13 deaths. At an average, how many died
cost to Jojo, how much did Jojo paid for
the unit? C. 134 n. 14 per day?
D. 144
A. p 4,000 U'7: CE Board May 1995 A 15
B. P 4,100 U.J:1!:E Board Marc:h 1998 \:'Jh:1! 1s tt·"' sum of the progression 4, 9, B. 16
C. P 4,200 :t:;ravity causes a body to fall 16.1 ft in the 14; ·19 ... up to the 20th term? c. 17
D. P 4,300 first second, 48.3 in the 2"d second, 80.5 D. 18
in the 3'd second. How far did the body A 1030
fall during the 101h second? II 1035
<; II 140
A 248.7 ft II ,:145

.{
Day 5 - Clock, Variation, Miscellaneous Prob~J & ProJ.l!ession 92'
96 1001 Solved Problems in Engineering Mathematics (2nd Edition) by T'ong & Rojas
C. 0.102 ,:zj"(;~ ME Board October 1996
;atZ: GE Board July X99:5 "pf;blem 226: D. 0.099 A product has a current selling of P
·"A Geodetic Engineering student got a · When all odd numbers from 1 to 101 are 325.00. If its selling price is expected to
score of 30% on Test 1 of the five number added, the' result is 2~i;' Find the 9 1h term of the harmonic decline at the rate of 10% per annum
test in Surveying. On the last number he progressio11 3, 2, 3/2 ..... because of obsolescence, what w\11 be its
got 90% in which a constant difference . A. 2500 selling price four years hence?
more on each number that he had ·on the B. 2601 A. 3/5
immediately preceding one. What was his C. 2501 B. 3/8 A. P 213.23
average score in Surveying? D. 3500 C. 4/5 B. p 202.'15
D. 4/9 C. P 302.75
A. 50 ~z71'"How many limes will a grandfather's ..... -~
D. P 156.00
/clock strikes in one day if it strikes only at :a3'i: Find the sum of 4 geometric means
B.
c.
55
60 the hours and strike once at 1 o'clock, between 160 and 5. ~: CE Board May 1995
twice at 2 o'clock , thrice at 3 o'clock and The numbers 28, x + 2, 112 form a G. P.
D.
/,<'
65
so on? I A.
B.
130
140
What is the 101n term?
~~b: ME Board April 1999
:If the sum is 220 and the first term is 10, A. 210 C. 150 A. 14336
find the common difference if the last term B. 24 D. 160 B. 13463
is 30. c. 156 c. 16433
D. 300 233: EE Board October :199:1 D. 16344
A. 2 <>"~ The fourth term of a G. P. is 216 and the
B. 5 "-'. Uth CE Board May :1992 61h term is 1944. Find the 81h tem1: ,.A:J's: ECE Board April :1998
c. 3 · To conserve energy due to the present The sum of the first 10 terms of a
D. 2/3 energy crisis, the Meralco tried to re- A. 17649 geometric progression 2, 4, 8, ... is
adjust their charges to electrical energy B. 17496
2:1'.ci;·'EE Board April 1997 users who consume more than 2000 kw- c 16749 A 1023
Once a month, a man puts some money hrs. For the first 100 kw-hr, they charged I> 17964 B. 2046
into the cookie jar. Each month he puts 40 centavos and increasing at a constant C. 225
50 centavos more into the jar than the rate more than the preceding one until the ZJ4: ECE Board April 1999 D. 1596
month before. After 12 years, he counted fifth 100 kw-hr, the charge is 76 centavos. Determine x so that: x, 2x + 7, 1Ox - 7 will
his money, he had P 5,436. How much How much is the average charge for the he a geometric progression. :t:J9f'if the first term of a G.P. is 9 and
money did he put in the jar in the last electrical energy per 100 kw~hr? p 1he common ratio is -213, find the fifth
I
1\. 7, -7/12 term.
month?
A. 58 centavos II. 7, -5/6 .
A. P 73.50 B. 60 centavos (' 7, -14/5 A. 8/5
B. P 75.50 C. 62 centavos D 7, - 716 B. 16/9
c. p 74.50 D. 64 centavos C. 1517
D. P 72.50 Z}Sr ECE Board April 1999 D. 1314
/" ·.
... -2291 CE Board November :199:5 II one third of the air in a tank is removed
/If hy each stroke of an air pump, what .,2401 EE Board April 1997
.,&25: EE Board AJiril1997 · The 3'd term of a harmonic progression is
A girl on a bicycle coasts downhill 15 and the 91h term is 6. Find the 11th ''action a I part of the total air is removed in The sev~nth term is 56 and the twelfth
covering 4 feet the first second, 12 feet teim. · · 1; strokes? term is --1792 of a geometric progression.
the second second, and in general, 8 feet Find the comro~,..., ral,..; i;jnd the first term.
more each second than the previous A. 4 (\ 0.7122 Assume the ratios .are equal.
second. If she reaches the bottom at the B. 5 II 0.9122
end of 14 seconds, how far did she C. 6 ,.. 0.6122 A. -2, 5/8

I
D. 7 ll 0.8122 B. -1, 5/8
coasts?
C. -1, 7/8
A. 782 feet .•. ~;;: ECE Board November 1995 D. -2, 7/8
B. 780 feet Find the .fourth term of the progression
C. 784 feet 1/2' 0.2, 0.125, ~ .. I
b. 786 feet ·~
A.. 1/10
B. 1/11
98 .1 00 1. Solved Problems in Engineering Mathematicl:' (2nd Edition) by Tiong & Rojas
Day 5 - Cl<>.c:1c, Y:ariation, Misc~U~n~ous Pro~lems & Progression 99
,~.u; A person has 2 parents, 4 -~Zlf$:
EE Board March J:998
B. 5/2 ~: ECE Board Aprill998
grandparents, 8 great grandparents and Determine the sum of the infinite series:
C. 712 r Find the 198ih digit in the decimal
so on. How many ancestors during the 15
generations preceding his own, assuming
no duplication?
s= i + ~ + 2~ + ... + ( i J D. 11/2

zsol CE Board May 1998


. Ient to -
eqUJva 1785 . from t h e
- startmg
9999

A.
B.
131070
65534
A.
B.
4/5
3/4
Find the sum of 1. - i, 2~ ....
decimal point.

A. 8
C. 32766 c. 2/3 B. 1
D. 16383 D. 1/2 A. 5/6 C. 7
..... / B. 2/3 D. 5
,..:viz: In the PBA three-point shootout .«:!&: Under favorable condition, a single C. 0.84
- contest, the committee decided to give a cell bacteria divided into two about every D. 0.72
prize in the following manner: A prize of 20 minutes. If the same rate of division is
P1 for the first basket made, P 2 for the maintained for 10 hours, how many 211: ECE Board November :1998
second, P 4 for the third, P8 for the fourth organisms is produced from a single cell? Find the ratio of an infinite geometric
and so on. If the contestant wants to win progression if the sum is 2 and the first
a prize of no less than a million pesos, A. 1,073,741 term is 1/2.
what is the minimum number of baskets B. 1,730,74
to be converted? c. 1,073,741,823 A. 1/3
D. 1,037,417 B. 1/2
A. 20 C. 3/4
B. 19 z.47: EE Board October 1:994 D. 1/4
C. 18 A rubber ball is made to fall from a height
D. 21 of 50 feet and is observed to rebound 2/3 zs:t(EE Board April 1:997 ·
A'
of the distance it falls. How far will the ball If equal spheres are piled in the form of a
24'3: CE Board November 1:994 travel before coming to rest if the ball complete pyramid with an equilateral
/In a benefit show, a number of wealthy continues to fall in this manner? triangle as base, find the total number of
men agreed that the first one to arrive spheres in the pile if each side of the
would pay 10 centavos to enter and each A. 200 feet base contains 4 spheres.
later arrive would pay twice as much as B. 225 feet
the preceding man. The total amount C. 250 feet A. 15
collected from all of them wasP D. 275 feet B. 20
104,857.50 How many wealthy men C. 18
paid? . -ziis: EE Board April 1:990 D. 21
What is the fraction in lowest term
A. 18 equivalent to 0.133133133? zs3: Find the 6th term of the sequence
B. 19 55, 40, 28, 19, 13, ...
c. 20 133
D. 21 A. 1\. 10
666
/ IJ. 9
133
z4~: A man mailed 10 chain letters to ten B. c 8
··of his friends with a request to continue 777 () 11
by sending a similar letter tci each of their 133
C.
ten friends. If this continue for 6 sets of 888 l:S4: EE Board October 1:997
letters and if all responded, how much will In the series 1, 1, 1/2, 1/6, 1/24, ... ,
133
the Phil. Postal office earn if minimum D. determine the 6th term. ·
999

l
postage costs P 4 per letter?
1\ 1/80
A. p 6,000,000 ~~ ECE Board April J:998 'p,_: It 1/74
;
B. P 60,000 .i1Find the sum of the infinite geometric (: 1/100
c. p 2,222,220 progression 6, -2, 2/3, ... ll 1/120
D. P 4,444,440
A. 9/2

I
ml
Day 5- Clock, Variation, Miscellaneous Problems & Progression 101

ml X --
' l
'
'
!
[
t

12 ---

Topics X

D Mon
Clock Problems
Variation Problems
Diophantine Equation
D Tue
Sequence
Series . t es
Note: 1200 ( 30 minutes) = 20 mmu
Arithmetic Progression x=40+~ 180°

D D
Theory Wed
Geometric Progression
Infinite Geometric Progression
12
x = 43.6 minutes X =~+20
12
Harmonic Progression Ill x=21.818 min

D D Figurate Numbers
Therefore, the time is 12:21.818


Problems Thu

Solutions
~
Fri

D D
Notes Sat
X
12

ANSWER KEY RATING


12
\
191. D 208. D 225. c 242.A c:J 55-65 Topnotcher x=5+~
192.C 209. A 226. B 243.C
c:J 42-54 Passer
12
x = 5.454 minutes
193. D 210.A
194. B 211. A
195. c 212. D
227.C
228.A
229. B
244.0
245. D o 32-42 Conditional
X=40+~
12

196. A 213. A 230. B


246.C
247.C
248. D
c:J 0-31 Failed mJ x = 43.6 min
197. D 214. D 231. A Therefore, the time is 8:43.6
198. A 215. c 232.C 249.A !


If FAILED, repeat the test.
199. c 216. D 233. B 250.A
m&Jiill~1:t!!!JlJiiill!i
200.A 217. A 234. D 251. c fiWliilk
X= 30+L 7 Eq.
:~
201. c 218. c 235. B 252. B 12
202.C 219. A 236.A 253.A
203. D 220.C 237.A 254. D y=15+~ ~ Eq.2
i 12
204. c 221. D 238. B 255.A
'
205. D 222.C 239. B
Substitute (2) in (1 ):
206.C 223.A 240. D
L'07.C 224.A 241. B xc15t-~- 15 + -~­
v~,
12 X= 30 + ·--· ! 2._
x 1G.:lG rnin ~2
llll't"lut<', lilt' lllll<' r•; :1 11·, :If>

I
102 100i Solved Problems in Engineering Mathematics (2nd Edition) by Tiong & Rojas Day 5- Clock, Varia!i<m, Miscellaneous Probl~ms & Progression 103

6: 15 = 6(60) + 15 = 375 minutes Note: When the wire was stretched, the

X
7:45 = 7(60) + 45 =465 minutes diameter was changed but the volume
remains constant assuming there was no
ED P=k VI
d
losses in the process. When P = 150, V = 115, I = 100 & d = 150
X=465-375
x = 90 minutes When R = 100 and L = 10
150 = k (115)(100)
2 150
100=k'(10)
k = 1.956
x=90min ( -180°
-) k' = 1
30 min
When V = 230, I= 150 and d = 100
X= 540° When L = 12:

Time when he left his home ml D=kC


R=1(12)
2 p = 1.956[(230)(150)]
100
R =144 Q P = 675 megawatts·
When D = 2 and C = 0.5 C
y
2 ~ k(0.5C)
ml w =k xy mJ t=k ws
p
z2
4
k=- When t = 30, W = 100, S =50 and P =10
c When w = 4, x = 2, y = 6 and z = 3
When C = 0.75 C:
4=k(2)(6) 30 = k[ (10~~50)]
(3)2
Y ; r D=( ~ )(o.75C) k=3
k = 0.06
12 - - : :-----
D= 3 days When t = 40, W = 800 and S = 40
When x = 1, y = 4 and z = 2:
Time when he returned home
ml R=k.!:_
A
~ Eq.1
w=3[(1)(4)]
40 = (0.06>[ (800~( 40)]
Multiple both sides by 12:
(2)2
p =48 hp
= volume of the wire
15+~] 12
Let: V W=3
x=30+~ V=AL ml Let: x = net col.'t
[
A=Lv ~ Eq.2
ml x=kr
2x = selling price
d =discount
2x (1 -d) =new selling price
z
12x = 360 + 15 + ~
12
Substitute equation 2 in equation 1: Wilen x = 14, y = 7 and z = 2 New selling price = Net cost + Gain
x = 31.4it min

R"k( ~]
2x(1- d)= x + 0.25x
14=k~
So the time when he left home was 2 2x - 2xd = 1.25x
3:31.47. k=4 d=0.375
r;;


d =-S:-7<5%


R=k'L2 Wilen y = 16 and z = 4,
Let: x = number of examinees
Let: x = the number of minute difference k'=~ (4}(~6) y = tutor's fee
between 6:15 and 7:45. v ii . X

X
c

16
x
y
original fee shared per examinee
~

Day 5- Clocls__'{_aria!i()n, Miscellaneous Problems & Progression lOS


l 04 1Ot) 1 S_olved Problems in Engineering Mathematics (2nd Edition) by Tiong & Rojas

If !here were 10 more examines who wm


join,
m ~~~~ a, =3; a2 = 5;

By inspection, d = 2
a3 =7
Let: x = number of men in the popuiC!tion Note: Arithmetic mean i? the same term
y =(X+ 10{~--2) y = number of female in the as average.

10y
population
Thus, the arithmetic mean of a and b is,
nr
S='2L2a 1 +(n-1)d
· J
y "' y - 2x + - - - 20
X X+ y = 1200 a+b
10y =2x + 20x
2 X= 1200- y ~ Eq. 1 2 2600 =~[ 2(3) + (n -1){2) J
2
y "'0.2x + 2x --• Eq. 1 0.5X+0.7y=0.6(1200) ~Eq.2 I
I5IIJI 34, a2, a3, a4,42
n
2600=-(6+2n-2)
2 .
If there were 5 examinees who will back- Substitute equation 1 in equation 2: n
a5 = a1 + 4d 2600= (4 + 2n)
out,
42 =34+4d
2
y=(x-5{~+2)
2
0.5(1200- y)- 0.7y = 720 . 2600 = 2n ;t n
d=2
600- 0.5y + 0.7y = 720 n2 + 2n - 2600 = 0
5y
y = y + 2x - · - -1 0 y = 600 females Thus, a2 = 36, a3 = 38 and _a4 == 40 (n + 51){n- 50)= 0 j
5y =2x 2 -10x
X

m Sum = 36 + 38 + 40
Sur:n = 114
Equating factors to zero: I
y = 0.4x 2 - 2x -~ Eq. 2 n ==-51 (absurd)
Let: x = number of radios sold out
y = number of clocks sold out B!J a,= 16.1; a2 = 48.3; a3·= 80.5 n=50
Equate equations 1 and 2: =
z number of flashlight sold out
d = a2 -a 1 ~~~~ a, = 1; d = 1; S = 105
2
0.2x 2 + 2x = 0.4x - 2x X+ y+ = 100
Z 7 Eq. 1 d=48.3-16.1
4x =0.2x 2 50x + 30y + z = 1000 7 Eq. 2 S =~[2a 1 +(n-1)d]
d = 32.2
x = 20 examinees
ti~'
m
Let
Subtract equation 1 from equation 2:

( 50x + 30y + z)- ( x + y + z) = 1000 -1 00


a 10 = a 1 +9d
a10 =16.1+9(32.2)
a 10 = 305.9
105 =%[2{1)+(n -1){1)]

n
105=-(2+n--1)
rl~
49x + 29y = 900 2 (,I
x = selling price without t!iscount
IIi'
0.8x = new selling price (with discount} Assume x = 16:
fJEI af = 25; a4= 13
105 = ~(1 + n)
2
ti;'
ljl
I Profit= Income- Expense!s 49(16) + 29y = 900
a 4 =a 1 +Gd
210=n+n 2
0.3(0.8x) = 0.8x -200 y=4
13=25+3d
n2 +n-210=0 'I'i\)l
~: 'i

0.24x = 0.8x ·- 200 d=-4 (n-14}(n+15)=0 Iilli


i
Substitute x = 20 and y =4 in equation 1:
Equating factors to zero:
X= 357.14
1!:1
a3 =a,+ 2d I
It!
Lc· x = price Jojo paid for the VCR
16-t-4-t-z = 100
z=80 a 3 = 25 + 2 ( -4)
n = -15 (absurd)
n = 14 layers of logs
i·'l
1;,1
1:::
1.4x = price Rudy paid for the VCR a 3 = 17
Substitute x =20, y =4 and z = 80 in
1 .2(14x) =price Noel paid for the VCR
equation 2:
mJ a,= 4; a2 =7; a3 = 10 Ell a,= 4; a2 = 9; a3 = 14;_a4
n = 20
:= 19;

1.2(1.4x) = x + 2856
1 .68x = x + 285
X= 4200
50(16) + 30( 4)+ 80 = 1000
1000 = 1000
By inspection, d = 3 By ir:~spection, d=5 I
It checks! cL 10 'a 1 ·1· 29d S=%[2a 1 +(n-1)d]
a,0 4 1 29(3)
Therefore, x= 16 radios
,I 111 ~) 1
106 1001 Solved Problems in Engineering Mathematics (2nd Edition) by Tiong & Rojas DayS- Clock, Variation, Miscellaneous Problems & Progression 107

20 37620 = 571n- n2 • 8 = 220; a1 = 10; an= 30


n
S= [2(4)+(20-1)5] 8=2(a1 +an)
2 n2 - 571n + 37620 = 0
s =1030 n 51
(n- 76)(n- 495) = 0 S= (a 1 +a 2 ) 8=-(1+101)
2 2
n = 495 (absurd)
Ell a1 = 61; a2·= 58; a3 =55; an= 10 n = 76 days 220 = ~(10 +30)
2
8=2601

By inspection, d = - 3
Ell n = 11
ml a1 = 1; a2 = 2; a3 = 3; ... a12 = 12

Let: x = number of days, the meal can a11 = a1 +10d n


an =a1+(n-1)d
last 30=10+10d 8=-(a,+an)
10=61+(n-1)(-3) 2
S = total number of provisions d=2 12 .
10=61-3n+3 n = number of days the total S=-(1+12)
provisions can last 2
n =18
d = number of soldiers died per day ml d = 0.50; n = 12(12) =144
8=78

S = %[2a 1+ (n -1)d] 216 Note: One day is equivalent to 24 hours.


X=-
3 S=%[2a 1 +(n-1)d]
1 Therefore, total = 2(78) = 156 times
S= :[2(6.1)+(18-1)(-3)] x = 72 days 144
5436 = [2a 1 +(144 -1)(0,5)]
S=639 2 • a1 = 40; as= 76

a 3, a2, a3, a4, as, a6, a7, aa, 35


s = 6400(72)
S = 460800 meals
5436 = 144a1+5148
a 1 =2
a5 =a 1+4d
76 =40+4d
By inspection: a1 = 3: an = 35; d=9
n='7+2=9 n =72+9 a144 =a,+ 143d
Therefore:
n =81 days a,4 4 =2+143(0.5)
n a,44 = 73.50 a2 =40+9=49
S=-(a 1+aJ S = %[2a 1+(n -1)d]
2 a3 = 49+9 =58
9
s =-(3+35)
2
s = 171
460800 = ~1 [2(6400)+80d] fm a1 = 4; d = 8; n = 14
a4 = 58+9 = 67

d= -18 40 + 49 + 58 + 67 + 76
A verage = - - - -- - - -
ml
Total provision= 5700 (66) = 376,200
Therefore, 18 soldiers died everyday 8 = %[2a 1 +(n-1)d]
Average = 58 centavos
5
1
&D

8= ;[2(4)+13(8)]
Note: a1 = 5700; d = - 20 a1 = 30; as = 90 1
8= 784 h3 = 15· a3 = -
' 15
as= a1+4d
S =%[2a1+(n-1)d]
90 =30+4d
• a,=1;an=101;d=2
hs = 6; as= 6'1
d =15
a 3 =a, +2d
376200 = %[ 2(5700) + (n -1)( -20)] S=%[2a 1 +(n-1)d] an= a1 +(n-1)d
1
101=1+(n-1)(2) = a 1 + 2d
15
376200 = ~[11400- 20n + 20) S=%[2(30)+4(15)] 101=1+2n-2 1
2 a1=--2d ~Eq.1
376200 = 571 On -1 On
2
8=300 n =51 15
a 9 =a 1 + 8d
300 1
Average score = - · ". 60 -- =a, + 8d -> Eq.2
5 6
Day 6- Clock, Variation, Miscellaneous Problems& Progression 109
108 100 l Solved Problems in Engineering Mathematics (2nd Edition) by Tiong & Rojas

Substitute equation 1 in equation 2:

1
6=15-2d+8d

d=~
1 • h 1 = 3; a, =
d = a 2 -a 1
1 1
d=---
1
3 h2 = 2; a2 = 21
al 1994
al = 216
r2 = 9
r=3
Volume left after 2"d stroke

=~x-i(~x )=~x
Volume left after 3'd stroke

60
2 3 Substitute r in equation 1:
=~x-i(~x)= : 7 x
d=2
Substitute d in equation 1: 6 3
216=a,(3)
Note: By inspection, the volume left after
a,= 8 each stroke forms a GP whose common
=1~-2(6~)
a9 =a, +8d
as= al7 ratio is:
a,
7
1 ag=i+8(i) a8 =8(3) 4
a,.,.,-
30 5
ag =-
a8 =17496 r=Jl_2
2--
3 - 3
a11 =a, +10d m::l a, = x; a2 =2x + 7; a3 =10x- 7 3

a,, =~+10(~)
30 60
hg = __!_ = ~
a9 5 r = az = a3
Solving for the volume left after the 61h
stroke:
3 a1 a2
1
a,,=- 2x + 7 1Qx -7
5
hg =s3 x 2x+7
as= als

Therefore:
2
(2x + 7) = x(10x -7) as= (~x )(~J
1
h,,=-a
BD a, = 160; as = 5 4x 2 +28x+49 = 10x2 -7x
6x 2 - 35x- 49 = 0
a6 = 0.08779x

11
aa = als Thus, the total volume removed after the
1 Using the quadratic formula: 61h stroke:
h,,=T 5 = 160r
5

-
r=0.5
35 ± ~5)
2
h,, =5
5
~ 4(6)( -49) y = x- 0.08779x

Thus,
X=
2{6)
y =0.9122x
a2 = 160(0.5) = 80
35±49
ml h, =
2
1
; a1 = 2 a3 = 80(0.5) = 40
=
a4 40(0.5) = 20
X=---
12
mt r =0.9
1 as= 20(0.5) = 10 a, = 325(0.9) = 292.5
h2 = 0.2 =- ; az = 5
5
Sum = 80 + 40 + 20 + 10
x, = ~~-+49
=7
1 12 a4 = a,r3
h3 = 0.125 =-; a3 = 8 Sum= 150
8 35-49 7 a4 =292.5(0.9t

By inspection, d =3
fJ!I a4 = 216; aa = 1994
x2 =-1-2- =-6
a 4 = 213.33

a 4 =a, +3d
a 4 = a 1r3 mt Bil a,= 28; a2 = x + 2; a3 = 112
216=al ~ Eq.1 Let: x = total volume of air in the·tank
a 4 =2+3(3) ' 5 y = total volume removed from the
a6 =a,r tank after the 61h stroke
a 4 = 11 r = a2 = a3
1
1994 = al ~ Eq. -2 a1 a2
Thus h4 = - = -
, a4 11
Divide equation 2 by equation 1:
Volur. ;e left after 1"1 stroke = x _13 x = -~3 x X+2 112
28 X+2
Day 5- Clock, Variation, Miscellaneous Problems & Progression 111
110 100 l Solved Problems in Engineeruig Mathematics (2nd Edition) by Tiong & Rojas

2 Divide equation 2 by equation 1: 0.1(2"-1) S a,(r" -1)


(x+2) =112{28) 104857.5 = _ _,__-----!._
2 -1 r-1
a r 11 -1792
2
(x+2) =56 _1_=-- 2" -1 = 1048575 1( 2030 -1)
X=54 al 56
2" = 1048576
s = --'------'-
2-1
rs = -32 S = 1,073741,823 organisms
Solving for r:
=-2 Take log on both sides:

• v\
r
r = a2 = 54+2 Substitute r in equation 1: log2" = log1048576

I
a, 28
8
nlog2 = log1 048576
r=2 56= a,(-2) log1048576
n=--"---
7
:-~·.

v ,, ,, .. h.~
a1=- log2
a,o = a,rg
,>··\ .-· · .
a10 = 28(2)
a, 0 = 14336
9

1111
8

a, =2; r = 2; n = 15 Bill
n = 20 wealthy men

a,= 10; r = 10; n =6


5o' ft

-
j ,f .Iv ........ 100/3 ft .

v·······v···\..-·-.,_.... -·

Ell a,= 2; a2 = 4; a3 = 8; n = 10
a,(r"
S=-'--~
r-1
-1) a,(r"-1)
S=--'----'-
r-1 a,= so[3.J3 = 100.
,n=oo
By inspection, r = 2 15 3
2(2 -1) . ~0(10 6
-1)
s = --'-2---1--'- So~---''---
10 -1
S = a1 (r" --1) s = 65534 ancestors S=~
S=1111110 r-1
r-1
2(2 -1) 10
s =--'-----.-:..
2-1.
a a,= 1; r=2.
Total cost= 4{1,111,110) = P 4,444,440
S=-3-
1--
100

2
s =2046 I!IIIIW 1 1·a- 1
a1 (r" -1) laiiil a,= 3 ; a2 = 9 , 3 - 27 3


s =--'---'- S= 100
·r-1
a,= 9; r = _ 3_
1(2"-1)
By inspection, r = 31
3 1000000=--
. 2-1 Let: D = total distance traveled by ball
S=~ D =50+ 2S
as= a,r4 2" - 1 = 1000000 1-r D =50+ 2(100) = 250 feet

as =9( -iJ 2" = 1000001


Take logarithm on both sides:
S=-3.
1- _!
-
1

tm
as =9G~) log2" = log1000001 1
3 0.133133133133 = 0.133 + 0.000133 +
0.000000133 + ......
S=-
16 nlog2 = log1000001 2
as=- Note: The numbers being added are in a
9 log1000001
n=--=---- B:l a1 =1;r=2
GP and 0.133133133133 ... is the sum of
an infinite GP.
~~ a7 =56; a12 =-1792
log2
n=19.93. 60 Solving for the common ratio: ·
n=-=30
a7 = a1r 6 Say n = 20 baskets 20 0.000133 1
56=al ----+Eq.1 EJI a, = 0.1; r =2
r=
0.133
=--
1000

a,2 =a/, S = a1(r"-1) S=~


1-r
-1792=<>r 11 ,r:-q.2 r -1
112. IOOl.Solved Problems in Engineering Mathematics (2"d Edition) by Tiong & Rojas_
Day 5- Clock, Variation, Miscellaneous Problems & Progression 11~

5=0.133

S=_9~33
1-
1- ___
1000

s9?r
2 = _1__

2=2(1-r)
1-r
1

1785
9999
=0.178517851...
'"'"'

The above number is a-repeating decimal


1000 1
4 =--- number.
s =: 133 1-- r
999 3
r=- The four repeated digits are 1, 7,8 & 5
4

~~~ =1984
a1 =6; a2=-2

a -2
ED 1987 = 496.7G;
4
496(4)

2
r=--=- Therefore, 19841h digit= 5
a1 6 19861h digit= 7
1
r=--
3
Layer I Layer II
£o
Layer Ill Layer IV
19851h digit =
19871h digit=
1
8.

a1
S=·---.
1-r Total spheres= 10 + 6 + 3 + 1
6
= 20 spheres
S=-(1)
-3
9
1-
Ell=
Let: x the 6th term of the number series
S=- ! j
2
55
'--y-~~
40 28 19 13 X
in I
B a1 = 1, a2 = - i, a3 =
2~ -15 -12 -9 -6 -3
II
i I
1 ~ _,·.~ I
By inspection, r =- 5 ; ~,·

1'.~:'I'
X=13+(-3)

:1 1 1 1!1!~
X =10
a1
S=-----
1-r
1
Ell ,r,l/,, 11
I
Let: y = the H
S=~-(--1~
6th term of the number series
,,,·_1

5; y, 1/6 1/24 y !I
S=~ \.'.- v··- .~".. .. y· } . . ·y. k y ·""-- ·y·--) I~,
6
X1 x1/2 x113 xY. X 1/5
'Iii
EDI S=2;a1= i y= 2~(i)
riJ'ii!
. ~'I !
.>!'
1 Ill ~;l
S=-~ y = 120 l,ilr,l
1-r
1
s =_1__
~~
1-r .JI':rl

l
',!
1
-
116. 1001 ·solved Problems ~n Engineering Mathematics (2nd Edition) by Tiong & Rojas

Topics
[J
"'" ,-,,.-~.~""'"-"'-'"-""'"'''" ·~u·m"'""""'"""'"""'"" '""""" ""'•'''""'"'"m""

Venn Diagram
Mon Combinatorics
Fundamental Principle of Counting
IJ
Tue
Permutation
Inversion
Cyclic Permutation
1 IJ
~-
....
Assortment
Theory Vl/ed Combination

!_j
Problems
DThu
Probability Theory
Mutually Exclusive Events
Independent Events

[]
Solutions
DFri
Binomial Distribution
Odds For and Odds Against
Mathematical Expectation

DNotes
~Sat

What is a Venn Diagram?

Venn diagram is a rectangle (the


universal set) that includes circles
depicting the subsets. This diagram,
named after the English logician John
Venn (1834- 1923) in 1880, is a way of
displaying the events or an experiment.
A Venn diagram can be employed for any
lhe Venn diagram below shows two number of subsets, but more tnan three
events A and B and their intersection. defeat the purpose of gaining increased
clarity.

Below is an example of a typical problem


that is given in the engineering licensure
examtnations.

Problem:
A survey was conducted in a graduating
ECE students in a certain university on
lilt• following Venn diagram shows two which board subject they like best. The
lltttltt.tily exclusive events A and B result is tabulated as follows:
118 _I 00 I Solved Problems in Engineering Mathematics (2nd Edition) by Tiong & Rojas ., Day 6 -V§!!l_llJ:)iagram, Permutation, Combination & Probability 119
. /f Element E precedes D, Band A, therefore 111
11 11
Mathematics 55 Mathematically, has 3 inversions. P = (11-11)12121211i111i 111i
Math & Electronics 32

II Element D precedes B and A, therefore 11p11 = 4989600


Electronics 50
Math & Communications 28 N=m-n has 2 inversions
Communications 51 What is an Assortment?
Electronics & Comm 25 Element B precedes A, therefore has 1
All three subjects 10 What is Permutation? ''l'/1 inversion. Assortment refers to a group of objects
't selected from a larger group in such a way
How many were there in the graduating Permutation is defined as an ordered j The total inversion is 2 + 3 + 2 + 1 =8. So that an object can be used more than
class? arrangement of a finite number of '.;.); permutation (C, E, D, B, A) is an even once.
elements, either all of the available n permutation
Solution: Use Venn diagram: elements or of a part of them. The The number of assortments is expressed·
mathematically as:
permutations that contains exactly the
same elements but not in the same order What is a Cyclic Permutation? Assortments = (No. of choices for 1st
are regarded as different. position) x (no. of choices for the 2nd
Cyclic permutation is the shifting of an position) x (no. of choices for the 3'd
The number of permutations of n different entire order of elements one or more steps position) x · · · x (no. of choices for last
things taken r at a time is forward or backward -the first element position)
taking the position of \he last, or vice
versa, without changing the order of the What is a Combination?
n! elements in the sequence.
nPr=(n-r)! Combination is an arrangement of the
selection of objects regardless of the
If taken all at once, npn =(n-1)! order.
Electronics Communications

npn = n! What is the Permutation with Identical The number of combinations of n different
N = 5 + 22 + 10 + 18 + 3 + 15 + 8 Elements? things taken r at a time is
N = 81 students The number of permutations is reduced
What is an Inversion?
n!
when a collection contains identical
What is Combinatorics? elements. The number of permutation n ncr= (n-r)!r!
If two elemen.ts in a permutation of distinct
elements are in reverse order relative to objects, p of one type, q of another, s of
Combinatorics is the branch of
their normal or natural order, they another, etc., is given by:
mathematics that concerns with the If taken all at once,
constitute an inversion.
selection of objects called elements. ll n!
p -
A permutation is said to be even if it n r-(n-r)!_p!q!s! ncr =1
Combinatorics traces its history back to
contains an even number of inversions; it
the ancient times when it was closely ·
is odd if the number of inversions is odd.
associated with number mysticism. This What is the relation between
branch of mathematics led to the creation I'~ Problem: Permutation and Combination?
The number of transpositions that are
and development of probability theory. Find the largest number of permutations of
required to return a sequence of elements
the letters in MATHEMATICS? The relation between permutation of n
to their natural order is even or odd
according to the number of inversions in thing taken r at a time to the combination
What is the "Fundamental Principle of Solution: of n things taken r at a time is expressed
the arrangement.
Counting? mathematically as follows:
The word MATHEMATICS has 11 letters.
Example:
The Fundamental Principle of Counting The letter M occurs 2 times, letter A occurs
In the permutation (C, E, D, B, A)
states that: 2 times, letter T occurs 2 times while the
Element C precedes B and A, therefore rest occurs once. nCr=~
r!
"If a thing can be done in m different ways
has 2 inversions.
and another thing can be done in n
different ways, then the two things can be
done in m times n different ways."
120 · 100 l Solved Problems in Engineering Mathematics (2nd Edition) by Tiong & Rojas Day 6- Venn Diaqram, Permutation, Combination & Probability 121

What is Probability? E. Frequency of the outcome - refers to The probability for a mutually exclusive may mean or is synonym<;>us to the word
the number of times a certain outcome event (E or F) is: "chance".
Probability is simple defined as the will occur. For example, if you roll a die
numerical assessment of likelihood. It is 50 times, you are conducting 50 trials If a coin is tossed, the probability of
. expressed as a number between 0 and 1, of the same experiment. The number Pe or F = Pe + PF coming up head is 50% and the probability
where 0 means an event is impossible of times a "6" comes up is known as of coming up tail is also 50%. This is an
while 1 means it is surely or certainly to the frequency of the outcome. example of 1-to-1 odds or even odds. For
occur or happen. What are independent events? a game with even odds, a one peso bet is
The relative frequency of the paid exactly one peso upon winning. This
The total probability of favorable and outcome is the ratio of the frequency An event is independent if the outcome of is known as the "true odds". However,
unfavorable outcomes is 1. of the outcome to the number of trials. one trial has no effect on the outcome of there are instances that 90 centavos is
Mathematically, any other trial. paid instead to a one peso bet, which
One of the earliest mathematical studies would give it an advantage on each game.
on probability was "On Casting the Die" RF =· no. of occurrences A good example is when a coin is tossed, This refers to the "payoff odds". The
written by the 16th century Italian no. oftrials the outcome has nothing to do with what payoff odds are not always the same as
mathematician and physician Gerolamo happened on the previous toss and will not the true odds.

I Cardano (1501 -1576). Cardano defined


probability as "the number of favorable
outcomes divided by the number of
possible outcomes". Cardano was
What is the Principle of Probabiitv?
affect the next one.

If two events are independent, the


probability that the two events occur is the
The odds of an event occurs is the ratio of
the probability that it will occur to the
probability that it will not occur.
If an experiment has a set of distinct
regarded as "the father of the theory of outcomes, each of which is equally likely product of their individual prob.abilities.
probability". to happen, then the probability of an event odds for E = prob. that E occurs
f E, is the ratio of the number of outcomes The probability of independent events (E prob. that E does not occur
The following are some of the important to the total number of possible outcomes. and F) is:
terms used in probability: ·
P.
odds for event E =-e-
A. Experiment - is a controlled study P. __ no. of outcomes Pe & F = Pe x PF . 1-Pe
whose outcome is uncertain but not e - total outcomes
entirely unknown. Problema
What is a Binomial Distribution?
What is the odds for throwing a total of 5
B. Trial -a recorded result of an . The probability that it is not E is: and 10 in rolling two dice?
experiment. If there are two possible outcomes of an
event and the-possibilities of the outcome
Pnot E = '1.- Pe Solution:
C. Outcome - is one of the possible are independent and constant, the
results from an experiment trial. It also distribution of probabilities is called
First, solve the probabilitY of throwing a
refers to the basic unit of possible binomial distribution.
total.of 5: For a total of 5, it must be (1 and
occurrences. For example, in. tossing a What are mutually exclusive events? 4), (4 and 1). (2 and 3) and (3 and 2).
coin, the outcomes are heads and tails. Binomial or repeated trial probability:

Two or more events are said to be p =ncr pr qn-r 4 1


D. Event- is some combination of p5 =-=-
mutually exclusive if no two of them can 36 g.
possible outcomes in one experiment possibly happen in the same trial.
trial. In picking a card from an ordinary
where: p = probability of success Then, solve the probab' ity ofthrowing a
deck of cards, the outcomes are the 52 For example, in picking a card from a deck q = probability of failure = 1 - p total of 10: For a total of 10, it must be (4
cards themselves. Examples of events of cards, it is not possible to pick a card n = number of trials and 6), (6 and 4), and ( ' and 5).
are: that is a diamond and a heart at the same r = number of successful trials
time. 3· 1
"Pick a 5" (one of the four cards
What is an Odd? P1o = 36 = 12
numbered 5) If two or more events are mutually
"Pick a heart" (one of the 13 cards in exclusive, the probability that either one or The term "odd" is an ambiguous word
the other will occur is the sum of their
1 1 7
the suit of heart)
probabilities.
that may refer to the probability that an Psopo = g + 12--36
-
event occurs, or it can be used to indicate
"Pick a face card (one of the 12 cards
the payoff on a winning bet. It sometimes
showing a face i.e. king, queen or jack)
122. 1001 Solved Problems in Engineering Mathematics (2"d Edition) by Tiong & Rojas Day 6 - Venn Diagram, Permutation, Combination & Probability 123

The odds for. throwing a total of 5 or 10 is: If the expectation is positive, the player will deck of cards contains 52 cards divided • Poker Hand with corresponding
win in the long run and lose if the into 4 suits; spades, hearts, diamonds and probability:
7 expectation is negative. clubs. Each suit contains 13 cards labeled
Ace, 2, 3, 4, 5, 6, 7, 8, 9, 10, jack, queen Hand Probability
odd=~ Problem: and king. Each of the four kings in a deck Royal flush 0.00000154
1- .2_ In tossing a single coin, P 100 bet is represents a great leader from history; Straight flush 0.00001385
36 placed on heads i.e. if heads comes up, Charlemagne (hearts), Alexander the Four of a kind · 0.0002401
7 the player wins P 100 and if tails comes Great (clubs), Julius Ceasar (diamonds),
odd= Full house 0.0014406
29 up, the player loses P 100. What is the and King David (spades). Flush 0.0019654
expectation? StraiQh! 0.0039246
This is expressed as 7 -to-29 odds. The playing cards were used also to
Three of a kind 0.0211285
Tails describe a calendar year. The 52 cards
Two pair 0,0475390
What is another formula for "odd for"? y, represent the 52 weeks in a year. The 4
Pair 0.4225690
-100 suits represent the four seasons of the
year and the 12 face cards (4 kings, 4 None of the above 0.5011774
If the·"odds for" an event are given as "a"
and "b", then the probability of that event queens and 4 jacks) represent the 12 Total 1.00000000
would be: Expectation= (~}1 oo) + (~ )c-1 oo) months ot' the year.
Poker Hand with corresponding expected
Expectation = 0 frequency:
odds for a·n event =-a-
a+b Hand
Problem: Expected frequency
In Casino Filipino, a roulette offers winning Royal flush 1 in 649740 hands
Problem:
a bet on a single number pays 35 to 1. The Straight flush 1 in 72192 hands
A player in a certain card game lays odds
actual probability is 1 in 38. What is the Four of a kind 1 in 4165 hands
of 3 to 5 for him to win. What would be the
expectation? Full house 1 in 694 hands
payoff for a bet of 3 pesos?
Flush 1 in 509 hands
Solution: Solution: Straight 1 in 255 hands
Poker Harid with corresponding number of Three of a kind 1 in 47.33 hands
The 35 to 1 is a house (payoff) odds of ways: Two pair 1 in 21 hands
3 -~
Odds for = 3 + 5 - 8 Casino Filipino. Pair 1 in 2.37 hands
Hand No. of ways None of the above 1 in 2 hands
Expectation can be obtained by taking the Royal flush 4
Therefore, it is a 3-to-8 odd. A bet of 3 Stra_ight flush · ·
difference betWeen the product of the first 36 What are Probabilities with Dices?
pesos will have a payoff of 8 pesos.
number in the house odds and probability ~our of a kind · 624
of success, and the product of the second Full house 3744 Dice were invented for sole purpose of
number of the house odds and the Flush .5108 gambling. As a matter of fact, dice gaines
What is "odd against"?
probability of failure. Straight 10200 have so little intrinsic interest that in the
The "odds against" an event are the . Three of a kind 54912 absence of wagering they would hardly be
reciprocal of the "odds for" the event.
The odds of 5 to 7 for an event is
translated into odds of 7to 5 against it.
Expectation= 35(_!__) -1(

Expectat1on
. =- 2
38

38
37
)
313 .
Two pair
Pair
None of the above
Total
123552
1098240
1302540
2598960
worth nl::>llinn

-
''' '',.
Expectation = -0.0526
What is a Mathematical Expectation?

A mathematical expectation is the Thus, a player expects to lose to Casino


average amount a player can expect to Filipino an average of 5.26 centavos for
win or lose on one play in any game of every peso bet.
chance. · Dice were first used by th~ Chinese. The
What are "Card Games"? sum of the opposite faces of~ die is
always equal to 7. And the sum of all the
It can be found by multiplying the
Card games are games which are played vertical faces of a die, no matter how it
probability of each possible outcomes by
using a standard deck of cards A standard rolls is always equal to 14. When two dice
its payoff, and then adding these results.
124 IO<h Solved Problems in Engineering Mathematics (2"d Edition) by Tiong & Rojas

are rolled and both will face up 1, this is Proceed to the next page for your 6th test
known as snake's eyes. Detach and use the answer sheet provided
at the last part of this book. Use penCil
number 2 in shading your answer.

G8 ~.8 [ZJ8 GOOD LUCK I Topics


Gr:J~r:J [ZJ[J
G rsJ ~ rsJ 1.··1 LSJ -m:ribia: D tvlon
Venn Diagram
Combinatorics
Did you know that .. the number 1 Fundamental Principle of Counting
l:l [i"7l
L:.J l!_!J
rel [i"7l
l!._j l!_!J
r:;][i"7]
l!.:J l!_!J
followed by 100 zeros is called "google"
and the term "google" was coined in the
1930s by the aine-year old nephew of the
D Tue
Permutation
Inversion
l:l r-;;1 rel ~ r:eJ fe.eJ Cyclic Permutation
D D
L:.J l!:.!J l!._j l!:.!J l!.:J l!:.!J American mathematician Edward Kasner
when he was asked to come up with a Assortment
•• .•
G [] ~
• []•
• •• 1.··1 [] ·name for a very large number !
Theory VVed Combination

1:·:1 G 1::1 G
~uote:
"Statistics - the mathematical theory of ~ D Probability Theory
Mutually Exclusive Events
~8 Problems Thu Independent Events
ignorance.
[i"7lr.l
l!_!J L!J
r.=--11-l
LUI L!J
I!!]
LUJ
lei
u - Morris Kline D D Binomial Distribution
Odds For and Odds Against


Solutions Fri Mathematical Expectation
l!!lr-::1
1: :11··.1 1:·:11··.1 LUJ ~
(QJ
fe"il [i"7l ~- reel I!!] reel
Notes Sat
l!_!J l!_!J LUl l!_!j LUJ L!_..!J

1: :11:·:1 1:·:11:·:1 1: il ~
~ []
•• • ~-.
••• []• •• •• []•
•• •• []• •• ••
:15&: EE Board October 199~
In a class of 40 students, 27 like Calculus
and 25 like Chemistry. How many like
.2$8: GE Board February 1994
·' A survey of 100 persons revealed that 72
of them had eaten at restaurant P and
both Calculus and Chemistry? that 52 of them had eaten at restaurant Q.
Which of the following.could not be the
The following are the two-dice A. 10 number of persons in the surveyed group
probabilities: B. 11 who ·had eaten at both P and Q?
C. 12
Two-dice sum Probability D. 13 A. 20
2 1/36 B. 22
3 2/36 %57: ECE Board November 1998 C. 24
4 3/36 A c!ub of 40 executives, 33 like to smoke D. 26
5 4/36 Marlboro and 20 like to smoke Philip
6 5/36 Morris. How many like both? 2591 ECE Board November 1992
7 6/36 'The probability for the ECE board
8 5/36 A. 10 examinees from a certain school to pass
9 4/36 B 11 the subject Mathematics is 3n and for the
10 3/36 c 12 subject Communications is 5/7. If none of
11 2/36 n 13 the examinees fails both subject and
12 1/36 there are 4 examinees who pass both
subject~. find the number of examinees
from that school who took the
examinations.
126 H>O I Solved Problems in Engineering Mathematics (2nd Edition) by Tiong & Rojas Day 6- Venn Diagram, Permutation, Combination & Probability 127

A. 20 26~: Five different mathematics books, 4 2&8: EE Board April199& 27~: EE Board June 1990
B. 25 different electronics books and 2 different In how many ways can 6 distinct books EE Board April199~
c. 30 communications books are to be placed be arranged in a bookshelf? CHE Board May 1994
D. 28 in a shelf with the books of the same In how many ways can 4 boys and 4 girls
subject together. Find the number of ways A. 720 be seated alternately in a row of 8 seats?
/ B. 120
,260: EE Board March 1998 in which the books can be placed.
In a commercial sur.iey involving 1000 C. 360 A. 1152
persons ori brand preference, 120 were A. 292 . D. 180 B. 2304
found to prefer brand x only, 200 prefer B. 5760 c.· ~
brandy only, 150 prefer•brand z only, 370 C. 34560 2&9: EE Board April1997 D. 2204
prefer either brand x or y but not z, 450 D. 12870 What is the number of permutations of the
prefer brand y or z but not xand 370 letters in the word BANANA? 274: EE Board October 1997
prefer either brand z or x but' not y. How 264: The number of ways can 3 nurses There are four balls of four different
many persons have no brand preference, and 4 engineers be seated on a bench A. 36 colors. Two balls are taken at a time and
satisfied with any of the three brands? with the nurses seated together is B. 60 arranged in a definite order. For example,
C. 52 if a white and a red balls are taken, one
A. 280 A. 144 D. 42 definite arrangement is white first, red
B. 230 B. 258 second, and another arrangement is red
c. 180 C. 720 270: ME Board April1994 first, white second. How many such
D. 130 D. 450 A PSME unit has 10 ME's, 8 PME's and 6 arrangements are possible?
CPM's. If a committee of 3 members, one
261: EE Board April i:997 265: ECE Board November 1998 from each group is to be formed,. how A. ~
A toothpaste firm claims that fn a survey If 15 peop!e won prizes in the state lottery many such committees can be formed? B. 6
of 54 people, they were using either (assuming that there are no ties), how C. 12
Colgate, Hapee or Close-up brand. The many ways can these 15 people win first, A 2,024 D. ~
following statistics were found: 6 people second, third, fourth and fifth prizes? B. 12,144
used all three brands, 5 used only Hapee C. 480 275: How many different ways can 5
and Close-up, 18 used Hapee or Close- A. 4,845 D. 360 boys and 5 girls form a circle with boys ,
up, 2 used Hapee, 2 used only Hapee B. 116,260 and girls alternate?
and Colgate, 1 used Close-up and C. 360,360 271: ME Board October 1992
Colgate, and 20 used only Colgate. Is the D. "3,003 In how many ways can a PSME Chapter A 28,800
survey worth paying for? with 15 directors choose a President, a B. 2,880
266: CE Board November 1996 Vice President, a Secretary, a Treasurer C. 5,600
A. Neither yes nor no How many 4 digit numbers can be formed and an Auditor, if no member can hold D. 14,400
B. Yes without repeating any digit from the more than one position?
C. No following digits: 1, 2, 3, 4 and 6? 276: EE Board October 1997
D. Either yes or no A 360,360 There are four balls of different colors.
A. 120 B. 32,760 Two balls at a time are taken and
262: How many four-letter words B. 130 C. 3,003 arranged any way. How many such
beginning and ending with a vowel c. 140 D. 3,603,600 combinations are possible?
without any letter repeated can be formed D. 150
from the word "personnel"? 272: EE Board October 1997 A 36
267: EE Board June 1990 Four different colored flags can be hung B. 3
A. 40 How many permutations are there ifthe in a row to make coded signal. How many C. 6
B. 480 letters PNRCSE are taken six at a time? signals can be made if a signal consists D. 12
c. 20 t of the display of one or more flags?
D. 312 A. 1440 ~ 277: EE Board March 1998
B. 480 I A. 64 How many 6-number combinations can
C. 720 ! B.
c.
66
68
be generated from the numbers from 1 to
D. 360 42 inclusive, without repetition and with
D. 62 no regards to the order of the numbers?

A 850,668

J
1Z8 1001 Solved Problems in Engineering Mathematics (2nd Edition) by Tiong & Rojas Dart- VexmDiagram, Permutation, Combination & Probability 1Z9

B. 5,245, 786 c. 1435 A. 1/36 %93: EE Board Apll'ill996


c. 188,848,296 D. 1365 B. 1/9 The probability of getting at !east 2 heads
D. 31,474,716 C. 1/18 when a coin is tossed four times is,
Z8~: ECE Board April1998 D. 1120
Z78: Find the total number of A semiconductor company will hire 7 men A. 11/16
combinations of three letters, J, R, T and 4 women. In how many ways can the Z88: Roll two dice once. What is the B. 13/16
taken 1, 2, 3 at a time. company choose from 9 men and 6 probability that the sum is 7? c. 114
women who qualified for the position? D. 3/8
A. 7 A. 1/6
B. 8 A. 680 B. 1/8 ::t94: A fair coin is tossed three ti.mes.
C. 9 B. 540 C. 1/4 What is the probability of gel!ing either 3
D. 10 C. 480 D. 1/7 heads or 3 tail?
D. 840
Z79: ME Board October 1997 289: In a throw of two dice, the A. 118
In how many ways can you invite one or Z84a ECE Board April1994 probability of obtaining a total of 10 or 12 B. 3/8
more of your five friends in a party? There are 13 teams in a tournament. is c. 1/4
· Each team is to play with each other only D. 112
A. 15 once. What is the minimum number of A. 1/6
B. 31 days can they all play without any team B. 1/9 295: ECE Board Mare»• 19'96
c. 36 playing more than one game in any day? c. 1/12 The probability of getting a credit in an
D. 25 D. 1/18 examination is 113. If three students are
A. 11 selected at random, what is the ·
zsoa CHE November 1996 B. 12 290: Determine. the probability of drawing probability that at least one of them got a
In how many ways can a committee of C. 13 eithel" a king or a diamond in a single credit?
three consisting of two chemical D. 14 draw from a pack of 52 playing cards.
engineers and one mechanical engineer A. 19/27
can be formed from four chemical Z85a EE Board October 1996 A. 2/13 B. 8/27
j;!n'gineers and three mechanical There are five main roads between the B. 3/13 C. 2/3
engineers? cities A and B, and four between B and C. c. 4/13 D. 1/3
In how many ways can a person di'ive D. 1/13
A. 18 from·A to C and return, going through B :Z9fl: There are 3 questions in a test. For
B. 64 on both trips without driving on the same :1911 A card is drawn from a deck of 52 each question 1 point is awarded for a
C. 32 road twice? playing cards. Find the probability of correct answer and none for a wrong
D. None of these drawing a king or a red card. answer. If the probability that Janine
A. 260 correctly answers a question in the test is
Z81a EE Board Aprii199S B. 240 A 0.5835 2/3, determine the probability that she
In Mathematics examination, a student c. 120 B. 0.5385 _gets zero in the test.
may select 7 problems from a set of 10 D. 160 C. 0.3585
problems. In how many ways can he 0. 0.8535 A. 8/27
make his choice? Z86: EE Board Aprll1991 B. 4/9
There are 50 tickets in a lottery in which 1:91:1 CE Board November 1998 C. 1/30
A. 120 there is a first and second prize. What is A coin is tossed 3 times. What is the D. 1/27
B. 530 the probability of a man drawing a prize if probability of getting 3 tails up?
c. 720 he owns 5 ti't:kets? · :Z97Z EE Board April :199:1
D. 320 A 1ffl In the ECE Board Examinations, the
A. 50% B. 1/16 probability t.hat an examinee will pass
zsza EE Board April 1997 B. 25% C. 1M each subject is 0.8. What is the
How many committees can be formed by c. 20% o. m probability that an examinee will pass at
choosing 4 men from an organization of a D. 40% least two subjects out of the three board
membership of 15 men? subje~ts?
Z87a Roll a pair of dice. What is the
A. 1390 probability that the sum of two numbers is A. 70.9%
B. 1240 11? B. 80.9%
130 . rOO! Solved Problems in Engineering Mathematics (2nct Edition) by Tiong & Rojas

C. 85.9% 302: ME Board April 1.99&


D. 89.6% An urn contains 4 black balls and 6 white
balls. What is the probability of getting 1
298: In a multiple choice test, each black and 1 white ball in two consecutive
question is to be answered by selecting 1 draws from the urn?
out of 5 choices, of which only 1 is right If
there are ·1 0 questions in a test, what is A. 0.24 Topics
the probability of getting 6 right of pure B. 0.27
guesswork? c 0.53 Venn Diagram
D. 0.04 tv1on Combinatorics
A. 10% Fundamental Principle of Counting

d
B.
C.
D.
6%
0.44%
0.55 %.
303: EE Board October 1.990
From a bag containing 4 black balls and 5
white balls, two balls are drawn one at a
0 I
Tue
Permutation
, Inversion
I
:&99: ME Board April1994
From a box containing 6 red balls, 8 white
balls and 10 blue balls, one ball is drawn
time. Find the probability that one ball is
white and one ball is black. Assume that
the first ball is returned before the second
ball is drawn.
u
Theory
0
\JVed
Cyclic Permutation
j Assortment
1 Combination
at random. Determine the probability that
it is red or white. A.
B.
16/81
25/81
0
Problems
0 I
Thu
1 Probability Theory
Mutually Exclusive Events
Independent Events
A.
B.
C.
1/3
7/12
5/12
C.
D.
20/18
40/81
Solutions
0Fri
Binomial Distribution
Odds For and Odds Against
Mathematical Expectation
D. 1/4 304: EE Board October 1.997

300: EE BoardOctober 1990


From a bag containing 4 black balls and 5
A group of 3 people enter a theater after
the lights had dimmed. They are shown to
the correct group of 3 seats by the usher.
0Notes
[QJ
Sat
white balls, two balls are drawn one at a Each person· holds a number stub. What
time. Find the probability that both balls is the probability that each is in the
are white. Assume that the first ball is correCt seat accordi•1g to the numbers on
returned before the second ball is drawn. seat and stub? ANSWER KEY RATING

A. 25/81 A. 1/6 256.C 269. B 282. D 295.A c:J 43-50 Topnotcher


B.
C.
16/81
5/18
B.
C.
1/4
1/2
257. D
258.c
270.c
271. A
283. B
284. c
296. D
297. D c:J 33-42 Passer
D. 40/81 D. 1/8 259. D
260. A
272.A
273.c
285. B
286.C
298. D
299. B
c:J 25-32 Conditional

301: CE Board May 1996


A bag contains 3 white and 5 black balls.
305: From 20 tickets marked with the
first 20 numerals, one is drawn at
261.c
262.A
274.c
275. B
287.
288.A
c 300.A
301. D
.,
0 0-25 Failed

If two balls are drawn in succession random. What is the chance that it will be 263.c 276.C 289. B 302.c If FAILED, repeat the test.
without replacement, what is the a multiple of 3 or of 7? 264.C 277. B 290. c 303. D
probability that both balls are black?
265.C 278.A 291. B 304.A
266.A 279. B 292.A 305 D
A. 1/2
267.C 280.A 293. D
A.· 5/16 B. 8/15
B. 5/28 c. 3/10
268. A 281. A 294. c
c. 5/32 D. 2/5
D. 5/14
,jil

132· lQOl Solved Problems in Engineering Mathematics (2"d Edition) by Tiong & Rojas


Let: x = number of students who like
both subjects

Calculus Chemistry
ml
Let: x = number of examinees who took
the examination

Math Communication
ED
Given word:
Day 6- Venn Diagram, Permutation, Combination &Probability 133

PERSONNEL

Number of vowels= 2 (E & 0)


N = (5!}(4!)(2!}(number of patterns)
N = (5!)(4!)(2!)(6)
N = 34,560 ways

Number of constants= 5 (P, R, S, N & L)


Ill
I 2 I s I 4 I 1 I n n n E. E ;E E.·
~ J E n n n E E E
v E E n n n E E
Four-letter word E E E n n n E
(27 ~ x)+ x +(25- x) = 40 Note: E E E E n n n J
x =Hx-4)*4+(%x-4) Any of the two vowels can be filled in the
27 +25-x = 40 J
first letter. ·
X= 12 8
·x=-x-4 Five patterns
7 Any of the five consonants can be filled in
X=28 the second letter. Number of ways the 3 nurses can be
Bil
Let: x = number of executives who
smoke both brand of cigarettes

Marlboro Philip Morris



- Let: x = number of persons who have no
brand preference

Brand X Brand Y
Any of the remaining four consonants can
be filled in the third letter.

The remaining bowel can be filled in the


fourth letter.
arranged = 3!

Number of ways the 4 engineers can be


arranged = 4!

Let: N = total number of ways

Let: N = number of words N = (3!)(4!)(number of patterns)


N = (3!)(4!)(5)
N = 2(5)(4)(1)
N =720 ways
N= 40 ways

(33- X)+ X +(20- X)= 40


33+20-x = 40
X= 13
Brand Z
Ell
MATH
MATH
ELEC
COMM
COMM
ELEC

Ell 1000 =X+ 120+ 50+ 200 + 100 ELEC
ELEC
MATH
COMM
COMM
MATH N;, 15(14)(13)(12)(11)
+ 150+100
Let: x = number of persons who have COMM MATH ELEC
eaten in both restaurants X=280 N = 360,360 ways
COMM ELEC MATH
p -~
Q
fll
\_
Six patterns
J
FD
Note: Number of ways the books in MATH can
be arranged = 5!
The survey is not worth paying for. One
error is that according to the said survey, Number of ways the books in ELEC can N = 5(4)(3)(2)
(72-x)+x+(52-x)=100 there are 6 people who used 9.!JJ.h_ree l1c arranged= 4! N= 120 ways
brands but only ~~ used !he brands
72 + 52 - X = 100 Hapee and Close-up. Number of ways the books in COMM can
X =24 l11• arranned = 2!

I '·I N total nwnber of wctys

I
j
:I

l34 100 l ·Solved Problems in Engineering Mathematics (2"ct Edition) by Tiong & Rojas Day 6- Venn Diagram, Permutation, Combination & Probability 135

N=(4!)(4!)
N = 576 ways !,
ml ml
Total number of games
N = npn = n! = 6! N = 2" -1 =23 - 1
131
N=720
ml N = 7 ways N= 13 C 2 = .
(13- 2)!2!
=78

N=nPn n!=6!
4!
N= 4p2 = ( 4 -2)!
ml Number of games that can be played per
day:
N = 12 ways
N = 2" - 1 =2 5
- 1


N= 720 ways
N = 31 ways 13
N = - = 6.5 ~ 6 games/day
G ml 2

Number of days needed to complete the


Given word: BANANA
Number of A's = 3 GoBO?
0 08oG Number of ways of selecting a chemical
engineer:
tournament:
Number of N's = 2
41 78
B0 08 N - C = =6 N=-=13 days
1- 4 2 (4- 2)!21
N=_I1J__=~ 6
p!q! 3!2! GO e OG
N= 60 ways
8 '~
Number of ways of selecting a
mechanical engineer:
ED
\'~~ N = C =
31
.
3 1 (3-1)!1!
=3
Number of ways to travel from A to B = 5

N = 10(8)(6)
This seat is permanently I 2
Number of ways to travel from 8 to C = 4
. occupied by one of the children! J

N=480 ways "'--..__ .~ N = 6(3) = 18 ways Number of ways to travel from C to 8


without using the same road to travel from
Number of ways the boys can be
8 to C =3
arranged = (5-1)! =4! Ell 10!
Cit! ~~ I. ~;c I Tr7;s I ~f] Number of ways the girls can be
arranged = 5!
N= 10c1 = ( 10 _7 )!7!

N=120 ways
Number of ways to travel from B to A
without using the same road to travel from
A to B =4
N = 15(14)(13)(12)(11) N=(4!)(5!)
N = 5(4)(3)(4) = 240 ways
ml

N= 360,360 N = 2880 ways

ml N= 15 C 4 =~
(15-4)!4!
4! N = 1,365 committees P = probability of winning a prize in the
N = 4p1 + 4P2 + 4P3 + 4P4 N= 4c2 = (4 -2)12! lo_ttery

N=--4_!_+ __4_!_+ __4_!_+ __4_!_


(4-1)! (4-2)! (4~3)! (4-4)!
N =6 ways ml 2
P=-=-
1
50 25
Number of ways of hiring men:
N = 64 signals
Ell N1 = C7 =
g
91
. =36
(9- 7)!7!
P = probability for the man to win
= number of tickets he bought x.
- c-~
N-42 6-(42-6)!6!
probability of winning a prize

Num_ber of ways the 4 boys can be N = 5,245,786 ways


Number of ways of hiring women:
6!
P= 5(_!_)
.25
arranged= 4! N.,"' .C 4 =------=15
• (> (6-4)14! p =0.20
Number of ways the 4 girls can be
arranged = 4! N :16( 15) = 540 ways

j !
136 1001 Solved Problem~ngineering Mathematics (2nd Edition) by Tiong & Rojas Day 6- Venn Diagram, Permutation, Combination & Probability 137


Total number of trials =36 2. I
3
I
2 .3

--+-----1
4 5 6


Let:
p = probability of getting a head in a
single tl)row of a fair coin
Ill
P=n CrPr qn-r

·r=a
. 1
where:p=2";q= 1 :n= 3:r= 3
2 3 4 5 6
r---- 2
4
5
L- L-+--J---1! I q = probability of getting a tail in a single
throw of a fair coin 3 0 3
p3H=3C3 (2 '(2
1) 1) (1)2
2
f-· 6 =(1)
3 P=j, CrPr qn-r
4 I 1
P = number of successful trials
where: p=
1
: 1
q= : n=3; r=3
p
3H =-
8
5
- total number trials 2 2
6 4 1
3c3(~J(~J =(1{iJ
. P=--=-
(1) (1) (1)2
~'--- 3 0 3
36 9
2 2 P3T =


Number of trials with a sum of 11 =2 P=3C3 =(1)
1
1 p3T :;;:_
P = number of successful trials_ P=-. 8
8


total number trials
Note: In a pack of 52 playing cards, there
2 1
are 4 king cards, 13 diamond c.ards and 1 1 1
P=-=-- p = p3H +P3T =B+S
36 18 king & diamond card at the same time
P=n CrPr qn-r p:;;:~
PK = probability of drawing a king 4
II!.'P.fJII 1
laiiill Po = probability of drawing a diamond
where: p = ; q = 1 ; n = 4; r=2
·PK & o = probability of drawing a king at 2 2
Total number of trials = :36

2
3
2
34!] the same time a diamond

PKorD =PK +Po-PK&D


4 13 1
R ·=-+-----=-
Koro 52 52 52
4
16
52
P2H =

p2H:;;:
4c2(iJ(iJ
4! ( 1)
(4-2)!2! 2
4

:;;:.S3
ED
Note:
Probability of getting a predit = 1/3
Probability of not getting any credit

Let:
= 2/3

RKoro =-
13


4 P1 = probability that only one student
5 --1 P3H = 4c3(~J (~J
got a credit

·~ 1) (2) 4)
1 2
6
= c (3 3 =
Note: In a pack of 52 playing cards, there 4 3! ( 12
4! ( 1) 1 P, 3 1 (3-1)!1! 21 = 21
Number of trials with a sum of 7 = 6
are 4 king cards, 26 red cards and 2 king p3H:;;: (4-3)!3! 2 =4
&.red cards at the same time ·
P2 = probability that exactly two students
P = number of successful _trials
(21)2(1)2
2
PK = probability of drawing a king got a credit
total number trials PR = probability of drawing a red card P4H=4c 4
6 1
P=-=-
36 6
PK & R = probability of drawing a king at
the same time a red card 1)
p4H:;: ( 2
4
1
:;;:16 P 2 3
1) (2)
= c (3 3 =2
2 1
3! (
(3-2)!2! 21
2) = 21.6
PKorD =PK +PR -PK&R
P3 = probability that all three students
F.ll R
Koro
4
=--+----
52
26
52 52
2
p"" Pm + P3H + p4H-
-
3 1 1
8 +4- +16- got a credit
Total number of trials = 36
Number of trials with a sum of 10 or 12
=4
PKorD = 0.5385
I'~
11
16 r3 =3 c3 (i j J( J = (1 { 2~) = 2~
138 IOOl·Solved\Problems in Engineering Mathematics (2nd Edition.) by Tiong & Rojas Day 6- Venn Diagram, Permutation, Combination & Probability 139

P = probability that at least one student P = probability of passing at least two Note P1 = Pblack X pwhite
got a credit Subjects The 1st ball was returned in the bag
before the 2"d ball was drawn
4 6
P =-x-
P=0.384+0512
1 10 9
_ __g+~+~ 24
p = p1 + p2 + p3 - 27 27 27 p = 0.896 p2 =~ p =-
1 90
19 9
p = 27
ml P = probability that both balls drawn are
all wh1te
Assume the first draw is white and the
second draw is black:
Let:
ED p = probability of getting a correct answer
p = 1/5 P=P1 X p2 =g-Xg
5 5 p2 = pwhite X pblack
Note: 6 4
P =-x-
The only way she c'an get zero is, if all her p = 25 1 10 9
q = probability of getting a wrong answer
3 answers were wrong. 81 p = 24
q = 4/5


1
90
Let:
P = probability of getting 6 correct
p = probability of getting a correct answer
=
p 2/3
answers out from 10 questions Let:
P = probability that one ball is black and
Let: the other is white ·
q = probability_.of getting a wrong answer 1 )v ( 4 ~4
q = 1/3 p = 10 c61 5 l5 ! P1 = probability of drawing a black ball in
the first draw 24 24
6 p = p1 + p2 = 90 X90
p = nC,p' qn-r 101 1\ 4r
3 0 3 P= (10-6)!61 5) 5; P1- -5- p =0.533


8
3 3
P=3C3 ( 1 ) ( 2 ) =(1) ( 1 ) =27
1 3 p = 0.0055
P2 = probability of drawing a black ball in

ED mJ Note:
the second draw
Assume the first draw is black and the
second draw is white:
Notes: Let: The 1st ball was not returned in the bag
P = probability of getting a red or a white before the 2"d ball was drawn
Probability of getting a passing score in ball from tile box p1 = pblack X pwhite
each subject is 0.8 4 5
_ number of red or white balls p2 =~ p =-x-
1 9 9
Probability of failing in any of the three P- -------~~--·-·-
7
total number of balls p = 20
subjects is 0.2
1
14 7 P = probability that both balls drawn are 81
P=-=-
Let: 24 12 all black
P1 = probability of passing exactly two Assume the first draw is white and the
subjects · 5 4 second draw is black:

2
1m P=PxP=-X-
1 2 8 7
P1 = 3C 2 (0.8) (o.2f Let P= 20 =~
P2 = pwhite X pblack
31 P1 = probability of drawing a white ball in 56 14 .. 5 4
pt- (0. 128) = 0.384 '")• =-x
9 9
(3 -2)121 the first draw

P2 = prol;>ability of passing all the three _5


P1 - -
mJ p2 = 20
81
subjects 9 1\ssume the first draw is black and the
:;econd draw is white:
Let:
3
P2 = 3C 3 (Q8) (02) =(1)(0512)
0 P2 = probability of drawing a white ball 111 P =probability that one ball is black and
the second draw the other is white
p2 = 0.512

j
140 100·1 Solved Problems in Engineering Mathematics (2nd Edition) by Tiong & Rojas [
P=P1 +P2-20 20 I

I~
--x -
81 81
P= 40
81
Topics

ID
Probability that A is correct= 1/3,
D Mon
Venn Diagram
Combinatorics
Fundamental Principle of"Counting
assuming he is to sit down first

Probability that B is correct= 1/2,


D Tue
Permutation
Inversion
assuming he is to sit down after A

D D
Cyclic Permutation
Assortment
Probability that C is correct = 1, assuming
he is the last to sit down Theory Wed Combination

Let:
P =probability that A; Band C ~re correct
D D
Problems Thu
Probability Theory
·Mutually Exclusive Events
Independent Events
1 1
P=-x- x 1

1
3 2 D D
Solutions Fri
Binomial Distribution
Odds For and Odds Against
Mathematical Expectation
P=-
6 [I] ~ '!


Notes Sat

Numbers from 1 to 20, which is divisible


by 3 = 6 numbers (3,6,9, 12, 15, 18)

Numbers from 1 to 20, which is divisible


by 7 = 2 numbers (7,14)

Total numbers from 1 to 20, which is


divisible by 3 or 7 = 8 numbers

Let:
P = probability that the ticket nu!llber is
divisible by 3 or 7

P = successful outcomes
total outcomes
8

T
P=-
20
2
P=-
5

T
j
142 100 l Solved Problems in Engineering Mathematics (2nd Edition) by Tiong & Rojas

Topics
' " < ·' < ? ~ '< ~ 'o '
[QJ Branches of Geometry
Basic Postulates of Euclid
Mon
Geometric Elements and Figures
0
,"<•> 'tfi'•,<~'''"">~<i·A'<":<' i>.·.··~.~··•·:~

(• <-, -'<- X ~ ·, "' ), ;_ ... « 4- ~ .~ ~ ~ ~ ~ <• .,. .i , ' .{ .. ~ ·'It~ '< q ~


Types of Angles
Tue Units of Angles
Polygons
L 0
Theory Wed
Classifications of Triangles
Other Types of Triangles

' ,·? ~ . • < ' ~ ' " ~;., •


' ... ~ '" ~ ... 0
Problems
0
Thu
Types of Quadrilaterals
Bramaguptha's Theorem
Ptolemy's Theorem

0
Solutions
0
Fri
Terms Related to Circles
Theorems Involving Circles
Ellipses

0
Notes
0
Sat

What is Geometry? 2. Solid Geometry - deals with the


-t ••
,,. " ' " v ... ' • < ·' <; ~ ' < •., .< ,, .< ' .,
properties of geometrical shapes of
' > ; ., ,. ·+ ,. <; ,. ~ </ ... ·' "' ,_ ~ » , . .,. ~ -~ .(. Geometry is the branch of mathematics three dimensions, such as cones,
which deals with the properties of shapes pyramids, cylinders, prisms, spheres,
and spaces. The term "geqmetry" was etc.
derived from the Greek words, "ge"
meaning earth and "metria" meaning 3. Euclidean Geometry- a geometry
)0 i> o < )' ~ 0 :0 0 "< '• <• N ), ? 0 ". X ~ « ( •>. '0 ~ ( '")<

measurement. that is based on the assumptions of


Euclid, who, about 300 B. C., collected
Euclid (c. 330- c 275 B.C.) in his best the mathematical knowledge of that
known book in geometry "Elements" give time in the 13-volume "Elements". The
more emphasis on Plane geometry while problems of this type of geometry can
Archimedes (287- 212 B.C.) contributed be solvd by logical reasoning from an
so much to Solid geometry. initial core of postulates (axioms), a
method commonly referred to as the
What are the different branches of classical' axiomatic method of Euclid.
Q~s>metry?
4. Non-Euclidan Geometry- a
1. Plane Geometry- deals with the geometry that is not based on the
pmperties of plane figures or assumption of Euclid ..
qt~ometncal shapes of two dimensions,
:;11d1 <ls angles, tnangles, squ;1rr!, 5. Projective Geometry- deals with the
f"llyqow;, Ulllic s(•dron~;, (~!c, study of those properties of plane
frqures that arf' rmch;mqed when ;1
144 1001 oSo1ved Problems in Engineering Mathematics (2nd Edition) by Tiong & Rojas ··- Da_y_I -- Plane Qeq_m~.!Dt:..~

~
given set of points is projected onto a What are the Basic Geometric Elements 3. Supplementary angles -two angles
second plane. and Figures? whose sum is a straight angle (180°).

---'~
6. Trigonometry - a geometry which 1. Point- a.dimensionless geometric
specializes on the study of triangles. figure having no properties other than
This subject is divided into two
branches, namely Plane. Trigonometry
location or place. 5. Straight angle- equal to 180° or
n radians.
-- ..-
and Spherical Trigonometry. 2. line -.the shortest distance between 4. Explementary angles -two angles

--~
any two points. A line is implicitly a whose sum is <l perigon (360').
7. Analytic Geometry- a geometry straight line. A line segment is a piece
which deals with geometric problems
by using the coordinates systems, and
transforming them into algebraic ·
of a line with definite endpoints.

3. Angle - The opening between two


6. Reflex angle- angle great than ·180°
or n radians but less than 360° or
/
( ~-o2:__).
problems. This type of geometry was lines or two planes that meet. The point 2rr radians. \___,..J - - -
270"

r cr
invented by Rene Descartes. Plane where the two line me~t is known as
analytic geometry deals primarily with the vertex. Angle comes from Latin 5. Vertical angles - angles formed by two
the analysis of equations in two "angulus'' for "sharp bend". · intersecting lines. Vertical angles are
variables while Solid analytic geometry
deals with equations hi three variables. What are the different types of angles
according to their measure?
equal', s_/
8. Differential Geometry - a geometry
-........,·:r-£ ( ~\A

~
that applies differential and integral 1. Null or Zero angle -equal to 0.
calculus to cuiVes, surfaces and other A\ / ;r 1-
~
0~
7"~
geometric entities.

What are the Basic Postulates of


Euclid?
J
/
' 2. Acute angle- greater than 0 but less 7. Full angle or Perigon- equal to 360"
Euclid's Elements is based upon five basic Angles A and B are vertical angles.
than goo or 2: radians or 2rr radians.
postulates or assumptions which are as These angles are called vertical angles
2
follows: because each side of one is an

1. A unique straight line can be drawn


between any two points.

2. Such a line can be extended


.&__ G extension through the vertex of a side
of the ot~1er

Wh~..!!..tl!.~
What are the other types of angles? A bisector is a straight line which divides
indefinitely in either direction.
3. Right angle- equal to goo or ~ a geometric figure into two equal figures.
1. Adjacent angles -two angles with a An angle bisector is a light which divides
3. A circle can be drawn in a plane using common leg.
radians an angle into two equal angles.

k_
a given point (a center) and a given
distance (a radius).

4. All right angles are equal.

5. Given a line and a point not on the line,


there .exists exactly one line parallel to
l:_ 2. Complementary angles- two angles ;·
the original line passing through the
whose surn is a right angle (90°).
given point 4. Obtuse angle- greater than 90° or ~
2
Postulate number 5 above is commonly
known as the "parallel postulate"
radians but less than 180° or n radians
L, / Angle bisector

LL~--
l
I
146 . I 00 Solved Problems in Engineering Mat!tematics (2nd Edition) by Tiong & Rojas D~Plane Geometry 147

What are the different units of angles? A regular polygon is a polygon having all Reentrant angle is the inward-pointmg What is a triangle?
sides equal and a!: angles equal. angle (anglt= A 1n the f1gure) of the
1. Degree- The degree is a non-S! unit concave polygon wh1le the other angles A triangle is a polygon with three sides.
approved in 1969 by the General Table showing th•; regular polygons: are called salient angles. All the three sides are .contamed in a
Conference on Weights and Measures. plane.
The diagonal of a polygon is the line
This unit was established by the Name Sides Angle
6Qo .' connecting two opposite vertices. The
Babylonians more than 4000 years Equilateral triangle 3 What are the classifications of triangles
number of d1agonals in a polygon can be
ago. A degree is divided into 60 Square 4 goo I according to their sides?
calculated using the following formula:
minutes; a minute is divided into 60 Regular pentagon 5 108°
seconds. Reqular hexagon 6 120° 1. Equilateral triangle- all sides are
Regular heptagon 7 "'128.57° equal and all angles are equal. An
2. Radian -The radian is the standard ReQular octaQon 8 135° equilateral triangle is also equiangular.
angular measure in the International 9 140° Each interior angle is 60°
Regular nonagon
System.of Units (SI). A radian is the ReQular decagon 10 144°
angle between two radii of a circle 2. Isosceles triangle- two sides equal
Regular hectagon 100 176.4°
which cut off on the circumference an and corresponding two angles are also
· ReQular meqagon. 106 179.99964°
arc equal in length to the radius. equal
ReQular googolgon 100100 "'180°
3. Gon - The gon or centesimal degree is 3. Scalene triangle - no two sides are
1/4001h of the full circle. A right angle is Polygons are named according to the Diagonals= -'2(n- 3) equal.
100 gons A gon is divided into 100 number of sides or vertices. 2
centesimal minutes; one centesimal What are the classifications of triangles
Number of sides Name where n = number of sides according to their angles?
minutes is divided into 100 centesimal
seconds. This unit is also known as 3 Triangle
The interior angles of a polygon are the
grad or grade and is used in surveying 4 Quadrilateral or TetraQon 1. Acute triangle.- each interior angle is
angles inside the polygon. The sum of the
works and aircraft navigation. 5 Pentagon less than a right angle.
mterior angles of the polygon can be
6 Hexaqon
calculated using the following formula:
4. Mil- The mil is a·measure of an angle 7 Heptagon 2.. Right triangle -one angle is a right
which is 1/6400 of the full circle. A right 8 OctaQon angle.
angle is 1600 mils. 9 Nonagon
10 Decagon 3. Obtuse triangle - one angle is greater
The following is a table showing 11 UndecaQon than right angle.
different measure of an angle with 12 Dodecagon
corresponding value in one revolution. 100 Hectaqon Note: A triangle which is not a right
MeqaQon triangle is known as oblique triangle.
1000000
Unit One Revolution 10100 Googoigon -Both acute triangle and obtuse triangle
Degree 360 are oblique triangles.
n n-gon Sum of interior angles= ( n- 2)180°
Radian 2n
What are other types of triangles?
Gon 400 A convex polygon is a polygon having
Mil 6400 each interior angle less than 180° while a
where n =number of s1des
1. Egyptian tri;mgle- a triangle with
concave polygon is a polygon having an I he deflection angle of a polygon is the
sides 3, 4, 5 units.
interior angle greater than 180° ,mgle subtended by the prolongation of
What is a Polygon? 'n1e s1de to the next s1de.
2. Pedal triangle- a triangle inscribed in
a given triangle whose vertices are the
A polygon is a plane figure with three or
feet of the three perpendiculars,to the
more angles. It has as many sides as
sides from some point inside a g1ven
angles. The sides of the polygon are
triangle.
straight lines. The term "polygon" comes
from Greek words "poly" which means
3. Golden triangle- an isosceles
"many" and "gonia" which means "angles".
triangle with sides is to its base in the
golden ratio; its angles are 72°, 72",
Convex polygon Concave polygon ~;lllll of dofll)C[IOII cHID IUS '·' 360" 36"
148 100 l Solved Problems in Engineering_ Mathematics (znct Edition) by Tiong & Rojas Day 7 - Plane Geometry 149

2. Rectangle 4. Parallelogram or Rhomboid 6. Trapezium (General quadrilateral)


Note: The formulas to use in solving

aD
problems involving triangles are found in
A. Given base and altitude: A. Given two diagonals and included
Chapter 9 (Plane Trigonometry) of this
angle:
book
A= ab
What is a quadrilateral?

A quadrilateral is a polygon with four 'h


b i
sides. A quadrilateral is also known as ------- d~·
quadrangle or tetragon. 3. Rhombus b

What are the types of quadrilateral? A Given base and altitude A= bh


A:= ~(d 1 }(d 2 )sin0

0
1. Square- a regular quadrilateral. All its B. Given two diagonals and their
sides are equal and all angles are h included angle:
equal to right angle. A square is a where: d1 and d 2 are lengths of
rectangle with all sides equal. diagonals
b
2. Rectangle - a right-angled B. Given four sides and opposite
parallelogram. A =bh angles: (Using Bramaguptha's
formula)
3. Rhombus - all sides are equal but no B. Given two diagonals
angle equal to right angle.

4. Parallelogram - both pairs ot opposite


sides are parallel. Another term for
parallelogram is rhomboid.

5. Trapezoid-- only two sides are


parallel.
{-"><for
·

b
7
...... d1 ..... ·

dz \ ..
A= -~(d 1 )(d 2 )sine
2

where: d 1 and d2 are lengths of


diagonals

C Given two sides and included angle:


·For rhombus, 0 is a right angle.

£" 7
6. Trapezium- if no two sides are
parallel. A= ~(s-a)(s-b)(s-c)(s-d) -abcdcos 0
2
1
A= 2(d 1 )(d 2 )
7. Kite- convex quadrilateral whose
adjacent sides are equal in pair.
where: d1 and ch are lengths of where: t) =A+C or t) = ~_:+:_Q
a 2 2
8. Deltoid -concave quadrilateral whose diagonals
a+b+c+d
adjacent sides are equal in pair. S=-------
C. Given side and included angle: A= absine 2
9. Cyclic quadrilateral- a quadrilateral
whose vertices lie on a circle
I ---: 5. Trapezoid
7. Cyclic quadrilateral

tJ
a
b

GLih
What are the formulas used in solving
!!!e areq_of a 9!J.?drilateral? -
a

aD
1. Square a 2
·A=a sin0
8
2
A= a 1
A.= --(B + b)h
7

A
Day 7- Plane GeolllEillY 151
150 1001 Solved Problems in Engineering Mathematics (2nd Edition) by Tiong & Rojas
where: n = number of sides perpendicular to the radius of the
What is Ptolemy's Theorem?
circle.
A= .J(s-=a)(s- b)(s- c)(s- d) "The sum of the products of two pairs of 2. Regular polygon inscribed in a
3. Secant of a circle - a !ine cutting the
opposite sides of a convex cyclic circle
circle in two places.
where: s =a+ b + c + d quadrilateral is equal to the product of the
2 lengths of the diagonals." 4. Diameter of a circle -the longest
chord of a circle that passes through
The radius of the circle circumscribing This was named after the mathematician the center.
the quadrilateral may be cal<;ulated astronomer, and geographer, Ptolemy cl
using the following formula: Alexandria. 5. Radius -the distance from the center
to the circle. It is one-half of the
What are the formulas used in solving diameter.
J( ab + cd)( ac + bd)( ad+ be) the areas and perimeters of other
r=~--~~~~--~
4R regular polygons?
A=
1 . (360)
2 nr
2
n sm
6. Chord- the segment of a secant
bounded by the circle.
where: A = area of quadrilateral 1. Regular polygon
a
8. Quadrilateral circumscrib;ng a circle P =2nrsin( 1~0)
secant

where: n = number of sides


a
Wl:tiit is a perimeter?

A perimeter is the distance around a two-


a dimensional shape. The perimeter of the
square is the sum of the length of its sides. I
The formula for perimeter will vary
i
~na 2 cot( ~ )
1 0
A= depending on the geometric figure or
poly~ on. 6. Circumference- the perimeter of a
circle. This is also known as periphery ..
A = rs "' v'abed P=na The circumference of a circle is
What is a circle? 2n(radius) or n( diameter) .
where: s = a + b + c + d where: n = number of sides
2 A circle is a plane curve that is the locus 7. Radian- the measure of an angle
2. Regular polygon circumscribing a of all points in the plane equidistant from a whose arc length is equal to the radius
What is Bramaguptha's Theorem? circle given point, called center. The term "circle" of the circle.
comes from the Latin ·'circus", which
"In a cyclic quadrilateral having refers to a large round or rounded oblong 8. Sector of a circle- area bounded by
perpendicular diagonals, the perpendicular enclosure in which the famous Roman two radii and the included arc.
to a side from the point of intersection of chariot races were held. The circle is a
the diagonals always bisects the opposite conic section whose eccentricity is zero. 9. Segment of a circle- area bounded
side" by a chord and the arc subtending the
What are common terms related to a chord.
Bramaguptha (A.D. 598- 665) was a circle?
Hindu astronomer and mathematician who

tanC~O)
became the head of the observatory at 2 1. Arc of a circle- length of circle
Ujjain- the· foremost mathematical center A= nr between to points on the circle or
in ancient India. between two radii.

P = 2nrtan(. ~0 I
1 2. Tangent of a circle- a line touching
'- n J the ctrcle in one place A tangent is

j
~9b 1 Solved Problemsi!l Engineering Mat!:ematics (2~~ Editio~by Tiong & Rojas Day 7 -Plane Geometry 153

10. Central angle --an angle whose 3. Inscribe angles subtended by the 6. Secant- Tangent Theorem:
vertex is at the center of a circle and A= _:<r2e diameter of a circle are right angles.
whose sides are the radii. The central 360
angle is an angle subtended by an arc.

11. Angle subtended by a chord - an


where: e is in degrees
angle whose vertex is along the
4. Area of segment of a circle:
periphery or circumference and its
sides are chords.

'-- =-ts
e =a(a+b)
e = 13 =goo What is an ellipse?

A.= Area of sector - Area of triangle AOB Therefore, the triangle formed in the Ellipse is a locus of a point which moves
figure is a right triangle. so that the surn of its distances to the fixed
points (foci, denoted as F in the figure) is
The angle 0 is the central angle while p What are useful theorem involving a 4. Chord Theore;n: constant and is equal to the length of the
is the angle subtended by chord "x". major axis.
circle?
What are the for111ulas used in sqjyin.Q_<! 1. If a central angle and a peripheral
circle1 angle are subtended by the same arc,
1. Area of a circle:
then the central angie is twice as large ab = cd v v
as the peripheral angle

A=
A= ltr~ or
.
rt02
4 ----r--.. . .·
~ ·-·~

where: r = radius
a a
D =diameter
Ellipse has two axis of symmetry, a major
2. Circumference of a circle: 5. Secant Theorem: axis (2a), betvveen the vertices (V) of the
ellipse, and a minor axis (2b), which
intersect at the center of the ellipse.
C =2nr or C=:rcD
2, Inscribed angles subtend the same arc
are equal. Area of an ellipse, A

A= n:ab
3. Area of sector of a circle:
c 0=13 where:
1 a = semi-major axis
A= --rc b = semi-minor axis
2
a(a+b)=c(c+d)
i
A ""-r2e
2

where: e is in radians

t,l,
154 1001 Solved Problems in Engineering Mathematics (2"d Edition) by Tiong & Rojas
II
,.

i
·i
,I..
Proceed to the next page for your 7th test.
Detach and use the answer sheet provided
at the last part of this book. Use pencil
number 2 in shading your answer.
'··
,t',
GOOD LUCK I Topics

~ribia:: ~
Mon
Branches of Geometry
Basic Postulates of Euclid
Did you know that. .. the symbols + and - Geometric Elements and Figures
for plus and minus, respectively were
introduced by German mathematician and
astronomer, Johannes Regiomontanus in
'/-: D Tue
Types of Angles
Units of Angles
Polygons

D D
1456! ,:'',
Classifications of Triangles
<!auote: I
!
Theory Wed Other Types of Triangles.
"Man is like a fraction whose numerator is
what he is and whose denominator is what
he thinks of himself. The larger the
denominator the smaller the fraction."
D
Problems Thu
Types of Quadrilaterals
Bramaguptha's Theorem
Ptolemy's Theorem

-Tolstoy D DI
Solutions Fri
Terms Related to Circles
Theorems Involving Circles
1 Ellipses

D D
Notes Sat

30&: ECE Board November 1998 A. 16.85 em


Find the angle in mils subtended by a line B. 17.85 em
10 yards long at a distance of 5000 yards. C. 18.85 em
D. 19.85 em
A. 1
B. 2 309: ME Board April1990
C. 2.5 A rat fell on a bucket of a water wheel with
D. 4 diameter of 600 em which traveled an
angle of 190° before it dropped from the
307: ECE Board April1999 bucket. Calculate for the linear em that the
Assuming that the earth is a sphere whose rat was carried by the bucket before it fell.
radius is 6400 km. Find the distance along
a 3 degree arc at the equator of the earth's A 950
surface. B. 965
C. 985
A. 335.10 km D. 995
B. 533.10 km
C. 353.10km 310: ECE Board November 199:1
D 353.01 km Given a circle whose diameter AB equals
2 m. If two points C and D lie on the circle
:J08: EE Board April199:1 and angles ABC and BAD are 18° and 36°,
1111' <Hl\Jie subtended by an arc is ?-1". If respectively, find the length of the major
llu· r<~dllrs ot thr~ circlr~ is 4~, em, turd llw arc CD.
II'IHJth "' <IIC
156 100 l Solved Problems in Engineering Mathematics (2"d Edition) by Tiong & Rojas _ _ _ _ _ _ _ _ _ _ _ _ _ _ _ _ _ _ _ _ _ _ _.:::D~a::..;yc....:..7_-Plane Geometry 157

A. 1.26 m A. 12 3:t:Z: EE Board April 1991 3::&7: ECE Board March J99&
B. 1.36 m B. 24 From a point outside of an equilateral A circle with radius 6 em has half its area
C. 1.63m C. 20 triangle, the distances to the vertices are removed by cutting off a border of uniform
D. 1.45 m D. 48 10m, 18m and 10m, respectively. What width. Find the width of the border.
is the length of one side of a triangle?
3u:: A certain angle has a supplement 5 317: How many diagonals are there in a A. 1.76cm
times its complement What is the angle? polygon of 20 sides? A. 17.75 m B. 1.35 em
B. 18.50 m C. 1.98cm
A. 67.5° A. 200 C. 19.95 m D. 2.03 em
B. 58.5° B. 170 D. 20.50 m
C. 30° C. 100 3%8: ME Board April :199&
D. 27" D. 158 3::&3: EE Board April1991 The area of a circle is 89.42 sq. inches.
The sides of a triangle are 8 em, 10 em What is its circumference?
31::&: ECE Board November 1998 318: ME Board April1999 and 14 em. Determine the radius of the
Each angle of a ·egular dodecagon is Find each interior angle of a hexagon. inscribed circle. A. 32.25 in.
equal to B. 33.52 in.
A. 90° A. 2.25 em C. 35.33 in.
A. 135° 120° B. 2.35 em D. 35.55 in.
B. 150°
B.
C. 150° C. 2.45cm
I
C. 125° D. 180° D. 2.55 em 3::&9: ECE Board April 1991
D. 105° A square section ABCD has one of its
319: EE Board April1994 3::&4: CE Board May 199& sides equal to x. Point E is inside the
313: CE Board May 1997 Given a triangle, C = 100°, a= 15m, b = What is the radius of the circle square forming an equilateral triangle BEC
How many sides have a polygon if the sum 20m. Find c. circumscribing an isosceles right triangle having one side equal in length to the side
of the interior angles is 1080°? having an area of 162 sq. em.? of the square. Find the angle AED.
A. 26m
A. 5 B. 27m A 12.73 m A. 130°
B. 6 C. 28m B. 13.52 m B. "140°
C. 7 D. 29m C. 14.18m C. 150°
D. 8 D. 15.55 m D. 160°
3::&0: CE Board November 1994
314: ECE Board March 199& In triangle ABC, angle A = 45° and C = 3ZS: EE Board April 1991 330: CE Board November :1.995
The sum of the interior angles of a polygon 70°: The side opposite angle Cis 40 m The sides of a triangle are 8 em, 10 em The area of a circle circumscribing about
is 540°. Find the number of sides. long. What is the length of the side and 14 em. Determine the radius of the an equilateral triangle is 254.47 sq. m. ii r
opposite angle A? circumscribing circle. What is the area of the triangle in sq. m?
A. 3
B. 4 A. 26.1 m A. 7.14 em A. 100.25
C. 5 B. 27.1 m B. 7.34 em B. 102.25
D. 6 C. 29.1 m C. 7.54 em C. 104.25
D. 30.1 m D. 7.74 em D. 105.25
315: ECE Board April :1991
Fioo the sum of the interior angles of the 3::&1: CE Board May i995 3261: CE Board May 1996 3311 CE Board May 1.995
vertices of a five pointed star inscribed in a In triangle ABC, angle C = 70°, A= 45°, Two sides of a triangle are 50 m and 60 m What is the area in sq. em of the circle
'if
circle. AB = 40 m. What is the length of the long. The angle included between these circumscribed about an equilateral triangle
median drawn from vertex A to side BC? sides is 30° What is the interior angle with a side 10 em long?
A. 150° opposite the longest side?
B. 160° A. ~.3m A. 104.7
C. 170° B. ~.6m A. 93."14" B. 105.7
D. 180° C. ~.9m n 9) l4° C. 106.7
D. ~.2m c 90.? 11" D. 107.7
31&: ME Board April1999 (l llCI J()'
How many sides are in a polygon if each
interior angle is 165 degrees?
1~8 1001 Solved Problems in Engineering Mathematics (2"d Edition) by Tiong & Rojas Day 7- Plane Geometry 159

33:11 CE Board November 199:1 C. 64 1t 341: ECE Board November 1995 Problem 346: EE Board October 199:1
The area of a triangle inscribed in a circle D. 16n A rectangle ABCD which measures 18 em. Determine the area of the quadrilateral •
is 39.19 cm 2 and the radius of the by 24 em. is folded once, perpendicular to shown. OB = 80 em, AO = 120 em, OD =
circumscribed circle is 7.14 em. If the two 3371 ECE Board }lllovember 1993 diagonal AC, so that the opposite vertices 150 em c;nd q, = 25•.
sides of the inscribed triangle are 8 em The arc of a sector is 9 units and its radius A and C coincide. Find the length of the
and 10 em, respectively, find the third side. is 3 units. What is the area of the sector in fold.
square units?
A. 11 em A. W.5~
B. 12 em A. 12.5 B. 21.5cm
C. 13 em B. 13.5 c. n.5~
Tf::>,o
D. 14 em C. 14.5 D. n.5~
'\1
< ;V

D. 15.5
3331 CE Board November 1994 342: ECE Board April 1998
The area of a triangle is 8346 sq. m and 338: CE Board May 1998 A trapezoid has an area of 36 m 2 and an A. 2721.66 cm 2
two of its interior angles are 37"25' and A circle having an area of 452 sq. m is cut altitude of 2 m. Its two bases have ratio of B. 2271.66 cm 2
56° 17'. What is the length of the longest into two segments by a chord which is 6 m . 4:5. What are the lengths of the bases? C. 2172.66 cm 2
side? 2
from the center of the circle. Compute the D ... 2217.66 cm
area of the bigger segment. A. 12, 15
A. 171.5 m B. 7, 11 347: CE Board Oc:tober 1997
B. 181.5 m A. 354.89 sq. m C. 8, 10 Find the area of a quadrilateral have sides
C. 191.5m B. 363.68 sq. m D. 16, 20 12m, 20m, 8 m and 16.97 m. if the sum !:!i,
'I
D. 200.5 in C. 378.42 sq. m of the opposite angles is equal to 225°, 'r'·j
I
1!,,,
D. 383.64 sq. m 343: EE Board Marc:h 1998 find the area of the quadrilateral. i 1
1·i
334: ECE Board April 1998 A rhombus has diagonals of 32 and 20 'I'
'j:
The angle of a sector is 30° and the radius 2
339: ECE Board April 199:1 inches. Determine its area. A. 100m i
is 15 em. What is the area of the sector in A swimming pool is constructed in the B. 124m2 1,

cm 2? shape of two partially overlapping identical A. 360 in 2


C. 168m2
circles. Each of the circles has a radius of B. 280 in 2 D. 158m2
2
A. 59.8 9 m and each circle passes through the C. 320in
B. 89.5 center of the other. Find the area of the D. 400 in 2 348: ME Board Oc:tober 1996
C. 58.9 swimming pool. ME Board April :1997
D. 81:;CJ 344: ECE Board April 1998 The area of a regular hexagon inscribed in
A. 380m 2 If the sides of a parallelogram and an a circle of radius 1 is
3351 EE Board Ai"ll'il 199:1 B. 390m
2
included angle are 6, 10 and 100°, ,,
Two perpendicular chords both 5 em from 2
C. 400m respectively, find the length of the shorter A. 1.316
the center of a circle divide the circle into 2
D. 410m diagonal. B. 2.945 'I
four parts. If the radius of the circle is 13 c. 2.598
em, find the area of the smallest part. 340: ME Board April 1991 A. 10.63 D. 3.816
Find the difference of the area of the B. 10.37
2
A. 30 cm square inscribed in a semi-circle having a C. 10.73 349: EE Board April 1990
B. 31 cm 2 radius of 15 m. The base of the square lies D. 10.23 2
Find the area (in em ) of a regular octagon
I
C. 32 cm 2 on the diameter of the semi-circle. inscribed in a circle of radius 10 em?
2
D. 33 cm 3451 CE Board Nove.mber 1996 .
A. 171.5 cm 2 Find the area of a quadrilateral having A. 283
33fn ECE Board April 1998
The distance between the centers of the
B.
C.
172.5 cm 2
173.5 cm 2
sides AB = 10 em, BC = 5 em, CD= 14.14
em and DA = 15 em, if the sum of the
B.
C.
289
298 I
three circles which are mutually tangent to D. 174.5 cm 2 opposite angles is equal to 225•. D. 238
each other externally are 10, 12 and 14
units. The area of the largest circle is A. 96 sq. em 350: GE Board February 199:1
B. 100 sq. em A regular hexagon is inscribed in a circle
A. 727t (' 94 sq. em whose 'diameter is 20m. Find the area of
B. 23 n qg sq. em the 6 segments of the circle formed by the
1t
sides of the hfiXilCJOil llnl"l''l:;;'r' l
iiJj!
I '•I!
I
:_
.:
'.l.__ ·.
~-h
1
I

Ill'~ ·
160 1001 ~olved Proble!ns in Engineering Mathematics (2nd Edition) by Tiong & Rojas
1
A.
B.
36.45
63.54
sq.
sq.
m
m
355: EE Board March 1999
Determine the area of a regular 6-star
li'
C. 45.63 sq. m polygon if the innerregular hexagon has
D. 54.36 sq. m 10 em sides. II

351: EE Board Aprii199:J A. 441.66 cm 2


Find the area of a regular pentagon whose B. 467.64 cm 2 Topics
side is 25m and apothem is 17.2 m. C. ·519.60 cm 2 ;I
D. 493.62 cm 2 Branches of Geometry
I~

2
A. 1075 rn
B. 1085 m
2 Mon Basic Postulates of Euclid
2
Geometric Elements and-Figures
0
C. 1080 m
2
D. 1095 m Types of Angles
TtJe Units of Angles '
3S:l: ME Board October 1!.99&
Polygons
The area of a circle is 89.42 sq. inches.
What is the length of the side of a regular
hexagon inscribed in a circle? 0 [WedJ Classifications of Triangles
Other Types of Triangles
Theory

A.
B.
5.533
5.335
in.
in. [1 [l Types of Quadrilaterals
Bramaguptha's Theorem
c. 6.335 iio. Problems Thu Ptolemy's Theorem
D. 7.335 in.

353: EE Board April1990


In a circle of diameter of 10m, a regular
Solutions
0
Fri
Terms Related to Circles
Theorems Involving Circles
Ellipses
five-pointed star touching its circumference
is inscribed. What is the area of that part
not covered by the star?
0Notes
[]
Sat
A. 40.5 sq. m
B. 45.5 sq. m
ANSWER KEY RATING
C. 50.5 sq. m
319. B 332. D 345.8
D. 55.5 sq. m
306. B
307.A 320. D 333.8 346.A I 0 ~3-50 Topnotcher·
354: EE Board March 1998
A regular pentagon has sides of 20 em. An
308.C
309. D
321. A 334. c
347.
322.C 335. B 348.
c
c '
; c::J33-42 Passer
inner pentagon with sides of 10 em is
310.A
311. A
323.C 336.C 349. A
324.A 337. B •350.0 c::J25-32 Conditional
inside and concentric to the larger
pentagon. Determine the area inside and
concentric to the larger pentagon but
312. B
313. D
325.A 338. B 351. A
326. D 339. D 352. B
0 0-25 failed

outside of the smaller pentagon.


314.C 327.A 340.C 353. c If FAILED, repeat the test.
315. D 328. B 341. c
354. D
A. 430.70 cm
3 316. B c
329. 342. D 355. c
B. 573.26 cm
3 317. B 330. D 343. c
C. 473.77 cm
3 318. B 331. A 344. c
3
D. 516.14 cm

:l
il. iI•l
II
i
i
t
j il :'
;,j[,,
162 100.1 Solved Problef!lS in Engineering Mathematics (2nd Edition) by Tiong & Rojas Day 7- Plane Geometry 163

Ill
5
Ell
Note: A dodecagon has 12 sides, n =12.
a
S= (n-2)(180')
n
5 9 = (n- 2)(180')
= (n-2)(180")
n 165
5 'looo<..-,.: n
tan8=-- 8 = (12-2)(180")
5000 165n = 180n- 360
12
5 15n = 360
8 = tan·
1
( --)= 0.0573 c a= 1so·
5000 n =24 sides
Theory:
28 = 2(0.0573) = 0.1146"
= .
0 1146
x 6400 mils For the same intercepted arc (arc CD),
Ell a
360' the value of the central angle is twice that
S=(n-2)(180")

-
20 = 2.037 mils of the inscribed angle. n
1080 = (n- 2)(180) N=-(n-3)
If one side of the triangle inscribed m a n·2=6 2
circle is equal to the diameter of the 20
n=8sides N =-(20-3)
C =r9 circle, then the said triangle is a right
triangle.
2
N = 170 diagonals
C = 6400.(3° x
2
n rad)
360° Refer to the first figure: Ill
C =335.10km
18+ e + 36 =go
S=(n-2)(180")
540=(n-2)(180)
a
Ill 8=36'
28=72"
n-2 =3
n = 5 sides
Note: A hexagon has 6 sides, n = 6.

C=re 8= (n-2)(180")

C=45(24~x 2nrad)
360°
Let:
c = length of. arc CD
C=r8
a n
8 = (6-2)(180')
6
C = 18.85cm
e =120·
C=1(n·x~)
Ell
c =re
. 360°
C=1.26m
• c

C = 3oo(190o x 2n: rad)

C=995cm

Elll
360°
Ell
Let:
e =the angle
go• - e = the complement of angle e
180•- e =the supplement of angle e
28 = 360

0=36

Let:
5 Az=s:B
By cosine law:
c =?

180 - e = 5(90 -e) S = sum of the interior angles of the five


vertice!O 2
c=Ja +b -2abcosC
2
180-8 = 450-58
48= 270
s;:; 50-:;; 5(36') c=~(15) 2 2
+(20) -2(15)(20)cos100'
e = 67.5' :; - 11l0 c =27
c

j
!'',1
,j:l,
I'
I
'. '.·I'

""
164 1001 Solved Problems in Engineering Mathematics (2nd Edition) by Tiong & Rojas Day 7- Plane Geometry 1&5

• b
c
x. = fc 2 + (at2/ - 2( c)( a/2)GOS B
x=~(40) 2 +(15) 2 -2(40)(15)cos65'
x=36.3m
A= Js(s-a)(s-b)(s-c)
A= J16(16- 8)(16 -1 0)(16 -14)
A=39.19cm

A=rs
2
S=
a+b+c

S = 16 em
2

A= Js(s-a)(s-b)(s-c)
=----
8+10+14
2

~·,I
Mf _......_---:c--~40
\ _ ___: 'o B
ml 39.19 = r(16) A= J16(16 -8)(16 -1 0)(16 -14)
I
r=2.45 em A=39.19cm 2
II
By sine law:

sinA
a

a=c -
sinG

sinAJ
-
c
• A= abc
4r
39.19 = 8(10)(14)
4r
I

II
II
II
\l.,ll' .
( r=7.14cm
sinC :[ .

sin45 J
a=40 - - • B J
(

·~
sin70 Note:
By sine law: Since the isosceles right triangle, is
a-= 30.1m r
inscribed in the circle, then its hypotenuse I',,II:
sin 30 sin (60 + ~)
ml c 10 18
must be equal to the diameter of the circle
C
30"
A
11,1
'i :1
1 b =60 II'!;
~=4.158'
1
A= -(r<~se)(height)
2 1!:1'
By cosine law:
a + ~ +60 + 30 = 180 1 Iii.'

lL
162 = -(x)(x)
B a+ 4.158 + 90 = 180
2 c=Ja 2 +b 2 -2abcosC
a =85.842' x =18 em
c = ~ (50) + (60)
2 2
- 2(50)(60)co:;30'
By sine law: Using Pythagorean theorem: c = 30.06m

By sine law: sin 85.842• sin 30• By sine law:


d=Jx 2 +x 2
X 10
sinC sinA 2r = .J2x 2. sillB sin30"
x =19.95m
II''

c a 2r=J2(1W 60 30.06 1:,.
sin 10• sin45" r=12.73cm B=86.38 r i


It
40 a
a=30 m
a/2=15 m
ml
A+B+C=180
45 + B + 70 = 180
b = 10
B=65° a+b+c 8+10+14
S= =----
2 2
By cosine law:
S=:16cm ac:8

j
166. 100 I Solved Problems in Engineering Mathematics (2"d Edition) by Tiong & Rojas Day 7 - Plane Geometry 167

Note: A c
Since half of the area was removed, then
the area (A} left is also one-half of the
30' +a+a=180o
a=75 111 Ill
total area.
60' +2a+ 8=360°

A=.:!_[ n(R/ J=:_[n(6) J 2


60 + 2(75) + El = 360. Bd" ' ' 'DA
2 2 9 =150'
c =?
A= 18n

But "A" is also equal to the area of the


small circle. (Refer to figur~

A= nr 2
18n = nr 2
• A
Note: Since an equilateral triangle, then
angles A, B and C are equal to 60°.

.1 2
A =-(x) sin A
A= 3r25' = 37.416°
B = 56°17' = 56.283°

.A +B+C = 180
37.416 +56.283 +C = 180
c =86.301'
:f

r =4.24cm 2
1 By sine law:
2
A= -(10) sin 60°
x=R-r
2 sin86.301• sin37.416• i
x=6-4.24
2 A=43.3cm 2
l
'!l'r
r=1.76cm A= nr c a
254.7 = nr 2
A= abc a=
sin86:301 Jc
(
El r=9m
4r
43.3 = (10)(10)(10)
sin37.416
a =0.609c
360°
A= nr 2 9=-
3 4r 1
89.42 = nr 2
r =5.335in
e= 120° r =5.774cm Atriangle ::;:-acsinB
2
Solving for area of circle: 1
By cosine law: 8346 =-(0.609c)(c)sin 56.283)
Let: C = circumference of the circle
2
A= nr 2
C = 2nr x=) r 2 + r 2 - 2(r)(r)(cos 9) 16,692 =0.5065 c 2
C = 2n(5.335) A= 7t(5.774) 2
2 2
x.=A9) +(9) -2(9)(9)(cos120°) c=181.5
C=33.52in A =104.7cm 2
x=15.59m
Ill
• n.-- X J:JA
Note: Since the triangle is an equilateral
triangle, then angles A, B 'and C are each
equal to 60 degrees.

1
A= ~(x)(x)(sln A)
.
Ill
b = 10
'I
II'
I!
1

X X 2
1
A= ~(15.59) 2 (sin 60°) 1 .
A= abc
A =-r 2 e , I'I::if
2
2
2 I r:
-- .,.- tJs A =105.24 m 4r
c~
X
39.19 = -~O)c
1
A =--.(15) 2(30. x--J 7t

Note: 2 180'
4(7.14)
Triangle CDE is an isosceles triangle. A =58.9cm 2 I
cco14crn
l lflilll'
,11'1
,
,l
1
I
j
I

j II: LLJ
ttl
168 ._LOOl Solved Problems_in Eng_!~eering Mathematics (2nd Edition) by Tiong & Rojas Day 7 -Plane Geometry 169

• 5
B
12 = y+5

y=7

·A =As -2A 1
• A 2 = Ac - A1 = 452-84.44
A 2 = 363.56 m 2

A= 66-2[;(base)(height)]

A= 66-(7)(5) = 31 cm 2
1
A=-rC
2
1 .
A=-(3)(9)=13.5sq. umts
2

~
.
stna =--
5 Ell 4.5
cose=-=60°
9
13
o; =22.62" 6
Let:
A = area of the pool 4.5
2a+0=90
A, = area of triangle ABC
2(22.62) + e = 90 Ac = area of the sector
.9=44.76° As= area of the segment (shaded portion.)
Let:
Let: r1 +r2 =12-tEq.1 A1 = area of the smaller segment A. =Ac -A 1
=
A area of the shaded part r2 +r3 =10-tEq.2
A2 = area of the bigger segment 1 1
As == area of sector ABC Ac = area of the circle =-r 2 (29)--r 2 sin29
A, = area of triangle COB r1 +r3 =14-tEq.3 As = area of the sector 2 2
A, = area of the triangle ABC

1 9=-(13)
As =-r 1 2 2[ 44.76•x--
1t ]
Subtract Eq.3 by Eq.2:
A. = 1tr 2
=_!.(9) 2 (120"
2 .
x-n:-}
180"
2 2 180" (r1 +r3 )T(r2 +r3 )=14-10 2 1
452 = 7tr --<W sin120"
A 5 =66cm 2 r1 -r2 =4-+Eq.4 r=12 2
A, =49.75 m 2
Add Eq.4 and Eq.1:
cos8=~=~
r 12 A'=2A- -2A.
(r1 ·~r2 )+(r1 -r2 )=10+4
9=600 =2(1tr 2 )-2A.
2r1 = 16
=21t<W -2(49.75)
r1 =8
1 2 1 2 • A =409.44 m 2
r2 =12-8=4 A 1 =A. -A, =-r (29)--r stn29
' ' ~c r3 =10-4=6 2 2
5

Let: A = area of the largest circle


=_!.(12) 2 (120/' x-1t-} Ill

By Pythagorean theorem: 2 180'
1
{13) 2 =(5) 2 +(y+5) 2 A= 1tr/ =1t(8) 2
= 647t --(12) 2 sin 120"
2
144=(y+5) 2 .A 1 ::::84.44
x/2 x/2

.'~
:11!

170 1001 Solved Problems in Engineering Mathematics (2"d Edition) by Tiong & Rojas Day_J-:()l_ane Geometry 171

r2 = x2 +(;r a a II Substitute: Ill


i',l

2
15 =1.25x 2 A=
(s- a)(s- b)(s -c )(s- d) ii'I'
-abcdcos 2 9
x
= 13.416
(22.07- 5)(22.07 -14.14)
Let: b
A, = area of the semi-circle 1 10 A= 1(22.07-15)(22.07 -10)
Az = area of the square A=-(a+b)h~Eq.l
A = difference of A1 and Az 2 Total interior angle= (n- 2)180 -(5)(14.14)(15}(10)cos 2 112.5"
=(4-2)180 A=100cm 2
1 a 4
A=A 1 -A 2 =-1tr 2 -x 2


Total interior angle= 360'
2 b 5
1 4 29 + 100 + 100 = 360
A =-1t(15) 2 -(13.416) 2 a=-a~Eq.2
2 5 9=8o·
A =173.44
Substitute Eq.2 in Eq.1: By cosine law:

Ill 36=;(~b+b}2) d2 =6 2 +10 2 -2(6)(10)cos80.


24 d = 10.73 '!1:'1
A~B ~ t'J V( :-=oo i ~I
4 5
36 =-b+-b
5 5 Ill By secant law:
l
,:1
r'

18 9 a =5 li
36=-b
5 .AO • BO =CO • DO
b=20m llillllil!ili
o~c
120• 80 =C0•150 L 'Ill,
1',
24-x X 4 C0=64 em ~~~
l1

a=-(20)==16m
5
A Let:
d=-J18 2 +24 2 =30 c =15 A = area of the quadr lateral

x2 =18 2 +(24-x) 2
Ell X
(s- a)(s- b)(s- c)(s- d)
A, = area of triangle 1100
Az = area of triangle FOG I

2 2 A=
x =324+576-48x+x abcdcos 2 9 A =A 1 -A 2
X= 18.75 ,,'i
,,
1 1
A = -(AO)(OO)(sin E - -(BO)(CO)(sin 9)
a+b+c+d

x2 =(;r +(y)2 1
A=-d 1 d2
S=----

s=
2
5+14.14+15+10
=22.07
2
·1
A= -(120)(150)(sin '5")
2
2

;I,!
ir
I'
':
(18.75) 2 =(15) 2 +y 2 2
1 2 1
y = 11.25 A= -(32)(20) --(80)(64 )(sin 25")
,,

2 2
A +C 225"
Length of the fold= 2y = 2(11.25) A=320in 2 9=--=-- A= 2721.66 cm 2
2 2
Length.of the fold= 22.5cm
0=112.5"
172 1001 Solved Problems in Engineering Mathematics (2nd Edition) by Tiong & Rojas . Day 7 -Plane Geometry 173

II A1 = (21)
A 1 =.2,(1) sin60°
2
r 2 Sln9

2
.
Let:
Ah = area of the hexagon
At = area of triangle
Ac = area of the circle
A = area of the 6 segments of the circle

A 1 =0.433 360°
9=-=60°
A0~'~--7b-=~2~0--- oo 6
Atotal =6At = 6(0.433)
2
A total = 2.598 A 1 =(;} sin9 360°
(s- a)(s- b}(s- c)(s- d) 9=--=60°


A= 6
1
abcdcos 2 9 2
At =-(10) sin6o·
Ac= 7tr2
2
a+b+c+d At =43.3m 2 89.42:;; 1tr 2
S=---- r = 5.335in
2
Ah = 6A 1 = 6(43.3)
12 +20+8 + 16. 97 = 28.485 By cosine law:
S= Ah = 259.8 m 2
2
x=Jr 2 +r 2 -2(r)(r)cos9
A =Ac -Ah
9- A+C
5
- . -_ 22 • =112.5•
Let: x = b(5.335)
2 2
2(5.335) cos60
Atotat :;; area of the octagon A:: 1tr 2 - Ah = 1t(10)2 - 259.8 -
- 2 2 x =5.335in
At = area of triangle A= 54.36 m2
Substitute:

A=
(28.485 -12)(28.485- 20)

I (28.485- 8){26.485 -16.97)


A1
360°
9=--=45°
8

=(1)r 2 stn9
.
Ill •
-(12)(20)(6)(16.97)cos 2 112'.5" 2
2 1 2
A=168m A 1 =-(10) sin45°


2
. A 1 = 35.355 cm 2
b=25
.Let:
A total =SA, = 8(35.355) A = area not covered by the star
Let: As = area of the star
A 10181 = 262.84 cm 2 Ah = area of the hexagon Ac = area of the circle
At = area of triangle

Let:
Atota~ =area of the hexagon
At = area of triangle
• 1

A 1 =215m 2
1
A 1 =-(base}(height)=-(25)(17.2)
2 2
360"
29=-

8=36c

8/2 18 =
5
9

I A= 5A 1 = 5(215)
360° ! 9/2
A=1075 m 2
9=-=60°
6 I
j
174 . 100 1 Solved Problems in Engineering Mathematics (2nd Edition) by Tiong & Rojas "'-~ ________ [}liy_7,_ !'lal\f? Qeometry 175

e
9+-+a =180
A= n(;-(base)(height)J = n(;xh J
2
36 +18 +a= 180
a= 126~ A =;xCt:neJ
By sine law: nx 2 X= 10
A=--
4tan9
sin18" sin126"
180 180
9=-=~=30"
X 5
n 6
x = 1.91 m
6(10) 2
A1=---
1 1 4tan30"
A 1 = -rx sine= -(5)(1.91 )sin 36
2 2 A 1 = 259.8 cm 2
A 1 =2.806m 2
x= 10
X2= 20

\ll
A=Ac -10A 1
=7tr 2 -10A 1 180" 180°
9=--=--=36"
= 1t(5)2 -1 0(2.806) n 5
A=50.5m 2 Note:
Let: By inspection, the triangle must be
A1 = area of the inner pentagon equilateraL
Ell A2 = area of the outer pentagon
A = area of the shaded portion 1
A 2 = -x 2 sin 60
A =A 2 -A 1 2
nx ___
nx2 2 1 il'
=--2 . _1_ = -(10) 2 sin 60 ll·il!l

2 II,
I'
4tane 4tane I
A 2 =43.3
5(20) 2 5(10) 2
= ---
4 tan 36 4tan36 A=A 1 +6A 2
x/2 x/2 A= 516.14 cm 2 = 259.8 + 6(43.3)

x/2 X A =519.6 cm 2
tan9=-=~
h 2h Ell
X
h=--
2tane

360" 180'
9=-=-
2n n
Let:
Let: A = area of an •n• sided polygon A1 = area of the hexagon
A2 = area of the triangle IIIII:
A = total area
, ... 1

j I
d
178 l 00 l Solved Problems in Engineering Mathematics (2nd Edition) by 'riong & Rojas

' ' ~ ;'"T(., :,;. '

N .0< <' '> "<- ~ ' " < f',

' .,.. .
Topics

0
~

Polyhedrons
!v!on Platonic Solids
Properties of Platonic Solids
[QJ Prisms
Cubes & Parallelepiped
Tue
Cylinders

,, ... .', '' ~ :. ""N 0 "' V.


~
Theory
0Wed
Pyramids
Cones
.;. ...&

"v .,.
>"" .• .;. .:. '-0 ~ ., ~ ~ ~ .... ·'· ,..:,:,. ~i'"

> "!' ~ :<-.;. '

'
~

,,.~
0 ,' "' «

i'< • .., " «: «


0 '<- ~ <·

* ~ :t 't ·~
* ;t ~

)<"" 0 ~
-:;..

:>.,

" ..,_·
~,

~ " ' ~
o>;,ifj!>.-$Mio;•

• ~ • "'!" "' "


:· " ..·

" "
....
'·''
"~ ;:

~-." 4: ~
~~

.,:.

'f ., " • •
." ~ '
~**~'1<-i<'..f$-<·

~ " ,. . . . . '
0
Problems
0Thu
Frustum of Pyramid or Cone
Prismatoid
Sphere
" " :!<"' "<-... ~· '" ·' ' < ..... "" ,,: .;. ",. -~

•• "
'· ~ .... ~ "-'<- ~ ., '

,, ~ l• ·""' "? ~ I .. , v. ~ ~ .: ~ • •· •
0
Solutions
0 Fri
Zone
Spherical Segment & Sector
Spherical Pyramid & Wedge

., ~ •·.;, ::> f • .'<' '<· J' ::>


~ 'i

o· ~
>X~ ;< •·

$- .., ' • <- :<- ·' "'


> <" '>. ~ 'o

'•· ,_ ,< ~ ~ .. ~ • ~ •· 0Notes


0 Sat
Torus
Ellipsoids
• " ~ .. <·:•".,, ' ... ;: "->:.. ' "0 .. ' '!'" ·' / · , ·' ; , ., ~"'. " .

\'' ""v, ~"' » •' ·' ·<•., •• ".••

What is a polyhedron? Every polyhedron in three-dimensional


space consists of (two-dimensional)
Polyhedron is a solid whose faces are faces, (one-dimensional) edges and
•~'<">0~{~~-.j.-l:> -~1<-<v< r·<.j·• plane polygons. The term "polyhedron" (zero-dimensional) vertices. The faces of
,, ""' ·' ~- .., ~ ;. ·' • '• ' .,. * • ·.,.• < •
comes from the Greel "poly" for "many" the polyhedron intersect each other along
and "hedron" for "base or face". Polyhedra the edges, which meet at the corners
;, < 0 <'- < "<- o.· + ,. 0 + ~ '' 0' <· 0 X < « 0 <• <· ~ <> 0 0 < •'
are named according to the number of known as vertic.es
::>'I<"' .., < ' ... « •. "'"" >' ,' ~ ~ ~ ;< ~
faces. A polyhedron with six faces is
called hexahedron while the one with What are the 5 regular polyhedrons?
'<• ~-$ <f: -i. 0 ~A,~ '$ ~ ~ « -<· : 0. ,_,.. 6:.-:f • ,. ~ • t '< < ~ P > ~- ~ -., P 4 • .;. ~ <o ~-" :f •'
three faces is called tetrahedron.
«~. .,0:<<:<~0{"'>'0"'J<{o.{"'~"'~A""' "<-.(<< ... :.c-~~{'<'->i"'«~~''"=<'-""<'
A regular polyhedron is a solid with all its
Convex polyhedron is one that lies faces identical regular polygons. There are
' ~ ~ 0 ?: ~ "" ~ !> <· z< '• ··' -~ ,,: • p..,."' ~ ~.·,., '~'."' -.!"<-~"~"·-"'~~... .,.,.,,..• -~.? '"'"'' ~-~"'
r~ntirely on one side of a plane that contain only five regular convex polyhedra,
.. ~,,.~,~~~·*0"~1'< ·~.,~~-$¢~'~"' ..... -~.~,~·~~""~"·'
;111y of its faces while a concave namely the tetrahedron, the hexahedron
polyhedron has at leart one face so (cube), the octahedron, the dodecahedron
~
4• { .< > • -> X { X ->. < ": ' "j' » -l< H
located that there are parts of the and the icosahedron. These solids are
,,,·l"?' l'olyhedron on both sides of a plane also known as Platonic solids in honor of
Lorrt.Hninn that face. If a line that connects Plato (427- 348 B.C)
~;. ,. ~ ~ ., .,.
,;
.'"Y lw<J JH >rrrts on the surface of a
• ...., ... ' . . . . ,. ""' ' ' ' v " ~ ~ ' ' ...,
1" >lylir ·dr '>ll 1s cornpletely rnside or on the The following are the Platonic solids with
i'' 1lylu ·rlr orr, lilr, polylwdron IS convc:x, its properties:
"ll11·1wr:;r rl r:; <.<•llC.lVI'

1
Day 8- Solid Geometry 181
180 1001 S~lved Problems in Engineering Mathematics (2"d Edition) by Tiong & Rojas
3. Octahedron 5. Icosahedron Surface area, A:
1. Tetrahedron
A=2(ab+bc+ca)

Right prism is one which has its lateral


faces perpendicular to the base.

e Faces Trianqle
Faces Trianqle No. of faces 20
Faces Triangle No. of faces 8 No.ofedqes ~----1 Ih
No. of faces 4 No. of edges 12 No. of vertices 12
No.ofedqes 6 No. of vertices 6 Polygon angle 60°
No. of vertices 4 Polygon angle 60° Sum of anqles 300°
Polyqon angle 60° Volume of prism, V
Sum of angles 240° Radius of
Sum of anqles 180° Radius of e../2 circumscribed ~~2(5+J5) V=Bh
Radius of eJ6 circumscribed sphere - - sphere
- - ___?__
circumscribed sphere Radius of inscribed
4 Radius of inscribed e.J6
- - sphere ~t+3.J5 Lateral area of prism, A
Radius of ir.scribed e.J6
--
sphere
6 2 6
A= h Pb
sphere
12 Total area 2e 2
./3 Total area 5e 2.f3
Total area e2.f3 Volume Volume
Volume
~e3.J2
!e3.J2
3 ~~3 (3 + .J5) where:
B = area of the base
12
4. Dodecahedron h =altitude of prism
What is a prism? Pb =perimeter of base
2. Hexahedron
A prism is a polyhedron with two faces Oblique prism is one in which the lateral
(bases) parallel and congruent and whose faces are not perpendicular to the base.
remaining faces (lateral faces) are
parallelograms.

~,Ih
Cube is a prism with all six faces a
square. It is a regular hexahedron.and one
e e of the five Platonic solids.
Faces Pentagon
Rectangular parallelepiped is a prism
Faces Square No. of faces 12
with all six faces a rectangle.
No. of edges 30
No. of faces 6
20
--
No. of edges 12 -~o. of vertices -·- Volume of prism, V
~_of vertices
,.l'_l)lygon angle
8
goo
~..Qt:l_angle
Sum of angles
108°
324°
-· / 71 c V =Bh = Ke
-Radius of
~ofanqies 270°
~J3(1 + JS)
Radius of
circumscribed sphere
.!-Radius of inscribed
e~
e
J circumscribed
~_her~---~ ----·-------------
Radius of inscribed
sphere ~ J.~9.~~~-~I5
'---------"b
a
Lateral area of prism, A

A= e Pk
sphere -
2 i
~--·· ---·--·------·-·-··· Volume, V:
Total area
Total area 6e 2 !--.------·----··---·
Gt/ where:
~olume eJ Volume e:~
'I ( 1 1) 7 J~,)
V =abc B = area of the base
------ ~- l ~-~--- I h = altitude of prism
I

j
182 ·100 l Solved Problems in Engineering Mathematics (2"d Edition) by Tiong & Rojas
183
K =area of right section Oblique cylinder is one which has its

_,}
e =lateral edge cylindrical surface not perpendicular to the Volume of the frustum of cone, V
PK =perimeter of right section base.

Truncated prism is a portion of a prism


,_ v "' ~( 81 + 82 + ~8182)
contained between the base and a plane
that is not parallel to the base.
where:

)
r Volume of cone, V

V=-Bh
1
3
81 and 82 = area of the bases
h = altitude of the frustum of cone

or
V = 1t3h (Rz +rz + Rr)
Volume of cylinder, V
where:
V = 8h = Ke B = area of the base
h = altitude of cone where:
where: R = radius of the lower base
8 = area of the base What is a frustum of a pyramid? r~ r = radius of the upper base
=altitude of cylinder h = altitude of the frustum of a cone
v"' s(h1 + hz: h3 + h·-) h
K = area of right section Frustum uf a pyramid is a portion oft~- - .
e = lateral edge pyramid included between the base and a what is a prismatoid?
section parallel to the base.
What is a pyramid? Prismatoid is a polyhedron having for
What is a c_ylinder?

--lh
bases two polygons in parallel planes and
Pyramid is a polyhedron of which one for lateral faces triangles or trapezoids with
Cyiir;ader is a solid bounded by a closed one side lying in one base, and the
face, called the base, is a polygon of any
cylindrical surface and two parallel planes. opposite vertex or side lying in the other
number of sides and the other faces are
triangles which have a common vertex. base of the polyhedron.
Right cylinder is one which has its
cylindrical surface perpendicular to the

~l·.L
base.
Volume of the frustum of pyramid, V
Ih
v =~(81 +82 + ~8182) A \ J
)" Volume of pyramid, V where:
81 and 82 = area of the bases Volume of the prismatoid, V
Volume of cylinder, V h = altitude of the frustum of pyramid
V=_!Bh L
V=8h 3 What is a frustum of a cone? V=s{A 1 +4Am+A2 )
where:
Lateral area of cylinder, A Frustum of a cone is a portion of the
B = area of the base
h = altitude of pyramid cone included between the base and a where:
A = (circumference of base )(h) s.ection parallel to the base. At and A2 = end areas
What is a cone? Am = area at the midsection (at half oft)
L = distance between end areas

l
where:
Cone is a solid bounded by a conical
8 = area of the base This formula is known as the Prismoidlll
surface (lateral surface) whose directrix is
h = altitude of cylinder Formula.
a closed curve, and a plane (base) which
cuts all the elements.

d
184 · lOQ 1 Solved Problems in Engineering Mathematics (2nd Edition) by Tiong & Rojas Dav 8- _SoU!iGeometrv 185
What is a sphere? B Cross-section of torus:
Sphere is a solid bounded by a closed
lh :+-- generating axis
surface every point of which is equidistant
from a fixed point called center.
€J,~~=t)

Volume of spherical segment, V Volume of torus, V


Volume of spherical pyramid:
rch2
- 1 Great circle V =-{3R-h)
3 V=
3
itR E
=
V 2'lt 2Rr2
540 Lateral area of torus:

Volume of sphere, V What is a spherical sector? where:


E = spherical excess of polygon ABCD A =4tt2Rr
4 Spherical sector is a solid generated by
V=-nR 3 in degrees
3 rotating a sector of a circle about an axis where:
which passes through the center of the What is a spherical wedge? R = distance from axis to center of
Surface area of sphere, A circle but which contains no point inside generating circle
the sector. Spherical wedge is a portion of a sphere R = radius of generating circle
A=4'ltR 2
Jh
bounded by two half great circles and an
included arc.
What is an ellipsoid?
What is a zone?
Ellipsoid (Spheroid) is a solid formed by
Zone is that portion of the surface of a revolving an ellipse about its axis.
sphere included between two parallel
planes.

.!h Volume of spherical sector:

1
V=-AR Volume of spherical wedge:
3

where : A = area of zone V= 1tR39


270
. .
maJOr axiS
..
Volume of general ellipsoid, V
Area of zone, A What is a spherical pyramid? What is a torus?

Spherical pyramid is a pyramid formed Torus is a solid formed by revolving a 4


A= 2nRh V =-ltabc
by a portion of a surface of a sphere as circle about a line not intersecting it. 3
base and whose elements are the edges
What is a spherical segment? from the vertices of the base to the center Prolate spheroid is a solid formed by
of the sphere. revolving an ellipse about its major axis.
Spherical segment is a solid bounded by
a zone and the planes of the zone's base. 4 2
V ::,...nab
3
·"
186 1001- Solved Problems in Engineering Mathematics (2nd Edition) by Tiong & Rojas

Oblate spheroid is a solid formed by


revolving an ellipse about its minor axis.

4 2
V=-na b
3
Topics
Proceed to the next page for your 8th test.
Detach and use the answer sheet provided
at the last part of this book. Use pencil
0tvlon
Polyhedrons
Platonic Solids
Properties of Platonic Solids
number 2 in shading your answer.

GOOD LUCK I
[QJ Prisms
Tue Cubes and Parallelepiped
Cylinders
~ribia:
Did you know that ... the Pascal's triangle
0
Theory
0
Wed
Pyramids
Cohes
which is used to determine the coefficient
of a binomial expansion was named after
the French mathematician, philosopher Problems
0Thu
Frustum of Pyramid or Cone
Prismatoid
Sphere
and physicist E31aise Pascal but die:! not
claim recognition for the discovery
because such triangle was first introduced
by a Chinese mathematician, Chu Shih-
0
Solutions
0Fri
Zone
Spherical Segment & Sector
Spherical Pyramid & Wedge
chieh in 1303 !

~uote:
0Notes
0Sat
Torus
Ellipsoids

"Mathematics consists of proving th~ most


obvious things in the least obvious way." 31!0&: ME Board October 1.99:1 358: CE Board May 1995
A circular piece of cardboard with a A sphere having a diameter of 30 em is cut
- George Polya diameter of 1 m will be made into a conical into 2 segments. The altitude of the first
hat 40 em high by cutting a sector off and segment is 6 ern. What is the ratio of the
JOining the edges to form a cone. area of the second segment to that of the
Determine the angle subtended by the first?
sector removed.
. A. 4:1
A. 144° B. 3:1
B. 148° C. 2:1
C. 152° D. 3:2
D. 154°
359: CE Board November :199&
:li>S7: CE Board November :1994 If the edge of a cube is increased by 30%,
What is the area in sq. rn of the zone of a wj how much is the surface area
spherical segment having a volume of increased?
1470.265 cu. rn if the diameter of the
sphere is 30 m? A. W%
B. ~%
2
A 465.5 m C. ~%
5G5.5 m 2
,.ll f!G~>-~i m 2
D. W%
IJ !l~lfifJ rn 2

j LL
188· 1001 Solved-Problems in Engineering Mathematics (2nd Edition) by Tiong & Rojas
Day 8- Solid Geometry· 189
3601 ECE' Board April1995 3&51 CE Board November 1997
Find the volume of a cone to be 3701 CE Board November 1994 the triangular base and have the height of
Each side of a cube is increased by 1%.
constructed from a sector having a A regular triangular pyramid has an 8.6 ft., 7.1 ft. and 5.5 ft. respectively.
By what percent is the volume of the cube
altitude of 9 m and a volume of 187.06 cu.
increased? diameter of 72 em and a central angle of
210". m. What is the base edge in meters? A. 413 te3
B. 311 ft
A. 1.21%
B. 2.8% A. 12367.2 cm
3 A. 1~ c. 313 ft3
3 B. 13 I}: 391 ft3
c. 3.03% B. 13232.6 cm
3 C. 14
D. 3.5% C. 13503.4 cm
D. 14682.5 cm
3 D. 15 375: CE BOard November 1995
A circular cylinder with a volume of 6.54
3&11 ECE Board November 199Z
3&61 CE Board May 1998 371: CE Board November 1995 cu. m is circumscribed about a right prism
Given a sphere of diameter, d. What is the
Find the volume of a cone to be The volume of the frustum of a regular whose base is an equilateral triangle of
percentage increase in its diameter when
constructed from a sector having a triangular pyramid is 135 cu. m. The lower side 1.25 m. What is the altitude of the
the surface area increases by 21 %?
diameter of 72 em and a central angle of base is an equilateral triangle with an edge cylinder in meters?
150" of 9 m. The upper base is 8 m above the
A. 5%
lower base. What is the upper base edge A. 3.50
B. 10%
3 in meters? B. 3.75
c. 21% A. 5533.32 cm
3 C. 4.00
D. 33% B. 6622.44 cm
C. 7710.82 cm
3 A. 2 D. 4.25
3&ZI ECE Board November 199Z D. 8866.44 cm
3 B. 3
C. 4 3761 CE Board May 199&
Given a sphere of diameter, d. What is the
D. 5 A circular cylinder is circumscribed about a
percentage increase in. its volume when 3&71 CE Board November 199&
right prism having a square base one
the surface area increases by 21 %? A conical vessel has a height of 24 em and
a base diameter of 12 em. It holds water to 37Z.: EE Board April 199Z meter on an edge. 'T:he volume of the
What is the volume of a frustum of a cone cylinder is 6.283 cu.m. Find its altitude in
A. 5% a depth of 18 em above its vertex. Find the
whose upper base is 15 em in diameter meters.
a 10%
3
volume (in cm ) of its content.
and lower base 10 em. in diameter with an
~ 21%
altitude of 25 em? A. 4.00
~ ~% A. 188.40
B. 298.40 B. 3.75
3
C. 381.70 A. 3018.87 cm C. 3.50
3&31 EE Board October 1991 3
D. 412.60 B. 3180.87 cm D. 3.25
How many times do the volume of a
sphere increases if the radius is doubled? C. 3108.87 cm 3
3
368: CE Board May 1995 D. 3081.87 cm 377: CE Board November 1997
What is the height of a right circular cone The bases of a right prism are hexagons
A. 4 times
3731 EE Board April1993 :•. with one of each side equal to 6 em. The
B. 2 times having a slant height of J10x and a base In a portion of an electrical railway .cutting, bases are 12 em apart. What is the volume
C. 6times
diameter of 2x? the areas of cross section taken every 50 of the right prism?
D. 8 times
mare 2556, 2619, 2700, 2610 and 2484
A. 2x sq. m. Find its volume. A. 1211.6cm3
3&41 CE Board May 1997 B. 3x B. 2211.7 cm 3
A circular cone having an altitude of 9 m is
C. 3.317x A. 522,600 m 3 c. 1212.5 cm 3
divided into 2 segments having the same
D. 3.162x B. 520,500 m 3 D. 11.22.4 cm 3
vertex. If the smaller altitude is 6 m, find
the ratio of the volume of the small cone to
c. 540,600 r'n
3

3&9: CE Board November 1995 D. 534,200 m 3 378: EE Board April199&


the big cone. The ratio of the volume to the lateral area Two vertical conical tanks are joined at the
of a right circular cone is 2:1. If the altitude 374r ME Board April199& vertices by a pipe. Initially the bigg~r tank
A. 0.186 is 15 em, what is the ratio of the slant Determine the volume of a right truncated is full of water. The pipe valve is open to
B. 0.296
height to the radius? triangular prism with the following allow the water to flow to the smaller tank
C. 0.386
definitions: Let the corners of the triangular until it is full. At this moment, how deep is
D. 0.486
A. 5:6 base be defined by A, B and C. The length the water in the bigger tank? The bigger
B. 5:4 of AB = 10ft., BC =9ft. and CA =12ft. tank has, a diameter of 6 ft and a height of
C. 5:3 The sides A, 8 and C are perpendicular to 10 ft, the smaller tank has a diameter of 6
D. 5:2
190 1001 Solved Problems in Engineering Mathematics (2"d Edition) by Tiong & Rojas

ft and a height of 8 feet. Neglect the 38~: An ice cream cone is filled with ice
volume of water in the pipel.ine. cream and a surmounted ice cream in the
form of a hemisphere on top of the cone. If
A. '1200 the hemispherical surface is equal to the
lateral area of the cone, find the total
B. ~ volume (in cubic inches) of ice cream if the Topics
C.~ radius of the hemisphere is 1 inch and

D.~
assuming the diameter of hemisphere is
equal to the diameter of the cone. D
tvlon
Polyhedrons
Platonic Solids
Properties of Platonic Solids
~79: The central angle of a spherical
wedge is 1 radian. Find its volume if its
radius is 1 unit.
A.
B.
C.
3.45
3.91
4.12
[QJ Prisms
Cubes and Parallelepiped
Tue

·u
D. 4.25
Cylinders
A.
B.
C.
2/3
1/2
3/4.
384: ME Board April1997
A cubical container that measures 2
D
Theory VVed
Pyramids
Cones
D.

D D
2/5 inches on a side is tightly packed with 8 Frustum of Pyramid or Cone
marbles and is filled with water. All 8
Prismatoid
~80: A regular octahedron has an edge marbles are in contact with the walls of the
2m. Find its volume (in m 3 ). container and the adjacent marbles. All of Problems Thu Sphere
I Zone
~ D
the marbles are of the same size. What is
A.
B.
3.77
1.88
the volume of water in the container? I Spherical Segment & Sector
Solutions Fri j Spherical Pyramid & Wedge

D Di
c. 3.22 A. 0.38 in 3
D. 2.44 B. 2.5 in 3 Torus
C. 3.8 in 3 . Ellipsoids
~81: CE Board May 1996 D. 4.2 in 3 Notes Sat
A mixture compound of equal parts of two
liquids, one white and the other black, was ~8S: CE Board May 1997
placed in a hemispherical bowl. The total The corners of a cubical block touched the
depth of the two liquids is 6 inches. After closed spherical shell that encloses it. The ANSWER KEY ' RATING
standing for a short time, the mixture
separated, the white liquid settiing below
volurrte of the box is 2744 cubic em. What
volume in cubic centimeter inside the shell 356. A 366. c 376.A c:J 26-30 Topnotcher
the black. If the thickness of the segment
of the black liquid is 2 inches, find the
is not occupied by the block? 357. B
358.A
367.
368.
c
B
377. D
378.A
,, c:J 20-25 Passer
radius of the bowl in inches. A. 2714.56 359. D
360.c
369.
370.
D
A
379.A
380.A
c:J 15-19 Conditional

A.
B.
7.33
7.53
B.
C.
D.
3714.65
4713.56
4613.74
361. B
362. D
371.
372.
B
c
381. A
382. A
0 D-14 Failed

C. 7.73 363. D 373. A 383. B If FAILED, repeat the test.


D. 7.93 364. B 374. B 384.C
365.c 375. c 385.c
~82: CE Board November 1996 ,, ~

The volume of water in a spherical tank


having a diameter of 4 m is 5.236 rna
Determine the depth of the water in the
tank.

A. 1.0
B. 1.2
C. 1.4
D. 1.8

I
4
11111;:'11,11:.·,11111

:II
192 1001 Solved Problems in Engineering Mathematics (2nd Edition) by Tiong & Rojas .193 I I

• A= 21trh
= 21t(15)(6)
A= 565.49 m 2 ••
@
v2=[~)
v1 r,
=(
V 2 =SV1
2\ =8
\)
3

r
AI A2= 1.21A1
Ill
- -r·~;l2
A2 - -
A1· . d1
x=~ =f<Sol-(40) 2

x=30

c1 = circumference of the circle


A2 2--m:.h 2 24
-=--=-=4
A1 2--m:.h 1 6
:: = J:: =r~~ =11

~ = [-~) = (~)
3 3
c2 = circumference of the base of the d2 =1.1d1 .
cone A 2 :A 1 =4:1 = 0.296

•~
C = length of arc Thus, d2 is increased by 10% v2 112 9
C=C 1 -C 2
= 27tr- 21tX
= 21t(50)- 21t(30)
c =401t
/

X1
.
X1
X1

X2=1.3x1
X2

• ~· •
\:E;}
4
C=re

~=[2)2 =(1.3']2 ·=1.69


401t =(50)0
401t 180 ..
0=--x-
50
0=144'
1t A
1
x
A 2 =1.69A 1
1
'
[::)' t:J'

Let:
Thus, A 2 is increased by 69% . c1 =circumference of the circle
=(1.21~1
312 312
= circumference of the base of the
v2 - -
- -[A
2 )
c2
cone
II
_
-1.331
V1 A1 . ~ C = length of arc of the sector

~
X1
.. X2 v2 =1.331V1 C =C 1 -C 2
. X2 Thus, V2 is increased by 33.1% re =2n:r-2:n:x
X1 X2=·1.01 X1 36(210 x-1t-)· =21t(36)-27t(X)
1th2
V=-(3r-h)

1th2
3 1 01
3

V2 =[2) =( · ' ] =1.0303


3
ml!f\ 180"
x=15cm
·II

1470.265 = -[3(15)-h] 1
x
1
h=~
vV2 = 1.0303 v, ' II'

h=6 m
3
1404 = 45h 2 -h 3
Thus, V2 is increased by 3.03% C9 r, r2"' 2r,
h = .[362 -15.
h=32.726cm
2 I
illl
~!
i
194 I 00 i Solved Problems in Engineering Mathematics (2"d Edition) by Tiong & Rojas
195
1

V =7710.88cm 3
1
V = -nx 2 h = -1t(15) 2 (32.726)
3 3
By ratio & proportion:

6
24 18
• Substitute:

8[35074+0.433x
135 =-
2
+ l
• r =4.5 2
3 )(35.074)(0.433x )
2
50.625 = 35.074 + 0.433x
1 2 1 2
V = -nr h 2 = -n(4.5) (18) + 3.897x
X
3 3
0 = x 2 + 9x -36
V =381.7cm 3

~
Note: e "'60°, since equilateral triangle. 0 =(x-3)(x +12)
r
.
1 x=3 em

ml
@
V=-Bh
Ox 3
mJ
=;-1[1;x SinSJh
'

Let: .. V 2 .
c1 = circumference of the circle X
c2
T25
= circumference of the base of the
cone By Pythagorean theorem: 187.06 = ~(; } 2
sin 60°(9)
C = length of arc of the sector
x=12m
C=C 1 -C 2
(...fi0x)2 =h2 +x2
10x 2 =h 2 +x 2
re=2nr-2nx
h 2 = 9x 2
ED Let:
36(150" x-n-)=2n(36)-2n(x)
h=3x --- A1 =area of the lower base
180° A2 = area of the upper base

h
x=21cm

=-/r 2 -x 2
h = -/36 2 -21 2
h=29.24cm
•& '
'

.
A 1 = nr,Z = n(5) 2 = 25 n
A2

h
=nr/ =n(7.5) 2

V=-(A 1 +A 2 + -/A 1 A 2
= 56.25n

AL = nrl )

1 3
1
V =-nx 2 h V =-nr 2 h Note: 8 = 60°, since equilateral triangle.
3 3 Substitute:
v Let:
1 -=2~asgiven
V =-n(21) 2 (29.24) A1 = area of the lower base 25
3 AL A2 = area of the upper base V = - ( 25 1t + 56.25 n +-} (25n )(56.25n:))
3
V = 13,503.44 cm 3 V=2AL
1 V =3108.87 cm 3

• •
2
..!_ 'n/\h=2'7tt:_L A 1 =-(9) sin60°=35.074
3 2
rh =6L 1 2 2
A 2 = -(x) sin60o = 0.433x
L h 15 2 Note:
-----
r 6 6 Since the areas being cut is at the same
h1= 24
h distance, then the given solid is a
L 5 V-= (A 1 +A 2 +-}A 1 A 2 ) prismatoid. And since there are five
:l different areas being cut then, this solid is
2
equivalent to two rrisrnatoids
196 1001 Solved Problems in Engineering Mathematics (2nd Edition) by Tiong & Rojas Day 8 - Solid Geometry 197

h
V =-(A 1 +4Am +Az)

where:
6

A1 = area of the first base (base 1)


• h
v =(~d
2
)h

6.283=.::.(1.4142) h

h=4m
4
2
v1 =-1 ( -
nd,Z
3
V1 = 94.247 tt
4
1t
- ) h1=-(6) 2

3
(10)
12

A2 = area of the second base (base 2)


1(1td/)
V2 =- 1t
- - h 2 =-(6) 2 (8)
Am = area of the middle section
3 4 12
h = distance between base 1 and base 2
V2 = 75.398 ft 3
Let
V1 = total volume of the two prismatoid V = V1 - V2 = 94.247-75.398

100
v =18.849 ft 3 .
v, =-[2556+4(2619)+2700]
6 360° By similar solids:

r
100 9=-=120°
+-[2700 +4(2610)+ 2484] x=6
3
6
V1 =522,600m 3 By cosine law:
Let:
A = area of one base
x = length of each side of the base
~ =(h~
3
x2 =r 2 +r 2 -2r 2 cos9 94.247 = (10)
Ill (1.25)2 = 2r 2 - 2r 2 cos 120
nx 2
A=-- 18.849 h
1.5625 = 3r 2 4tan9 h =~200ft


r=0.72m
18o· 180°
h21 9=-=--=30°
h1 V=1tr 2 h n 6
.li... a
. c .......··· B ..."···
. . •••••··•••. ·· ··1h3 6.54 = 1t(0.72)h
...••-·····...~ ····t:: h=4m 6 6 2
A= ( )( ) = 93.53 cm 2
A b c


4 tan 30•
V = Ah = 93.53(12) 1tR 3 9
a+b+c 9+12+10 V:-:1122.4cm 3
V=--
S=--- 2700


2 2
180
1t(1 )3 1 rad x - -)
s =15.5 V=
(
1t ·~ 180

A =)s(s-a)(s-b)(s-c) 270' 270


2
A= j15.5(15.5- 9)(15.5 -12)(15.5 -10) V =- cubic units
2 3
A = 44.039 ft
Top view
V =A(h 1 +h; +h3 J d=~(1)
d=1.4142
2
+(1)
2
Ell
v = 44.039( 8 ·6 + 7~1+ 5.5 J Let:
v1 =total volume of the bigger tank
V2 = total volume of the smaller tank
V=311ft 3 V := volume left in the bigger tnnk

. '.!·.~
J i •
198 1001 Solved Problems in Engineering Mathematics (2nd Edition) by Tiong & Rojas Day 8- Solid Geometry 199

J2 Jth2
.J x 2 + x 2 + x 2
V=-x 3
3
V=-(3r-h)
3 Ell d=

7th2
x=2 d= .[3(14)2
J2(2)3 5.236 = --(3(2)- h)
V=-__:___:_-
3 ' d = 24.24 em
3 2 3
5 = 6h - h r=12.12cm
V=3.77m 3
h=1m
x=2 V=Vs -Vc
ED ml Let:
r = radius of each marble
3
v=(;nr .)-x3
Vw = volume of water inside the cube

,y
Vc = volume of the cube 3 3
Vm = volufl1e of each marble v=(;n(12.12) )-14

x =4r V =4,713.55em 3

2 =4r
Let:
vb = volume of the black mixture r = 1t2
Vw = volume of the white mixture
!Vw =Vc -8Vm
Let:
Vb =Vw

V 1 =Vb+Vw
Ac = surface area of the cone
Ah = surface area of the hemisphere
Yw =x
3
-s(;nr3
)

V1 = total volume
V 1 =2Vw Vc =volume of the cone
Vh = volume of the hemisphere
Vw =(2)
3
-s(;n(0.5) 3
)

Substitute:
V., =3.8in 3
Ac =Ah
V 1 =2Vw
~L =2(4\rl)
2
-1r(h1-(3r-h
) [ 1r(h )
1) = 2 -
2
-(3r-h 2 )
2
J 2 Ill
l
3 3 L = 2r = 2(1)
L =2in r=1
1t(6)2 [ 1t( 4}2
- -(3r-6)=2 - -(3r-4)
3 3 h=~
36(3r-6) = 32(3r-4) = )(2)2 -(1)2
108r-216=96r-128
h = 1.732in
12r = 88 Let:
V = volume inside the sphere but outside
r=7.33in V1 =Vc+Vh
the box
V. = volume of the sphere
2
V 1 = 1 nr h+;
1(4 nr 3J Vb = volume of the box
3 3
2 3 vw = x3
v, =(;}1) {1.732)+ :1t(1)
2744 = x 3
V1 = 3.91in 3 ,,
I,

x 14 em ;,.,,lr

-~"Et.
202 1001 Solved Problems in Engineering Mathematics (2nd Edition) by Tiong & Rojas

Topics

0Mon
Plane Trigonometry
Solutions to Right Triangles
The Pythagorean Theorem
0Tue
Special Triangles
Solutions to Oblique Triangles
Law of Sines & Cosines

Theory
~
VVed
Law of Tangents
Trigonometric Identities

0
Problems
0Thu
Exponential Form of Identities
Other Parts of Plane Triangle
Radius of Inscribed Circle

0
Solutions
0Fri
Radius of Circumscribing Circle
Properties of Triangles
Points in a Triangle

0Notes
0Sat
Conditions for Congruency
Conditions for Similarity

What is Trigonometry? What are the two general classifications


of plane triangles?
Trigonometry is the study of triangles by
applying the relations between ·the sides There are two general types of triangles,
and the angles. The term "trigonometry" namely:
I comes from the Greek words "trigonon"
which means "triangle" and "metria" Right triangle - a triangle that has a right
meaning "measurements. angle.

What are the two branches of Oblique triangle- a triangle that does not
trigon(lmetry? have a right angle. Acute triangle and
obtuse triangle :;~re oblique triangles.
Trigonometry is divided into two branches,
'i namely: Isosceles triangle (triangle with two sides
!i er;ual and two angles equal) could be a
Plane Trigonometry deals with triangles right triangle or an oblique triangle.
in the two diiT!::nsions of the plane.

Spherical Trigonometry concerns with


triangles extracted from the surface of a
sphere

I
j
'

204 1061 Sohred Problems in Engineering Mathematics (2nd Edition) by Tiong & Ko]as ' Day 9- Plane Trigonometry 205
What are solutions to a plane right 3. Special Triangles . 1. Reciprocal relations
triangle? a b c
A. Egyptian triangle: sinA = sinB = sinC

LJ.
1. Fundamental Trigonometric 1 1
Functions 2. Laws of Cosines
sin'A= cscA cotA =tanA
1 1
This solution to an oblique triangle is .cosA=secA secA= cosA
side opposite used when the given are: 1 1
tan A"' cotA cscA= sinA
3 A. two sides and the included angle or
side adjacent B. three sides
B. 45°- 45° Right Triangle 2. Even-odd identities
. side opposite . This was demonstrated by the French
s1n 9 = -:---'-'--- mathematician Francois Viete (1540-
hypotenuse
1603). sin(-e) "".~sine.
side adjace. nt cos'(-9) =cose
cos e = ---=--,-- 1
hypotenuse · a2 = b2 + c 2 - 2bccosA
f tan(-e) =~tan a
tane = side opposite b =a2 +c2 -2accosB
2
cot(-e) =-cote
side adjacent e2 = a2 + b2 - 2abcosc
te side adjacent sec(-e) =sece
co side opposite esc( -9) =-esc a
_ hypotenuse C. 30° - 60° Right Triangle 3. Laws of Tangents
sec e - . . 3. Cofunction identities
'2~
s1de adJacent This solution to an oblique triangle is
hypotenuSe used when the given are two sides and
sine= cos(90~e)
~.1
esc e = --:-''-'------,... the included angle.
· side opposite
cose = sin(90- 9)
This was demonstrated by the French
2. The Pythagorean Theorem .J3 mathematician Francois Viete (1540- tane =cot{90- 9)
Pythagorean theorem is the most
1603). cote= tan(90 -e)
renowned of all mathematical · What are solutions to a plane oblique
triangle? This was first described by a Danish seQe ::;;cs9(90- e)
theorems. It is considered as the most
proved theorem in mathematics. This
mathematician and Physician Thomas esc a... s~~(9o -.e)
1. Laws of Sines Fincke (1561-1656) in 1583.
was formulated by Pythagoras .(c.580
- c 500 B.C.) about 500 B.C. 4. Pythagorean relations
This solution to an oblique triangle is 1
used when the given are: _a_-b_ = tan 2(A-B) il
II
Pythagorean theorem states that "In a
a+ b -~1.____ sin 2 A+ cos2 A =1 f
right triangle, the sum of the squares of A. two angles and one opposite side
=
1+ cot2 A cse2 A
I
tan (A+B) t,l

hypotenuse• /"1
. the sides is equal to the square of the or
B. two sides and an opposite angle
\ 2
What are trigonometric identities?
1+tan2 A "'sec2 A
i
0 This was demonstrated by Ptolemy of
L_ja Alexandria in 150AD.
c
Trigonometric identities are equations
that express relations among ttigonometric
5. Sum of angles formulas

functions which are true for all values of sin(A+B)= sinAcosB+c:oSAsinB

~
.b the variables involved. cos(A +B)= cosAcosB-sinAsinB
2 2 tan( A+ B)= tan A+ tanB
a +'b*•c · The following are the trigonometric
· 1-tanAtanB
identities:
B c A

1 - 11
20& 1001 Solved Problems in Engineering Mathematics (2nd Edition) by Tiong & Rojas Day 9 - Plane Trigonometry 207

6. Difference of angles formula 11. Difference of two functions triangles. This phenomenon is known as

~
the ambiguous case.

sin(A- B)= sinAcosB -cosAsinB sin A- sinS= 2cos~(A + B)sin~(A- B)


cos( :A ·B)"' cos A, cos 8 + sin A sin 8
tan(A .. B)= tl;inA.~tan B cos A- cosB = 2sin1(A+ B)sin.!(A- B)
2 2
· · · 1+tanA tans c
tan A-· tanS= _s_in_,(_A_-_B..:.)

=./~b[1-(a:bJ]
cosAcosB

Note: sinp = sin(180- f3)


tc
7. Double angle formulas

12. Product of two functions What are the other parts of general
sin2A = 2sinA.cosA triangle? What are values of the radius of
cos2A = cos2 A - sin2 A 2sinAsinB =cos( A· B)- cos( A +B) inscribed circle and the radius of
2sinAcosB =sin( A+ B)+ sin( A -B) 1. Height of a general triangle circumscribing circle in a triangle?
tan2A = 2tanA
1·tan2 A 2cosAcosB =cos( A+ B)+ cos( A -B)

~
let R be the radius of the circumscribing
circle and r be the radius of the inscribed
8. Powers of functions circle.

What are the exponential forms of the 1. General triangle


sin 2 A"" .2(1•-cos2A) fundamental trigonometric functions?
2
c
cos 2 A =~·(1+cos2A)
2 , The following are the trigonometric
functions expressed in terms of h -. 2Jf>(s- a)(s'-b}(s- c:)
2
tan,
A ,t- C;QS2A
.,, . exponential functions and the imaginary . c- c
· · 1+cos2A unit.
a+b+c
9. Functions of half angles where: s =
sinx = eix- e·ix 2
2i Note that he is perpendicular to side c.
sin~ =±l~cosA COSX = eix + e-•x
2. Median of a general triangle
2 2 2
A ·l~cos.A
cos-=± tanx = -i(eix -e-ix}
2 . 2 e•x + e-ix abc
r = ArRIANGLE
R=
tan~= 1- cosA = sin A
cotx = i( eix + e-ix}
a+b+c 4ATRIANGLE
2 sin A 1+cos A
e•x -eix
2. Equilateral triangle
10. Sum of two angles c c
where: e = base of the natural logarithm
i = imaginary unit
2 2
sin A+ sinB = 2sin1(A +B)cos.!(A -B)
2 2
' .1 1
What is an ambiguous case? me =1.J2a
2
2
+ 2b~-
· . c.
2
cosA+cosB"" 2cos-(A +B)cos-(A -B)
2 . 2 If you are given a triangle with two sides
sin(A+B) and an opposite angle known, you can
tan A +an
t 8= . . . solve one of the two unknown angles 3. Angel bisector of a general triangle
co~AcosB
using the law of sines. Since sin e = sin
(180- 8), a unique solution is obtained
only in right triangles. When rlmlinCJ with
oblique triangles, you will f111d lw"

j
208 · 100 I SOlved Problems in Engineering Mathematics (284 Edition) py Tiong & Rojas
-------------- Day 9 •· Plan£.!!!9.!?nOI!!_et!)!'~~J.

.~
,,.aJi .... a,/3
R=- where: s = - b -+ c an d a 1s
a+- · th e s1·de
6 3 2 fi.--·------·
A= y\s- a)(s- b)(s- c)(s -d)
physically tangent to the circle
3. Right triangle
Note: A+ C = 1so•
c B + 0 = 180"
A= .Js<s- a)(s- b)(s ·c) 1. Given two diagonals and an Ptolemy's Theorem states that • The sum
included angle of the two pairs of opposite sides of a
where: s= a-t:b+c convex quadrilateral inscribed in a circle is
2 equal to the product of th1:1 lengths of the
diagonals"
4. Triangle inscribed In a circle
ac + bd "' did2

where : d1 and d2 are diagonals of a

A= Id 1d2 sine
quadrilateral

This theorem w<)s named after the


geographer, mathematician and
2. Given 4 sides and 2 opposite angles astronomer. Ptolemy or Claudh.1s
What are the formulas for the area of a Pto:~maeus (<:.100- c.16U A.D.) of
triangle? b c Alexandria.
abC
1. Given bllse and altitude A=-
. 4r Whi!t~l.f!m.Q..rtant ~arQJ.!2~
triangle?

LJ·£
5•. Triangle circumscribing a circle
1. The sum of two sides of a triangle is
AL _ ___.JD greater than the third side and their
d difference is less than the third side.

b b A= J(s :a)(s -b)(s- c)(s- d) .. abcdcos2 9 .2. The perpendicular bisectors of the
sides and the blsectors of the angles
A .. t ...._ a
.... l
•. ......_WIT where: of a triangle, meet in points which are
a+b+c+d the center of the circumscribing circle
A=rs S=----·- the inscribed circle, respectively.
2. Given 2 sides and Included angle 2
O='~~= B+D

~
3. The altitudes of a. triangle meet in a
where: s= a+b+c 2 2 point.
2
3. Cyclic quadrilateral (All vertices lie 4. The median of a triangle are
6. Triangle with escribed circle on a circle) concurrent at a point which is 2/3 of
b the distance from any vertex to the
···················
midpoint of the opposite sides.

~
A=tabstne
r
3. Given 3 sides . ...•.
c
Using Heron's Formula: Named after Circumcenter- the point of concurrency
Heron of Alexandria (1'1 Century AD) of the perpendicular
A= r(s-a)
bisectors of the sides of a
/./ triangle.

j
!I
!['

210 .1 00 1 Solved Problems in Engineering Mathematics ~2nd Editioll) by Tiong & Rojas

lncenter- the point of concurrency of the Proceed to the next page for your 9th test.
angle bisector of a triangle. Detach and use the answer sheet provided
.at the last part of this book.· Use pencil
Orthocenter - the point of concurrency of ?
number in shading your ansWer.
the altitudes of a triangle.
GOOD LUCK!
What are the conditions for two
Topics
triangles to be congruent(:::~)?

Two triangles are congruent if the


~ribia:
Did you know that... Pythagoras was the
D
Mon
Plane Trigonometry
Solutions to Right Triangles
elements of one triangle are equal to the The Pythagorean Theorem
corresponding parts of the other triangle.

The following are the conditions for


first to connect mathematicS and music. In
fact, he is credited with discovering octave
and the fifth interval of a note !
D
Tue
Special Triangles ·
Solutions to Oblique Triangles
congruency:

1. Two sides and the included


<muote: D [g La"'\S of Sines & Cosines
Law of Tangents
"Do not worry about your difficulties in Theory Wed Trigonometric Identities
angle are equal.

2. Three sides are equal.


mathematics, I assure you that mine are
still greater."

- Albert Einstein
D
Problems Thu
Exponential Form of Identities
Other Parts of Plane Triangle
Radius of Inscribed Circle
On~

D D
3. side and adjoining angles Radius of Circumscribing Circle
are equal.
Properties of Triangles
What are the conditions for two
Solutions Fri Points in a Triangle
triangles to be similar?

The following are the conditions fbr similar


D D
Notes Sat
Conditions for Congruency
Conditions for Similarity ·
triangles:

1. Two corresponding sides are


proportional; included angles are 386: ECE Board April :1999 D. sin 4A
Sin (B -A) is equal to _ _ , when B =
equal.
270 degrees and A is an acute angle. 389: ECE Board April :1999
2. Three sides are proportional. Of what quadrant is A, if sec A is positive
A. -cosA and esc A is negative?
B. cos A
3. Two angles are equal.
C. -sinA A. IV
D. sinA B. II
C. Ill
387: ECE Board "pril :1999 D. I
2
If sec A is 5/2, the quantity 1 -sin 2 A is
equivalent to 390: ME Board October :1996
Angles are measured from the positive
A. 2.5 horizontal axis, and the positive direction is
B. 1.5 counterclockwise. What are the values of
C. 0.4 sin B and cos B in the 4th quadrant?
D. 0.6
A. sin B > 0 and cos B<0
:188: ECE Board April :1999 B. sin B <0 and cos B<0
(cos A) 4 - (sin A) 4 is equal to _ _ . C. sin B > 0 and cos B>0
D. sin' B < 0 and cos B>0
1\ r:o~; '1/\
ll C()~; :)/\

c ~~Ill ~)/\

j . . t!
212. 100 t Solved Problems in Engineering Mathematics (2"d Edition) by Tiong & Rojas Da~ Plan_eTrigonometry 213

:!91: ECE Board November 1998 397: EE Board April1992 B. 1/6 408: EE Board October 1992
Csc 520° is equal to Find the value of A between 270° and 360° C. 2 Evaluate arc cot [2cos (arc sin 0.5
if 2 sin 2 A- sin A =1. D. 1
A. cos 20° A. 30"
B. esc 20° A 300° 403: CE Board November 199:! B. 45°
C. tan 45° B. 320° Find the value of y in the given: y = (1 + C. 60°
D. sin 20° C. 310° cos 28) tan 8. D. 90°
D. 330°
:!921 ECE Board Aprii199:J A. sin 8 409: ECE Board March 199&
Solve for 8 in the following equation: Sin :J98: CE Board November 199:! Solve for x in the given equation: Arc tan
B. cos 8
28 =cos 8 If cos 65° + cos 55° = cos 8, find 8 in
C. sin 28 7t
radians. (2x) + arc tan (x) = -
D. cos 28 4
A. 30°
B. 45° A. 0.765
404: CE Board May 1992 A. 0.149
C. 60° B. 0.087
C. 1.213 Find the value of sine+ cos e tan e B. 0.281
D. 15°
D. 1.421 cose C. 0.421
:J93: CE Board November 1993 D. 0.316
If sin 3A = cos 68, then 399: CE Board November 1992 A. 2 sin 8

A. A+ B = 90°
Find the value of sin (arc cos fj ). B.
C.
2
2
cos 8
tan 8
410: EE Board March 1998
Solve for x in the equation: arc tan (x + 1)
+ arc tan (x- 1) = arc tan ( 12).
B. A+ 2B =30° D. 2 cot 8
C. A+ B =180° A. 8/11 A. 1.5
D. None of these B. 8/19 40S: ME Board April1996 B. 1.34
C. 8/15 2 2
Simplify the equation sin 8 ( 1 + cot 8) C. 1.20
394: EE Board October 1996 D. 8/17 D. 1.25
So lye for x, if tan 3x = 5 tan x.
A.
400: EE Board October 1991 411: ECE.Board November 1998
A 20.705° The sine of a certain angle is 0.6, calculate B. sin 2 8 Solve for A for the given equation cos 2 A =
B. 30.705° the cotangent of the angle.
C. sin
2
e sec 28 1 - cos2 A.
C. 35.705°
D. 15.705° 2
A. 4/3 D. sec e A. 45, 125, 225, 335 degrees
B. 5/4 B. 45, 125, 225, 315 degrees
39S: EE Board October 1997 c. 4/5 406: ME Board October 199S
If sin x cos x + sin 2x = 1, what are the C. 45, 135, 225, 315 degrees
D. 3/4 Simplify the expression D. 45, 150, 220, 315 degrees
values ofx?
sec 8- (sec e) sin 28
40_1: EE Board March 1998 412: ECE Board April1991
A. 32.2°, 69.3°
B. -20.67", 69.3° If sec 2A = _ __.:!___ , determine the angle Evaluate the following:
2
sin 13A · A. cos 8 sin oo +sin 1 o +sin 2° + ... +sin 89° + sin90°
C. 20.90°, 69.1 o
D. -32.2°, 69.3° A in degrees. B. cos e cosoo +cos 1° +cos 2° + ... + co.s89° + cos90°
2
C. sin 8
396: EE Board April1991 A. so A. 1
D. sin 8
Solve for G is esc (11G -16 degrees)= B. 6° B. 0
sec (5G + 26 degrees). C. 3° C. 45.5
D. yo 407: ME Board April1998
112
Arc tan [2 cos (arc sin [(3 ) I 2]) is equal
D. 10
A. 7 degrees
402: CE Board November 199:l't to
B. 5 degrees 413: ECE Board April1991
C. 6 degrees Simplify the following:
If tan x = _.!_ tan y = _.!_ what is the value A. n/3
D. 4 degrees 2' 3' cos A +cosB sin A+ sinB
R. n/4 - - - - , - - - + ---,----=-
of tan (x + y)? c )[/16 sin A -sinB cosA-cosB
IJ 11/?
A '1/7 A. 0

J
ZI 4 1001 Solved Problems in Engineering Mathematics (2"ct Edition) by Tiong & Rojas Day 9 - Plane Trigonomet!Y 215

B. sinA A. 39.49 A. 19.7", 307.4 mph D. 90.7 m


C. 1 B. 35.50 B. 20.1•, 309.4 mph
D. cosA C. 30.74 C. 21.1•. 321.8 mph 427: CE Board November 1997
D. 42.55 D. 22 3", 319.2 mph Points A and B are 100 m apart and are of
4141 ECE Board April1991 the same elevation as the foot of a
2sin8cos8-cos8 419: ECE Board April 1998 42~: ECE Board April 1998 building. The angles of elevation of the top
Eva Iuate: -------;:,....-----,;o---
2 2 Points A and B 1000 m apart are plotted A man finds the angle of elevation of the of the building from points A and Bare 21"
1- sine+ sin 8 - cos 8 on a straight highway running East and and 32• respectively. How far is A from the
top of a tower to be 30°. He walks 85m
West. From A, the bearing of a tower C is nearer the tower and finds its angle of building in meters?
A. sin 8 32• W of N and from B the bearing of C is elevation to be so•. What is the height of
B. cos 8 26• N of E. Approximate the shortest the tower? A. 259.28
C. tan 8 distance of tower C to the highway. B. 265.42
A. 76.31 m C. 271.64
D. cot 8 D. 277.29
A. 364m B. 73.31 m
B. 374m C. 73.16 m
4151 ECE Board April1994 C. 384m 4281 ECE Board November 1991
D. 73.61 m
Solve for the value of "A" when sin A = 3.5 D. 394m The captain of a ship views the top of a
x and cos A= 5.5x. ~; ECE Board April 1994 lighthouse at an angle of so· with the
4201 ECE Board November 1998 A pole cast a shadow 15 m long when the horizontal at an elevation of 6 meters
A. 32.47• Two triangles have equal bases. The angle of elevation of the sun is 61 •. If the above sea level. Five minutes later, the
B. 33.68• altitude of one triangle is 3 units more than pole is leaned 15" from the vertical directly same captain of the ship views the top of
C. 34.12• its base and the altitude of the other towards the sun, determine the length of the same lighthouse at an angle of 30•
D. 35.21• triangle is.3 units less than its base. Find with the horizontal. Determine the speed of
the pole.
the altitudes, if the areas of the triangles the ship if the lighthouse is known to be 50
416: ECE Board November 1996 differ by 21 square units. meters above sea level.
A. 54.23 m
If sin A= 2.5J 1x, cos A= 3.06x and sin 2A
B. 48.23 m
= 3.939x, find the value of x? A. 6 and 12 A. 0.265 m/sec
C. 42.44 m
B. 3 and 9 D. 46.21 m B. 0.155 m/sec
A. 0.265 C. 5 and 11 C. 0.169 m/sec
B. 0.256 D. 4and10 D. 0.210 m/sec
4251 ME Board November 1994
C. 0.562
A wire supporting a pole is fastened to it
D. 0.625 421: GE Board August 1994 429: ME Board April1997
20 feet from the ground and to the ground
A ship started sailing S 42.35' W at the 15 feet from the pole. Determine the length An observer wishes to determine the
417: CE Board May 1994 rate of 5 kph. After 2 hours, ship B started of the wire.and the angle it makes with the height of a tower. He takes sights at the
If conversed sin 8 = 0.134, find the value at the same port going N 46.20' W at the top of the tower from A and B, which are
pole.
of8. · rate of 7 kph. After how many hours will 50 feet apart, at the same elevation on a
the second ship be exactly north bf ship A. 24 ft, 53.13° direct line with the tower. The vertical
A. 30" A? B. 24 ft, 36.87° · angle at point A is 30• and at point B is
B. 45° c. 25 ft, 53.13° 40•. What is the height of the tower?
c. 60° A. 3.68 D. 25ft, 36.87"
o. go· B. 4.03 A. 85.60 feet
C. 5.12 4261 CE Board November 1997 B. 92.54 feet
418: ME Board April1991 D. 4.83 The angle of elevation of the top of tower C. 110.29 feet
A man standing on a 48.5 meter building B from the top of tower A is 28• and the D. 143.97 feet
high, has an eyesight height of 1.5 m from 422: ME Board April :i99~ angle of elevation of the top of tower A
the top of the building, took a depression An aerolift airplane can fly at an airspeed from the base of tower B is 45•. The two 4~0: ME Board April199~
reading from the top of another nearby of 300 mph. If there is a wind blowing towers lie in the same horizontal plane. If A PLDT tower and a monument stand on a
building and nearest wall, which are so· towards the cast at at 50 mph, what should the height of tower B is 120 m, fine! the level plane. The angles of depression of
and so• respectively. Find the height of the be the plane's compass heading in order height of tower A. the top and bottom of the monument
nearby building in meters. The man is for its course to be 30•? What will be the viewed from the top of the PLOT tower at
standing at the edge of the building and plane's ground speed if it flies in this A. 66.3 m 13° and 35" respectively. The height of the
both buildings lie on the same horizontal course? B. 79.3 m tower is 50 m. Find the height of the
plane. C 872 m monument

A
Zll 1001 oSolved Pr~blems in Engineering Mathematics (2"d Edition) by Tiong & Rojas

A 29.13 m A. 240
B. 30.11 m B. 420 ·
c. ·32:t2m' C. 320
D. ~ 33.51 m D. 200

431: ECE .Boar«t November 1:998


If an equilateral triangle is circumscribed
about a circle of radius 10 em, determine
Topics
~-~-----·---·-·
the side of the triangle.

A. 34.64 em
D
Mon
Plane Trigonometry
Solutions to Right Triangles
B. 64.12 em The Pythagorean Theorem
C.
D.
36.44 em
32 ..10 em D
Tue
Special Triangles
Solutions to Oblique Triangles

~
oz: EE .Board Oetober 1997 Laws of Sines & Cosines
The two legs of a triangle are 300 and 150.
m each, respectively. The angle opposite
the 150m side is 26". What is the third
D
Theory Wed I
Law of Tangents
Trigonometric Identities
side?

A. 197.49 m
D
[=]
Problems Thu
' Exponential Form of Identities
Other Parts of Plane Triangle
Radius of Inscribed Circle
B. 218.61 m
ra,,
C.
D.
34{78
282.15
m
m DI
Solutions Fri
Radius of Circumscribing Circle
Properties ofTriang les
Points in a Triangle
43~ EE Board October 1997
The sides of a triangular lot are 130 rn.,
180m and 190m. The lot is to bl'l divided
by a line bisecting the longest side and
D DI
r-Jotes Sat
1 Conditions for Congruency
Conditions for Similarity

drawn from the opposite vertex. Find the


length of the line. -----:--··--~
ANSWER KEY ·· ~---·----·--·----
. RATING
A
B.
120m
130m 386. A
c
399. D 412. A 425. D 0 1
13-50 Topnotchel'
C.
D.
125m
128m
387.
388. B
389. A
400.A
40'1. B
402. D
413. A
414. D
426. B
427.A 0 33-42 Passer·
415. A 428.C
390. D 103.c 416. B 429. B
I [ ] 25-32 Conditional
434: EE Board October 1:997
The sides of a triangle are 195, 157 and
210, respectively. What is the area of the
391. B
392.A
404.C
405.A
417.
418.A
c 430. D
431. A
0 0-25 Failed

tria~~gle? 393. B 406. B 419. B 432.C


c '
IfFAILED, repeat the test.
394. A 407. B 420. D 433.
A. 73,250 sq. units 395.c 408.A 421. B 434.C
B. 10,250 sq. units 396. B 409. 8 422. c 435.A
C. 14,586 sq. units 397. D 410. B 423. D
D. 11 ,260 sq. units 398. B 411.c 424.A

435: ECE Board April1998


The sides of a triangle are 8, 15 and 17
unies. If each side is doubled, how many
square units will the area of the new
trioogle be?

j
rull
!l,i(

218 lOOi Solved Problems in Engineering Mathematics (2nd Edition) by Tiong & Rojas· · Day 9- Plane 'I'l"igonometry 219 I

II
sin (270. -A)= sin 270' cos A
-$in A cos 270'
= (-1 )cos A- sin A (0)
Ill
In the 4th quadrant:
a

-b

sin3A =cos6B
sin 3A =sin (90'' - 68)
2x=41.8'
X =20.9'

Complementary angles have the same


values of their sine functions. Thus, the
3A =90' -68 other angle is equal to: 90" - 20.9" = §JU.:


sin(270' -A)=-cosA
A =30' -28
. Opposite side -b b

a s1nB=
hypotenuse
=-=--
c
a
c
A+2B=30'

sin 2 A 2
+cos A= 1 cos B -
adjacent side
hypotenuse
.
c
ml esc(11G-16" )=sec(5G+26')
1 1
1-sin 2 A=cos 2 A ·--
tan 3x = 5 tan x sin(11G-16') cos(5G+26')
- 12 -
-------- Thus, sin B < 0 and cos 8 >0 tan (2x + x) = 5 tan x cos(5G+26' )=sin(11G-16' )~Eq.1
sec A 5/2 tan2x+tan x

- ml - - - - - = 5 tan x
1-sin 2 A=0.4 Note: sine = cos (90° - a )
1-tan 2x tan x
tan 2x +tan x = 5 tan x- 5 tan 2x tan 2 x Let: 9 = 11G-16"
lliiiiill tan 2x = 4 tan x -5 tan 2x tan 2 x
sin (11G -16' ) =cos (90 -(11G -16)
4 tan x =tan 2x + 5 tan 2x tan 2 x
cos 4 A-sin 4 A=cos 2 Acos 2 A
sin(11G-16' )=cos(106' -11G)~Eq.2
4 tan x =tan 2x (1 + 5 tan 2 x)
- sin 2 A sin 2 A
= cos 2 A(1- sin 2 A) esc 520' =esc (520' - 360' ) 4~=( 2 ~ J(1+5tan 2 x) Substitute Eq.2 in Eq.1:
2
- sin A(1- cos A) 2
=esc160' 1-tan 2 x
Cbs..,(5G+26' )=~(106' -11G)
4-4tan 2 x =2+10tan 2 x
=cos'2A-~ esc160' =esc(180' -160')
5G+26' =106' -11G
2=14tan 2 x
-sin 2 A+~ esc 160' = C$C 20'
16G=80'
tan x = 0.377964473
= cos 2 A -sin 2 A G=5'
Thus esc 520° = esc 20° . X= 20.7048° !llllllillll
cos 4 A- sin 4 A= cos 2A


,,

Note: cos 2A = cos2 A- sin2 A Ell IDI


2 sin 2 A- sin A= 1
sin 29 =cos 9 sin x cos x + sin2x = 1 ~ Eq.1
sin 2 A- 0.5 sin A= 0.5
b Note:
Note: sin 29 = 2 sin 9 cos 9 By completing square:
2 sin x cos x =sin 2x
-a Substitute: sin x cos x = 0.5 sin 2x ~ Eq.2 (sin A- 0.25) 2 = 0.5 + (0.25) 2
In the 4th quadrant: 1' 1111''1'

2 sin 8cos8=cosa (sin" A- 0.25) 2 = 0.5625 1'1 ,,:


Substitute Eq.2 in Eq.1:
hypotenuse c 2sin8 ;,1 sin A -0.25 = ±0.75 ,I
1

secA= , =- 0.5 sin 2x +sin 2x = 1 sin A =-0.75 + 0.25


adjacent side b sin9=0.5
8=30' 1.5 sin 2x = 1 A =sin·1 (-0.5)
hypotenuse c c
escA= =-=-- A =-30' or
opposite side -a a sin 2x = 0.6667
A= 360-30 = 330'
ill
j '
220. l 001 Solved Problems in Engineering Mathematics (2nd Edition) by Tiong & Rojas Day 9- Plane Trigonometry 221


cos 65 + cos 55 = cos e
0.99619=cose
a=~ =J5 2 -3 2
a=4

.
cote=
adjacent side a
=-
Note:
cos 29 = cos 2 8- sin 2 e
=(1-sin 2 8)-sin 2 8
cos 2e = 1- 2sin 2 8
x=sece(cos 2
1
= - -- ( cos 'a e)
Cbs-a_
X= COS8
e)

e = 5" opposite side b


Substitute:
1t 4
cote=~
e=5" x - - y= (1 +cos 28) tan 8
1800 3 \

=(1+1-2sin 2 8)tan8

• J]
e = 0.087 rad
e = tan-1 [ 2 cos( sin- 1 ;


= (2- 2sin 2 8) tan 8
2
=2(1-sin 8)tan8
e = tan-1 [ 2 cos (60" l]

b~?~
1
sec2A=---
sin 13A I
.: i
= 2(cos ) 8) ( Cbse_
sinS J 8=tal')-1 [2cos(60' )]

1 =2cos8sin8 8=tan- 1 (1)


1t 1t
cos 2A sin 13A y =sin28 8=45' x - - = - rad


a =15 sin 13A = cos2A 180' 4

e=cos-

cose=-=
17
1
(
15)
~ j~Asgiven

15 adjacent side
·
hypotenuse
Note: sin

Substitute:
e = cos (90• - 9}

sin 13A = cos 2A


Cbs._ (90 -13A} =do&_ 2A
x=
sin 8+cos8 tan 8

cos8
sine Cos-e_tan8
=--+
• a= cot-1 [ 2 cos (sin-1 0.5)]
8 = cot- 1 [ 2 cos (30 l]
b=~ =J17 2
-15 2
90-13A =2A cos a Cos-a_ 1
8=cot- (1.732)
b=8 90=15A = tan8+tan 8 cot 8 = 1.732
A=6" x =2tan8 1


. opposite side b --=1.732
stne = =- tan8

8
hypotenuse c D tane=(--
1
J
sine=- tan (x + y) = tan x +tan y 1.732
17
x = sin 2 8(1 + cot 2 8)


1-tan xtan Y 8=30"
= sin 2 8 + sin 2 8 cot 2 8

sine= 0.6 or~= opposite side


5 hypotenuse
=~

tan (x + y) = 1
1
-+-

1-(;)(;J
1
=sin 2 8+ ~[-
cos-

=sin 2 8+cm~ 2 8
X= 1
eJ
~
2


tan-1 2x+tan-1 x =2:or45'
4

b~3bh • •
Let:
A= tan-1 2x
tanA=2x
X= sec; 8- SeC tl sin 2 8 B=>=tan-1 x
y = ( 1 +cos 2e} tan e
7
a=? -csec0(1-sin 0) tan B = x
Z22. 100-1 Solved Problems in Engineering Mathematics (2"a Edition) by 'l'iong & Rojas Day 9- Plane Trigonometry 223

Substitute:

A+B=45
Using the quadratic formula:

2
-2± J(2) -4(12)(-24)
1111
x=
cos A +cosB sin A +sin B
+~·-------
a
tan (A+ B)= tan 45 X = --'------'---'---'- sin A= 2.511x
sin A - sin B cos A - cos B
tan A +tanB 2(12) cos A =3:06x
-----1 (cos A+ cos B)(cos A· cos B)J
sin 2A = 3.939x
1-tanA tanB -2±34 ( +(sin A+ sin B)(sin A- sin B)
x=--- x=~----------~
2x+x Note: sin 2A = 2 sin A cos A
--=1 24 (sin A- sin B)(cos A- cos B)
1- 2x(x) -2+34
x=--=1.33 (cos 2 A- cos 2 B) +(sin 2 A- sin 2 8) Substitute:
3x = 1-2x 2 24 x=--------------~
(sin A- sin B)(cos A· cos B) sin 2A = 2 sin A cos A
2x 2 +3x-1::r0

Using the quadratic formula:


Ill 2 2 2
(cos A+ sin A)- (sin B + cos B)
x=----------~-------~
(sin A- sin B)(cos A- cos B)
2
3.939x = 2(2.511 x)(3.06x)
3.939\ = 15.367x 1.
2
2
cos A= 1-cos A 2 1-1 3.939
X: -3±J(3) -4(2)(-1) = -3±4.123 x=---------------- x=--
2cos 2 A =1 (sin A -sin B)(cosA -cos B) 15.367
2(2) 4 X =0.256
cos 2 A = 0.5 x=O
-3+4.123
X

X =0.28
4
cosA=±0.707
cosA =0.707
A =45" or335
a a
2 sine cos e -cos a
a cos A =-0.707
A =135" or 225°
x=
1-sin e + sin 29 -cos 29
coversed sin e = 0.134


cos 9(2 sine -1) Note: coversed sin 9 = 1 - sin 6
tan·1 (x + 1) + tan·1 (x -1) = tan-1 12
~= (1- cos 2 9) + sin 2 9- sinS
Substitute:
Let: cos 9(2 sin e -1 )
X=---~----~-
A =ta'ril-1 (x+fj' sin 2 0 + sin 2 1+ ... sin 2 89 + sin 2 90 sin29 + sin 2 e- sine
coversed sine= 0.314
X= .
•II cos 9(2 sin e -1) 1-sin9=0.314
tan A =(li! + 1' 2 2
eos O+cos 1+ ... cos 2 89+cos 2 90 !
1 I X= " sin9=1-0.314
B = tan - (x -1) 2 sin 2 e- sin e
Note: sin9=0.866
tanB = x-1 cos9(~) 'I
sin 2 A +cos 2 B =1andcos 2 A +cos2 B = 1 X=--------- 9=60' I'
II
sinS(~)
I
Substitute: provided, A+ B = 90
ml
'
x =cote •I

A+B=tan-1 12 Thus, the above equation can be written f;!:'I

--~-=12
tan(A+B)=tan(tan- 1 12)
tan A +tanB
in the following format:

(sin 2 0 + sin 2 90) + (sin 2 1+ sin 2 89)


a ,1,.!
,I

sin A= 3.5x; cos A= 5.5x


1-tanA tanB
(x + 1) + (x -1 ) = +... +(sin 2 44+sin 2 46)(sin 2 45)
x=--~------~--~-
12 sin A 3.5x
1- (x + 1)(x -1) (cos2 0 + cos 2 90) + (cos 2 1+ cos 2 89)
cosA 5.5x
2x = 12-12(x 2 -1) +... + (cos 2 44 + cos 4 46)(cos 2 45) tan A= 0.636363
2x=12-12x 2 +12 X= 1
A= 32.4T
12x 2 +2x-24=0
!'iillil
1: 1~1!
j
224 J_Q_OJ oSolv~d Pro~em~ in Engineering Mathef!lati~~nd Edition) by Tiong & R~~ - - - - - - - - - - - - - - - - - - - - = D a y 9-~ane Trigonome~!!_ Iii,
~ ill

~h~h
X X
sin42.35')

\l\j50 ~Jso-h {1 0 + 5t) -_-.----- :: 7t


(

1\
stn46.20'
9.354 + 4.6771 = 7t

I :h, \ L ih\
A 2.3231 = 9.354
= 4.03 hrs
85 +X X

L ____~ -
_t
I"


h
50 tan30=-- i
tan 80 = --· B5+x
X.
b b {~ h = (85 + x) tan 30-+ Eq.1
x=8.816m "'
50-·h h1 = b + 3 h
!an 50 - -----·· tan60=-
X h2 =b-3 x
50-h h = x tan60 -+Eq.2
tan 50 ---·--·--
A 1 =A 2 +21
8.816 I
1 1 Equate Eq.1 to Eq.2: ,,I
tan50° (8.816):c::50-h --bh1 =--bh2 +21
10.506 '0 50- h 2 2 (85 + x) tan 30 = x tan 60
h = 39.49m b(b + 3)::: b(b- 3) + 42 By sine law:
tyz_ + 3b = ~•. - 3b + 42 85+x = x(~~ 60 )
a N
c
6b = 42
b=7
50

sin 13
300
sin 60
\ tan30
85+x =3x
x =42.5m
_;;;. h 1 =7+3:::1Qunits 13 =8.3

/
,.-<_t\•:
ei
h 2 =7-3=4units Substitute x in Eq.2:


./ i 32° a +p +60 = 180
d. a+8.3+60=180 h = 42.5 (tan 60)
h = "73.61m
N a=111.7

*
B cf I--
' l>A
1000 B ..~···..··
9 + 26 + 58 = 180 ?t
By sine law:
ml
46°20' v 50
-----------

-~-
0=96 sin111.7 :>in8.3

By sine law: sin 111.7


V=--(50)
10
sin.3
sin 96 sin 58 V = 321.8 mph
15
1000

sin 26
,d

sc
BC
BC = 852.'119m

d
Note:
7t = total distance traveled by ship B
10 + 5t = total distance traveled by ship A
• h
e + 61 + 90 + 15 = 180

By sine law:
6=14

sin26 By sine law: sin14" sin61°


852.719
d =: 852.719(sin26 sin 42.35' sin 46"70' r~r 15 X
x = 54.23m
II
I
(j o: :·\7·~ I'll 71 I() I 'ol II·
II
j:
i

lb
ZZ6 . 100 1 Solved Problems tn Engineering Mathematics (2nd Edition) by Tiong & Rojas _ _ _ _ _ _ _ _ _ _ _ _ _ _ _ _ _ _ _D_a..._y_9_-_Pl_a_ne Trigonometry 227


I_
Equate Eq.1 to Eq.2:

120-h h
• / l
~h~h
11
I

----
tan 28' tan 46
5(@+ X X

120-h= - tan28'
- h J·
(
· tan46' 41
120-h =0.513h
tan 40' =a--
6X
h=79.31m h

• s X =m--- ~ Eq.1
By Pythagorean theorem: tan40

~44
h ·I
tan30' = - -
x=J(15) 2 +(20) 2 1144 50+x nil
X= 25ft
15
tane=-
~~ S +X X
h
X =----50~
tan 30 ·
Eq.2

20
Equate Eq.1 to Eq.2:
9=36.87 44


tan60=-
x h h
- - =----50

~h
x =25.4m tan400 tan30"
11h 119175h=1.73205h-50

~&J
44
tan30=-- h = 92.54 ft.

120
100 +X

tan21° = - -
100+x
h

h=(100+x)tan21' ~Eq.1
X

V=-
s
t
S+25.4
s + 25.4 = 76.21
S=50.81m
• 35°

50.81
X h V=·--
tan32' =- 5(60)

~
......---1,20-h
&_j
./"1 h h
x
=x tan 32' ~ Eq.2
V =0.169m/s

X X
Equate Eq.1 to Eq.2: ·
ml
I
120-h X X
tan28 = - -

~~
(100 + x) tan 21' = x tan 32' 3°
X h
120-h
x=--~Eq.1 100+x=x - -
tan 32' J 50
50
(
tan28" tan 21'
h 100 + x = 1.6278x
tan46' =- 50
x=159.286m tan35' = -
x
h Thus: x
x=--~Eq.2 ¥: + 100 = 259.286 m X'"71.407m
tan46"
228 I 00 ["Solved Problems in Engineering Mathematics (2nd Edition) by Tiong & Rojas Day 9 -Plane Trigonometry 229

50-h By sine law:

-~
tan13. = - -
x 150 c
50-h -------- b=30
tan13'' = - - · sin 26° sin 92.75°
71.407 c =341.78m
tan13. (71.407) =50- h
c= 34
h =33.514m
m1 c
A a +b +c 16 + 30 +34
1111 S=
2
-----
2
8=40
Be{ \ ~·- w b vu.. "oA
A= .J s(s- a)(s- b)(s- c)
c=190
Bd "'"¢ < _'_?QC A= .J 40(40 -16)(40- 30)(40 -34)
By cosine law:
A= 240 square units
2 2 2
b =a +c -2ac cosB
Note:
180 2 =130 2 +190 2 -2(130)(190)cosB
Since equilateral triangle, A = B ::: C = 60°
B =65.35'"
r
tan30" = - - By cosine law:
0.5x
10
tan30' = - -
r~ x 2 = a 2 + (c/2) 2 - 2a(c/2)cos B
0.5x 0.5x. x2 = 130 2 + 95 2 - 2(130)(95) cos 65.35'
x =34.64cm x =125m

• c

Bcf ----=c==-:-?-~r::.J_,:' 'o A c=210

By sine law: a+b+c 195+157+210


S= =-----
150 300 2 2
sin26' sinB s =281
B = 61.2!5
A+B+C=f80 A= .Js(s-a)(s-b)(s-c)

26 +61.25 +C = 180 A= .J281 (281-195)(281-157)(281- 210)

c =92.75 A= 14,586.2 square units


"'l
!IIi

232 100 !'Solved Problems in Engineering Mathematics (2nd Edition) by Tionsz & Rojas il

I
Topies
[]Mon
Spherical Trigonometry
Great Circle & Small Circle
Poles and Polar Distance
0
,, < • ~ • \ '

Spherical Wedge & Lunes


I

' , •. .,. ..~ ~- ·< -'· :v "~ >

·,. .
~.
I 101 0
---'
Tue

Wed
Propositions of Spherical Triangle
Solutions to Right Spherical
Triangle
Napier's Rules
I
[ii

'r:

r
Theory

[j [QJ Quadrantal Spherical Triangle


Solutions to Oblique Spherical
:!11
',

- .- Problem<; Thu Triangle

·. ' ~ 0
Solutions
0 Fri
Law of Sines & Cosines
Area of Spherical Triangle
Terrestrial Sphere
111,1
I

0
Notes
0 Sat
GMT & UTC
Latitudes and Longitudes
'I
:11

What is SQherical Trigonometry?

Spherical Trigonometry is the part of


trigonometry which deals with triangles on
the sphere. The object of this subject is to
study the relations connecting the sides
·,,._,..,,,., :I and angles of a spherical triangle'. This is Circle A
I of great importance for calculations in
·' astronomy and navigation.

i
1{
c'
What is a Great Circle'?

Great circle is a circle whose center


coincides with the center of the sphere On
the surface of the sphere, the closest
I
.I.
II Circles A and circle B are example of
analogue to straight lines is great circles.
A great circle is a circle on the surface of a
sphere that has the same diameter as the
great circles because their centers
coincide with the center of the sphere. The
m~idians and the equator of the earth , .,
~ !I

I
sphere, dividing tl1e sphere into two equal are also examples of the great circles. ~~.~~ I,
I!
hemispheres. A great circle is the
intersection of a sphere with a plane going Great circle paths are used by ships and
aircmfts where currents and winds are not :lillii
tluouqh its center. J\ great circle is the
l<ifl)"~,t circlr' tlrdt c::rrr ht~ dr:1wn on CJI)ivrm
a siqnificant factor. For aircraft traveling
·;plrr·rr• w<>;l\;rly between r:on11nents in the
234 1001 Solved Problems in Engineering Mathematics (2nd Edition) by Tiong & Rojas Day I 0 -Spherical Trigonometry 235 '111111111'11

northern hemisphere these paths will What is a Polar Distance? What is right spherical triangle? I

extend northward near or into the arctic ill I


I
region, while easterly flights will often The polar distance of a circle is the least A right spherical triangle is one with a
fly a more southerly tract to take distance on a sphere from a point on the right angle.
advar>+age of the jet stream. circle to its pole. The polar distance of a
great circle is goo. A spherical triangle with two right angles is
What is a Small Circle? ;!, called birectangular spherical triangle
r:
while the one with three right angles is
A small circle of the sphere is the circle known as trirectangular spherical
constructed by a plane crossing the •
sphere not in its center. Circle C belowis
an example of a small circle. '
j triangle.
A
:I

Center of Sphere

What are the important propositions of I

a spherical triangle?
B il
The following are the important proposition
of a spherical triangle: C a
1. If two angles of a spherical triangle What are the solutions to right
are equal, the sides opposite are spherical triangle? I
p1 is the polar distance of the small circle equal; and conversely.
and P2 is the polar distance of the great A right triangle can be solved by using the
circle. P2 is an arc that measures goo. 2. If two angles of a spherical triangle Napier's Rules.
are unequal, the sides opposite are
What is a Spherical Wedge? unequal, and the greater side lies Napier circle (sometimes called Neper's
opposite the greater angle; and circle or Neper's pentagon) is a mnemonic
Two diametral planes that form an angle conversely. aid to easily find all relations between the
divided the sphere into four spherical angles and sides in a right spherical
What is a Pole?
wedges, opposite wedges being ,II' I 3. The sum of two sides of a spherical triangle.
congruent. The surfaces of the we_dges are triangle is greater than the third .side.
A line through the center 0 of a sphere
called sphericallunes or digons.
perpendicular to the plane of a circle on
the sphere cuts the sphere into two points a+b >C
tune
called poles.
4. The sum of the sides of a spherical
Pole triangle is less than 360 degrees.

0° < a + b + c < 360°

5. The sum of the angles of a spherical


triangle is greater than 180 degrees
and less than 540 degrees. Napier's Rules:

Hule no. 1 (Tan-ad Rule):


180° < A + B + C < 540" The sine of any middle part is equal to the
product of the cosines of the opposite
What is a Spherical Triangle? 6. The sum of any two angles of a parts.
spherical triangle is less than 180
Spherical triangle consists of three arcs degrees plus the third angle. Rule no. 2 (Co-op Rule):
Pole
of great circles that form the boundaries of The sine of any middle part is equal to the
a portion of a SP.herical surface. A +B< 180° +C of
product the tangent of the adjacent
parts.

j
Z36 1001 Solved Problems in Engineering Mathematics (2nd Edition) by Tiong & Rojas Day 10 -Spherical Trigonometry 237

How to apply the Napier's Rules? sinb "'sinBsinc A. Law of Sines: The equator is the great circle whose
plane is perpendicular to the axis. The
Consider the right spherical triangle below: Important Rules: sina sinb sine equator has a latitude of zero degree.
A I sinA = sinB =sinG
1. In a right spherical triangle an oblique A meridian is half of a great circle on the
angle and the side opposite are of the B. Law of Cosines for sides: earth terminated by the north pole and the
same quadrant. south pole.
cosa =cosbcosc + sinbsinccosA In October 1884, it was agreed upon that
B 2. When the hypotenuse of a right
spherical triangle is less than 90°, the cosb == cosacosc + sinasinccosB the reference meridian is the one that
two legs are of the same quadrant cosc =cosacosb + sinasinbcosC passes through the Royal Greenwich
C a and conversely. Observatory at Greenwich, England. This
meridian is called .the Prime Meridian or
Convert it to its equivalent Napier's circle: 3. When the hypotenuse of a right C. Law of Cosines for angles: sometimes known as the Greenwich
spherical triangle is greater than 90°, Meridian.
one leg is of the first quadrant and the cos A= -cos8cosC + sirt8sinG~cosa.
other of the second and conversely.
cosB = -cosAcosC + sinAsinCcosb
What is a quadrantal spherical triangle? cosC = -cosAcos8 + sinAsin8cosc

A quadrantal spherical triangle is a


spherical triangle having a side equal to What are the formulas for area of
goo. spherical triangle?

If "b" is considered as the middle part, then A A


the opposite parts are c and B while the
adjacent parts are A and a.

Applying rule no. 1:


B ~8
sinb =tan A tan a ~ Eq. 1
C a c
But TanA=tan(90-A)
What Is an oblique spherical triangle? 1tR2E
A=-.-·-·
taoo
Substitute Tan A in equation 1 An oblique spherical triangle is one with
sinb = tan(90- A)tana no angle equal to a right angle. where: R = ~adius of the sphere
=
E spherical excess in degrees The meridian on the side of the earth the
lie opposite to the prime meridian is known
What are the solutions to an oblique E =A+ 8 + C -180°
. 1 as the international date line. This
But tan(90-A)=cotA=-- spherical triangle?
tanA What Is a Terrestrial Sphere? imaginary line on the surface of the earth
. 1 offsets the hours that are added or
smb =--tana A subtracted as one travels east or west
tan A Terrestrial sphere refers to the earth
(though slightly ellipsoid) as sphere with a through successive time zones.
sinbtanA = tana
radius·of 3959 statute miles.
If one crosses the international dateline
Applying rule no. 2: precisely midnight, going westward one
The terrestrial sphere rotates about a
cjiameter called its axis which pierces the skips and entire day while going eastward,
sinb = cosBcosc B .sphere in the north pole and the south the day repeats.
sinb = cos(90- B)cm:(90- c) pole.
c a
But cos(90- B)= sinS
cos{90- c)= sine
l
~
238 1001 Solved Problems in Engineering Mathematics (2"ct Edition) by Tiong & Rojas 1l DayJO- SpliE!rical Trigonometry 239

What is a Greenwich Mean Time (GMT)? A parallel of lati~de or briefly parallel, is


the small circle cut from the earth by a
I
.!
What are Terrestrial Sphere constants? 'Ql:ribia:
Did you know that..: in the ancient times,
-:~
Greenwich Mean Time I GMT) is the plane parallel to the equatorial plane. the number 40 was used to indicate "many
mean solar time at the Royal Greenwich latitude or too many" and the number 1001
: 1minute:
Observatory, in Greenwich, England.
Theoretically, noon Greenwich Mean Time
is the moment when the sun crosses the
31)
N,,rth
(+)
; angle

·~~-·L
'\ ·signifies a kind of "finite infinite" as in
Alibaba and the 40 thieves, Moses leave
his people for 40'days and 40 nights and
Greenwich meridian (and reaches its
highest point in the sky in Greenwich).
····:·:r>~~:··,_.;.,:.~.~·· · :.-:.·.~· \· · · · · ·. . . . the famous Arabian "A thousand and one
nights"
Because of the earth's uneven speed in its \ ''(
elliptic orbit, this even may be up to 16 \1 Nautical mile ®uotr:
minutes off apparent solar time. GMT "To be a scholar of mathematics, you must
became a world time and date standard Equator If 1:. ':1 .• J ..· T'~':'''l I 1
Lf',t o be born with talent, insight, concentration,
because it was used by the Britain's Royal taste, luck, drive and the ability to visualize
Navy and merchant fleet during the and guess"
nineteenth century.
Radius of earth = 3959 statute miles
- Paul Halmos
In January 1, 1972, GMT was replaced as 1 minute on the great circle arc = 1 NM
the international time reference by 1 NM (nautical mile)= 6080 feet
Coordinated Universal Time (UTC}. The
')(!
S·>Ut.L
= 1852 meters
UTC is maintained by an ensemble of 1 statute mile = 5280 feet = 1760 yards
atomic clocks (a much more stable 1 statute mile = 8 furlongs = 80 chains ·
timebase) around the world. · Tl<le longitude of a point is the angular
distance between the prime meridian and The!. statute mile (sometimes known as the
UTC uses a 24-hour system of time the meridian through the point. The international mile) is typically meant
notation, "1:00am" in UTC is expressed as angular measurement ranges from 0° at when the w9rd "mile" is used without
0100, pronounced as "zero one hundred". the prime meridian to +180° eastward and qualification.
Fifteen minutes after 0100 is expressed as -180° westward. A degree of longitude
0115; thirty-eight minutes after 0100 is corresponds to a distance from 0 to 111 The nautical mile is used universally in
0138 (usually pronounced as "zero one km. The degree of longitude computed as aviation, naval and maritime purposes.
thirty-eight"). The time one minute after 111 km times the cosine of the latitude,
0159 is 0200. The time one minute after when the distance ir.> laid out on a circle of
1259 is 1300 (pronounced "thirteen constant latitude. Proceed to the next page for your 1Oth
hundred"). This continues until 2359. One l.Dngitude test. Detach and use the answer sheet
minute later is 0000 ("zero hundred"}, and provided at the last part of this book. Use
the start of a new UTC day. 180 pencil number 2 in shading your answer.

The UTC is also kncwn as "Zulu Time" or GOOD LUCK I


"World Time".

The latitude of a point is the angular ·; •. ! 90


distance of the point from the equator, and
will be considered positive if the point is '. ·/~~- .. ' East
north of the equator and negative if the (+)
point is south of the equator. This angular
measurement ranges from 0° at the
equator to 90° at the poles. One minute of
the of the arc of latitllde is approximately
one nautical mile or 1852 meters. A 0
degree of latitude always corresponds to Prime
about 111 km or 69 miles. meridian

Colatitude is the complement of latitude.


·-----'=D-"'aY.JQ.- SEherical Trig_9~!.Y 24_1

441: Solve for side b of a right spherical c. 90°57'


triangle ABC whose parts are a ::: 46°, c "' D. 98°45'
75° and C =90°
447: What is the spherical excess of <>
A. 74° spherical triangle whose angles are all
Topics B. 68° right angles?
C. 48°

0
tv! on
Spherical Trigonometry
Great Circle & Small Circle .~.·
··'
D. 74°

44:.t: Given a right spherical triangle whose


A.
B.
C.
45"
90°
60°
Poles and Polar Distance parts are a = 82°, b = 62° pnd C = 90°. D 30°

0 Tue
Spherical Wedge & Lunes
Propositions of Spherical Triangle
What is the value of the side opposite the
right angle? 448: ECE Boar4 April 11.997
The area of sphencal tnangle ABC whose
Solutions to Right Spherical

lJ 0
A. 83°30' parts are A= 9::1°40'. B = 64°12', C =
Triangle B. 84°45' i 16°51' and the radius of the spl1ere i~ 100
V'./ed Napier's Rules C. 86°15' rn is:
Theory
D. 85°15'
[Q] Quadrantal Spherical Triangle
Solutions to Oblique Spherical 443: Determine the value of the angle B of
A.
B.
15613 sq. m.
16531 sq. m.
Problems Thu Triangle an isosceles spherical triangle ABC whose C. !8645 sq. m.
=
0
Solutions
0Fri
Laws of Sines & Cosmes
Area of Spherical Triangle
Terrestrial Sphere
given parts are b = c 54°28' and a =
92°30'.

A. 89°45'
D. 25612 sq. rn

449: A spherical triangle has an· area of


327.25 sq. km. What is the radius of the

0Notes
0Sat
GMT & UTC
Latitudes and Longitudes
B.
C.
D.
55°45'
84°25'
41°45'
sphere if its spherical excess is 30°?

A. 20 !\m
B. 22 krn
444: Solve the angle A in the spherical C. 25 krn
triangle ABC, given a = 106°25', c = 42°16' D. 28 krn
436: If Greenwich Mean Time (GMT) is 6 C. 30 minutes
and B = 114°53'.
A.M., what is the time at a place located D. 1 hour
4$0: EE Board. April :1:99•1
30° East longitude?
A. 45°54' A ship on a certain day is at latitude 20°N
439: A spherical triangle ABC has an
B. 80°42' and longitude i 40°E. After sai!ing for 150
A. 7A.M. =
angle C 90° and sides a = 50° and c =
C. 97"09' hours at a uniform speed along a great
B. BA.M. 80° Find the value of "b" in degrees.
D. 72°43' circle route, it reaches a point at latitude
C. 9A.M.
10°S and longitude 170°E. If the radius of
D. 4A.M. A. 73.22
445: Solve for angle C of the oblique the earth is 3959 miies. find the speed in
B. 74.33
437: If the longitude of Tokyo is 139°E and C. 75.44
spherical triangle ABC given, a = 80°, c = miles per hour.
115° and A = 72°
that of Manila is 121°E, what is the time D. 76.55
A 17.4
difference between Tokvo and Manila?
A. 61° B. 15.4
440: Solve the remaining side of the
B. 85° C. 16.4
A. 1 hour and 12 minutes spherical triangle whose given parts are A
C. 95° D. 19.4
B. 1 hour and 5 minutes · = B = 80° and a = b =89° D. 119°
C. 1 hour and 8 minutes
D. 1 hour and 10 minutes A. 158°12'
441&1 Determine the spherical excess of
B. 162°21'
C. 168°31' the spherical triangle ABC given a = 56°, b
438: One degree on the equator of the
earth is equivalent to _ _ in time. D. 172°12' 65" and c 78° =
A :n"33'
A. 1 minute
ll (ill":l7'
B. 4 minutes
Day 10- Spherical Trigonometry 243

Topics
•diff. in til11e =diff. in longitude
sin co-c = cos a cos b
cos c = cos a cos b
cos 80 = cos 50 cos b
b = 74.33''


24 360°
diff. in time 30°- oo
D Mon
Spherical Trigonometry
GreatCircle & Small Circle
-------
24 360"
diff. in time = 2 hours
c
Poles and Polar Distance
D Tue
Spherical Wedge & Lunes
Propositions of Spherical Triangle
Note: The time in the place is 4 hours
ahead of GMT because the place is at the il:·
Solutions to Right Spherical East. Thus, the time is 8 AM.

D D
Theory

D ~
Problems
Wed

Thu
. Triangle
Napier's Rules
Quadrantal Spherical Triangle
Solutions "to Oblique Spherical
Triangle
••
diff. in time , diff. in longitude

24 360°
sin co-B =tan co-a tan c/2
cos f3
1
=cot a tan c/2

D
Solutions Fri
Laws of Sines & Cosines
Area of Spherical Triangle
Terrestrial Sphere
diff. in time
----=~---
24
139°-121 o

360°
cos B =--(tan c/2)
tan a
1
cos 80 = --(tan c/2)
diff. in time= 1.2 hours

D D
Notes Sat
GMT & UTC
Latitudes and· Longitudes
= 1 hour and 0.2(60) min
= 1hour and 12min
tan 89
tan c/2 = cos 80 tan 89

436. B
ANSWER KEY

441. B 446.A c:J 13-15


RATING
Topnotcher

Note: 360 degrees = 24 hours Ill
c/2 =84.26°
c = 168.52" or 168° 31'

A
437. A
438. B
442.
443. D
c 447.8
448. B c:J 9-12 Passer
' time=1 \ 241m;_)
x (- - x [60
- minJ
-.
. 1 ~
439. B
440.C
444.C
445. D
449.
450.C
c c:J b-8 Conditional time =4 minutes
360\

0 0-b Failed

If FAILED, repeat the test.


• A
c a=46°

sin co-c = cos a cos b


cos c = cos a cos b
cos 75 =cos 46 cos b
b=68.12' or68'07'

a=so·

.,,1'1'11111
l '
II
I

•I

It
244. 1001 Solved Problems in Engineering Mathematics (2"d Edition) by Tiong & Rojas;
Day 10 - Spherical Trigonometry 245
'
A cos b = cos a cos c + sin a sin c cos B
cosb = (cos 106 25')( cos 42 16')

+ (Sir 106
(cos114 53')
25' )(sin 42 16' ) • B=72°


E=(A+B+C)-180
E = (93'40' +54· 12' + 116° 51') -180
E =94°43'
c b =11843'
a=82° A
b=65°
c
Using law of sines: 1tR 2 E ~t(100) 2 (94.43')
A=--------
sin co-c = cos a cos b Using law of cosines for sides: 180" 180°
cos c = cos 82 cos 62 sinA sinS
------- A.= 16,531.17m 2
c=86.25° or86'15' cos a= cosb cosc +sin b sin ccosfl,.
sin 106°25' sin b


I


sin A sin 114°53' cos 56 =(cos65 cos 78) ·· ,I,,
----
+ (sin 65 sin 78 cos A)
sin 106°25' sin 118°43'
A A =5T53' I
A=82'51'or ~tR 2 E 1.,
A= - I
A= 97"09' (its supplement) Using law of sines: !',,I
180°

• ~tR
2
sine sinA 327.25 = (30°)
8=72° 180°
sine sin a
c sin C sin 57°53'
R=25km


a=92°30'
sin 78° sin 56°
sin co-B = tan co-c tan a/2 C =>8T52' North

:: \h;~~~·~:~}"N)
cos B = cot c tan a/2
A sinS sinA
1 b
cos B =--(tan a/2) I
Using law of sines: sinb sina I,
tan c ,II
sin B sin 57" 53'
1 [ t a92'30') sinC sinA

u
cosB= n-- "
sin65" sin 56"
tan 54'28' 2 i:li·llll.i'
sine sina 8=67"48' \···... _ C B(170J'E, 10°8) 1'.1
B =41"75'or41"45' sin C sin 72° 'i

~>b-l
E = (A + B +G) -180
sin 115° sin 80°
II 8=114°53' C=61"04'or E =(5T53'+67 48' +8T52')-180
E=33"33' South
C = 118 ° 56' (its supplement)

A
b
Note:
Since side c is greater than side a then
angle C is greater than angle A. Thus use
c = 118°56'

E =(A+ B + C)-180
E=(90 +90' +90 )-180
A

E=90 B
Using law of cosines for sides: C a=3o•

cos b = cos a cos c + sin a sin c cos B

;11:

,,
246 100 1 Solved Problems in Engineering Mathematics (2nd Edition) by Tiong & Rojas

sin co-c = cos a cos b


cos c = cos 30 cos 30
c =41.4096°

1\
C=41.4096\ X ( 60NMJ
Topics
c = 2,484.58 NM
v
distance
time
D Mon
Spherical Trigonometry
Great Circle & Small Circle
Poles and Polar Distance
V
2484.58
150
.
= 16.56NM/hror D Tue
Spherical Wedge & Lunes
Propositions of Spherical Triang~e
V = 16.56 knots Solutions to Right Spherical
D D
Theory Wed
Triangle
Napier's Rules

D ~
Quadrantal Spherical Triangle
Solutions to Oblique Spherical
Problems Thu Triangle

D D
Solutions Fri
Laws of Sines & Cosines
Area of Spherical Triangle
Terrestrial Sphere


Notes Sat
GMT & UTC
Latitudes and Longitudes

"' ,A,,,;,

,,.•,,-o·,.., , , ,,. ' ····'00 '*'"'"•"< ,.,.

? ,, ~ ·' ' ,, ;; ' ·,. •. • • . ~ < < '~ ~ <


248 100 l Solv:ed Problems in Eng!!'!~~g Mathematics (2nd Edition) by Tiong; & Rojas

E'ii'IW I

Topics

0 Mon
Analytic Geometry
Rectangular Coordinates System
'I ~ ..: ;,, ~ < '
Distance Between Two Points
~· 0'

0 Tue
Slope of Line
Angie Between Two Lines
Distance BetWeen a Point & a Line
~il
~J 0
Theory V\fed
Distance Bel.\-IJeen 2 Parallel Lines
Division of Line Segment

l] 0
Problems Thu
Area of Polygons by Coordinates
Equations of Lines
Conic Sections or Conics

0 ~
Solutions Fri
General Equation of Conics
Geometric Properties of Conics
Equations of Circles

0 0
Notes Sat

'
Wha!Js Analytic Geometry?

Analytic geometry deals with geometric


problems using coordinates system Second Quadrant First Quadran!
thereby converting it into algebraic P(5,3)
problems. abscissa

Rene Descartes (1596- 1650, Cartesius 2 ordinate
in Latin language) is regarded as the
founder of analytic geometry by 0
introducing coordinates system in 1637. I I I t--4>~---t I I l•x
-4 -3 -2 -1 2 3 4 5
-1
Rectangular Coordinates System (Also
known as Cartesian Coordinates System)

Th;ro ""''""' ~ I Fomth Qoed"ot


250 1001 Solved Problems in Engineering Mathemati<;:s (2"d Edition) by Tiong & Rojas Day 11 -Analytic Geometry (Points, Line11 & Circles) 251

Z· A line parallel to the x-axis has a slope of (Xt, Yt)


Point 0 is the origin and has coordinates
(0,0). The x-coordinate or ~bscissa is
zero while a line parallel to the y-axis has •...
always measured from the y-axis while the
y-coordinate or ordinate is always
a slope of infinity {co).
"······-~-~
mei;lsured from the x-ax1s. The point P has
(X2, Y2, Z2) For parallel lines with slopes of m1 and m2,
respectively, the slopes are the same.
5 and 3 as abscissa and ordinate,
respectively. ~--~--~--~--~x Ax+By+C=O
m~=m2
What is distance between two points in
i elane?
For perpendicular lines with slopes of m,
y d=Ax +By1+C
Consider two points whose coordinates ,
are (Xt, y,) and (x2, y2), respectively. A
d = J(x2- x1)
2 2
+ (Y2- Y1) + (z2- z1)
2
and m2, respectively, the slope of one is
the negative reciprocal of the other. 4
± A~ +82
right triangle is formed with the distance
between two points being the hypotenuse 1 use
m2 =--.
m, + if B is positive and the point is above.
of the right triangle.
the line or to the right of the line
What is slope of a line (m)? I + if B is negative and the point is below
y
(x2, Y2) What is angle between two lines? the line or to the left of the line
Consider two points whose coordinates

.~//./'!'-
If otherwise
are (x,, y1) and {x2. y2}, respectively. A line
Consider two lines with slopes of m, and
is formed by connecting the two points.
The slope of the line is defined as the rise m2.
What is distance between two.Qarallel
{vertical) per run (horizontal) Line 2
··········· lines?

(XI, Y1
) I... y
Consider two parallel lines with equations
X2- XI •I
as shown in the figure.

d ..·
~---+-----------------------X Line 1
...···········'
~~
,l The angle, e between these lines (line 1
(X1, Y1) X2- X1 1: and line 2) may be calculated using the
Using Pythagorean theorem, the distance
between two points can be calculated ~
,j
. following formula:
using:
----~-----------------------X m2 -m~
Ax;t-By+C1 =0
d;:J{; -xt) 2
+(Ya -Y,) 2
J tane "' 1 + m m
2 1

t~
The (perpendicular) distance, d, between
This formula is known as the distance · the two lines is:
rise t:..y
formula. slope=m = - =--
run t:..x
where: !!.. denotes an increment c,":'c2
±.JA2 +B~
What is distance between two I?Oints in
d=
space?
tane = Y2 - Y2
x2
I What is distance between a point and a
!!.!!!1
Consider three axes namely x,. y and z and
two points with coor<;linates (x,, Yt. Zt) and
-x~
,I
'.',
Consider a point with coordinates (x1, y,)
Use the sign (either+ or-) that would
make the distance positive.
(x2. y2, z2), respectively. But tane = m \ and a line with equation Ax + By + C = 0.

m = Y2 -y2
x2 -x~


I
252 1001 Solved Problems in Engineering Mathematics (2nd Edition) by Tiong & Rojas Day 11 -Analytic Geometry (Points, Lines & Circles) 253

What are the coordinates of a point that E. Intercept Form: Circle is produced when the cuttiryg plane
(XJ, Yt) is parallel to the base of the cone.
divides a line segment?
~+1'.=1
Consider two points with coordinates (x,, a .. b Ellipse is produced when the cutting
y,) and (x2. y2). The line segment formed plane is not parallel (o~ inclined) to the
by these two points is divided by a point P base of the cone.
whose coordinates are (x, y). What is a Conic Section?
~z.Y0 ~~y~ Parabola is produced when the cutting
Let r, and rz be· the corresponding ratio of Conic section (or simply Conic) is the plane is parallel to the element (or
its length to the total distance between two The arrow shown in tire figure moving in a locus of a point which moves so that its generatrix) of the cone.
points. counterclockwise direction ir1dicates that distance from a fixed point (focus) is in
the vertices must be written in the equation constant ratio, e (eccentricity) to its Hyperbola is produced when the cutting

Y./\ . . (h Y2)
below in counterclockwise direction.
.~ ..~ .~
distance from a fixed straight line
(directrix) .
plane is parallel to the axis of the cone.

What is the General Equation of a


A-~lx, . ~2·.··/~3 )'1r·· The term "conic" was first introduced by a Conic Section:
,\ ~P(x,y} - 2 Y1/ Y2"1_-a ~Y1 . . ~ renowned mathematician and astronomer

// of antiquity, Apollonius (c.255- 170 B.C.)


Also, the term ~conic section" was due to Ax2 + Bxy + Cy 2 +Ox+ Ey + F =0
(x,, y,) A= .!((XWz +i<2Y3 +x3y,) 1 the fact that the section is formed by a
plane made to intersect a cone.
2 -(y,x2 +y2x3 +yax,)j Circle When B is not eq·ual to zero, then the
The abs.cissa of the point, P is: principal axes of the conic are inclined (not

K= (Xl2)+(XifQ
r1 + r2
What is line?

A line is defined as the shortest distance


$ parallel to the coordinates axes). The
curve can be identified from the equation
given by determining the value of the
determinant; 8 2 - 4AC.

$
between two points. The followi'ng are the
The ordinate of the point, P is: equa~ons of the lines: 8 2 -4AC Conic Eccentricity
Section
A. General Equation: <0 Ell~ < 1.0
Y"' (y,r2)+(Y:l1) Ellipse
=0 Parabola = 1.0
r1 +r2 >0 Hyperbola > 1.0
Ax+By+C=O
If the point, P is at the mid-point of the line When B is equal to zero, then the principal
segment, then the abscissa and ordinate B. Point-Slope Form: axes of the conic are parallel to the
of the point are the following: coordinates axes (x and y axes). To '
y-y,=m{x-x 1) identify the curve, compare the
x, +X2 and y:::::Y2+Y1 coefficients of A and C.
X= 2 2 Parabola
C. Slope-Intercept Form: If A = C, the conic is a circle.
*
If A C but the same signs, the conic is an

What is the formula for the area of a


polygon using the coordinates of its
vertices?
y=mx+b

D. Two-Point Form:
1E ellipse.
If A and C have different signs, the conic
is a hyperbola.
'If either A or C is zero, the conic is a
Hyperbola parabola.
Consider a polygon whose vertices have
coordinates of (x,, y,), (Xz. Yz) and (x3, y3). _ Yc y, (x ... x,)
y-y,-·xz-x,
~~
The conic sections have geometric
properties that can be used for some
engineering application such as beams of
sound and reflection of rays of light.
254. 100 1 Solved Problems in EngineeljC,Q Mathematics (2nd Edition) by Tiong & Rojas Day 11 -Analytic Geometry (Points, Lines & Circles) 255
Center (h,k)
Circle reflects rays issued from the focu;s: If D & E = 0, center is at the origin (0,0)
back to the center of the circle. · ·
If either D or E, or both D & E "" 0, the h"' -D k= -E
center is at (h,k). 2A 2A

2. Standard Equations: Radius ( r)


y
C(O,O)
D2 +E 2 -4AF
r = .1---.--
Parabola reflects rays issued from the 4A 2
focus as a parallel (with respect to I~ I ~I/ ;• I X
axis) outgoing beam.

Proceed to the next page for your 11th


test. Detach and use the answer sheet
x2 *·Y2 =rt· provided at the last part of this book. Use
pencil number 2 in shading your answer.
C(h,k)
GOOD LUCK I
Ellipse reflects rays issued frorn the
y
focus Into the pther focus.
'(ll;ribia:
Did you know that. .. nature seems to

8···/······~
know its math and never wastes its ·
'•.,
resources! The reason why bubbles are
k spherical because with this shape, bubbles
can enclose the most volume with the
0 i X least material. This was first shown by
Archimedes.
Hyperbola reflects rays issued from the
focus as if coming from the o~er h <!euote:
focus. "Can we actually know the universe? My
2
{-J(-.h) +(y-kf = r2 God, it's hard enough finding your way
around Chinatown!"
When the equation given is general
equation rather than standard equation, -Woody Allen
the center (h,k) of the circle and its radius
(r) can be determine by converting the.
general equation to standard using the
process known as completing the
What Is a Circle? square. Or using the following formulas:

A circle is a locus of a point that which General equation:


moves so that it is equidistant from a fixed
point called center.
·Afi!.· +Qy2 +'Ox +Ey + F =0
1. General Equation:

J!- +Yt +DX+Ey+F = 0


Day 11 -Analytic Geometry (Points, Lines & Circles) 257

45•1 If (-2,-4) is the midpoint of (6.-7) and 4•2: Find the angie formed by the lines 2x
(x,y), then the values of x and yare =
+ y -8 = 0 and x + 3y + 4 0.

A x= 2, y =1 A. 30"
B. X= -10, y =-1 B. 35"
Topics C. X:: 10, y = -1 c. 45°
0. X:::; =
·8, y -1 D. so·
D Mon
Analytic Geometry
Rectangular Coordinates System 45'71 ECE Board November 1998
Determine the coordinates of the point
4•:J• Find the angle between the lines 3x +
2y =
6 and x + y = 6.
Distance Between Two Points
D
which is three-fifths of the way from the
Slope of Line point (2,-5) to the point (-3,5). A. 12•20'
Tue Angle Betw~en Two Lines B. 11•19'
Distance Between a Point & a Line
A. (-1,1) c. 14°25'

D D
B. (-2,-1) D. 13.06'
Distance Between 2 Parallel Lines c. (-1,-2)
Theory Wed Division of Line Segment D. (1,-1) 4•4a What is the acute angle between the

Problems
D
Thu
Area of Polygons by Coordinates.
Equations of Lines
Conic Sections or Conics
4S8a ECE Board April 1998
The segment from (-1 ,4) to (2,-2) is
extended three times its own length. The
lines y·= 3x + 2 andy= 4x + 9?

A.
B.
4.4"
28.3"

D ~
General Equation of Conics terminal point is C. 5.2c
Geometric Properties of Conics D. 18.6"
Solutions Fri A. (11,-24)
Equations of Circles B. (-11,-20)
[] D
Notes Sat
c.
D.
(11,-18)
(11,-20)
465: EE Board October 1997
Find the distance of the line 3x.+ 4y
from the origin.
=5

4S9t The points (a, 1), (b,2) and (c;3) are A. 4


collinear. Which of the following is true? B, 3
451: ECE Board A.pril :t999 C. 19 c. 2
The linear distance between -4 and 17 on D. -5or19 A. c-b=c-a D. 1
the number line is B. c-b b-a =
454: Find the coordinates of a point C. c-a =·a-b 4 ... CE Board November :1'!'92
A.
B.
13
21
equidistant from (1, -6), (5, -6) and (6, -1 ). D. c-a = b- a The two points on the lines 2x = 3y + 4 0 =
which are at a distance 2 from the line 3x +
c. --17 A. (2, -2) 4601 If the slope of the line connecting the 4y-6 = 0 are
D. -13 B. (3, -2) origin and point P is 3/4, find the abscissa
c. (3, -3) of P if its ordinate is 6. A. (-5, 1) and (-5,2)
452: EE Board Apri!l 1.994 D. (2, -3) B. (64,-44) and (4,-4)
Find the distance between A (4,-3) and B A. 2 C. (8,8) and (12, 12)
(-2,5). 455: lEE Board April 1995 B. 6 D. (44,-64) and (-4,4)
The line segment connecting (x,6) and c. 7
A. 11 (9,y) is bisected by the point (7,3). Find the D. 8 4•71 CE Board November :1992
B. 9 values of x and y. The distance from the point (2, 1) to the
C. 10 4••a ECE Board April 1999 =
line 4x- 3y + 5 0 is ·
D. 8 A. 14,6 Find the inclination of the line passing
B. 33, 12 through (-5,3) and (10,7). A. 1
453: If the distance between points (3,y) C. 5, 0 B. 2
and (8,7) is 13, then y is equal to D. 14,6 . A. 14.73 c. 3
B. 14.93 D. 4
A. 5 or-5 C. 14.83
B. ·5 or 19 D. 14.63
258 . 100 1 Solved Problems in Engineering Mathematics. (2nd Edition) by Tiong & Rojas
Day 11 -Analytic Geometry (Points, Lines & Circles) 259
4681 CE Board November 1996 47~1 ME Board April 1998 A . ., -4 =X 484: Find the equation of the line passing
Determine the distance from (5, 10) to the Find the slope of the line defined by y - x = B. y =-X - 2 through the origin and with a slope of 6?
linex- y = 0. 5. C. y =X- 4
-D. y- 2 =X A. y- 6x =0
A. 3.33 A. 1
B. y = -6 l
B. 3.54 B. 1/4 479: EE Board October 1997 C. X+ y = -6
c. 4.23 C. -1/2 What is the x-intercept of the line passing D. 6x + y =0
D. 5.45 D. 5+x through (1,4) and (4,1)?
485: Find the equation of the line if the x-
4691 The distance from a point (1 ,3) to the 4741 CE Board November 1995 A. 4.5 intercept and y-intercept are -2 and 4,
line 4x + 3"y + 12 = 0 is What is the slope of the line 3x + 2y + 1 = B. 5
0? . respectively.
c. 4
A. 4 units D. 6 A. y- 2x-4 =0
B. 5 units A. 3/2
B. y + 2x-4 =0
C. 6 units B. 2/3 480: ME Board April1997 C. y- 2x + 4 =0
D. 7 units C. -3/2 Find the equation of a straight line with a D. y + 2x + 4 =0
D. -2/3 slope of 3 and a y-intercept of 1.
4701 CE Board May 199:1
486: ECE Board April 1998
Find the distance between the given lines 4751 ECE Board November 1990 A. 3x + y- 1 =0
4x- 3y = 12 and 4x- 3y -8.= In a cartesian coordinates, the vertices of B. 3x- y + 1=0
Determine B such that 3x+ 2y ·- 7 = 0 is
perpendicular to 2x- By + 2 = 0.
a triangle are defined by the following C. X+ 3y + 1 =0
A. 3 points: (-2,0), (4,0) and (3,3). What is the D. X- 3y- 1 =0 A. 5
B. 4 area? B. 4
c. 5 481: ECE Board April1999 C. 3
D. 6. A. 8 sq. units If the points (-2,3), (x,y) and (-3,5) lie on a D. 2
B. 9 sq. units straight line, then the equation of the line is
4711 EE Board April 1995 C. 10 sq. units
487: The line 2x - 3y + 2 = 0 is
Find the distance between the lines, 3x + y D. 11 sq. units
perpendicular to another line L 1 of
- 12 = 0 and 3x + y - 4 = 0. A. X- 2y -1 = 0 unknown equation. Find the slope of l.,.
4761 EE Board April 1994 B. 2x + y -1 = 0
16 Given three vertices of a triangle whose C. X+ 2y -1 = 0
A. A. 312
J10 coordinates are A (1, 1), B(3,-3) and D. 2x + y + 1 =0 B. -3/2
(5,-3). Find the area of the triangle.
C. 2/3
12 48ZI ME Board April :1998
B. D. -2/3
A. 3
J10 B. 4
The equation of a line that intercepts the x-
4 axis at x = 4 and the y - axis at 488: A line through (-5,2) and (1 ,-4) is
c. C. 5 y =- 6 is, perpendicular to the line through (x,-7) and
J10 D. 6
(8,7). Find X.
8 A. 3x + 2y = 12
D. 4771 ECE Board November x99o B. 2x- 3y = 12 A. -4
J10 In a cartesian coordinates, the vertices of C. 3x- 2y = 12 B. -5
a square are: ( 1,1 ), (0,8), (4,5) and D. 2x- 3y = 12 C: -6
47:&1 ME Board October 1996 (-3,4)~ What is the area?
D. -19/3
What is the length of the line with a slope 483: A line with an inclination of 45•
of 4/3 from a point (6,4) to they-axis? A. 20 sq. blnits passes through (-5/2,-9/2). What is the x- 489: CE Board May 1996
B. · 30 sq. units coordinate of a point on the line if its What is the equation of the line that
A. 10 C. 25 sq. units corresponding y-coordinate is 6? passes thru (4,0) and is parallel to the line
B. 25 D. 35 sq. units X- y- 2 = 0?
C. 50 A. 6
D. 75 478: EE Board April1997 B. 7 A. X- y + 4 = 0
A line passes thru (1 ,-3) and (-4,2). Write C. 8 B. x+'y+4=0
the equation of the line in slope-intercept f) <) C. X .... y --4 =.- 0
form.
D X y 0
260 100 l Solved Problems in Engineering Mathematics (2"d Edition) by Tiong & Rojas

49e: Find the equation of the line through 49&: CE Board May 1.997
point (3, 1) and is perpendicular to the line Find the slope of a line having a
X+ 5y + 5 = 0. parametric equation of x =2 + t and y = 5
-3t.
A. 5x- 2y = 14
B. 5x- y = 14 A. 2
C. 2x- 5y = 14 B. 3 Topics
D. 2x + 5y = 14 C. -2

491.: Find the equation of the


perpendicular bisector of the line joining
D. -3

497: CE Board May 1.998


0Mon
Analytic Geometry
Rectangular Coordinates System
(5,0) and (-7,3) Find the slope of the line having a Distance Between Two Points
A.
B.
8x + 2y + 11 = 0
8x - 2y + 11 = 0
parametric equation y = 4t + 6 and x = t +
1. 0Tue
Slope of Line
Angle Between Two Lines
C.
D.
8x - y + 11 = 0
8x + y + 11 = 0
A.
B.
C.
1
2
3 0Theory
0
Wed
Distance Between a Point & a Line
Distance Between 2 Parallel Lines
Division of Line Segment
492: Which of the following lines is parallel D. 4
to the line 3x- 2y + 6 = 0?

A. 3x + 2y - 12 = 0
498: ECE Board Aprill:999
Two vertices of a triangle are (2,4) and(-
0
Problems
0Thu
Area of Polygons by Coordinates
Equations of Lines
Conic Sections or Conics
B. 4x- 9y = 6 2,3) and the area is 2 square units, the
C.
D.
12x + 18y = 15
15x-10y-9=0
locus of the third vertex is [IJ General Equation of Conics
Geometric Properties of Conics
A. 4x- y = 14
Solutions Fri Equations of Circles
49:J: The equation of the line through (-3,-
5) parallel to 7x + 2y - 4 = 0 is
B.
C.
D.
4x + 4y = 14
X+ 4y = 12
x-4y=-14
0Notes
0Sat
A. 7x + 2y + 31 = 0
B. 7x - 2y + 30 = 0 499: ECE Board April 1.998
C. 7x + 2y - 4 = 0 Find the area of the triangle which the line
D. 2x + 7y + 30 = 0 2x - 3y + 6 = 0 forms with the coordinate
axis. ANSWER KEY RATING
494: What is the equation of the line
joining the points (3,-2) and (-7,6)? A. 3 451.B 464.A 477.C 490. B c:J 43-50 T~pnotcher
A. 2x + 3y = 0
B.
C.
4
5
452.C
453.D
465. D
466. B
478. B . 491. B
479. 8 492. D c:J 33-42 Passer
B.
C.
4x- 5y = 22
4x + 5y = 2
D. 2 454.C
455.C
467. 8
468. 8
480. 8 493.A
481. D 494.C
c:J 25-32 Conditional
D. 5x + 4Y = 7 soo: ECE Board November 1998
A line passes through point (2,2). Find the
456. B
457.A
469. 8
470. 8
482.C 495. D
483. C . 496. D
0 0-25 Failed

49S: What is the equation of the line equation of the line if the length of the !ine· 458. D 471. D 484. A 497. D IfFAILED, repeat the test.
passing through (-2,6) with the x-intercept segment intercepted by the coordinates 459. B 472. A 485. 8 498. D
half the y-intercept? axes is the square root of 5. 460. D 473.A 486.C 499.A
461. 8 474.C 487. B 500. 8
A. x-y=6 A. 2x+y-2=0 462.C 475. 8 488.C
B. 2x + 2y + 2 = 0 B. 2x - Y,- 2 = ·o 463.8 476. 8 489.C
C. 3x- y + 2 = 0 C. 2x- y + 2 = 0
D. 2x + y- 2 = 0 D. 2x + y + 2 = 0
262 l 00 l Solved Problems in Engineering Mathematics (2nd Edition) by Tiong & Rojas
Day 11 -Analytic Geometry (Points, Lines & Circles) 263

ml ml Let
4 0 17
Xm and Ym. the coordinates of the midpoint
Iii P2(-3,5)
- x, +X.2
'----y---__) Xm- __

2.1 2
The linear distance from -4 to 17 is equal 7= x+9
to 21. 2
x=5

P2(-2,5) (1 ,~6) (5,-6) Ym = Y1 + Y2


2
d, =d2 3= 6+y
J'"(x---1)::--
2 +-(-y-+6) 2 = Jr-(x---5)7"
2 -+(-y-+-6)7'
2
2 P,(2,-5)
y=O
(x-1) + ~ =(x-5) +~
2 2

P,(4,-3)
'JR.- 2x + 1 ='JR. -1 Ox+ 25
8x=24
x=3

d2 ::: d3
• 2 4 6 8
X

X=
x 1 r2 + x2r1

-1
r1 +r2

y= Y1r2 +y 2 r1
2(2/5) + ( -3)(3/5)
-~~~~~

(3/5)+(2/5)

= -5(2/5)+5(3/5)
J,-(x---5--=)2:-+_(_y-+-6)-=-
2 = J,-(x---6--=)2:-+-(-y-+-1)"""
2 r1 + r2 (3/5) + (2/5)
Using distance formula:
y =1
2


2 2 2
(x-5) +(y+6) =(x-6) +(y+1)
d=~(x2 -x, )2 +(y2 -Y, )2
P,(6,-7)
d=J(-2~4} 2 +(5+3) 2 Substitute x = 3:
d=10 P,(-1,4)


2 2 2 2 Let:
(x-5) +(y+6) =(x-6) +(y+1)
Xm and Ym. the coordinates of the midpoint
2 2 2 2
(3-5) +.(y+6) =(3-6) +(y+1)

4+ ) \ +12y+36=9+ ) \ +2y+1 Xm = x, +x2


P1 (x 1 , y 1 ) = (3, y) 2
10y::: -30
P2 (x 2 ,y 2 )=(8,7) • ::: 6+ X
y =-3 2
Using distance formula:
2
Thus the pointis at (3, -3) X =-10
x1 r2 + x2 r1
d=~(x2 -x, )2 +(y2 -y, )2 x-
13 2 =(8-3) 2 +(7·y) 2
mJ P,(x,6)
Ym = y, + Y2 r1 +r2
6 2
169:::25 + 49 -14y + y 2 ::: -'-(--'-1)("'--3-'-)_+
d _:X2:..:(d-'-)
4 -7 + y 2
2 -4= - d+3d
O=y -14y··95
2
-3'li.+'Cl_x 2
O=(y-19)(y+5) I I o I I I 10.-1 1---.. y::: -1 2=---
y=19or 2 4 6 4'Cl_
P2(9,y)
y =·-5 X1 =11

~
. ' .
264 . 100 I Solved Problems in Engirieering Math:emaacs (a" Edition) by Ttong~ Rojas Day ll -Analytic Geometry (Points, Lines & Circles) 265

y= Y1r2. + Y2r1

-2=
r1 +r2
4(3d)+y2(d)
d+3d
.
Substitute:

3 6-0
----
4
3x=24
x-0

3x+2y=6

Y=~-x+3
3
-2= 12M_+ M,y2 2
x=8 m1 =-3/2
4)(
Y2 =-20
a x+y=6
y=-x+6

• _ ry<c.3)
'Y (b,2)
Let 9 = angle of inclination

Note: P1 (-5,3); P2 (10,7)

Y2 -y, 7-3
m=--=--=4/15
m 2 =-1

Let: 9 = angle ~tween the two lines

9 =tan·1 m2 -m 1 )
d= A(xt)+B(y 1 )+C
__:..±....:..J.:.,:A=:2::::+::8=;:2:--

d= 3(0)+4(0)-5 = -5
t-13 2 +4 2 5
[ 1+m m
x 2 -x 1 10+5 1 2 =
d (-)1
tan9=m -1- (-3/2) )
9=tan -1 ( Note:
9 = tan· 1 ( 4/15) . 1 + (-3/2)(-1) The negative sign only denotes that the
9 = 14.93' point is below',the line.


9=11.3099" or 11" 18' 35"


Given two points, the slope of the line is,

m=Y2·Y1 Ill
x2 -x1 2x+y-8=0
Y=~x+2
y = -2x +8 m1 =3
Since collinear, m1 = m2.
m1 =-2
y=4x+9
m 1 =m 2 x+3y+4 =0 m2 =4
2-1 3-2 1 4
--=-- y=--x-.-
b-a c-b 3 3 Let: 9 = angle between the two lines
1 1 m 2 = ·113
--=--
b-a c-b
Let: 9 = angle between the two line£!
9=tan·1[ m2 -m, )
c-b=b-a 1+m1m 2


whenx~O.y = -J.JJ

-m~
4 3 when y=O, x
= • 2.0
9=tan·1[ m2 ) e =tan· 1
( -----)
1+m,m2 1+(4)(3)

m=Y2·Y1
'9=4.398'" +d, =A(x 1)+B(yd+C.
9=tan·1 [ -113-(-2) ) .JA2 +82
X2 -x 1
9=45
1+ (-~)(-1/3)
Ill 2= .
3(x 1)+4(y 1 )+(-6)
./32 +42
P1 (0,0)-+x 1 =Oandy 1 =0
Givenline:3x +4y-5 =0 10=3x1 +4y 1 -6
P2 (x,6)-+X 2 =xandy 2 =6
A =3;B=4;C::: -5 16=3x1 +4y 1·
m=3/4
Given point: (0,0) y1 :t 4-0. 75x 1 -+ Eq.1
XI "'0;y1 tt:Q

J
266 . .I 001 Solved Problems in Engineering Mathematics (2"d Edition) by Tiong & Rojas Day II -:-Analytic Geometry (Points, Lines & Circles) 267

2x 1 +3y 1 +4 =0 --)Eq.2

Substitute Eq.1 in Eq.2:

2x 1 +3y 1 +4=0
[4x- 3y + s =
~
whenx~O.y ~1.67
!Q
· when y-o. x "" -/.25
d

P(2,1)

Given line: 4x + 3y + 12 = 0
A= 4; 8 =3; C = 12
•Given line 1: 3x + y -12 = 0
A=3;8=1;C 1 =-12
2x 1 +3(4-0.75x 1 )+4;::0 Given point: (1,3) Given line 2 : 3x + y - 4 =0
x 1 =1;y 1 =3 A= 3; B = 1; C 2 = -4
-0.'25x 1 =-16
x 1 =64
y 1 =4-0.75(64) A(x 1)+8(y 1)+C
d= c 2 -C 1 =--4_-(_-1_2_)
Y1 = -44 d = _..:__~===--­ _j A2 +82 J32;"12
±-JA2 +82
8
4(2)+(-3)(1)+5 d=--units
Thus, the first point is at (64,- 44)
d- --'--7::=====- ../16
-~42 +(-3)2
A(x 1)+8(y 1)+C
-d1 = -.J-,.A=::=2=+=8::.=2=--

_2 = 3(xd+4(yd+(-6)
10
d=-
-5
d = (-) 2 units
a 4


2
-)32 +42
-10=3x 1 +4y 1 -6
A(x 1)+8(y 1)+C
-4=3x 1 +4y 1 d =--'--';::===='==-- (0,-y)
Y1 =-1-0.75x 1 -+Eq.3 ±-JA2 +82
Given line : x - y = 0
4(1) + 3(3) + 12
Substitute Eq.3 in Eq.2:
A=1;8=-1;C=O d= __:_'--========- Using point slope form:
Given point: (5,1 0)· +J42 +32
x;=5;y 1 =10 25 y- y 1 = m(x- x 1 )
2x 1 +3y 1 +4=0 d=-
2x 1 +3(-1-0.75x 1 )+4=0 5 4
y- 4 = -(x- 6)--+ Equation of line
-0.25x 1 = -1 d = 5 units 3
10
x1 =4
y 1 =-1-0.75(4)

Y1 = -4

Thus, the second point is (4, -4).


8
6
when x "(), y ·0
whenx~J.y of

Given line 1 : 4x- 3y -12 = 0
A=4;8=-3;C 1 =-12
Thus, at x = 0; y = - 4

Using the distance formula:

d=~(x2 2
-x1 ) +(y2 -y1 )2

Given line 2 : 4x - 3y + 8 = 0 d=~(6-0) 2 +(4+4) 2


ml A(x 1)+8(y 1)+C
A =4; 8 =-3; C 2 = 8 d = 10 units

Given line : 4x- 3y + 5 = 0

A =4; 8 =3; C = 5

Given point: (2,1)

x1 = 2; Y1 = 1
d = ---;::==:::=:~

d=-
±-JA2 +82
d = 1(5)+(-1)(10)+0
-~f
-5
+(-1) 2
d=

d=-.
20

5
C 2 -C 1
-JA2+B2
8-(-12)
= --;=:====
~42+(-3)2

y-x = 5
y=x+5

-.fi d = 4 units
From the point-slope form:. y = mx + b
d = 3.54 units
lly inspection, the slope (m) is equal to 1
Day 11 - ~ytic Geometry (Points, Lines & Circles} 269
268 100 1 Solved Problems in Engineering Mathematics (2nd Edition) by Tiong & Rojas

•3x:t-2y+1=0
1 [(1(-3)+3(-3)+5(1))
A=;- -(1(3)+(-3)(5)+(-3)(1))
A = 4 square units
l •
P1 (1,4)-+x 1 =1andy 1 =4
P2 (4,1)-+x 2 =4andy 2 =1
•• P1 (-2,3)-+x 1 =-2andy 1 =3 ·
P2 (-3,5)-+x 2 =-3andy 2 =5


2y = -3x-1
3 1
y=--x-- Using two-point form: Using two point form:
Pa(0,8)
2 2
Y2 - Yt Y2- Yt
From the point-slope form: y =mx + b y-y 1 =--(x-xd
x2 - x1
y-y 1 =--(x-x 1 )
x2 - x1
By inspection, the slope is equal to -3/2. P2(4,5)
1-4 5-3
y-4 =-(x-1) y-3=--(x+2)


4-1 -3+2
y- 4 = -1 (X -1) 2
y-3 =-(X +2)
y-4 = -x+1
Pa(3,3) -1
x+y=5 -y+3=2x+4
2x+y+1=0

A=~l
P1(-2,0)

x,
2 Yt
x2
Y2
x3
Y3
x,
Yt
I
P2(4,0)
11 x 1 x 2
A=-
2 Yt. Y2
_1,140-311
A--
215841
x3
Y3
x4
Y4
x1
Yt
~
....
(4, 1)
• a

x-interc~pt ~~
. A--
_1,-243-21 1[(1{5)+4(8)+0(4)+(-3)(1) )]
" .
2 0 0 3 0
A =~[·(-2(0)+4(3J+3(0)) l A=; -(1(4)+5(0)+8(-3)+4(1))
A = 25 square units Substitute y =Oto solve for the x-
b

r
[Y-~


intercept:
2 -(0(4)+0(3)+3(-2))
A =9 square units x+y=5

• Pt(1, 1)
P1 (1,-3)-+x 1 =1andy 1 =-3
P2 (-4,2)-+x 2 =-4andy 2 =2

Using two point form:


x+0=5


x=5

As given: m =3; b =1
By inspection, a =4 and b =6
Using the intercept form:

X y
y-y 1 =Y2-
- Yt
-( x-x 1 ) -+-=1
x2 -x 1 a b
Using point slope form:
P2(3,-3) P3(5,-3) 2+3 X y
y.+3=-(x-1) -+-=1
y=mx+b 4 -6
-4-1
_ 11 x 1 x2 x3 x1 ,. y+3=-1(x-1) y=3x+1 6x-4y=24
A--
2 Yt Y2 Y3 Yt y+3 =-x+1 ~x-y+1=0 3x-2y=12

A-111 3 5 .1, y =-x-2


21-3-31
270 1001 Solved Problems in Engineering Mathematics (2nd Edition) by Tiong & Rojas Day 11 -Analytic Geometry (Points, Lines & Circles) 271

•P1 (-5/2,-9/2) - H 1 = -5/2 and y 1


P2 (x,6)~x 2 =xandy 2 =6
= -9/2
3x +2y -7 = 0
3 7
y=--x+-
2

By inspection, m, = -3/2
2
Y2 -y, -4-2
m1 = - - - = - - = - 1
x2 - x1 1-(-5)
1
Since perpendicular, m 2 = - -
m,
By inspection, m 1 = -1/5

1 1
1
Since perpendicular: m 2 = - -
m,

9=45° 1 1 m2 =--=---=5
2x-By+2=0 m 2 =--=--=1 m1 (-1/5)
m1 (-1)
tan9= Y2 -y 1 2 2
y=-x+- P(3, 1), thus x; = 3 <md y, = 1
X2 -x 1 B B P3 (x,-7) ~ x 3 = x and y 3 = -7
tan45° = 6-(-9/2) Using point-slope form:
P4 (8,7)~x 4 =8andy 4 =7
By inspectiQn, niz = 2/B
X -(-5/2) y-y 1 =m(x-x 1 )
1 = 21/2 1
Since perpendicular, m 2 = - -
m2 = Y4 -y3 y-1=5(x-3)
x+5/2 m, x4 -x3 y -1 = 5x -15
x=8 2 1= 7+1 5x-y = 14


P(O,Q)~x 1
m=6
=Oandy 1 =0
B
B=3


-3/2


8-x
x=-6


P1 (5,0)~x 1 =5andy 1 =0
P2 (-7,3) ~ x 2 = -7 and y 2 = 3
Using slope-intercept form: y = mx + b Using slope-intercept form: y = mx + b
Using point-slope fonn:
x-y-2=0
y-y 1 =m(x-x 1 ) 2x-3y+2 = 0 m, =Y2 -y 1 = 3-0 =-~
3y =2x+2 y =x-2
y -0 = 6(x- 0) x2 - x1 -7-5 4
· y=6x 2 2
y =-x+- By inspection, m = 1
y -6x =0 3 3 1
Since perpendicular: m 2 = - -


Since parallel: mz = m, m,
By inspection, m, = 2/3
P(4,0), thus x1 = 4 and y, = 0
1 1
1 m 2 =--=---=4
Using the intercept fonn: Since perpendicular, m 2 = - - Using point-slope fonn:
m1 (-1/4)
m,
X y y-y 1 =m(x-x 1 )
-+-=1 - -1- -
m2 - - -1-
y-0=1(x-4)
a b m1 2/3
X y
y =x-4
m2 =-3/2


-+-=1


-2 4
-2x + y = 4
y -2x-4 = 0


Using slope-intercept form: y = nix + b
P1 (-5,2)~x,-=-5andy 1 =2
P2 (1,-4)~x 2 =1andy 2 =-4 X +5y +5 = 0
1 5
y =----X--·
Using slope-intercept form: y = mx + b
5 5
272 100 1--Solved Problema in Engineering Mathematics (2"ll Edition) by 'l'iong &: Rojas Dayll-.AnlllYlic GeometrY <PointS. Lines & Circles) Zf3

Solving for the midpoint of P, and P2:

x 1 -x 2 5-7
x=---.=--=-1
2
_y,-y2 0+3 3
y---=-=-
2
P(-3,-5), thus x, =- 3 andy,= -5
Using point-slope form:

y-y 1 =m(x-x 1 )
7
y.+5=--(x+3)
I

·I
-2
-+-=1
a 2a
-4+6
--=1
2a
6

a=1

2 2 2 b=2
2
Using point-slope form: 2y +10 = -7x-21
7x+2y+31=0
Substitute:
I

y-y 1 =m(x-x 1 ) - 11 x1
A-- x2 x3 x1
=
y-312 4(x +1) ~.!=1· 2 Y1 Y2 Y3 Y1
2y-3=8x+8
P1 (3,-2)-.x 1 =3andy 1 =-2
a b
X y
2 =_:12 -2 x 21
8x-2y+11=0 -+-=1 2 4 3 y 4


P2 (-7,6)--+x 2 =-7andy 2 =6 1 2 (2(3)-2y.f.4x) ]
2x+y=2 4
Using two poi.ntform: 2x+r-2=o
=[ -(4(-2)+3x+2y)
4 =6 -2y +4x+8-3x-2y


Using slope-intercept form: y =mx + b Y2 -y, O=x-4y+10
y-y 1 =--(x-x1 )
3x-2y+6 =0 x 2 -x 1
3 6+2 x=2+t
y=-x+3 y+i=-·-(x-3)
2 -7-3 t •x-2 --+Eq.1
8 y =5-3t--+Eq.2
By inspection. m, =312 y+2=-(x-3)
. -10
Substitute Eq.1 in Eq.2:
Since parallel: m2 =m, -10y-20=8x-24
8x+10y-4=0 y=5-3(x-2)
From the choices, the answer is choice D.
4x+5y-2=0 y=5-3x+6

• =.!1 x1 I
15x-10y-9=0 y=-3x+11 A x2 x3 x1
3 9 2 Y1 Y2 Y3 Y1
y=-x-- By inspection, m -3= 2 -2,

2 10 _ 1,-2
2-- X
1 2 '3 .4 y 3
a =_;b
By inspection, m2 =.312 (-2(4)+2y+3x)]
2


Using slope-intercept form: y

7x+2y-4 =0
=mx + b
b=2a

Using the intercept form:

X
-+-=1
a b
y
y =4t+6 --+Eq.1
x=t+1
t • x -1--+ Eq.2

Substitute Eq.2 in Eq.1:


4=
· [ -(3(2)+4x-2y)
4 =-8+2y+3x -6-4x+2y
O=-x+4y-16
O=x-4y-18

7 4 Suggested answer Is choice D.


y=--x+- y •4(x-1)+6
2 2 Substitute: x =- 2, y = 6 and b = 2a:
y •4x-4+6
By inspection, m, -712 = -2 6
-+-=1
y•4x+2

Since parallel: mz • m, a 2a By lnapectlon. m• 4


27 4 100 1 Solved Problems in Engineering Mathematics (2nd· Edition) by Tiong & Rojas


Given line--+ 2x-3y +6 =0

Atx =0:
X y
-+-=1

X
a
-+-=1
-2 1
y
b

-x +2y = 2 Topics
2x-3y+6=0 x-2y+2 = 0
2(0)-3y +6·= 0
y=2
0 Mon
Analytic Geometry
Rectangular Coordinates System
Distance Between Two Points
At y =0: 0 Tue
Slope of Line
Angle Between Two Lines
2x-3y+6=0 Distance Between a Point & a Line
2x-3(0)+6 = 0
X =-3
0 0 VVed
Distance Between 2 Parallel Lines
Division of Line Segment
Thus the x-intercept (a) is 3 units and the
y-intercept (b) is 2 units.
By inspection, the x and y intercepts are:
a= 1; b = -2
0
Problems
0 Thu
Area of Polygons by Coordinates
Equations of Lines
Conic Sections or Conics

0 ~
General Equation of Conics
Using the intercept form: Geometric Properties of Conics
Solutions Fri Equations of Circles ·
X y
-+-=1
a b
[I] 0
Notes Sat
X y
-+-=1
1 1 1 -2
A =-ab =-(3)(2)
2x-y=2
2 2
A = 3 sq. units 2x-y-2=0
* .,., . . ·, .. "'' ~ ~ i ·' < ~ "' .,


... ., '< 0 ,, ' ,. • • • ,. ... "' "' ,, ' "

'\'"'.

" .. ·•' .., ' ,, ~ ' ' .., ., <. ' " ·•' " "· '

·}.· <. "'' "' ~ ~ , • • • ·""·' "·,',

T
By inspection, the x and y intercepts are:
a= -2; b = 1

Using the intercept form:

T
276· 100 1 Sol'ved Problems in Engineering Mathematics {2nd Edition) by Tiong & Rojas

-·~~ .<"' ~,,~< ,,. '"~"·~·~·"·.>"<·

Topics

D Mon
Parabolas
- General & Standard Equations
- Eccentricity & Latus Rectum
D Tue
Ellipses
- General & Standard Equations
- Major & Minor Axis
[I] D
Theory Wed
- Eccentricity & Latus Rectum
Hyperbolas
- General & Standard Equations
D D
Problems Thu
- Transverse & Conjugate Axis
- Eccentricity & Latus Rectum

,,.,·_. ,.,,
D D
Solutions Fri
Polar Coordinates
-Radius Vector
- Polar Angle

' .,. « ~. { '• o. ' y « ,, "" 4 ' •


D ~
Notes Sat
Relation Between Rectangular
and Polar Coordinates

What is a Parabola? 1, General equations:


A. Axis parallel to the y-axis:
A parabola is a locus of a point which
moves so that it is always equidistant to a y
fix,ed point called focus and it a fixed
straight line called directrix.
y
~
----t----t------~x
l •

directrix

- X Ax2 + Ox + Ey + F =0
B. Axis parallel to the x-axis:
<"!"'""'"'

··~ whQre a = distance from vertex V to

I
"<····
focus F
ci di~;(;IIICi' fiOIII fHlillllo diWc!riX . ----x
I fo• • If d1· .I dill "
278 1001 Solved Problems in Engineering Mathematics (2nd Edition) by Tiong & Rojas
Day 12 -An(llytic Geometry (Parabola, Ellipse & Hyperbola) 279

C/ + Ox + Ey + F 0 = Opens down y B. Axis parallel to the y-axis: When the equation given is a general
equation rather than standard equation,
Opens upward the vertex V(h,k) of the parabola and its
2. Standard equations: --------~~~~--~ X focal h;mgth or focal radius "a" can be
y calculated by converting the general
Ve1iex (V) at origin (0,0)
equation to standard using the process
known as completing the square.
A. Axis along x-axis:
Opens to the rrght The following formulas can be obtained
. x2 =-4ay
y ~~------~r----. X For axis horizontal: C/ + Dx + Ey + F = 0
Vertex (V) at (h,k)
2
h = _·E_-_4_C_F · k = -E -D
2 A. Axis parallel to the x-axis: 2C a= 4G
y =4ax, 4CD

Opens to the right (x.., h)2 =4a(y - k)


---+------·~ X
y For axis vertical: A~ + Dx + Ey + F = 0
Opens downward

h= -0 k:: 0
2
""4AF -E
y 2A .. 4AE a== 4A
Opens to the left
y
X
What is an Ellipse?

An ellipse is a locus of a point which


---4---------+----~ X
y 2 "'~4ax moves so that the sum of its distance to
(y-k) 2 =4a(x-h) the fixed points (foci) is constant and is
------~~---·~ X equal to the length of the major axis (2a).

Opens to the ~eft (x- h)2 =-4a(y- k)


P(x~ I ._
y
B. Axis along y-axis:
The eccentricity of the parabol;. is the v v
Opens up ratio of the distance to the focus to the
y distance to the directrix.

X e=.!.
d
a
Since f = d, then:
D
(y- k)2 ""-4a(x- h) e=1 directrices

The latus rectum of a parabola is a line


x~ ':'<lay that passes through the focus and 1. General Equation:
perpendicular to the axis of the conic.
Ax2 +Cy2 +Ox +Ey +F =0
LR=4a
281 1001 Solved Problems in Engineering Mathematics (2nci Edition) by Tiong & Rojas Day 12 -Analytic Geometzy(P~ill:>oJ~Ellipse &_!ln>_er:l:>ola) 281

=
Nole: d1 + d2 2a. The major axis =2a, is Major axis is vertical: Since a < 0 and c < a, then the
the distance from V1 to V2. y (x-ht +(y-kf = 1 eccentricity of an ellipse is always less
b2 a2 than 1.
When the point is located along the minor
axis as shown in the following figure:. e <1'
The eccentricity of .an ellipse is the ratio
P(x,y) of the distance to the focus to the distance The latus rectum of an ellipse is a line
to the directrix. that passes through the focus and
X perpendicular to the axis of the conic.

~[ .

LJ· c
The relationship between a, b and c is
x2·
z+z=1
b a

Center, C at (h,k)
y2

directrices

.Et-_J:.d
~ri=~
2w2
a
where: a = semi-major axis
2
b +C~ =a 2 Major axis is horizontal: b = semi-minor axis
y
Whe'l the point P(x, y) is the minor axis: When the equatiOn given is a general
will •a• is always greater than "b". equation rather·tl'lan standard equation,
the center (h,k) of an ellipse and its focal
=
If D & E 0, center is at the origin (0,0). If length "c" can be calculated by converting
either 0 or E, of both 0 & E "' 0, the r..enter the· general equation to standard using the
is at (h,k). process known as completing the
----~r---~--~-----------x square. ·
2. Standard Equations:
· The following formulas can be obtained:
Ce!lter, C at (0,0)
General equation:
Mljor axis is horizontal: (x~bf + (Y-:f =1 c
y a2 b a Ax'- + Cy2 + Ox + Ey + F =0

Major axis is vertical:


D
directrices
h::~
2A
k= -a:
2G
c=.Ja2 -..tr

--~----------.----------r---x
y
What is a Hyperbola?
e=-
a
Q A hyperbola is a locus of a point which
moves so that the difference of the
Iff= c, then eccentricity of an ellipse, e is distanGes to the fixed points (foci) is
x2 2 constant and is equal to the length of the
-+y . c· transverse axis (2a) .
a2 ·b2 = 1
e=-a
X
282 100 1 .Solved Problems in Engineering Mathematics (2nd Edition) by Tiong & Rojas Day 12 -Analytic Geometry (Parabola, Ellipse & Hyperbola) 283

directrices Transverse axis vertical:


asymptot' Transverse axis 2
X - 2 =1
- i_ Transverse axis
a2 b
y
Transverse axis vertical:

I • • ., )I(' I Y6 • • ... X
Transverse axis y

a• i
Conjugate axis ~-7' ---t-----------+---~x
--· ··-~-·- _... _ : )I{ i ~ X

Transverse axis
··f<· ··~···.-r'\1\
• .~Y · A.:~ + ·o"~+
• eY:+-
·· F·=v
· ". (y-k)2- (x-h)2 =1
1. General equations: a2 b2

A Transverse axis - horizontal


Length of the transverse axis = 2a = 2.../A
The eccentricity of a hyperbola is the
Length of the conjugate axis = 2b = 2.JC ratio of the distance to the focus to the
Ax
2
- Gy2 +OX,+ Ey + F 0' = 2
Y
. 2
X
a2-b2='1 distance to the directrix.
where: A and C are the numerical y
Transverse axis is the axis that passes coefficients (absolute value) of x2
through the foci, vertices and the center of and y2 , respectively.
the hyperbola while the conjugate axis is Center, C is at (h,k)
the one that is perpendicular to the Also, the relationship between a, b and c is
transverse axis. Transverse axis horizontal:
• E!-2 + b,~: =42 b
y
Transverse I • • VI "*' I Y6 • ~ X
Length of the transverse axis = 2a = 2.JC
2. Standard equations:

Length of the conjugate axis = 2b = 2.../A

.--
Center, C at (0,0) .·
Transverse axis horizontal:
a
where: A and C are the numerical
y c
coefficients (absolute value) of x2
Transverse axis
and y2, respectively.
c
Also, the relationship between a, band cis
b
--+-------~~----~x
e=-
a
or e=oa
B. Transverse axis -vertical:
I • • VI )I(" I 116 • ' .. X
Since a > c and D > a, then the
(x-h)2 _(y-k)2 =1
eccentricity of a hyperbola is always
a2 b2 greater than 1.

e >1
.
Conjugate axis
I a .
·.., ~
Z84 . 100 1 Solved Problems in Engineering Mathematics (2nd Edition) by Tiong & Rojas Day 12 - Analyti(: QE!Qrnetry (Parabola, Ellipse & Hyperbola) 28J;

The latus rectum of an ellipse is a line Proceed to the next page for your 12th
(r.e)
that passes through the focus and test. Detach and use the answer sheet
(x,y)
perpendicular to the axis of the conic. provided at the last part of this book. Use
pencil number 2 in shading your answer.
y GOODLUCKJ

X QI:ribia:
Did you know that... the oldest known
example of numeration using place value
b
is not the Roman nor the Arabic
Polar angle is sometimes called the numeration but the Babylonian or
vectoral angle, the argument, the Mesopotamians' sexagesimal system of
amplitude, or the azimuth of a point. numeration which dates back to the 2nd
millennium B. C. ! This system of
numeration still survives today (i.e. 1 hour
~ = 60 minutes and 1 minute = 60 seconds,
=
and 1 degree 60 minutes)

,;t.R=2~2- I ....- r . • ~uott:


. 'a' po e polar ax1s "Every new body of discovery is
mathematical in form, because there is no
where: a = semi-major axis other guidance we can have. •
b = semi-minor axis What is the relation between polar
coordinates and rectangular - Charles DarWin
When .the equation given is a general coordinates?
equation rather than standard equation,
the center (h,k) of a hyperbola can be Relationship between polar coordinates
calculated by converting the general and rectangular coordinates: ·
equation to standard using the process
known as completing the square.
x:rC0$6 y =rt~ne
The following formulas can be obtained:

r·=Jx*~y2
h= -Q
2A
k= -e
2t
I
What are Polar Coordinates?

Polar coordinates (r,e) refers to the


coordinates of a point in a system of
coordinates where the position of a point
is determined by the length of ray segment
(the radius vector) from a fixed origin (the
pole) and the angle (the polar angle) the
ray (the. vector) makes with a fixed line
(the polar axis).
~

Day 12- Analytic Geometry (Parabola, Ellipse & Hyperbola) 287

506: CE Board May 1:996 C. 25 1t


How far from the y-axis is the center of the D. 27 rr
curve 2~ + 2y + 10 x - 6y - 55 0?
2
=
SJ:2r Determine the equation of the circle
A. -2.5 whose radius is 5, center on the line x = 2
B. -3.0 and tangent to the line 3x - 4y + 11 = 0.
C. -2.75
Topics D. -3.25 (X - 2) + (y - 2)2 ::: 5
2
A.

DMan
Parabolas
- General & Standard Equations
507: What is the distance between the
centers of the circles x2 +l + 2x + 4y - 3
B.
C.
D.
(x - 2) 2 + (y + 2) 2 = 25
2
2 2
(x- 2) + (y- 2) = 25
2
(X - 2) + (y + 2) = 5

- Eccentricity & Latus Rectum = o and x 2 +l- ax - 6y + 7 = 0?

DTue
Ellipses
- General & Standard Equations
A.
B.
7.07
7.77
SJ:~r Find the equation of the circle with
the center at (-4,-5} and tangent to the line
2x + 7y - 10 = 0.
-Major & Minor Axes
D D
C. 8.07
- Eccentricity & Latus Rectum D. 7.87 A. 2
x + / + 8x - 1Oy - 12 = 0
Theory Wed Hyperbola B. ~+ l
+ax- 10y + 12 = 0
508: CE Board November 1:99~ ~ + / + 8x + 10y- 12::: 0
[I] D
C.
- General & Standard Equations
-Transverse & Conjugate Axes
The shortest distance from A (3,8) to the
circle~+/+ 4x- 6y = 12 is equal to
D. ~+l-ax+ 10y + 12 o =
Problems Thu - Eccentricity & Latus Rectum 51:4: ECE Board April 1:998

D D
Solutions Fri
Polar Coordinates
- Radius Vector
- Polar Angle
A.
B.
C.
D.
2.1
2.3
2.5
2.7
Find the value of k for which the equation
~ + y2 + 4x - 2y - k = 0 represents a point
circle.

DNotes
~Sat
Relation Between Rectangular
and Polar Coordinates
509: ME Board Oetober J:996
2
The equation x + y2 - 4x + 2y - 20 =0
A.
B.
C.
6
5

-6
describes: D.. -5

SOJ:: CE Board May 1:995 C. (-3,2) A. A circle of radius 5 centered at the 515: ECE Board April 1:999
What is the radius of the circle x 2 + y2 - 6y D. (-2,-3) origin. 3x2 + 2x- 5y + 7 = 0. Determine the curve.
= 0? B. An ellipse centered at (2, -1 ).
504: ME Board April 1:998 C. A sphere centered at the origin. A. Parabola
A. 2 What is the radius of a circle with the ff. D. A circle of radius 5 centered.at (2,-1 ). B. Ellipse
B. 3 equation: ~- 6x + y2 - 4y- 12 = 0 C. Circle
c. 4 uo: EE Board April 1:997. D. Hyperbola
D. 5 A. 3.46 The center of a circle is at (1, 1) and one
B. 7 point on its circumference is (-1 ,.-3). Find 51:6: CE Board May 1:99~, CE Board
502: CE Board November 1:995 C. 5 the other end of the diameter through (-1 ,- November 1:99~, ECE Board April
What are the coordinates of the center of D. 6 3). 1:994
the curve~+/- 2x- 4y- 31 = 0? The focus of the parabola y2 = 4x is at
A. (2,4)
SOS: ECE Board April 1:998
A. (-1 ,-1) B. (3,5) A. (4,0)
The diameter of a circle described by 9~ +
B. (-2.-2) 9y2 = 16 is C. (3,6) B. (0,4)
C. (1,2) D. (1,3) C. (3,0)
D. (2,1) A. 4/3 D. (0,3)
B. 16/9 sur Find the area (in square units) of the
so~: A circle whose equation is ~ + y2 + C: 8/3 circle whose equation is~+ y2 = 6x- By. 51:7: CE Board November 1:994
4x + 6y -'-23 = 0 has its center at: D. 4 Where is the vertex of the parabola ~ =
A 20 rr 4(y- 2)?
A. (2,3) R 7/rr
B. (3,2)
288 100 1 Solved Problems in Engineering Mathematics (2nd Edition) by Tiong & Rojas __________ pay .~.:~nalylic Geornetry__(~ar~~~~Eilips~!_f:!.'[f~f::~E-~!<!L:~~~
2
A ~~ SZ:JI ECE Board Aprll1998 8, 8 - 4AC = 0 S34: CE Board May lt99~
a ~~ Find the equation of the axis of symmetry C. 8 2 - 4AC > 0 The length of the latus rectum for \lh':
~ ~~ of the function y =2x2 -7x + 5. D. 8 2 - 4AC = 1 . x2 y2 .
n ~~ ellipse ··- •·--- , 1 IS equal to.
64 16
A. 7x + 4 = 0 S:l9: CE Board November 1.994
518: ECE Board Aprll1994, ECE B. 4x + 7 0= What is the area enclosed by the curve 9x2
A 2
Board Aprll1999 · C. 4x-7 =0 + 25y 2 - 225 ::: 0?
x-2 = 0
B. 3
Find the e~uation of the directrix of the D.
parabola y = 16x. A ~1 c. 4
B. 50.2 D. 5
5Z4& A parabola has its focus at (7. 4) and
A x=2 directrix y =2. Find its equation. c. ~.8
a x=~ D. n.3 53$1 An ellipse with an eccentrir..:ity of 0.6!:i
~ x=4 A. x2 + 12y -14x + 61 = 0 and has one of its foci 2 units from the
n x=4 B. x2- 14y + 1·2x + 61 0= 5301 ECE Board Aprill.998 center The length of the latus rectum is
C. x2- 12x + 14y + 61 0 = Point P(x,y) moves with a distance from nearest to
5191 Given the equation of a parabola 3x D. None of the above point (0, 1) one-half of its distance from line
A.
=
+ 2y2 - 4y + 7 0. Locate its vertex. y = 4. The equation of its locus is
B.
3.5 units
3.8 units
5Z5r A parabola has its axis parallel to the
A.
2
2x - 4y 2 = 5 C. 4.2 units
A. (5/3, 1) x-axis, vertex at (-1, 7) and one end ofthe
B. 4x 2 + 3y 2 = 12 D. 3.2 units
(5/3, -1) latus rectum at (-15/4, 3/2). Find its
B.
C. (-5/3, -1) equation. C.
D. 2
2
2x + st
x + 2y· = 4
=3
S361 An earth satellite has an apogee of
D. (-5/3, 1)
A. y2 - 11y + 11x- 60 = 0 40,000 km and a perigee of 6,600 km.
5ZOI ME Board Aprll1997 B. y2 -11y + 14x- 60 = 0 S3l.: The lengths of the major and minor Assuming the radius of the earth ns 6,4CO
In the equation y = -x2 + x + 1, where is the C. y2 -14y + 11x + 60 0 = axes of an ellipse are 10m and 8 m, km, what will be the eccentricity of the
elliptical path described by the satellite
curve facing? D. None of the above respectively. Find the distance between
the foci. with the center of the earth at one of the
foci?
A.
B.
Upward
Facing left
5Zftl ECE Board Noveaaber 1997
Compute the focal length and the length of
I
'I A. 3
I
C. Facing right the latus rectum of the parabola y2 + 8x - B. 4 A 0.46
D. Downward =
6y + 25 0. . C. 5 B.
c
0.49
0.52
D. 6
A D. 0.56
5z:ll CE Board ala,- 1995 2,8
What is the length of the length of the latus B. 4, 16 S3jt: The equation 25x2 + 16y2 - 150 x +
rectum of the curve x2 20y?= c. 16,64 128 y + 81 =0 has its center at S~7: ECE Board Aprill998
The major axis of the elliptical path in
D. 1, 4
A (3,-4) which the earth moves around the sun is
A. .J20 sa?• Given a parabola ( y- 2 )2 =-8(x- B. (3,4) approximately 186,000,000 miles and the
B. 20 C. (4,-3) eccentricity of the ellipse is 1/60.
1). What is the equation of its directrix?
C. 5 D. (3,5) Determine the apogee of the earth.
D. .J5 A. x=-3
533: EE Board October 1.997 A 93,000,000 miles
B. x= 3
5ZZI EE Board October 1997 c. y =-3 Find the major axis of the ellipse x2 + 4/ ~ B.
C.
91,450,000 miles
94,335, iOO miles
D. y=3 2x- By+ 1 = 0.
Find the location of the focus of the D. 94,550,000 miles
parabola y2 + 4x- 4y- 8 0. = SZ8& ME Board October 199'7 A. 2
The general equation of a eonic section is B. 10 S38: CE Board November :199%
A. (2.5, -2)
B. (3,1) given by the following equation: A>t! + Bxy c 4 The earth's orb1t is an ellipse with the sun

c. (2,2) =
+ Cy2 + Ox + Ey + F 0. A curve maybe D. 6 at one of the foci. If the farthest distance of
the sun from the earth is 105 . 50 million km
D. (-2.5,-2) identified as an ellipse by which ofthe
following cOndition&? and the nearest distance of the sun from
the earth is 78.25 million km, find the
eccentricity of the ellipse.
A. B2 -4AC <0
I
11
290· roo l Solved Problems in Engineering Mathematics (2nd Edition) by Tiong & Rojas Day 12 -Analytic Geometry{Parabola, Ellipse & Hyperbola) 291

A. 0.15 A. 2 sso: ME Board April1997 555: EE Board October 1997


B. 0.25 B. 3 What is the radius of the sphere center at Given the polar equation r = 5 sin a.
c. 0.35 C. 4 the origin that passes the point 8,1 ,6? Determine the rectangular coordinates
D. 0.45 D. 5 (x,y) of a point in the curve when a is 30•.
A. 10
539: An ellipse with center at the origin 545: CE Board May 1996 B. 9 A (2.17, 1.25)
has a length of major axis 20 units. If the What is the equation of the asymptote of B. (3.08, 1.5)
xz yz C. JjQ1
distance from center of ellipse to its focus C. (2.51, 4.12)
is 5, what is the equation of its directrix? the hyperbola - - - = 1 ? D. 10.5
9 4 D. (6,3)

A. X= 18 551: The equation of a sphere with center


A. 2x- 3y = 0 at (-3,2,4) and of radius 6 units is
B·: X= 20
B. 3x- 2y = 0
C. X= 15
D. X= 16
C. 2x- y = 0 A. ,(! + l +z
2
+ 6x - 4y - 8z = 36
D. 2x + y = 0 B. J2! + / + z2 + 6x - 4y - 8z =7
540: What is the length of the latus rectum C. ,(! + y2 + z2 + 6x- 4y + 8z =6
of 4J2! + 9l + Bx - 32 = 0?
546: EE Board A.pril1994 D.
2
x + l +z
2
+ 6x- 4y + Bz = 36
Find the equation of the hyperbola whose
asymptotes are y = ± 2x and which passes 5521 EE Board April1997
A. 2.5 through (5/2, 3).
B. 2.7 Find the polar equation of the circle, if its
c. 2.3 center is at (4,0) and the radius 4.
A. 4J2!+/+16=0
D. 2.9 B. 4J2! +l- 16 = 0 A. r - 8 cos e = o
C. ,(! - 4/- 16 = 0 B. r - 6 cos e = o
541: EE Board Odober 1993 D. 4x
2
-l = 16
4J2! - / ='16 is the equation of a/an c. r- 12 cos e = o
D. r- 4 cos e = o
5471 Find the equation of the hyperbola
A. parabola with vertices at (-4,2) and (0,2) and foci at
B. hyperbola 5531 ME Board October 1996
(~5.2) and (1 ,2).
C. circle What are the x and y coordinates of the
D. ellipse focus of the conic section described by the
A. 5J2! - 4y 2 + 20x + 16y - 16 = 0
following equation? (Angle e corresponds
542: EE Board October .1993
B. 5J2!- 4y2 + 20x- 16y- 16 = o to a right triangle with adjacent side x,
C. 5J2!-4y2 -20x+16y+16=0
Find the eccentricity of the curve 9J2! - 4l opposite side y and the hypotenuse r.)
D. 5J2! + 4y2 - 20x + 16y- 16 = o
-36x +By= 4 rsin 2
a= cos a
548: Find the distance between P1(6,-2,-3)
A. 1.00 A. (1/4, 0)
and P2 {5,1,-4).
a 1.~ B. (0, rr12)
~ 1.~ C. (0,0)
A. 11
D. 1.ro D. (-1/2,0)
B. J11
543t CE Beard November 1995 c. 12
sS4: Find the polar equation of the
How far from the x-axis is the focus F of
the hyperbola J2! - 2l + 4x + 4y + 4 = 0?
D. f12 circle of radius 3 units and center a.t
(3,0).
549: The point of intersection of the planes
A. 4.5
B. 3.4
x + 5y - 2z = 9; 3x - 2y + z = 3 and x + y + A. r=3cose
c. 2.7
z = 2 is at B. r = 3 sine
D. 2.1 C. r=6cose
A. (2, 1-1)
D. r =9 sine
B. (2,0,-1)
5441 EE Board. October 1994
C. (-1,1,-1)
The semi-transverse axis of the hyperbola
D. (-1,2,1)
x2 Yz .
---=11S
9 4

l
Day 12 -Analytic Geometry (Parabola, ElliiJse &Hyperbola) 293

·--
Topics

x 2 +y 2 -6y=O

By completing square:
ED
x2 +y 2 -6x-4y-12=0

0tvlon
Parabolas
- General & Standard Equations x 2 +(y-3) 2 =0+(3) 2
By completing square:

- Eccentricity & Latus Rectum x2 +(y-3)2 =(3)2 (x-3) 2 +(y-2) 2 =12+(3) 2 +(2) 2

0Tue
Ellipses
-General & Standard Equations I Standard equation with center at (h,k):
(x-3) 2 +(y-2) 2 =25
(x-3) 2 +(y-2) 2 =(5) 2
- Major & Minor Axes (X- h) 2 + (y- k) 2 = ,.Z

0Theory
0
Wed
- Eccentricity & Latus Rectum
Hyperbola
By inspection, r =3 Standard equation: (x- h) 2 + (y- k) 2 = r"

By inspection, r = 5.

0 0 -General & Standard Equations


- Transverse & Conjugate Axes mJ
Problems Thu - Eccentricity & Latus Rectum
x 2 +y 2 -2x-4y-31=0 a
Solutions
0Fri
Polar Coordinates
- Radius Vector
- Polar Angle
By completing square:
9x 2 +9y 2 =16
16
x2 +y2 = -

0Notes
~Sat
Relation Between Rectangular
ahd Polar Coordinates
(x-1) 2 +(y-2) 2 =31+(1) 2 +(2) 2
(X -1 ) 2 + (y- 2) 2 = 36 2
9

x +y2=(;r
2 2
Stanaaro equation: (x- h) + (y- k) = r"

By inspection, h = 1 and k = 2, thus the Standard equation with center at (0,0):


x +~l=r"
2
ANSWER KEY RATING center is at (1 ,2)

501. B 516. l\
502.C 517.8
503. D ·5"18. 0
504.C 519. D
505.C 520. 0
506.A 521. B
507.A 522.C
531. D
532.A
533. c
534. c
535. A
536. D
537. D
546. D
547. A
548. B
549.A
550. c
551. 8
552.A
c:J 46-55
c::J
0
0
Topnotcher

33-45 Passer

27-32 Conditional

0-26 Failed

x 2 +y 2 +4x+6y-23=0

By completing square:
4
By inspection, r =- and d = 2r =-


3
8
3

508.A 523.C 538.A 553.A If FAILED, repeat the test. 2x 2 +2y 2 +10x-6y-55=0
509. D 524.A 539. B 554.C (x + 2) 2 + (y + 3) 2 = 23 + (2) 2 + ( 3 )2 2
510. B 525. c 540. B 555.A x +l
+5x-3y-27.5=0
(x + 2) 2 + (y + 3) 2 = 36
511 c 526.A 541. B
512. B 527. 8 542. A By completing square:
5i3. c 528.A 543. c Standard equation: (x- h) 2 + (y- k) 2 = r"
514. D 529.A 544. B (x+2.5) 2 +(y-1.5) 2 =27.5+(2.5) 2
515.A 530. B 545./\ By inspection, h = - 2 and k = - 3, thus the
center it at (-2, -3). +(1.5 )2
(x+2.5) 2 +(y-1.5) 2 =36

Standard equation: (x - h) 2 + (Y - k) 2 = r" /

'I
294 1001 Solved Problems in Engineering Mathematics (2nd Edition) by Tiong & Rojas Day 12- Analytic Geometry (Parabola, Ellipse & Hyperbola) 295

By inspection, h = - 2.5 and k = 1.5 x =d-r


y-axis C2(4,3) X =7.071-5
X= 2.071
x 2 +y 2 -6x+8y=O

Ill By completing square:

x 2 +y 2 -4x+2y-20=0 (x-3) 2 +(y+4) 2 =(3) 2 +(4) 2

x2 + y2 - 4x + 2y = 20 (x-3) 2 +(y+4) 2 =25or(5) 2


C,(-1,-2)

By completing square: By inspection, the radius is 5.

d = ~(x2 -x,)2 +(y2- y,)2


Note: The distance of the center of the (x-2) 2 +(y+1) 2 =20+(2) 2 +(1)
2 A= nr 2 = n(5)2
circle from they-axis is equal to h. d = ~(4 + 1)2 + (3 + 2)2 (x-2) 2 +(y+1) 2 =25or(5) 2 . A = 25n square units

Thus, the answer is 2.5 unit length.


d=7.071


By inspection, the center is at C(2, -1)
and the radius, r = 5 .

x 2 +y 2 +2x+4y-3=0
x 2 +y 2 +2x+4y=3

By completing square:
x 2 +y 2 +4x-6y=12

By completing square:

(x + 2) 2 + (y- 3) 2 = 12 + (2) 2 + (3) 2



(x+1) 2 +(y+2) 2 =3+(1) 2 +(2) 2 (x + 2) 2 + (y- 3) 2 = 25 or (5) 2
2
(X + 1 ) 2 + (y + 2) =8
By inspecJion, the center is at (-2, 3) and
the radius, r = 5.
By inspection, the center of the first circle
is at C,(-1, -2).
P,(-1 ,-3)
(3,8) Note: There are two possible circles
2 2
x +y -8x-6y+7 =0 Using midpoint formula:
Try circle 1:
x 2 +y 2 -8x-6y=-7
x = x1 +x2
-d= A(x1)+B(y1)+C
By completing square: 2
1 = -1 + x2
±~A2 +B2
(x-4) 2 +(y-3) 2 =-7+(4) 2 +(3) 2 2
2 Note: d is negative since the point is
(X- 4 ) 2 + (y- 3) = 18 Solving for distance between (-2,3) and
X2 =3
below the line.
(3,8): y = y, +y2
By inspection, the center of the second 2 Given line: 3x -4y + 11 = 0
circle is at C2(4, 3). f 2
d=v(x2 -x1) +(y2-Y1)
. 2 1= - 3 +Y2 A=3;B=-4;C=11
2 Center of circle 1: (2, y)
Using distance formula to solve .for the
d = ~(3 + 2)2 + (8- 3)2 Y2 =5
distance between c1 & c2.: X1 = 2; Y1 = Y
d = 7.071
Thus, the point is at (3,5)
Let: x = shortest distance

't
!~!!'"J.99J_.§_'?}v~d I:r.£!?~~_in Engi~!l_l9 M~pematics (2nd Edi!ionl_ey Tiong ~ Rojas Day 12- Analytic Geometry (Parabola, Ellips~ & Hyperbola) 297
Substitute:

~ ::l(2) + H>(y) + 11
Solving for the equation of the circle with
center at (-4, -5):
• By inspection, the equation of the directrix
of the given parabola is x = -4.


Given equation:/= 16x
-5·- -~~;)2-~(-4?~- (x-hf +(y-k) 2 =r 2
(x+4) 2 +(y+5) 2 =7.28 2 Standard equation: y2 = 4ax
6-4y+11
-5 ·;: ,,,_, ____ _
··5 x2 +8x+16+y 2 +10y+25=53 3x + 2y2 - 4y + 7 = 0
By inspection 4a = 16, then a= 4.
y =-2 x2 +y 2 +8x+10y-12=0 2 3 7
y -2y+-x+-=O
2 2
SCJiving for the equation of the circle with
center at (2, -2): 1111 By completing square:

{x- h) 2 + (y- k) 2 ""r 2 x2 +y 2 +4x-2y-k=O ~- 2 3 7


Y -2y=--x--
(X- 2f + (y + 2) 2 ::5 2 2 2
x +y +4x-2y=::k 2 2
(X- 2)
2
+ (y + ;?) 2 = 25 (y -1 )2 ::-~X .. .?. +(1 )2
By completing square: 2 2 .
Note: Since this equation is in the Thus, the focus is at (4,0) 3 5
:::::--x--


2
chOices, there is no need to get the (x + 2) + (y -1 ) 2 = k + (2) 2 + ( 1) 2 2 2
equation of the second circle. 2
(x+2) +(y-1) 2 =k+5 or (....rk+5f (y -1)
2:: -%(X+~)
Ill [2x+ 2'-t~ 1o= EJ By inspection, r = .Jk + 5 .
Given equation: x2 = 4(y - 2)
Standard equation: (y -kf = -4a(x -h)
~-- t whenx=O,y"/.43 Note: For a point circle, r = 0. Standard equation: (x- h)2 = 4a(y- k)
~. t whenrO.x~·5
where: h and k are the coordinates of the
By inspection, h = - 5/3 and k = 1, thus
the vertex is at (-5/3, 1)


0 =.Jk +5 vertex
--...: k:: -5
By inspection h = 0 and k = 2, thus the

Given line : 2x + 7y -1 0 = 0

General equation of a conic section:
Ax 2 + cy2 + Dx + Ey + F = 0
vertex is at (0,2)


Given equation: y2 = 16x
2
y=-x 2 +x+1
x - x = -y + 1

By completing square:

Note:
A=2;B=7;C=-10 Standard equation: y2 = 4ax (x -1/2l = -y + 1+ (1/2)2
.. If A = C, the conic is a circle.
Center of circle: (-4,-5) • If A 'I C but the same sign, the conic =-y+5/4
x1 =4;y 1 :::-5 is an ellipse. By inspection 4a = 16, thus a= 4. (x-1/2)2 =-1(y-5/4)
• If A and C have opposite signs, the

~-+u
Solving for the distance from the center of conic is a hyperbola.
the circle to the line: " If either A or C is zero, the conic is a
parabola.
r = j\(x1) + B(y1) + C
JA2 +fil - Given equation: 3x 2 + 2x- 5y + 7 = 0 a· =
{X..f1)2 t4~(ywk~ (x.-h)-z =: -4~{y4<+
r= 2(-4)+7(-5)-10
~
~-~2 =(-)7.28 By inspection C = 0, thus the curve is a
paratJola. By comparison to the standard equations,
+\/\<') +(7) the given parabola is facing downward!
298 1001 Solved Problems in Engineering Mathematics (2nd Edition) by Tiong & Rojas Day 12- Analytic Gee>rn~try (f'ara):)o@L~se & Hyperbol~ 299


Given equation: x 2 = 20y

Standard equatian: x = 4ay 2


By completing square:

(x-~r =t-%+(~r
=r+~
Substitute:

x 2 -14x+12y+61=0
2
(X- 7) = -4(3)(y + 1)

x 2 -14x + 49 = -12y -12


Focal length = a = 2
Length of latus rectum = 4a = 8

Ill


2 16 (y- 2)2 = -8(x -1)
Length of latus rectum = 4a

By inspection, 4a = 20
(x--~r =i(Y+~) Standard equation: (y- k)2 = -4a(x- h)


By inspection, the vertex is at (1, 2).
Standard equation: (x- h)2 = 4a(y- k)
Also 4a = 8, thus a = 2 .
V(-1,7)
By inspection the vertex is at (7/4, -9/8)
·-/ +4x -_4y -8 = 0
~
y2 - 4y = -4x + 8

Solving for a:
By completing square:

15 11
(y- 2)2 = -4x + 8 + (2)2 a=--1=-
4 4
= -4x +12
(y- 2)2 = -4(x- 3) Standard equation: (y- k)2 = -4a(x- h)
714 V(7/4,-9/8) Refer to the figure, the equation of the
directrix is, x = 3.


Standard equation: (y- k)2 = -4a(x- h) Substitute:
Refer to the figure, the axis of symmetry
By inspection the vertex is at (3, 2).
Also 4a = 4, thus a = 1.
is,
(y- 7)
2
=-4( ~1 }x + 1)
x=- 7
4
y 2 -14y + 49 = -11 X -11
For an ellipse, the discriminant must be
less than zero (B2 - 4AC < 0).

V(3,2)
4x-7 =0

• ~
y 2 -14y+11x+60=0

•y 2 + 8x - 6y + 25 = 0
y 2
6y = -8x - 25
•Note: This is an equation of an ellipse

9x 2 + 25y2 = 225
x2 y2
a -
-+-=1
25 9
By completing square: x2 y2
Thus, the focus is at (2,2). --+-=1
(5)2 (3)2


2x 2 -7x
y =2x 2 -7x+5
=y-5
By inspection, 2a = 6 and a =3
The coordinates of the vertex is at (7,-1).

Standard equation: (x - h)2 = -4a(y- k)


(y- 3)2 = -8x- 25 + (3)2
= -8x -16
(y-3)2 =-8(x+2)

Standard equation: (y- k)2 = -4a(x- h)


By inspection, a

A= 1tab = 1t(5)(3)
x2
Standard equation: 2 + 2 = 1
a b
y2

= 5 and b = 3

2 7 y 5
x --x=---
2 2 2 IJy inspection, 4a = 8 thus, a = 2 A= 47.12 square units
'

Day 12 -Analytic Geometry (Parabola, Ellipse & Hyperbola) 301


300 1001 Solved Problems in Engineering Mathematics (2"d Edition) by Tiong & Rojas
mt
• d1
P(x,y)
~
(x-3)2 (y+4)2 _
--+----1
16 25

b
2
Standard equation: (x- 2h) + (y-
--
a2
2
k)- 1
_ e=-
c
a
..,
Earth
a a

c =ea
cb "'? (0,1)
By inspection, h = 3 and k = - 4, thus the 2=0.65a
center is at (3, -4 ). a= 3.0769


a-c

1
Solving for b: c
e=-
d2 =-d1 a
2
2
2
b = Fa~ = )3.0769 2
- 2 2
::: 2.33B c = ea
2 x + 4y2 - 2x- By + 1 = 0
J(x-O/+(y-1) =-i(4-y)
2b 2 2(2.338) 2 1
c = -(93,000,000)
x 2 - 2x + 4(y 2 - 2y) = -1 LR=--=----
60
a 3.0769
(xf + (y -1? = ..!.(4- y)2 c = 1,550,000
4 LR =3.55
By completing square:
2
x + y2 - 2y + 1 = ..!. (16 - 8y + l)
4
4x 2 + 4y2- By+ 4 = 16- By+ y2
2 2
(X -1? + 4(y -1 ) = -1 + ( 1) + 4(1)2
2 2
mt Assuming the radius of earth and sun to
be very small compared to a:
(X -1 ) + 4(y -1 ) = 4 a a
4x 2 +3y2 = 12 Apogee =a+c
1)2 "'


(X
---+(y-1)2 =1 Satellite 6,600 = 93,000,000 + 1,550,000
4
Apogee= 94,550,000 miles

2a =10
a=5
2b=B
b=4
Standard equation: (x-h)2 + (y-k)2 -
a2 ~-1

By inspection, a2 = 4 thus, a = 2

Length of major axis = 2a = 2(2) = 4


• a
Earth
a

40,000
Solving for c: 12,800

Ill 2a = 40,000 + 12,800 + 6,600


c=1a -b =15 2 -4 2 =3
2 2
a=29.700
Distance between foci= 2c = 2(3) = 6 (x- 3)2 + (y + 4 )2 = 1
c =a- (6.600 +6,400)
64 16
c ::29,700- (6,600 + 6,400)
ml 2
Standard equation: (x- h) + (y- k) _ 1
2 c=16,700
a+c a-c

25x + 16y2 -150x + ~ 2By + 81 = 0


2 a2 ~- c 16,700 a+c=105.5~Eq.1
e=-=--
2 2 a 29,700 a-c =78.25 ~ Eq.2
25(x -6x) + 16(y + 8y) = -81
By inspection: e =0.56

By completing square: Add Eq.1 and Eq.2:


2
a = 64 ~a= 8; b2 = 16 ~ b = 4
25(x- 3f + 16(y + 4)2 = -81 + 25(3)2
ml 2a = ~05.5 + 78.25
Length of latus rectum: a =91.875
+16(4)2 2a = 186,000,000
2 a" 93,000,000
25(x- 3)2 + 16(y + 4)2 =400 LR = LR = 2b _ 2(16)
a - --·a-·- = 4
j
302 100 1 Solved Problems in Engineering Mathematics (2nd Edition) by Tiong & Rojas Day 12- AnalyticQeometry (Parabola, Ellipse & Hyperbo~ 303

Substitute a in Eq.2: 2 2 By inspection, a2 =4 thus, a = 2 . Solving for c:


"Standard equation: (x- h) (y- k)
· a2 +~=1
Also,b 2 =9 thus, b = 3.
c=105.5-91.875
c = ~a 2 + b 2 =.,ff+2 =1.73
c=13.625
2
By inspection, a = 9 thus, a =3 .
2
c - 13.625 = 0'.148 Also, b = 4 thus, b = 2 .
e =;- 91.875
2b - 2(4)
LR=a- 3
2
= 2.667
zjb
ml a
-~--,,-s<.,~ I I )> x-axis

Ill
General equation of a conic section: Solving for c:
Refer to the figure:
2
Ax +Cy 2 +Dx +Ey +F=O
c=~a 2 +b 2 =../4+9 =3.605 Let:
Note: D1 = distance of focus 1 to the x-axis
• If A = C, the conic is a circle. Solving for e: D2 = distance of focus 2 to the x-axis
• If A '1- C but the same sign, the conic
is an ellipse. D1 =1+c=1+1.73=2.73
-d +d e- -c- 3.605
-- --
• If A and C have opposite signs, the a 2 D2 = c -1 = 1.73-1 = 0.73
2a=20 conic is a hyperbola. e=1.8
a=10

e=-=-
a d
c a
• If either A or C is zero, the conic is a
parabola.
Ill rm
Given equation: 4x 2 - y 2 = 16 2
a 2 =cd X
-g-- i_=1
x2 - 2y2 + 4x + 4y + 4 = 0 4
(10)2 = 5d By inspection A = 4 and C = -1 , and since
x 2 + 4x- 2(y 2 - 2y) = -4 x2 y2
d=20 A and C have opposite signs, the curve is Standard equation: 2 - 2 = 1
a hyperbola.· a b


By completing square:
By inspection the equation of the directrix
2
is, By inspection, a = 9 thus, a =3 ;
(x + 2)2 - 2(y -1 )2 = -4 + (2)2 - 2( 1)2 2
b = 4 thus, b = 2
x=±20 (X + 2)2 - 2(y -1 )2 = -2


2 2
9x -4y -36x+8y = 4
(x+2) 2 Length of semi-transverse axis: a = 3
---+(y-1) 2 =1


2 2
9(x - 4x)- 4(y - 2y) = 4 2
(y -1 )2 - (x + 2)2 = 1
2
4x2 + 9y + 8x - 32 = 0 By completing square:
2
2
9(x- 2) - 4(y -1 )2 = 4 + 9(2)2 - 4( 1 )2
4(x 2 + 2x) + 9y 2 = 32 X
2
L:1
2
2 2
9(x- 2) - 4(y -1 )2 = 36 Standard equation· (y- k) (x- h)2 -g· 4
>
· a2 -~=1
By completing square: (X- 2)
2
_ (y -1l = 1 x2 y2
Standard equation: 2 - 2 = 1
4 9 a b
2
4(x + 1) + 9y 2 = 32 + 4( 1)2 By inspection, the center of the hyperbola
isat(-2,1); a2 =1thus,a=1;
2
4(x + 1) + 9y 2 =36 Standard equation: (x- h) _ (y- k)
2 2
=1 By inspection, a2 = 9 thus, a = 3 ;
'(x +1)2
y2
a2 ---b-2- b
2
' 2 thus, b = J2 . b
2
=4 thus, b = 2
--+-=1
9 4

I
,j
fI
q

304 1001 Solved Problems in Engineering Mathematics (2nil Edition) by Tiong & Rojas
Day 12 -Analytic Geometry (Parabola, F.1l!Ese & H.:meEola) 305.
Thus, the equation is,

x2
---=1
y2 a Substitute z and

x+(1)+(-1)=2
y in Eq.3:

22 42
P,(6,-2,-3)..-+x 1 =6;y 1 =-2;z 1 =-3 x=2
x2 y2
----=1 P2 (5,1,-4)--~x 2 =5;y 2 =1;z 2 =-4
4 16 Thus the point is {2, 1, -1)
4x2 -y 2 =16 ~--2 2 . 2
d=v\~z-x1) +(y2-y,) +(z2-z1)
Equation of the asymptotes of a
hyperbola with center at (0,0): Ill 2 2
d = J(5-6) + (1 + 2) + ( -4 +3)
2

Standard equation of a sphere with center


d=Ji1 at (0,0): r 2 = x 2 + y2 + z 2


b
y=±-x
a Substitute the coordinates of the given
2 point to the standard equation:
y=±-x
3
2x-3y = 0 or
=
x + 5y- 2z 9 ...-+ Eq.1
r2=x2+y2+z2
3x- 2y + z"' 3---+ Eq.2
2x +3y =0 r2 "'(8)2 + (1 )2 + (6)2
x + y + z = 2---+ Eq.3

• By inspection, the coordinates of the


Subtract Eq.3 from Eq.1:
r 2 =101
p:..fi'01
=


center is at (-2,2); a = 2 and c = 3. (x + 5y -2z)-(x + y + z) 9-2
Standard equation of the ~symptotes of a
4y -3z = 7 -l- Eq.4
hyperbola with center at (0,0): y =±E.a x b = ~c 2 - a 2 = ~3 2 -2 2
b = -J5 Multiply Eq.3 by 3: (x- h)2 + (y- k}2 + (z -1)2 =r2
Given equation of asymptotes: y = ±2x
3x + 3y + 3z =6 ---+ Eq.S where: (h. k, I)= coordinates ofthe center
- -k)2
Standard equation: (x- 2h)2 -(y- =1
By inspection: E.a = 2 ---+ b = 2a a b2
Subtract Eq.2 from Eq.5: As given the center is at (-3, 2, 4), thus
Substitute:
h =- 3, k = 2 and I = 4 and the radius is 6.
(3x + 3y + 3z)- (3x- 2y + z) = 6- 3
xz y2
Standard equation: 2 - 2 =1 5y+2z=3 Substitute to the standard equation:
2 2
a b (X • h) ( y-k) _
3 5
- - 2-.. - - - 2 - - 1
a b
z=----y
2 2 (x + 3}2 +(y -2)2 + (z -4Y =62
Substitute b = 2a and the coordinates of
point (5/2, 3) to the standard equation: (x+2)
2
(y-2) _
2 x2 +6x+9+y2 -4y +4 + z2 -8z +16:::: 36
------1 Substitute in Eq.4:
22 ( -!5)2 x2 +y 2 +z2 +6x-4y-8z 7 =
(5/2)2 _K=1
a
2

25
4a
(2a)
9
- -2 - =2 1
4a
2

16 = 4a 2
(x+2f
------1
4
2
5
(y-2)

=
5(x + 2) - 4(y- 2)2 20
5(x + 4x + 4)- 4(y - 4y + 4) =20
2
2 2
2
_ 4y-3(%-%v)=7

9 15
4y--+-y=7
2 2
8y - 9 + 15y = 14
y=1

Standard equation: (x - h )2 + (y - k)2

Substitute coordinates of the center and


=r2
a=2 5x + 20x +20-4y 2 +16y -16 = 20
radius as given:
b =2a =2(2) 2 3 5
5x -4y2 + 20x +16y -16 = 0 z=---(1)=-1
2 2
b=4 (X-4)2 +(y -0)2 = 4 2
2
x - Bx + 16 + y2 16 =
.
II
306 lOO(Solved Problems in Engineering Mathematics (2nd Edition) by Tiong & Rojas '
I
:~

x 2 + y 2 = 8x ~ Rectangular equation
ml
. Standard equation: (x- h) 2 + (y- k) 2 = r 2

Substitute coordinates of the center and Topics


ra~ius as given:

(x- 3)2 + (y- 0) 2 = 32 D Mon


Parabolas
- General & Standard Equations
Refer to the triangle: x2 - 6x + 9 + y2 =9 - Eccentricity & Latus fjectum

x2+y2=r2
x =rcos9
y~ x 2 + y2 =·6x

Refer to the triangle:


D Tue
Ellipses
-General & Standard Equations

y~
- Major & Minor Axes
Substitute:
X

x2 + y2 =r2
x=rcos9
D D
Theory Wed
- Eccentricity & Latus Rectum
Hyperbola
x2 +y 2 =8x
l_j D
x. ... General & Standard Equations
2 -Transverse & Conjugate Axes
r'a = 8()1_ cos 9) x + y2 =6x
Problems Thu - Eccentricity & Latus Rectum
r =8cos9

D D
ra = 6()1_ cos 9) Polar Coordinates
r-8cos9=0 r = 6cos9


- Radius Vector


Solutions Fri - Polar Angle

~ Relation Between Rectangular

y~
and Polar Coordinates
Refer to the triangle: Notes Sat
Refer to the triangle:
r sin 2 e = cos e
X
x=rcos9
y~
r(~J =(7) y =r sine
.x
y2 =x Given: r = 5 sin 9 and 9 = 30°

Note: This equation is a parabola with Substitute:


vertex at (0,0) and opening to the right.
x=rcose
Standard equation: y2 = 4ax x = (5 sin e)cos e = 5 sin 30° cos 30;
X= 2.17
By inspection, 4a = 1 thus, a = 1/4
y =r sine
y = ( 5 sine) sine = 5 sin 2 30°
y =1.25

Thus, the point is at (2.17, 1.25).

Thus, the focus is at (1/4, 0).

+ ' A ~
308 . 100 1 Solved Problems in Engineering Mathematics (2nd Edition) by Tiong & Rojas

Topics
[QJ Differential Calculus
Limits
Mon
Theorems of Limits
0
Tue
One-Sid~d Limits
Continuity
Special Limits

Theory
0
Wed
Derivatives
-Algebraic Function
<";.<·, ,,,

0
Problems
0
Thu
-Exponential Functions
- Logarithmic Functions
-Trigonometric Functions

0
Solutions
0Fri
- Inverse Trigonometric
Functions
- Hyperbolic Functions

0
Notes
0
Sat
- Inverse Hyperbolic Functions

What is Calculus? What,is Differential Calculus?

The term "Calculus" was derived from a Differential Calculus is a branch of


Latin word "calx" which means "stone" mathematics which deals with derivatives
and from a Greek word "chalis" which and limits.
means "limestone".
What is a Limit?
~ ~ < In 1684, a German mathematician and
philosopher Gottfried Wilhelm von Let f be a function that is defined on an
Leibniz published his early work on open interval containing a, but possibly not
calculus, while an English astronomer, defined at a itself. Let L be a real number.
physicist and mathematician Isaac The statement
Newton made an early study on the
subject in 1665 but did not published his limf(x) = L
x->a
work until 1704. These two
rn<t!hematicians are now considered as the
defines the limit of the function f(x) at a
founders of today's calculus.
point a. L is the value that the function has
as the point a is approached.
1111 · ,;ulljed Calculus is divided into four
,,,,.,~.; lldllldy, differential calculus,
What are the Theorems of Limits?
inft·qra! calculus, differential equations
.l!rtl c.llculus of v<Jriiltioro.
lhu followin(J are important theorems and
pr<>Jll'tlr•·'; 11! lrr;~rt'; con:oirl.-:rinq thr: ·
fiiJJ(lWIII<J

j
310 100 ~ Solved Problems in Engineering Mathematics (2na Edition) by T1ong & Rojas Day_l3 -Differential Calculus (Limits anci Derivatives) 311

lim f(x) = L and lim g(x) = K What are One-Sided Limits? What is a Continuity? 6. For 0 < a < 2·, then:
X-Joa X-+a

Suppose f is a function such that it is not A function f is continuous at a point a if lim ax= 0
1. If lim f(x) exists then it is unique. defined for all values of x. Rather, it is X--++«·
X->8
defined in such a way that it "jumps" from lim f(x) = f(a)
·one y value to the next instead of smoothly x--+a
lim ax= +oo
2. lim[f(x)+g(x')] = limf(x)+ limg(x) going from one y value to the next. X->-oo
X-Joa , X-J-3 X-+3
The following are the three conditions
lim[f(x) + g(x)] = L + K y being satisfied:
X->8 lim IQg. x = -CJJ
X->+oo

3. lim[f(x)-g(x)]=limf(x)-limg(x)
~ 1. f(a) exists, that is, f is defined at a,
x-+a x-Joa X-Joa lim log. x = +oo
x->0
lim[f(x)- g(x)] = L- K 2. lim f(x) exists,
X->8 x--+a
What is a Derivative?
----------4r------------ X
4. lim[f(x)•g(x)] = limf(x)-limg(x) 3. the two numbers are equal.
X-43 X-Jo3 X-+8
The derivative of a function expresses its
= a: rate of change with respect to an
lim[f(x)·g(x)] = L·K
X->8
The figure above shows that y =f(x) is not Continuity test at a point x
independent variable. The first derivative
defined for all values of x.
of a function is the slope of the tangent
lim F(x) = lim F(x) = F(a)
y line to that curve defined by the function.
lim f(x) = ~i~ f(x) =!::
X-+3+ X-+3-

5. (K ot 0)
x->ag(x) limg(x) k The following are the derivatives of the
X->8
What are Special Limits? different functions:

6. lim--=~
1
(K ot 0) A. Algebraic Functions:
x->a g(x) K 1. lim sinx = 1
x-+0 X

1. de =O
7. lim[f(x)j"
x ...... a
= lim[f(x)J" fern>
x~~a
o 2. lim 1- cosx = 0 dx
X--+0 X d du dv
2. -(U+V)=-+-
The fi9ure above shows that y = f(x) dx dx dx
8. ~i~[cf(x)] = c[~~f(x)J "jumps" from a positive value to a negative d dv. du
value. 3. lim
n-+oo
(1 + ~)"
n =e
3. -(uv)=U-+V-
dx . dx dx
9. lim ex" = c lim x" = ca" du dv
The statement lim f(x) = R means that as
X--78 X---4-8
x-Joa+ 1
4. ~(~) = vdX -udx
10. Iff is a polynomial function then
x approaches "a" from the right or from 4. lim{h n)n = e
n-+0 •
dx v v2
positive infinity, the function f has the limit
d n n-1 du
R. 5. - u =nu -
lim f(x) = f(a) 5. For a > 1, then: dx dx
X-->8
Likewise, the statement lim f(x) = L
X->8 lim ax= +oo du
11. lim
X-----+a
'1fW = \Jnllim f(x)
X-+a
means that as x af?proaches "a" from the X-HOC

6. ~JU = d~
left hand side or from negative infinity, the
when n is positive integer dx 2vu
function f has the. limit L. lim ax= 0
X--+-oo 7.
12 If f(x)::; h(x)::; g(x) for all x in an open Iff is defined in <:~n open interval containing d (u)
interval containing a, except possibly a, except possible at a, then lim log)'= +oo
X H·OO
8.
dx c =c1 dxdu
at a, and if lim f(x) = L = lim g(x) then du
X-Joa X-+3
limf(x) = L lim log. x = -CJJ d c c dx
limh(x) = L
x->a x ~o 9.
X--+8 dxu=lT
if and only if lim, f(x) ·~ lim f(x)
X· ><I X >il

• J
3~2 1001 Solved Problems in Engineering Mathematics (2nd Edition) by Tiong & Rojas Day 13- Differential Calculus (Limits and Derivatives} 313

B. Exponential Functions: d -1 -1 du Proceed to the next page for your 13th


25. -(esc u ) = - - = -
dx uN -1 dx test Detach and use the answer sheet
9. -~(a")= a" Ina~-~ provided at the last part of this book. Use
pencil number 2 in shading your answer.
dx dx F. Hyperbolic Functions: .
10. _E_(e") =e" du GOOD LUCK I
dx _ dx
26. ~(sinhu) = coshu du
dx dx
C. logarithmic Functions: 27. ~(cosnu) = sinhu~~ m:ribia:
dx dx Did you know that. .. the most proved
d 2 du theorem in Mathematics is the
28. --,(tanhu)=sech u-- Pythagorean Theorem which has more
11 d log du dx . dx
dx .u =-~
. ·-·(log ) a e-
u d 2 du
than 370 different proofs! All the 370
29. -(cothu) = -csch u - different proofs are found in the book
dx dx entitled "The Pythagorean Proposition"
12 d log du d du
. -(log ) 10 e - which was published in 1940
dx 1ou = .u dx 30. -(sechu) = -sechutanhu-
dx .dx
du
31.
d
-(cschu) =-cschucothu-
du c!euote:
13 d -- dx dx "The mathematician does not study pure
. dx(lnu)=·dx
u mathematics because it is useful; he
G. Inverse Hyperbolic Functions: studies it because he delights in it and he
D. Trigonometric Functions: delights in it because it is beautiful."

d . du
32. ~(arcsinhu) = ~ - Henri Poincare
14, -(s1nu) =cosu- dx u2 + 1
dx dx
d 1
. du
d (COSU ) =- -SinU-· 33. -{arccoshu) = ~; Jxl > 1
15. - dx 2
u -1
dx dx
d 1
16.
d
-(tanu) =sec u-- 2 du 34. -(arctanhu) =--· lxl < 1
dx dx
dx · 1- u2 '
d 1
H.
d ·
--(cotu) =-esc u -
2 du 35. -(arccothu) =- - ; lxl > 1
dx dx dx · 1- u2
d du d 1
18. -(secu) = secutanu- 36. -(arcsechu) = - - - ; O<u<1
dx dx dx x~1-x2
d du
19. --(cscu) = -cscucotu--
dx dx d 1
37. -{arccschu) = ~; x *0
dx JxJ 1+ x2
E. Inverse Trigonometric Functions:
20. ~(sin-1 u) = _1__ du
dx ~1-u2 dx

21. -~(cos- 1 u) =- c--1 du


. -u2 dx
dx · "1/l-
1
d _1 _ _ _1___ du
'""'
22. dx (tan u)- 1+ u2 dx

d~ (cor-1 u) = t+ u2 dx
--1 du
23.

24. :x (sec-1 u) = uf:l1 1


du
a;
_1
:111

Day 13- Differential Calculus (Limits and Derivatives) 315

5&o: EE Board April :1995 5&5: EE Board October :1997


1- cosx Differentiate y = ex' cos -?-.
Evaluate: Lim _x_2_
X--+0 A -ex sin x!-
B. ex (cos x!- - 2x sin x!-)
A. 0 C. ex cos x!- - 2x sin x!- 1r
B. 1/2
Topics C. 2
D. -2xex sin x
II.,
D. -1/2
Differential Calculus 5&&: EE Board October :1997 ,I

Differentiate y = sec (x!- + 2).


Mon
limits 5&:1: ME Board October :1997
Theorems of Limits Compute the following limit:

D
Tue
One-Sided Limits
Continuity .
Limit:
X+4
X-+oc x-4
A
B.
C.
D.
2x cos (x!- + 2)
- cos (x!- + 2) cot (x2 + 2)
2x sec (x2 + 2 ) tan (x!- + 2)
cos (x!- + 2)
Special Limits

D D
Theory Wed
Derivatives
- Algebraic Functions
A.
B.
t.
1
0
2
5&7: CE Board November :1994
What is the derivative with respect to x of
(x + 1)3 - x3 ? ·

D
Problems Thu
- Exponential Functions
- Logarithmic Functions
-Trigonometric Functions
D. Infinite

56:&: EE Board Octuber :1994


A.
B.
3x + 6
3x- 3
3x 4 - 2x 2 + 7
D D - Inverse Trigonometric . C. 6x- 3
Evaluate: Lrm ---,3, - - - - D. 6x + 3
Functions X-+oc
5x +x-3
Soi~Jons Fri - Hyperbolic Functions 5&8: EE Board October :1997

D D
Notes Sat
- Inverse Hyperbolic Functions A.
B.
C.
Undefined
3/5
Infinity
Differentiate y = log 1o (x!- + 1)2

A. 4x{x2 + 1)
D. Zero 4xlog10 e
B.
x2 + 1
5&~: ECE Board November :199:1 C. log e(x)(x!- + 1}
55&: CE Board November :1997 558: ME Board April 1998 Evaluate: Lim (x!- + 3x - 4) D. 2x(x!- + 1)
x2 -1 2
x -16 X--+1
Evaluate: Lim Evaluate the Lim
2
x +3x-4 x-4 5&9: EE Board October :1997
112
X--+1 X--+4 A. 24 Differentiate (x2 + 2)
A. 0 B. 26
A 1/5 C. 28
B. 1 (x 2 +2)1/2
B. 2/5 D. 30 A.
C. 8 2
C. 3/5
D. 16
D. 4/5 5&4: ECE Board November :1994 X
B.
559: ECE Board April :199~ tan~ (x 2 +2)1'2
557: ECE Board April :1998 Evaluate: Lim (2- x) 2
(x-4) x2 -4 C.
2x
Evaluate: M = Lim - -
Evaluate: Lim ------o:'-----'-
2 x-2 X--~ 1 (x 2 + 2)1'2
x--+ 4 (x - x -12} X--+2
ez. D. (x!- + 2)3/2
A.
A 0
A. Undefined
B. 2 B. e'l/Jr
B. 0 570: EE Board October :1997
C. 4 C. 0
C. Infinity If y = (f + 2)2 and t = x 112 • determine dy .
D. 6 D.
D. 1/7 I dx
:

I
!I
1
l

316· 1001 Solved Problems in Engineering Mathematics (2nd Edition) byTiong & Rojas Day 13- Differential Calculus (Limits anciJJerivatives) 317

3 4 9 3 584: ECE Board November :199:1


A. - B. B. 4y Find the slope of the line tangent to the
2 (1-16x2 )o5
curve y = x 3 2x + 1 at x = 1.
2x 2 +2x 32 xy
C. -
--4
B.
3
c. (1- 4x2)o.s 16 3
D. -y A. 1
c.. 2(x + 2)
4
9
B. 1/2
D. x5'2 +x1'2 D. C. 1/3
(1- 4x2 )o5 579: ME Board April :1998
D. 1/4.
57:1: ME Board April :1.997 Find the partial derivatives with respect to
What is the first derivative of the 575: CE Board November :1.996 x of the function xy2 - 5y + 6.
585: ECE Board November :199:1
3
expression (xy)x = e? Find the derivative of (x + 1) A. y2-5
Give the slope of the curve at the point
X B. y2 x3
(1,1): y=--2x+1
A. 0 C. xy- 5y 4
X (x + 1)2 _ (x + 1)3 D. 2xy
B. -y A. A 1~
X X
580: ME Board October :1997 B. ~~
4(x + 1)2 _ 2(x + 1)3
c. -y (1+1nxy). B. Find the second derivative of x 3 - 5~ + x = C. 1 1~
X X X 0. ~ ~1~
-y (1-lnxy) 2(x + 1)3 (x + 1)3
D.
x2
c. --x---r A.
B.
10x- 5
6x-10
58fn ECE Board November :1998
Find the slope of x2 y = 8 at the point (2,2).
2 3
3(X+ 1) 1) (X+
572: ME Board April <:f98
Find the derivative with respect to x the
D. --x----xz C.
D.
3x + 10
3~-5x. A. 2
B. -1
function .J2- 3x 2 576: ECE Board November :199:1 58:1: ME Board April :1998 C. -1/2
2 Given the function f(x) = x to the 3rd power D. -2
-2x 2 Differentiate the equation: y =_x_ - 6x + 2. Find the first derivative at
A. X+1
X= 2. 587: CE Board May :1.998
.J2-3x2 x2+2x
A. Find the slope of the curve ~ + l- 6x +
-3x (x+1t A. 6 10y + 5 = 0 at point (1 ,0).
B. B. 7
.J2-3x2 X
B. - C. 3x2 -5 A 1/5
-3x 2 X+1 D. 8 B. 2/5
C.
.J2-3X2 c. 2x C. 1/4
2x 2 582: CE Board May :1996 D. 2
3x -
D. D. Find the slope of the ellipse ~ + 4y2- 1Ox
X+1
.J2-3x2 - 16y + 5 = 0 at the point where 588: CE Board May :1996
y = 2 + 8° 5 and X = 7. Find the slope of the tangent to the curve,
577: CE Board November :1995
573: EE Board April :1.995 2 y = 2x- ~ + x3 at (0,2).
The derivative with respect to x of2 cos
Find y' if y = arc sin cos x A. -0.1463
(~ + 2) is
B. -0.1538 A. 1
A. -1 C. -0.1654 B. 2
A. 2 sin (x2 + 2) c~s (~ + 2)
B. -2 D. -0.1768 C. 3
B. -2 sin (~ + 2) cos (~ + 2)
C. 1 D. 4
C. 8x sin (~ + 2) cos (~ + 2)
D. 2 S8:Jz EE Board October :1997
D. -8x sin (x2 + 2) cos (~ + 2)
If y = 4 cos x + sin 2x, what is the slope of 589: ECE Board April :1999
574: CE Board May :1997
578: CE Board November :1993
=
the curve when x 2 radians? .Find the coordinates of the vertex of the
Find the derivative of arc cos 4x. parabola y = ~ - 4x + 1 by making use of
Find the second derivative of y by implicit
A 2.21 the fact that at the vertex, the slope of the
differentiation from the equation 4~ + 8y2
--4 B -4.94 tangent is zero.
A. = 36. c; 3.25
(1-16x 2 ) 0 5
A. 64~
ll /21 A. (2,-3)

I
J
:Jls 1001 Solved Problems in Engineering Mathematfcs (2nd Edition) by Tiong & Rojas

B. (3,-2) 595: ECE Board November 1.996


c. (-1 ,-3) Find the radius of curvature at any
D. ' (-2,-3) point in the curve y + In cos x = 0.

590: ECE Board April 1:999 . A. cos X


Find the equation-of the normal to
. 5 at the point (2, 1).
x? + y2 = 8. 1.5707
C. secx ......,.,._,_____ ~, . ______
Topics

~
D._ 1
A. y = 2x Differential Calculus
B. X= 2y lv1on Limits
C. 2x + 3y = 3 Theorems of Limits
D. X+ y = 1

59:11 CE Board May :1995


i D Tue
One-Sided Limits
Continuity
Special Limits
D D
What is the equation of the normal to the ·,·,I

curve x? + y2 = 25 at (4,3)? .. Derivatives


A. 5x + 3y = 0 Theory Wed - Algebraic Functions
B.
C.
D.
3x-4y = 0
3x + 4y = 0
!}x- 3y = 0
D D
Problems Thu
-Exponential Functions
- Logarithmic Functions
- Trigonometric Functions
59%: EE Board April :1997
Locate the points
y = f(x) =
X: e•
of inflection of the curve D
Solutions Fri
- Inverse Trigonometric
Functions
-Hyperbolic Functions
A. -2± J3 i.;
D D - Inverse Hyperbolic Functions

2 ± J2
l
B. Notes Sat
C. -2 ± J2 .\
D. 2 ± J3
ANSWER KEY RATING
593: ECE Board November :199:1
In the curve 2 + 12x- x 3 , find the critical 556. B 566.C 576.A 586. D :,"
p
c:J 34-40 Topnotcher
587. B
points. 557. D
558.c
567. D
568. B
577. D
578. B 588. 8 c:J 24-33 Passer
A.
B.
(2,18) & (-2,-14)
(2,18)&(2,-14)
559.c
560. B
569. B
570.C
579. B
580. B
589.A
590. B
0 20-23 Conditional

C.
D.
(-2,18) & (2,-14)
(-2,18) & (-2,14)
561. A
562.c
571.c
572. B
581. A
582. D
591. B
592.c
0 0-19 Failed

563.A 573.A 583. B 593.A If FAILED, repeat the test.


5941 CE Board November :1997 564. B 574.A 584.A 594.A
Find the radius of curvature of a parabola 565. B 575. D 585. D 595.C
y2 - 4x = 0 at point (4,4).
";' ' -~

A. 22.36 units
B. 25.78 units
C. 20.33 units
D. 15.42 units

I
~~
Day 13- Differential Calculus ~Limits and Deriv!!_tives) 321


320 1001 Solved Problems in Engineering Mathematics (2nd Edition} by Tiong & Rojas

•. x2 -1
L1m -..--::--
x-.1
2
x +3X --4
Apply L'Hospital's rule & substitute x = 4:

. 2x
L 1m=-= 2 x
1
lim= 2(4) =8
•. x+4
Ltm - -
x-->«> x-4
lim x2 + 3x- 4
X·->4

Substitute x = 4:


Substitute x = 1: Substitute x = co:
Lim=
2
(1) ~1 0 Lim =(4)2 + 3(4)- 4
. oo+4 co 'd .
(1)1 +3( ).
1 4
=0 .indetenninate L tm =---=-,
oo-4 oc.
m etermmate, Lim= 24
. X2 -4
Llm.--
Note: Using L'Hospital's rule, differentiate x-+2 x-2 Apply L'Hospital's rule & substitute x = oo:
separately the numerator and
denominator and substitute the value of Substitute x = 2:
Ill
limit to the variable 1 Lim=2=1 Lim (2 _ X)tan 1t~
l 1 X-+1 2


. (2)2 - 4 0 . d t . •
L1m = - - · =- m e ermma,e
Lim= 2x 2-2 o· -
Substitute x ::: 1:
2x+3
Apply L'Hospital's rule & substitute x = 2: .I
Lim=~-2
2(1)+3 -5 . 3x 4 - 2x 2 ·+ 7 Lim=(2~1)1an.'!~ =1"', indete1minate
. 2x ltm ·--~-­


3 5x +X -3
L1m=-= 2x x->oo
1 Take In on both sides:
lim =2(2}=4 Substitute x = oo:


. tanJI!I
In Ltm =In (2-x) 2
. x-4 Lim= 3(oo)4- 2(oo)2 + 7.
Ll m2- - -
x-+4 x -x-12 5(oo)3 +oo--3
In lim= tan _1tx (In (2- x))
2
'1\:
Lim 1-cosx lim == .~. indeterminate ..: ln(2- x)
Substitute x = 4: -~
x-+o -x-2 ~r_, 00 - 1

Lim= 4-4 0 tan n:x


(4~ -4-12 = 0· indeterminate . 1-cosO 0 . d t .
Apply L'Hospital's rule & substitute x = oo: 2
L1m = - -2- =- 1n e ermmate
(0) o· . ln(2--x)
in Lim ::: ------
12x 3 - 4x
Apply L'Hospital's rule & substitute x = 4: Lim = - 15x2 + 1 cot2:.~
Apply L'Hospital's rule & substitute x = 0: 2
3
1 . 12(oo ) - 4( oo) = 00 • indeterminate
Lim= 2x-1 . sinx Ltm == 15( 00 )2 + 1 oo Note:
Llm=-- du 2 ·
2x
1 :..! d(lnu) = - and d(tan u) =-esc u du
u
Lim= 2 (4 )-1 7 Lim = sin 2(0) = ~ indeterminate Apply L'Hospital's rule again:
2(0) o·

• 36x 2 -4 Apply L'Hospital's rule:


Lim= 30x -1 -1
Apply L'Hospital's rule again:
lnlim=---.£:._x_ = · 2-1
2 Lim =
36( oo l-
4 _ '!!._ indeterminate
30( oo)- - <X> ' csc2 n;(~) -csc2~~)(~)
Lim x -16 Lim= COSX
X-+4 X -4 2
Lim= cosO
-1 2
Substitute x = 4; ---2-=21 Apply again L'Hospital's rule:
:n: 1t

l2x 72(co) 2
ltm '- .:: .. --'~ oo
4 2 16 30 30 In Lim= 2/n
Lim= ( ) " =.Q. indeterminate
4-4 o·

j
322. 1OOJ Solved Problems in Engineering Mathematics (2nd Edition) by Tiong & Rojas
I
!I Day 13- Qifferential Calculus (Limits and Derivatives) 323.

Take exponential on both sides:

eln Lim = e2/n


Lim= e2 /n,

Y = log1o (x2 + 1)2

(xy)x =e.

Take in on both sides:


,
y=

, -sin x
y=-.-
Sin X
-sin x
~1- cos 2
x
=--
-sin x
~sin 2 x


y' = -1
Note: d(log10 u) = log10 e ( duu)
In (xy)x = In e

where: x In (xy) = 1 ml
y =ex cos x 2 u = (x 2 + 1)2
Note: d (uv) = udv + vdu y =cos-14x
du = 2(x 2 + 1)(2x) = 4x(x 2 -t·1)
Note: d(uv) = udv + vdu
Differentiating both sides:
where: y' =log1o e
2
4x(x +
2
1)] Note: d{ cos-1u) = ~1- u2
-du

u =ex; du = ex; v = cos x 2 ; dv = -2x sinx2 • [ ( x2 + 1} \ [ x\:y ]+In xy(1) = 0

where: u = 4x; u2 =16x


2
y' = 4x log10 e ; du = 4
y' = ex(-2x sin x 2)+ cos x 2 ex xy'+y+ylnxy=O
x 2 +1 xy' = - y - y In xy
y'= ex(cosx 2 -2xsin x 2) -4
=- y(1 +In xy)

• ~1-16x 2
xy'
Ill y' = -y (1 +In xy)
y'=

X
-4
r

5
Y= (xz + 2)1/2 v· = (1-16x2
2
y = sec(x + 2)

Note: d sec u = sec u tan u du

where: u = x 2 + 2; du = 2x

y' = sec (x 2 + 2) tan (x 2 + 2)(2x)


Note: d(u)" = nu"·1 du

where: n=112;u=x 2 +2;du=2x

y' = ~(>.<z + 2 )112-1( 2 x)


Y =~2-3x

Note:
2

dJU = 2JU
du
•y = (x + 1)
X
3

y' = 2x sec (x 2 + 2) tan (x 2 + 2)


~)
y' = x(x2 + 2f112 vdu-udv
where: u = 2-3x 2 ; du =- 6x Note: d( v2

=---X ··-
IB y
-{)x
where:
y' = 2~2 -3x 2
y = (x + 1)3 - x3

Note: d(u)" = nu"-1 du •


t=x 112 ~Eq.1
-3x
y'=-~2-3x 2
u = (x + 1)3 ; du = 3(x+1 )
v::: x; v 2 = x 2 ; dv = 1
2


2 3
y' = x(3)(x+1) -(x+1) (1)
2 2
y = (tz + 2)2 ~ Eq.2 x2
y' = 3(x + 1) (1)- 3{x) (1)
2 3
y' = 3(x + 1 )2 - 3(x)2 Substitute Eq.1 in Eq.2: ,_3\(x+1) (x+1)
y sin 1cosx y- - -2 -
y' = 3(x 2 + 2x + 1)- 3x 2 x2, x

y'= ~ +6x+3-}~
y = [(X112 )2 + 2 J N••terl(sin- 1
u)::: ~
, _ 3(x + 1)
y-------
2-
2
( x + 1)
3

y'=6x+3 Y = (x +2)2 x x
v1- u2
y' = 2(x +2) wtww u cosx; u2 =cos 2 x; du =-sin x

_J
324 1001 Solved Problems in Engineering Mathematics (2nd Edition) by Tiong & Rojas


Day 13.-_Q_if!~enti<ll Calculus (Limits and Derivatives) 325
J

2 2

•! (
y' = -4x[ 2cos( x + 2)sin( x + 2)

+ 2)sin{x + 2) J
2 2
x2 y' =-ax[ cos( x
y= x+1

Note: d(~) = vdu -.udv


v v2

4x 2 +8y 2 = 36
!(
xy

xy
2

2
-

- 5y + 6) = y
2
5y + 6) = y (1) + 0

2
y = 4 cos x +sin 2x

Differentiate:


y' = 4 (-sin x) +cos 2x (2)
where: x2 + 2y 2 = 9 ~ Eq.1 y' = 2 cos 2x - 4 sin x
u.,x2 ; du = 2x
Differentiate both sides: At x = 2 radians
v = x + 1; v 2 = (x + 1)2 ; dv = 1
y = x 3 -5x 2 + x
2x,+4yy' = 0
2 2 2 y' =3x2 - 1Ox + 1 1 00
y' = (x + 1)(2x)- x ( 1) =2x + 2x- x -2x y' = 2 cos 2[ 2( : ) ]-4 sin [ 2( 1:0")]
y'=- y" = 6x-10
(x+1)2 · (x+1) 2 4y
2x 2 + 2x- x 2
y' = .=_--=~-
(X+ 1)2
, x2 +2x
Y = -(x_+_1-)
2
-X
y'=-~Eq.2
2y

Take second derivative:



y = x3 -6x +x
y' = 2 cos 229.183°-4 sin 114.591"
y' =-4.94

rm.
• 2
y = 2cos (x + 2)2
y" = -M y(1~; xy']

Substitute Eq.2 in Eq.3:


~ Eq.3

.,
Differentiate and substitute x

y' = 3x 2 -6
y' =3(2)2 -6
y'=6
= 2:
Y=x3 -2x +1

Note: slope = y'

Differentiate and substitute x = 1:


Note: cos2 e = 1+cos 26
y"=-~
y-x( -x J] y' = 3x 2 -2.
.
S1mplifying:
2
2
[
y2
2y 1!1 y' = 3(1)2 -2
y' =1
2
Y = 2cos (x + 2)
2
x2 +4y2-10x -16y +5 = 0
y+ X
Y = ~[ 1+ cos i x +2)]
y"=-i[ /y2]
2
Note: Since the line is tangent to the
Note: slope = y'
curve at x = 1, then the slope of the line is
the same as the slope of the curve at the
As given: y = 2 + 8°· 5 = 4.828, x = 7
=
Y 1 +cos 2(x 2 + 2) given point.
y = 1 + cos (2x 2 + 4) y" = __1_. [2y2 + x2 J Differentiate & substitute x = 7 & y = 4.28
Thus, the slope of the line is equal to 1.
2l 2y
Note: d(cosu) = -s.inu du

where: u =2x 2 + 4; du

2
y' =-sin (2x + 4)(4x)
= 4x
y"= 2y2 +x2
4y3

Substitute Eq.1 in y":


2x + 8yy' -1 0 -16y' = 0
y'(8y -16) = 10- 2x
, 10-2x
y=--
8y-16
• x3
y=--2x+1
4
y' = -4x sin 2(x 2 + 2)
.9
y"=--3-
I
I
'- 10-2(7)
y- 8(4.828)-16 Note: slope = y'
Note: sin 29 = 2 cos. e sin e 4y y' = -0.1768
Differentiate and substitute x = 1:
I
iir'
,I
j !. {

i
326 1001 Solved Problems in Engineering Mathematics (2"d Edition) by Tiong & Rojas Day 13- Differential Calculus {l.ixnits a.11d Derivatives) 327

Note: slope = y' Note: Since the line is normal to the curve Note:
y' = _!(3x 2 )- 2 at the given point, the slope (m2) of the d(uv) = udv + vdu
4 Differentiate and subsfih.ite x c,.O. line is equal the negative reciprocal of the
d(eu) = eudu
slope of the given curve.
y'::: ~(3)(1 )2 - 2
4 y' = 2 - 2x + 3x 2
1 1 - 1 y' = x 2 (ex)+ex(2x)
y'=-1 ~ y' = 2- 2(0) + 3(0) 2 mz = - m1 = - -2 - 2
4 y" = x 2 (ex) + ex(2x) + ex(2) + 2x(e')


y' ~2
y" = x 2 ex + 4xex + 2ex
Using point slope form:
Note: Since the line is tangent to the
curve at (0,2), then the slope of the line is y-y1 =m(x-x 1) At point of inflection, y" = 0:
2
x y =8 the same as the slope of the curve at the
1
8
given point. y-1=-(x-2)
2
O=x 2 ~ +4x~ +2~
y=-
x2 Thus, the slope of the line is equal to 2. 2y-2 = x-2 O=x 2 +4x+2


x=2y


Note: slope = y' By quadratic formula:

Differentiate and substitute x = 2: 2


- -4±)(4) -4(1)(2) = -4±.J8
Y = x 2 - 4x + 1 x- 2(1) 2
, _ S(-2x) _ 16
y ------
x4 x3
x2 + y2 = 25
Note: slope = y' -4±2.J2
x=
' 16 Let: m1 = slope of the given curve 2
y=-(2)3 Differentiate and y' = 0: m2 = slope of the normal line X =-2±.J2
y' =-2
y' = 2x-4 Differentiate and substitute x = 4 & y = 3: Substitute the values of x to the given
0=2x-4
Ill x=2 2x +2yy' = 0
, X 4
equation to solve for the values of y:

y =-- =-- y =(-2+J2)2 e(·Z+J2) =0.19or


2 Substitute x = 2 to the given equation: y 3
x +f-6x+10y+5=0
y=(-2-J2) 2 e(·Z-J2) =0.38
4
y = (2)2 - 4(2) + 1 =- 3 m1 =-.-
Note: slope = y' 3
Note: From the choices, only values of
1 1 3 the x-coordinates are given. Thus the
Differentiate and substitute x = 1 & y = 0. Thus, the vertex is at (2, -3) mz=--=--=-
m1 -4/3 4 suggested answer is choice C.
2x + 2yy' - 6 + 1Oy' + 0 = 0
y'(2y + 10) = 6- 2x
6-2x
y' = 2y +10
'- 6-2(1) 4
•x2 + Yz = 1

Let: m1 = slope of the given curve


Using point slope form:

y-y1 =m(x-x1 )

y-3=-(x-4)
3
4
•Y= 2+12x-x 3

y- 2(0)+10 -10 4y -12 = 3x -12 Note: Critical points are points wherein
Differentiate and substitute x = 2 & y = 1: the slope of the curve is zero.
y'=~ 3x-4y=O
5 y' = 12 -3x 2
4~~ 2x +2yy' =0

~
,
y =--=--
X 2 1m 0 = 12 -3x 2
~ y 1 x2 =4 •
y' =-2 Y=x 1 ex X= ±2
3
Y =2x- x2 + x ml c·o •. ;:o

j
/

328 . 100 1 Solved Prob1Efms i!l Engineering Mathemati~_:C! Edition) by Tiong & Rojas

Substitute the values of~ to the general


equation to solve for the\values of y:

y = 2 + 12(2)- (2)3
y =2+12(-2)-(-2)3 =-14
=18 or •
R:: [ 1+(yiJ
jy''l -
12
~~
'{
&
Topia
Thus, the points are (2, 18) and (-2, ··14). Differential Calculus
y+ln COSX =0 Mon Limits
Ill y =-In cos X Theorems of Limits

Note: d(ln u) = \]u ,:'!'


D
rue
One-Sided Limits
Continuity
u Special Limits
R =I1+(y')2r2 ,,:{I ·0'
... ···
ly''l-· where: u = cosx; du =-sin x
~~·~
D Wed
... Derivatives
-Algebraic Functions
l-4x =P
2yy'-4=0
• 4 2
y, =- (-sinx)
- - =tanx
cosx
0 D
Problems Thu
-Exponential Functions
- Logarithmic Functions
- Trigonometric Functions
y=-=-

y -
2y y
.. _ y{O)- 2y' _ 2y'
y2
. ----
y2
y" =sec2 x

Substitute y' and y" to solve for R:


D D
Solutions Fri
- Inverse Trigonometric
Functions
- Hyperbolic Functions

y"y2 = -2(~)
r
RJ1+(tan4J
12
D Sat
- Inverse Hyperbolic Functions

· sec 2 ;
4
. y"--- y3
Note: 1+ tan 2 x = sec 2 x
t
Substitute y = 4 to solve for y' andy": 3/2
2
R= ( sec x)
y•::3::3:::.! . sec2 x
y 4 2 3
~ ~ : ~ .0 ~ ~ x.•• ~ ,< 0 '' '< ..... •,-< < -<•·> "- « '> "< " ' : :·,,..,x ~ ~ > )

R::: sec x
y" = --~ :: ___i_ = _...!_
., < ~ '., ~ t , ·, ,< "·: 'i- ~ ~· < ~-..;,:" ~ ~· • '" • "' ~ ~ _., •.,. ~ t•v.; ~ o < ' :! '<" ~ !" > 1o .; ·k ~"' ~

3 3
sec2 x
y (4) 16
R=secx
u ->' h j<- 0< •X X "{ > ;,,. 0. '>< o > .' "l '( H > < <, ;< « ,• < > 0 {, "<, A "0 : "-<: .<• _." V X 0 < '', ~ o' ' ""' 0 0.0 'A V ~

~ 1-: ,,. ' 0 --;. " ' " ·~ { 0 ~· f ' .... 0 ' <I ,. '

Substitute y' and y" to solve for R:

R----
_Hi}T
1
t:. • • .
0 > "( .0 X ~ "< )' '~'._, ~· ., ~ • ,->: [< _.,. t ' ·>

T
~ « o" <' < » X <•

16
' , ... , ,:·,·,
R =22.36

T
J
»I
I
t.
:~

Topics

0
Mon
Maximum and Minimum Values
Local Maximum and Local
Minimum
[QJ Steps in Solving Maxima/Minima
Problems
Tue
Steps in Solving Time Rates

Theory
0
\1\fed
Problems
Relation Between Variables

0
Problems
0 Thu
Under the Condition of
Maxima/Minima

~ : < ,< <:<


0
Solut;ons
0 Fri

~ ~ «:" (;- 9 "~· ::. , ;_


0Notes
0 Sat

f What are Maximum and Minimum


Values? y
« ~ .; "

~
>. """'" :<<,<';.< .~,.; If a function f is defined on an interval!,
then:

1. f is increasing on I if f(x,) < f(x2)


whenever x,, x2 are in I and x1 <. x2.
· !f(x2)
j X
y X·; X2

~ /(!~1
• f(x,)
Decreasing function

3. f is constant if f(x,)
x,, x2 in I.
=f(x2) for every
.--t-- • X
Suppose f is defined on an open interval!
Xi X2
and cis a number in I, then:
Increasing function
1. f( c) is a local maximum value if f(x)
;> f is decreasing on I iff(x,) > f(x2) :s: ~(c) for all x in I.
wlwnever x,, X:> in I. ·.,
2. f(c) is a local minimum value ifl(x)
> f(c) for all x in I.
Day 14.:.. Differential Calculus (Maxima-Minima & Time Rates) 333
332 l 00 1 Solved Problems in En
4 Largest rectangle that can be 8. Rectangle with given area but with
How to Solve Maxima and Minima 1. Draw a figure when necessary. inscribed in a right triangle with the minimum perimeter.
Problems? 2. Formulate equation. sides of the rectangle parallel to
3. Differentiate with respect to time. the legs of the triangle.
The following are the steps in solving
problems involving maxima and minima:

1. Draw a figure when necessary.


2. Determine which variabl~ (the·
dependent variable) is to be
maximized or minimized
4. Substitute the condition I instant
to the equation.

Important: Substitute the given values


only after differentiating.
:I, . ~
!_ b
b
X=-
2
h.
Y=2
9.
Dy X=y
X

Rectangle with given area and


What are some of the Relationships -'----~

3. Formulate equation between the Variables and the minimum perimeter to be fenced
4. Reduce to one variable. Maxima/Minima Values? along 3 sides only.
5. Differentiate 5. Largest rectangle that can be
6. Equate to zero The following are the relationships inscribed in an ellipse. no

In step no. 5, the formulas in finding


between the variables under the condition
of maxima/minima:
b
---~~~ ·fence
needed
derivatives will become very useful. It is
y
recommended to memorize all the 1. Largest rectangle inscribed in a
formulas as mentioned in the previous circle. y
chapter. i X
X=2y
When the first derivative (slope) fs
equated to zero, it results to either
maximum point of minimum point. a b 10. Right triangle with maximum
X= .fi y= .fi perimeter or maximum area.
slope= 0
·············Maximum point

~
6. Largest area of a triangle with
=45°

D
The maximum rectangle is a square. given perimeter.
y =Y

•~b=C=i
2. Largest rectangle that can be
inscribed in semicircle.
e
X

Minimum point···········' c 11. Maximum area with perimeter (P)


slope= 0 ·······"l~/2 given.
7. Sector with given area but equilateral triangle
minimum perimeter.
First Second 3. Largest rectangle that can be
I Maximum Point Derivative
0
Derivative
Negative
inscribed in a triangle with one
r=JA p
1
side lying on the base of the
X=--
Minimum Point 0 Positive trjangle. e = 2 rad y 4.268
Inflection Point 0
b
X=- X
How to Solve Time Rates Problems? 2
h
Another types of problems involving the Y=-2
use of derivatives are the Time Rates
problems.

In solving a problem under time rates, the


following steps are to b~considered:

j
334 100 l.So1v' Day 14 - DifferentialgJlJq_t11~_(Maxima-Minima & Time Rates) 335

12. Maximum light a~ittance for a 16. Stiffest beam that can be cut from 19. Most efficient trapezoidal section. 22. Best possible view of a picture or
rectangular win do surmounted
with an isosceles tri ngle.
a circuJar section of radius r.
~- "Maximum capacity with minimum
clock.

"Stiffness is proportional to the product perimeter"

dl:J
of breadth (x) and cube of width Cv)."
It is ~ of a regular hexagon.
X I \X

t<= width at top

~"V"~
~---- --= ~q'T~.w&s:<t20t~~-, r
y y = x.../3 : :
j X j

Best view means e is maximized,


X
-• Ym]Y
........... X= ~Y1Y2
13. Maximum light admittance-for a base (not side)
Norman wind 23. Parallelepiped with maximum
17. Strongest beam that can be cut width at top = sum of sides volume.
from an elliptical section. width at top = 2x

I....,
········a···········
················rx/2 e = 12oo

LJJ
"Strength is proportional to the product Z X=Y=Z
h ........ X= 2y of breadth (x) and square of the depth .
(y)"
:. a cube
20. Length of rigid beam that can pass
,..l' h·' b a perpendicular hallways.
X .
y

X
a, It r 1\ : ...
24. Open squar~ container with
maximum volume.
· 14. Maximum length of line segment X=2bH
tangent to an ellipse.
y
y=2a~ ·······
.... ··"
......... .

~

18. largest rectangle that can be


inscribed in a given ellipse.
' L = ~I{ a2/3 + b2/3 )2

21. Minimum length of ladder/rod to be


X
.
X
y X=2y

X=
's_u_rf=-a_c_e_a_r_e_a
3

extended from ground to a wall 2~. Location of single stake at ground


L=a+b ~ with an intervening fence. level to minimize length of wire.

15. Rectangle of maximum perimeter


inscribed in a circle of radius r. stake
h2
X
h1

~ r}
x=y Aellipse _ 1t iC?'"' d
,(\rectangle - 2 I L= ~(a2/3 +b2/3t x- --
dh1
- h1 +h2
i-i
336. 10.0 l Solved Problems in Engineering Mathematics (2ild Edition) b~g & Rojas Day 14 - Differential Calculus (Maxima-Minima & Time Rates) 337

26. least amount of material to be 30. least amount of material for a 34. Maximum volume of right
,;•,
used for a square base rectangular given volume. '.-\. circular cylinder inscribed in a
parallelepiped. sphere of radius r.

0
y
r= J2
-
.
X -
X =2y
x =t'2{Volume)
31. Maximum v<,>lume of cone with a V -~1tf3
J27
c-
given slant height.
27. Least amount of material to be
used for an open top cylindrical Proceed to the next page for your 14th
tank. test. Detach and use the .answer sheet
h=~ provided at the last part of this book. Use
.j3 pencil number 2 in shading your answer.
h
e =tan-1 J2
GOOD LUCK!
~
r

r=h
I 32. Volume of largest cone, Vc that can
be inscribed in a hemisphere.
m:ribia:
Did you know that. .. the word "Algebra"
comes from an Arabic word "al-jabr''
meaning to transpose terms from one side
28. Minimum cost for a given volume, of an equation to the other! This was
v.
l
introduced by a Persian mathematician, ai-
Khowarizmi in around 825 A.D.

1 clauote:
vc =-V.h
2 "The most beautiful thing we can
experience is the mysterious. It is the
33. Largest cylinder that can be source of all true art and scienCe. •
inscribed in a cone.

r=f{: -:\····--·------·--··r
- Albert Einstein

I
29. Ratio of the weight of heaviest
cylinder, We to the weight of the
y r··-·--··
r.
circumscribing sphere, W,..

h
we- 1 Y=3
w:··· ./3
Day 14- Differential Calculus (Maxima-Minima & Ti1;11e Rates) 339

&oo: CE Boart~J~ay 1.'9~ . A. 1.22


Find ttie minimum amount of~sheet that B. 1.64
can be made into a closed cylindB_r having C. 2.44
a volume of 108 cu. inches in square D. 2.68
Inches.
Topics A. 125.50
&os: EE Board April 1.997
The cost of fuel in running a locomotive is

0Mon
Maximum and Minimum Values
Local Maximum and Local
B.
C.
D.
127.50
129.50
123.50
proportional to the square of the speed
and is $ 25 per hour for a speed of 25
miles per hour. Other costs amount to $
Minimum
[QJ Steps in Solving Maxima/Minima
Problems
&OJ.I ME Board April 1.998
A box is to be constructed from a piece of
1QO·per hour, regardless of the speed.
What is the speed which will make the cost
per mile a minimum?
Tue zinc 20 sq.in by cutting equal squares from
Steps in Solving Time Rates
0
Theory
0Wed
Problems
Relation Between Variables
each corner and turning up the zinc to
form the side. What is the volume of the
largest box that can be so constructed?
A.
B.
C.
40
55
50
D. 45

Problems
0Thu
·Under the Condition of
Maxima/Minima
A.
B.
C.
599.95
592.59
579.50
cu
cu
cu
in.
in.
in.
&o&1 ME Board April199&
The cost C of a product is a function of the

0
D. quantity x of the product : C(x) = x2 - 4000
0
622.49 cu in.
x + 50. Find the quantity for which the cost
&0:&1 EE Board April 1.997 is minimum.
Solutions Fri A poster is to contain 300 (em square) of

0
Notes
0Sat
printed matter with margins of 10 em at the
top and bottom and 5 em at each side.
Find the overall dimensions if the total
A.
B.
C.
1000
1500
2000
area of the poster is minimum. D. 3000

59&: ECE Board April 1999 598: EE Board March 1.998 A. 27.76 em, 47.8 em &071 An open top rectangular tank with
Find the minimL,Jm distance from the point A triangle has variable sides x,y,z subject B. 20.45 em, 35.6 em square bases is to have a volume of 10 cu.
(4,2) to the parabola y 2 = 8x. to the constraint such that the perimeter is C. 22.24 em, 44.5 em m. The materials for its bottom are to cost
fixed to 18 em. What is the maximum D. 25.55 em, 46.7 em P 15 per square meter and that for the
sides, P6 per square meter. Find the most
A. 4/3 possible area for the triangle?
&031 CE Board No-vember 1.99& economical dimensions for the tank.
B. 2J2 A. 15.59 cm 2 A norman window is in the shape of a
B. 18.71 cm 2 rectangle surmounted by a semi-circle. A. 1.5m x 1.5m x 4.4m
c. f3 C. 17.15 cm
2 What is the ratio of the width of the B. 2m x 2m x 2.5m
cm 2 rectangle to the total h1eight so that it will
D. 2/3 D. 14.03
yield a window admitting the most light for
C.
D.
4m x 4m x 0.6m
3m x3m x Um
599: EE Board October 1997 a given perimeter?
597: EE Board April 1.990
A farmer has enough money to build only &o8: ME Board October 199&
The sum of two positive numbers is 50. A. 1
100 meters of fence. What are the What is the maximum profit when the
What are the numbers if their product is to B. 1./2
dimensions of the field he can enclose the profit-versus-production function is as
be the largest possible. C. 2
maximum area? given below? P is profit and x is unit of
D. 2/3 production.
A. ~&~
B. W&~
A. 25m x 25m 8
B. 15mx35m 6041 CE Board May 1.998 p = 200,00- X- ( -1·1
-)
C. 25&25
D. W&W c. 20mx30m I Jetermine the diameter of a closed \X+ 1
D. 22.5 m x 27.5 m •;ylindrical tank having a volume of 11.3 A. 285,000
cu. m to obtain minimum surface area. B. 200,000
C. 250,000
\.
340 1001 Solved Problems in Engineering Mathematics (2" 0 Edition) by Tiong & Rojas Day 14- Differential Calculus (Maxima-Mi~ma & Time Rates) 341

613: EE Board October 1993 Problem6171 6:u: ME Board October 1996


D. 305,000
At any distance x from the source of light, A man walks across a bridge at the rate of Water is pouring into a swimming pool.
609: EE Board October 1993 the j.r{tensity of illumination varies directly 5 fps as a boat passes directly beneath After t hours, there are t + Jt gallons in
A boatman is at A which is 4.5 km from the a~he intensity of the source and inversely him at 10 fps. If the bridge is 10 feet above
the pool. At what rate is the water pouring
nearest point B on a straight shore ~M. He · the square of x. Suppose that there is a the boat, how fast are the man and the
into the pool when t = 9 hours?
wishes to reach in minimum time a point C 1ght at A, and another at B, the one at B ·,i boat separating 1 second later?
~
situated on the shore 9 km from B. How far having an intensity 8 times that of A The A. 7/6 gph
from C should he land if he can row at the distance AB is 4 m. At what point from A A. 8 fps B. 8/7 gph
rate of 6 kph and can walk at the rate of -.Qil1 the line AB will the intensity of B. 8.25 fps C. 6/5 gph
7.5 kph? ··· mination be least? C. 8.33 fps D. 5/4 gph
D. 8.67 fps
A.
B.
4.15 km
3.0 km A
B. ~1.15 m
1.33 m 6181 An LRT train 6 m above the ground
6:Z:ZI A helicopter is rising vertically from
the ground at a constant rate of 4.5 meters
C. 3.25 km C. 1.50 m crosses a street at 9 m/s at the instant that
per second. When it is 75 m off the
D. . 2~ a car approaching at a speed of 4 m/s is
D. 4.0 km
~-~
il 12 m up the street. Find the rate of the
ground, a jeep passed beneath the
helicopter traveling in a straight line at a
610: EE Board March 1998 614: CE Board May 1995 LRT train and the car separating one constant rate of 80 kph. Determine how
A fencing is limited to 20 ft length. What is A wall "h" meters high is 2 m away from second later. fast the distance between them changing
the maximum rectangular area that can be the building. The shortest ladder that can after 1 second.
fenced in using two perpendicular corner reach the building with one end resting on A. 3.64 m/s
sides of an existing wall? the ground outside the wall is 6 m. How B. 3.94 m/s A. 12.34 m/s
high is the wall in meters? C. 4.24 m/s B. 11.10 m/s
A. 120 D. 4.46 m/s C. 10.32 m/s
B. 100 A 2.34 D. 9.85 m/s
C.· 140 B. 2.24 619: 'E Board October 1993
D. 190 C. 2.44 Water is flowin~ into a conical cistern at 6%31 ECE Board November 1991.
D. 2.14 the rate of 8 m /min. If the height of the A balloon is released from the ground 100
611: EE Board October 199% inverted cone is 12 m and the radius of its meters from an observer. The balloon
circular opening is 6 m. How fast is the

j
The cost per hour of running a motor boat 6J.S: EE Board April 1997 rises directly upward at the rate of 4
is proportional to the cube of the speed. At The coordinates (x,y) in feet of a moving water level rising when the water is 4 m meters per second. How fast is the balloon
what speed will the boat run against a particle P are given by x = cost- 1 and y = deep? receding from the observer 10 seconds
current of 8 km/hr in order to go a given 2 sin t + 1, where t is the time in seconds. 'I later?
distance most economically? At what extreme rates in fps is P moving A 0.64 m/min
along the curve? B. 0.56 m/min A. 1.68 m/sec
A. 10 kph C. 0.75 m/min B. 1.36 m/sec
B. 13 kph A. 3 and 2 D. 0.45 m/min C. 1.55 m/sec
C. 11 kph B. 3 and 1 D. 1.49 m/sec
D. 12 kph C. 2 and 0.5 6:zo: CE Board November 1998
D. 2 and 1 Water is pouring into a conical vessel 15
'&24: ECE Board April 1998
61%: ECE Board November 1998 em deep and having a radius of 3:75 em A balloon is rising vertically over a point A
Given a cone of diameter x and altitude of 61.6: ECE Board April 1998 across the top. If the rate at which the
on the ground at the rate of 15 ft./sec. A
h. What percent is the volume of the A statue 3 m high is standing on a base of water rises is 2 em/sec, how fast is the point B on the ground level with and 30 ft
largest cylinder which can be inscribed in 4 m high. If an observer's eye is 1.5 m water flowing into the conical vessel when
'from A. When the balloon is 40 ft. from A,
the cone to the volume of the cone? above the ground, how far should he stand the water is 4 em deep? at what rate is its distance from B
from the base in order that the angle changing?
3
A M% subtended by the statue is a maximum. A. 2.37 m /sec
B. ~% B. 5.73 m 3/sec
3 A. 13ft /s
C. ~% A. 3.41 m C. 6.28 m /sec
3 B. ,15ft/s
D. ~% B. 3.51.m D. 4.57 m /sec C. 12ft /s
C. 3.71 m D. 10ft/s
D. 4.41 m

\.
342 1001 Solved Problems in Engineering Mathematics (2nd Edition) by Tiong & Rojas

&25: CE Board May 1997 &29: ECE Board November 1998


Car A moves due East at 30 kph at the What is the allowable error in measuring
same instant car B. is moving S 30• E, with the edge of the cube that is intended to
a speed of 60 kph. The distance from A to hold 8 cu. m., if the error of the computed
B is 30 km. Find how fast is the di~tance volume is not to exceed 0.03 cu. m?
between them separating after one hour.
A. 0.002 Topics
A. 36 kph B. 0.003 ----·-.~---

B.
C.
D.
38
40
45
kph
kph
kph
C.
D.
0.0025
0.001 D
Mon
Maximum and Minimum Values
Local Maximum and Local
Minimum
&2&: CE Board November 199&
A car starting at 12:00 noon travels west at
&30: EE Board October 1993
A standard cell has an emf "E" of 1.2 volts.
lfthe resistance "R" of the circuit is
&Q] Steps in Solving Maxima/Minima
Problems
Tue
a speed of 30 kph. Another car starting increasing at the rate of 0.0 hm/sec, at
Steps in Solving Time Rates
from the same point at 2:00 P:M. travels ..
north at 45 kph. Find how (in kph) fast the
two are separating at 4:00P.M.?
what rate is the current "I" hanging at the
instant when the resist ce is 6 ohms?
Assume Ohm's law E IR.
D D
Theory Wed
Problems
Relation Between Variables
A.
B.
c.
49
51
53
A.
B.
C.
-0.002 amp/s
0.004 amp/se
-0.001 ampdsec
I:
,J
!~·

~~
D D
Problems Thu
Under the Condition of
Maxima/Minima

0.003 amp/~c
D. 55

&27: CE Board May 199&


Two railroad tracks are perpendicular to
D.
D
Solutions Fri
each other. At 12:00 P.M. there is a train
at each track approaching the crossing at
50 kph, one being 100 km and the other
D D
Notes Sat
150 km away from the crossing. How fast
in kph is the distance between the two
trains changing at 4:00P.M.?
ANSWER KEY RATING
A. 67.08
B. 68.08 596. B 606.C 616. c 626. B c:J 30-35 Topnotcher
C.
D.
69.08
70.08
597.C
598.A
607. B
608. B
617. c
618. A
627.A
628.C
c:J 21-29 Passer

&28: CE Board May 1995 11


I
599.A
600.A
609. B
610. B
619.A
620.C
629.c
630.C
c:J 18-20 Conditional
Water is running into a hemispherical bowl
· having a radius of 10 em at a constant rate
3
'
'i!!
601. B
602.C
611. D
612. A
621. A
622. c
c:J 0-17 Failed
of 3 cm /min. When the water is x em. t 603.A 613.C 623. D If FAILED, repeat the test
deep, the water level is rising at the rate of 604.C 614. B 624.C
0.0149 em/min. What is the value of x? 605.C 615. D 625. D

A. 3
B. 2
"1.
C. 4 .,
D. 5 1
11'

'>
344 1001 Solved Problems in Engineering Mathematics (2nd Edition) by Tiong & Rojas . Day 14- Differential Calculus (Maxima-M~-~'rilne Rates) 345

• (4,2)
2=L
32
y3=64
y=4
3

1 l
'I

' •
Note: For maximum area, the triangle
must be an equilateral triangle.
Thus, the size of the field is 25 m x 25 m.


Substitute y = 4 in Eq.2: h
' )2
(4 =2
y=-a

V=nr2h
Substitute x =2 and y =4 in Eq.1: X

3x=18 108 = nr2h


2
d=J<'X~4) +(y-2r ~Eq.1 d = ~(2- 4) +(4--.:-2)2
2 x=6 _ 108. E
h --~ q. 1
xr2
y2 =8x d=J8 =~4(2) A=.!x 2sin9
2 A= 2xrh + 2(nr2} ~ Eq.2
y2 d=2../2
x=--~Eq.2
8 A= i<6rsineo
Substitute Eq.1 in Eq.2:
Substitute Eq.2 in Eq.1: Ill A=15.59cm2

··K~ -·r +(y-2f


Let:
x and y = two positive numbers
P = product of x and y
:t
• y
X

y
A =2\~( ~~~ )+2(nr
216
A=-+2xr2 ~Eq.3
r
2
}

l
x+y=50
'
du Differentiate:
Note: d(-!U) =2Tu y =50-x ~Eq.1 X
P=xy ~Eq.2 !
2x+2y=100
':'J dA _ -216 +4xr
Differentiate: y=50-x ~Eq.1 --
dr r2
Substitute Eq.1 in Eq.2:
A=xy~Eq.2
-216 +4xr
0=-2-
d'= 2( f-4)(¥)+~(1) P = x(50-x)
P=50x-x 2
Substitute Eq.1 in Eq.2:
r
216 =4xr
r2
2./( ~ -4J +(y-2r
A= x(50-x)
Differentiate: _r3 =216
A=50x-x2
4x
dP r=2.58 in
2(f-4 )(¥ )+2(y -2)(1)
-=50-2x
dx
Differentiate:

f: -·r 0=50-2x Substitute r in Eq.3:


x=25 dA =50-2x
o- •. +(y-2f dx 216 >2
0=50-2x A = - + 2n(2.58,
Substitute x = 25. in Eq.1: 2.58
x=25
o=( ~ -4)( 2i)+<y-2)(1) y=50-25·
Substitute x =25 in Eq.1:
A= 125.544 in2

y=25
y3
0=--y+y-2 y = 50-25
32 '

,,
Thus, the numbers are 25 and 25.
y=25
.

346. 100 i Solved Problems in Engineering Mathematics (2nd Edition) by Tiong & Rojas Day 14- Pifferential Calc.ulus (Maxima-Minima & Time Rates) 347

Ill Ill II d

o· J:
10
X

20 +y h
:.···1''1
h
20
...'
f ). --::

•·
} 10
'.
'• . y

X 5.'
X
:5 A =.::_d2h
4
10 +X 2x
11.3 =.::.d2h
Let: P = perimeter 4
A=(10+x)(20+y)
X - 45.2 Eq. 1
A= 200+10y + 20x + xy--+ Eq.1 d -----?
1 1td2
P = 2x +2y +-(27tx)
AP =xy 2
20-2x 300 = xy
,,.,
'!.

P = 2x + 2y + 1tX A= 1tdh + 2( ~d2 )--+ Eq.2


300 ~j P-2X-1tX
V = (20- 2x)(20- 2x)(x) y=-. -?Eq.2 y= •.., 2
X Substitute Eq.1 in Eq.2:
V =(20- 2x)2 (x) I y = 0.5P- 2.57x--+ Eq.1
V =(400-80x +4x 2 )(x) Substitute Eq.2 in Eq.1: '
l'

V = 400x- 80x 2 + 4x 3 --+ Eq.1


·~
I
A = y(2x) +
1
2
nx2 --+ Eq.2 A=7tdh+2( ~d2 )
A =200+1o(_3 ~ 0 )+20x+ \( 3~0 ) ~ Substitute Eq.1 in Eq.2: A= 1td( :~;)+2(~ )d2
Differentiate: ,'~i

3000 }~
2
A= 500 + + 20x--? Eq.3 A = 45.2 + 1td
dV
- = 400-160x +12x
2 X A= 2x(0.5P- 2.57x)+ 2x2
dx 2 d 2
0 = 400 -160x +12x 2 Differentiate: A= Px -5.14x 2 +.::_x 2 Differentiate:
,li~
2
By factoring: dA =0-~+20 i A = Px- 3.57x 2
dA = -45.2 +.::_(2d)
dx x2
(4x -40)(3x -1 0) = 0
II Differentiate: dd d2 2
0 =- 3000 +20
4x=40 x2 -45.2 + d
0=-2- 1t.
dA d
x = 10 in (absurd) 3000 =20 -=P-7.14x
dx 3 45.2
3x =10 x2 d=~
O=P-7.14x 1t
x =3.33in x 2 = 150
x=0.14P d= 2.432 in


x =12.24cm
Substitute x = 3.33 in Eq.1: y = 0.5P- 2.57(0.14P)
Substitute x = 12.24 in Eq.2: y=0.14P
v = 400(3.33)- 80(3.33)2 +4(3.33)3
Solving for h: Let: C = total cost per hour
V = 592.59 in3 300 =24.5cm
=


y = 12.24 N speed in miles per hour
Ratio= 2X =~= 2(0.14'R_)
Thus, the dimension of the poster is, h x+y 0.14'R_+0.14"R: C = fuel cost + others
-~
Ratio= 1 C = kN2 100--+ Eq.1
+
Let: A = total area (1 0 + 12.24 )(20 + 24.5)
Ap =area with the picture or 22.24 em by 44.5 em


348 100 i Solved Problems in Engineering Mathematics (2nd Edition) by Tiong & Rojas Day 14 - Differ~ntial Calculus (Maxima-Minima & Time Rates) 349

7
~-(~9=-x:fo)== + _1 = 0
Fuel cost =kN2
2
C = x2 (15)+ 4(xh)(6)
C = 15x2 +'24xh--+ Eq.1
0= -1-8(~)
x +1
(~)
(x + 1)2 6~(45) 2 +(9-x)2 75
·
25 =k(25)
1 V=x 2h 0 = -1 + 8(1.1 )8 9-x _ 6
k=-
25 10= x 2h (X+ 1)9 ~(4,5f +(9-x)2 -7.5
10 •/1 (X + 1)9 = 8( 1.1 )8 75
h= \'~ J<4.5)2 +(9-.x)2 =-·- (9-x)
Substitute k in Eq.1: 2 ~Eq.2
! X+ 1 = ~8(1.1 ) 6
X 8

C=_!_N2 +100 Substitute Eq.2 in Eq.1: X =0.371 ~(4.5) 2 2


+ (9- x) = 1.25(9- x)
25 (1.25)2(9- x)2 = (4.5)2 + (9- x)2
~;,!lll1 Substitute x in Eq.1:
Let: x = total cost per mile C:; 15x2 + 24x(:~) 0.5625(9- x)2 = 20.25
11
8
(9-x)2 = 36
x = total cost per hour C=15x2 + 240 P=200,000-0.371-( · )
0.371 + 1 9-x =6
. miles X
speed I n - - p = 199,999.457 x=3km
hour
Differentiate: =200,000
_!_N2 +100 'i
p
a
x= 25

25
N
N 100
x=-+-
N
dx _ 1 (-100)
- - - + -2-
dN 25 N
dC = 30x- 240
dx
0=30x--·

240 =30x
x2
240
x2
·~
it
'll

• X
I

0 = _!_+ (-100) 7 4.5


L-'

3 __ _
240
25 N2 x+y=20
2 X - 30
N =2500 y=20-x~Eq.1
B
x=2m A= xy ~Eq.2
N=50mph
Substitute x = 2 in Eq.2:
Ill 10 =2.5m Let: T = total time needed
Substitute Eq.1 in Eq.2:

A= x(20-x)
2 h =(2)2 T = time to row + time to walk
C=x - 4000x + 50 A= 20x-x 2
dC d X
-=2x-4000 Thus, the dimension of the tank is, T=-+- Differentiate:
dx
2 mx2 m x2.5 m
6 7.5
0=2x~4000

2x =4000
~~

T=
F-5) +(9-4 2

6
X
+ 7.5
dA =20-2x
x=2000 Ill dx
0 = 20-2x

• h
P=200,000-x- ( - 1.1 ) -+Eq.1

Differenti.ate:
x+1
8

f
!
l Note: d( .JU) =2.JU
Differentiate:

dT -( 11
du

2(9-x){-1) 1
X =10

Substitute x = 10 in Eq.1:

y = 20 -10 =10ft

1)7( +1) )
X dP _ 1.1 -1.1 Substitute x and y in Eq.2:
1 8
dx - - - ( X + (X 2 <.ix-- 6') 2J(4s)2 +(9-x)2 +7.5
Let: C = total cost A= (10)(10) = 100 ft2
\
'

350 100 1 SOlved Problems in Engineering Mathematics (2"d Edition) by Tiong & Rojas ·, Day 14 ~Differential Calculus (Maxima-Minima & Time Rates) 351

Ill V =nr2h-+ Eq.1


Ratio=:!___= n(-;.., )~ a
Let: C =C(OSt per hour By ratio and proportion: vc ~7t~
V =speed of the motorboat
c, =total cost ~=-r­ Ratio= 0.44

C = kV 3 -+ Eq.1

where: k = proportionality constant

_ distanee = ~ -+ Eq.2
H H-h
Hr =RH-Rh
h _ RH-Hr
--R~-+
E

Substitute Eq.2 in Eq.1:


q. 2 • A
·~~
- ""' ~
• ~ x o 4-x
B
,,~
- l':'l. ~
•. •

t- speed
c1 = Ct -+ Eq.3
V-8
- 2 (RH-Hr)
,.. E4 )o I sine=~
V -nr --- Let: X
R
E = intensity of illumination x= _h
Substitute Eq.1 and Eq.2 in Eq.3: 3
V ._
A & B = luminous intensities of the two sine
-nr 2H -nr- -
H
light sources respectively.
R
COSO=~
c,=kv (~)
3
V-8 Differentiate:
k = proportionality constant y
Y= _2_
kV S 3 E=kA+~ cose
c,=-- dV
-=2
2 x2 (4-x)2
V-8 dr nrH ---
3nr H
R
dC1 _ (V- 8)(kS3V 2)- kSV 3 (1- 0) E = kA + k(8A) L=x+y
dV - (V -8)2 0 = 2nrH- 3nr2H x2 (4- x)2 h 2
R- L=--+-·--+Eq.1
sine cose
= (V- 8)(kS3V 2)- kSV 3
0
(V -8)2
3\r"»-H_ =2~)( Differentiate:
R Differentiate:
kSV3_ = (V- 8)(3R&_~) 2 dE= -kA(2x) + (-8kA)(2)(4-x)(·1)
r=-R dL -hcose -2(-sine)
V =3V-24 3 -=~+-~~.:..
dx x4 (4-x)4 de sin 2 e cos 2e
2V=24
Substitute r in Eq.2: = -kA(2x) + (8kA)(2)( 4- x} = -hcose + -2(-sine)
v =12 kph 0 0
4 4 sin 2 e cos2 e


x (4-x)
h = 'R_H- H(2/3 'R_) = _!H hcose 2(sine)
R_ 3 k1\('2_\)- (8k1\}('2_)~ sin 2 e = cos2 e
-~- (4·x)"- 3
h = 2(sin e)
Substitute r and h in Eq.1:
8 cos 3 e
H

h
V=n(~R iH)
2
r( --;(3"
--
x3
(4- x)3 = 8
--3
(4-x) h =2tan3 e-+ Eq.2

Substitute L =6 and h =2 tan 3 e in Eq.1:


V =n(_i.)R H

J(4~xr =~
27 3
R .) .. 3
Let: X 6 = 2tan e +-2-.
sine cose
=
R radius of the cone Let: Vc = volume of the cone
4-x =
2 3
= 2tan ecos8+2sine
H = height of the cone X 6
sinecose
r = radius of the inscribed cylinder
h = height of the inscribed cylinder
=
Vc .!.nR
3
2
H 4 _ x =2x
6sinecose = 2tan 3 ecos0+ 2sine
V = volume of cylinder x. l.JJ m
Day 14- Differential Calculus (Maxima-Minima~ Ti.~-R.~tes) 353

3
6sinecos9 = 2( sine ) ccis9 + 2sin9
cose
a du
(x2 + 13.75 )3- 3x(2x)
=-----·-------
(x2 +13.75t
Substitute t = 1 in Eq.1:

s2 = 125(1)2 + 1oo
sin9r
6cos9=2 ( - - +2 s = 15
cos a Substitute:
2 SubstituteS::: 15 in Eq.2:
3cos9= ( ~.
. 9 ) +1 de du
cos a
Cix"'1;u 2 dS 12~(!L "" 8.33 fps
sin2 9 + cos2 9 du
dt-=-15
3cos9= 2
cos9 0'~-:;-:;-_-;:l
3cos3 9=1 O=du m1 osition ~t th$ ii-ai~ .
r:P_.r
l_ aftert seconds

if;:/
..J
cos3 9 = 0.333 (x + 13.75 2
~x(2x) · )3_ -· ~:---~
9=46.1. 0 =---------
(x2+13."15f
Substitute 9 In Eq.2: 2x 2 = x 2 + 13.75 16
a
lb&
2
h = 2tan46.1. 55 X =13.75
tan(a+9)= · X= 3.-108
h=2.24 m X .:.::·:.o·

• ,;/sz= 9t
tan a+tan 9 5.5
=- .
1-tanatane x .~1 of!he ~~a:J s,=~t N2-4t
2.5 t 9
x+ an 5.5
v2 =5
r.PoSiiic)n
aftertseconds - .--1 . \.. _ _ _

=-x V1 = 4 [POl·.sitton of the c~r


/ £·----·
x = cost-1 ;;;::..-----------'---
2.5t 9
dx
dt
.
-=-s1nt
1--an
X
13.75
2.5 + xtane = 5.5 ---tane
_/IS2=?L
. I S2
. after t seconds
-----
=(12-4ti" +(9t)2 +6 2
X 2
s = 144 - em + 1st2 + 81t 2 + 36
Note: ~~ is maximum if sin t = - 1 13.75) 10
tane ( x+-.-x- =3 S 2 = 97f - 96t + 180 __,. Eq. 1

3 .···
dx=-(-1)=1
dt
tane = 13.75 L--~" Differentiate both .sides with respect to t:
X+ -X
__;1=1ot /~
y =2sint+1 3x
V1 = 10 ~-osition of the boat
l_· v~fter I seconds
2s( ~~}=194t(*)-s6(*) +O
dy =2cost
tan9= x2+13.75
I
'~
dS 194t-96
Jl
dt 1 - = - - - - - - _, Eq.2
9 = tan- (x2 +;31x3.75) 2
S '= S/ + S/ + 10 2 dt 2S
:11

Note: ~~ is maximum ifcos t = 1 :/ oc(10!)2 +{5tl +10 2 Substitute 1 = 1 in Eq.1:


du s7 .· ·125t2 + 100---+ Eq.1
Note: d(tan-1 u) = 1+ u2
s 2 = 97(1)2 -- 96(1) + 180
dy =2(1)= 2
where:
I )ltferentiate both sides with respect tot: s = 13_45
dt
2
if=
·:f(\~ ) 250t {1) + 0
3x . 9x .. Substitute S = 15 in Eq.2:
Thus, the extreme rates are 2 and 1. 2 +-1-:-3-.7-5'
u =-x""" (x 2 + 13.75)'
dS '194(1)- 96 = 3.64 m/s
d:; I )~it
,. >I q 7 d! = --2(13A's)
dt
"
354 1001 Solved Problems in Engineering Mathematics (2nd Edition) by Tiong & Rojas
Day 14- Differential Calculus (Maxima-Minima & Time Rates) 355

a R=6
V=
1
3
2
nr h ~ Eq. 1
rm 1m
By ratio and proportion: 'km_ X 1 hf:.. 1000 m
V = 80
1
hi:.. 3600 sec
X
1 'km.
s
3.75 r
.
s1 = 4t
--=- V1 = 22.22 m/sec
15 h
h
r= ~Eq.2 ~ V2=4.5
4
V=
1
nr 2h ~ Eq.1
Substitute Eq.2 n Eq.1: s2: 4.5! I I ' t
3 s 2 = 1oo2 +(S1 )2
s 2
By ratio and proportion: S =100 2 +(4t)2

-
V=in(%r h
75 v1 = 22.22 s 2
= 1o,ooo + 16t2 ~ Eq.1
6
-=-
r V = __!_h 3 ~ Eq.2
48
~
12 h Differentiate both sides with respect to t:
h .
r=-~t::q.2 Differentiate both sides with respect to t:
2 s1 = 22.22t
dV = 3n h2 dh
s 2 = (S 1 ) 2 +(75+S 2 )2
2s( ~~ )=0+32too
Substitute Eq.2 in Eq.1: dt 48 dt
2
S = (22.22t)2 + (75 + 4.5t)2
dS = 15t ~Eq. 2
2
Substitute dh/dt = 2 and h = 4: dt s
1 '( h ) h
V=3n2 s 2 = 493. 728t2 + 5625 + 675t + 20.25t 2
Substitute t =10 in Eq.1:
dV- 3n (4)2(2)
s 2 =513.978t2 + 675t + 5625 ~ Eq.1
V=~h 3 dt- 48
12
Differentiate both sides with respect to t: s 2 =1o,oooo+16(10)2
dV = 6.28 cm3/s 8=107.7 m
Differentiate both sides with respect to t: dt

~~) =2(513.978t)oo +67500


dV = 3n h2 dh
dt 12 dt

Substitute dV/dt = 8 and h = 4:



Let: Q = rate of discharge in gph
2s(

Substitute t
dS
dt
= 513.978t+337.5 ~E

=1 in Eq.1:
s q
_
2
Substitute t = 10 and S = 107.7 in Eq.2:

dS
dt =
16(1 0) = 1.485 m/s
107.7

3n
8=12(4)
2 dh
dt
O=t+Jt

s2 =513.978(1)2 + 675(1) +5625


1m
Differentiate both Sides with respect to t:
dh = 0.64 m/min s =82.547 m
dt dQ 1 s1 = 15t
1 = 82.547
dt= + 2Ji. Substitute t = 1 and S in Eq.2:
1m R=3.75
Substitute t =9 hours: dS 513.978(1) + 337.5 B A

H-1~ J-~-,
1
-~; -·
dt 82.547
2
dQ 1 1 dS S = 30 2 + (S, )2
-=1+--=1+- 10.315 m/sec
dt
dt 2.[9 6 S
2
= 30~ + (15t)2
dQ 7 s2 =goo+ 22st2 ~ Eq.1
t "~ - = - gph
dt 6
356 lO<U Solved Problems in Engineering Mathematics (2"d Edition) by Tiong & Rojas
Day 14- Differential Calculus (N.[a_xirna-Minim(l & Time Rates) 357
Differentiate both sides with respect to t· Differentiate both sides with respect to t: Substitute t = 2 hours (from 2 PM to 4 8 2 = 5, 000( 4 )2 - 25, 000( 4) + 32, 500
PM) in Eq.1:

~~)=0+4501~ 2S(~T-)=5400t~ -2700~


S = 111.8 km
2s(
' ,~1
s 2 2
= 2925(2) + 3600(2) + 3600 Substitute S = 111.8 and t = 4 in Eq.2:
dS = 225t ~ Eq. 2 dS 2700t -1350 E ,;~.,l ,i
S = 150 km
dt s dt
s ~ q. 2
I dS 5000( 4) -12500
Substitute S = 150 and t = 2 in Eq.2: dt 111.8
When S1 = 40 ft, Substitute t = 1 in Eq 1:
dS = 2925(2)+1800 dS = 67.084 k h
dt p
s 1 = 15t s 2
= 2700(1f - 2700(1) +goo dt 150
40 = 15t
t = 2.667 sec
S = 30 km

SubstituteS = 30 and t = 1 ir. Eq.2:


dS =51 k h
dt p
rm
=2.667 in Eq.1:
Substitute t
dS·= 2700(1)-1350 m.l A
s 2 = 9oo + 225(2.667)2
s =50ft
dt
dS =45 k h
dt p
30

N
;I
~~
VA= 50 l
2.667 and S = 50 in Eq.2:
·~
Substitute t :c 100

dS
dt
225(2.66 7 ) = 12 ft/s rm .r
>::>s
V=-(3r-x)
1tX2
3
1tX2
50 f,, Q
50t-150
6. ·-- V=-(3(10)-x)
3
U"1
-:
ml
0
N iO
"<!"
Vs=45 :: 0
Vs= 50 V=10nx 2 - -
1tX3
II 3
$ t
0
m
4ltt"'-vs=60 (/)
Differentiate both sides with respect to t
B
Ss =50!
-vA=3o SA = 30t A~
-~
60 B dV = ( 2onx _ 3nx
2
Jdxdt
~ SA=30t 9
A'a.:~,--~- Position of carA'i s 2 2
=(50t-100) +(50t-150) 2
dt 3

at2~~-j S 2
=2500t2 -1 OOOOt + 10000 + 2500t2 -dV = ( 20nx -nx 2 )dx
- ~ Eq.1
dt dt
-1 5000t + 22, 500
s2 = (60 + SA)2 +Ss2 Substitute dV/dt = 3 and dx/dt = 0.0149 in
s 2 = 5000t2 - 25000t + 32,500 ~ Eq.1
2
s = (60 + 30t) + (45t) 2 2 Eq.1:
By cosine law:
s2 = 36oo + 360at + 9Dat2 + 202st2 I lifferentiate both sides with respect to t:
il 3 = (20nx -nx 2 )(0.0149)
2 2 2
s = (30t) + (60t- 30) S 2
2925t + 3600t + 3600 -~ Eq.1
2

/S( ~~) 2(5000t)~ -25000~


"'

-2(30t)(60t- 30)cos60
s 2 = 9ooe + (60t- 30)2 - 30t(6ot- 30)
Differentiate both sides with respect to t:
I ~;
201.342 = 20nx -nx 2
64 = 20x- x2
" dS 5000t -12500 ~ Eq.2 2
s 2 =9ooe + 360at 2 - 360at + 9Do --s x - 20x + 64 = 0

~~) = 2(2925t)~ + 3600~1J,


dt
(x-4)(x-16)=0
-1800f + 900t 2s(
:;utdihr\(; I= 4 hours (from 12 PM to 4 x = 4 em or
s 2 =21om 2 - 21oot + 9oo ~ Eq.1 .!:!_~ "' _2925t + 1800 ~ Eq.2 I'M) Ill lq t· x = 16 em
dt s
358 1001 Solved Pcoblems In Englnee<lng M~ 0CL_.don) byTiong & Rojos


V = x 3 ~Eq.1

Differentiate both sides with respect to its Topics

D
individual variable:
Maximum and Minimum Values
dV = 3x dx ~ Eq.2
2
Mon Local Maximum and Local
Minimum
Substitute V = 8 in Eq.1: Steps in Solving Maxima/Minima
Tue Problems
8= x3 Steps in Solving Time Rates
x=2

Substitute dV = 0.03 and x = 2 in Eq..2:


D D
Theory Wed
Problems
Relation Between Variables

0.03 = 3(2)2 dx
dx =0.0025 m .·~
D D
Problems Thu
Under the Condition of
Maxima/Minima
il

D D

~~

'"'

l Solutions Fri

E=IR
1.2 = IR ~ Eq.1
D
Notes Sat

Differentiate both sides with respect to its


individual variable:

f\lote: d(uv) = udv +'vdu

o~{:)+R(~!)~Eq.2
Substitute R = 6 in Eq.1:

1.2=1(6) T
T
1=0.2
JJ
Substitute I = 0.2, dR/dt = 0.03 and ·.~
R = 6 in Eq.2: I

0 = (0.2)(0.03) +

dl
- = -0.001 amp/sec
dt
(6{ ~~)
il
li
T
T
. 0
1'opics

T.
,.;..r,.,
. integral Calculus
,, . .
Definite and Indefinite Integrals
Mon
,.~. .,.·~+.!•"\ ···<~=~"~<'<"''~ . .
Fundamental Theorem of Calculus
/' ;~ ',;. > S ·~ .« < 4 ' ' > , < '"-" I \> '· ( ·>

0 Basic Integrals Formulas

,T
Tue - Exponential, Logarithmic,
. ·rrigonometric, Hyperbolic,

..... l~J
.

Inverse Trigonometric, By Parts,


Theory VVecl Trigonometric Substitution,
. !•.
[] []
Problems Thu
Wallis Formula, etc.
Applications
- Area, Centroid, Arc Length

D []
Solutions Fri
Surface Area, Volume, Work
Moment of Inertia, etc.
Propositions of Pappus

t'llj
i'
[] 0
Notes Sat
Hooke's law
Multiple Integrals

~hat is Integra! Calwlus? b


Example: Jt(x)dx
Integral Calculus is the branch of
calculus which deals with functions to be
integrated. Integration is the reverse Indefinite integral is an integral with no
process of differentiation. The Junction to . restrictions imposed on its independent
be integrated is referred to as the variables. It is also called antiderivative or
integrand while the result of an integration primitive integral.
is called integraL
Example: Jt(x)dx
lhe integra! symbol or sign J is an
dongated S denoting sum (Latin: summa), What is the Fundamental Theorem of
was introduced by Leibniz, who named Calculus?
mtegral calculus as calculus
II smnmatorius. Suppose f is a continuous function in a
closed interval [a, b], then '
i! What are Definite and Indefinite
lnte.m~l.l~ F'(x) = f(x) for ali x in [a, b).

P<'luuk' tnlegral is an integral that is


The fundamental theorem of calculus
"' ~fuwd l•y IIH • ltm'f v; dtt"!~ a and b of the S!<iles U-;at the definite integral is:
.·,.. Huf<·p• 'IHil 'IIi \l.llf.liil<'
362 1001 Solved Problems in Engineering Mathematics (2"d Edition) by Tiong & Rojas Day 15- Integral Calculus 363

b A. Rectangular coordinates:
18. Jcsc 2 udu = -cotu+C 39. fsechutanhudu = -sechu+C
Jt(x)dx = F(b)- F(a)
Using a vertical differential strip:
19. Jsecut ~u~--- 40. fcschucothudu = -cschu+C

What are the Integrals of the Different 41. fsinh udu = ~sinh2u -iu+C
2
20. lcscucotudu = -cscu+C
Functions?
21 I Jsin 2 udu = .!u- .!sin2u + C 42. Jcosh 2 udu =.! sinh2u + !u + C
The following are the integrals arranged 2 4 4 2
according to its functions:

A. Basic Integrals:
!2_) Jcos 2
udu = iu + ~sin2u + C 43. Jtanh 2 udu = u- tanh u + C

23. Jtan2 udu =tan u- u + C 44. Jcoth 2 udu = u- cothu + C


1. fdu=U+C 0
24. Jcoeudu=-cotu-u+C F. Trigonometric Substitution:
2. Jadu =au +C
D. Inverse Trigonometric Functions: 44. J~a 2 -u 2 du let : u = a sin e
A= Jydx
un+1
3. u"du=--+C (n ,, -1)
f n+1
25. Jsin-'; udu = u sin- 1 u + ~ + C 45. J~a 2 +u2 du let : u = a tan e
du Using a horizontal differential strip:
4.
fu=lnu+C
26. Jcos- 1 udu =U cos- 1 u- ~1- u2 + C 46. 2
J~u -a du 2
let : u = a sec e
y
B. Exponential & Logarithmic 27. Jtan- 1 udu=Utan- 1 u-Jn~ +C G. Integration By Parts:
Functions:
28. Jcot- udu
1
= u cot- 1 u +In J1 + u2 + C
47. Judv uv- Jvdu
5. feudu = eu + c
29. Jsec- udu = usec- u -lnlu + N~~
1 1

au H. Wallis Formula: Formulated by John


6. audu=-+C +C
f Ina Wallis.
7. Jue"du = e"(u -1) + C
30. Jcsc- udu = ucsc- u + lnlu + -Ju 2
1 1
-11 . X

+C 2
8. Jlnudu = ulnu-u+C 48. fsinm ecos" ede =
E. Hyperbolic Functions: 0 A·= Jxdy
9.
f~=lnllnui+C
ulnu
[(m -1)(m ·- 3)· .. 1or2][(n -1)(n- ~)···1or2)
31. fsinh udu =cosh u + C '-'----'---'----~-'-'-----'---"•a
J
C. Trigonometric Functions: (m + n)(m + n- 2)(m + n- 4)···1or2 B. Polar coordinates:
32. Jcosh udu =sinh u + C
y
11. fsinudu=-COSU+C 33. Jtanh udu = In !cosh ul + C where: a = ~ if both m and n are even
12. Jcosudu=sinu+C 34. Jcothudu=lnlsinhui+C a = 1 if otherv:ise
13. Jtanudu=ln!secu!+C 35. Jsec hudu = tan- 1(sinh u) + C How to Find an Area using Integration?

14. Jcotudu=inlsinui+C
36. Jcschudu=lnltanh~~+C One import::>nt use of definite integrals is
the determination of area between two
15. Jsecudu = ln!secu + tanul +C curves with given equations. This is done 0 X
37. Jsech 2udu=tanhu+C by using. a differential area (horizontal or
16. Jcscudu = ln!cscu- cotul + C vertical strips for rectangular coordinates
38. Jcsch 2 udu = -coth u + C r2
17. Jsec2 udu=tanu+C
of a differential triangle for polar
c:o'oldlll<Jtes).
A= h·d9
_ _ _ _ _ _ _ _ _ _ _ _ _ _ _ _ _ _ _ _ _..:.D::.:a::.yL'.:.:lS::___-..::In:::t:.:::eSE~Calculus~
364 1001 Solved Problems in Engineering Mathematics (2nd Edition) by Tiong & Rojas
"\ ,tlow to Find the Location of the How to Find the Surface Area of a Solid
The Second Proposition of Pappus
states as follows:
Centroid using Integration? Figure by Integration?
"If an area is rotated about an axis, it will
The centroid is the center of mass of a The surface area of a solid generated by
generate a volume equal to the product of
• given figure. The figure below illustrates on revolving a curve about a certain axis may
the area and the circumference described
how to fin\l the location of the centroid of be calculated using the first proposition of
its centroid."
an arc .. bounded by curves of given Pappus.
equations. d
The First Proposition of Pappus states A
y

e.T;:tt{!tttl :~::18
as follows:
centroid
"If an arc is rotated about an axis, it will V ':" J2nyxdx
generate a surface area equal to the
product of the length of the arc and the
circumference described its centroid." X
0 B. Using a horizontal differential strip
~
....., y y (Ring Method):
y
0 X -~s,
: :
x : :
.
~
''

X f\ ········;···d V =A ·2nd or V= fdA ·2nd

_ fdA·X _ fdA·r 11 X
where : d = distance from centroid to axis
X;- A Y= -· _2 0 of rotation
A
>......!_.........
How to Find the Arc Length of a Graph?
\::I If the area is not given or cannot be
computed easily, the following will be used
~
X

v rn finding the volume of the solid:

A. Using a vertical differential strip


y ds A= S ·2nd (Shell Method):

~dy
dx (xz,Yz) where: S =length of arc
d = distance from centroid to the
The figure formed by rotating the
horizontal strip about the vertical axis is a
:······· cir.cular ring or washer.
axis of rotation

or
A= JJ1+(~~J dx ·2nd
+~
dY~ j'
:
: • • •,.,,.,,, ' ' .c' •
:
:

----4-------------------------~~
0
X ----~x "t"" . v.
...
-
,

How to Find the Volume of a Solid


Figure by Integration? J x/ - x/ ~y
S= J~1+(~~Jdx
x, The volume of a solid generated by
I IH~ tr~ 1 ure formed by rotating the vertical
. V = n(

revolving 11 curve about a certain axis may


·.trrp ;tlmut the veriical axis is a hollow
be calculated using thesecond proposition

f 1+(ddy )2 dy
'vlindrical shell
S=
y,.
X
of Pappus.
f
Yt
1'
366., 100 I Solved Problems in Engineering Mathematics (2"d Edition) by Tiong & Rojas

How to Solve for the Work Done by What are multiple integrals?
Integration?
An integral in which the integrand is
Work is defined as the force multiplied by integrated twice is a double integral.
·--~ce traversed. If the force i.s
constant throughout the distance, then
Example: fJf(x)dx 2

Topics
work is simply the product of the applied
jlnt;gral Calcul~
force and the distance traveled. However if
the applied force varies with the distance,
then the best solution is to apply ·
If integrated thrice, the integral is called
triple integral. 0
lvlon Definite and Indefinite Integrals
Fundamental Theorem of Calculus
integration.

Work= fF·dX
Example: ffJf(x)dx 3
0
Tue
Basic Integrals Formulas
- Exponential, Logarithmic,

~
Trigonometric, Hyperbolic,

where: F =force expressed in terms of x


An integral in which the integrand is
integrated to n times is n-fold iterated
integral. 0Theory Wed
Inverse Trigonometric, By Parts,
Trigonometric Substitution,
f J-· Jf(x)dx"
What is Hooke's Law?

Hooke's Law, named after the English


Example:

Functions of more than one variable may Problems


0
Thu
Wallis Formula, etc.
Applications
-Area, Centroid, Arc length
mathematician and scientist. Robert
Hooke (1635- 1703) is stated as follows:

'Within elastic limit, the force required to


be integrated with respect to one variable
at a time while the other variables are held
constant, reversing the process of partial
differentiation.
0
Solutions
0Fri
Surface Area, Volume, Work
Moment of Inertia, etc.
Propositions of Pappus
stre~ch (or compress) a spring is
proportional to the change in length."
Proceed to the next page for your 15th
0Notes
0Sat
Hooke's Law
Multiple Integrals
For spring: test. Detach and use the answer sheet
provided at the last part of this book. Use
F = kx pencil number 2 in shading your answer. b:U: ECE Board April 1:999 633: ECE Board November :1998,
where: k =spring constant What is the integral of (3t- 1)3 dt? ME Board April :1998
GOOD LUCK I
Integrate x cos ( 2x' + 7 ) dx.
How to Solve for the Moment of Inertia 1 ( 3t-1) 4 +C
'{[;ribia: A --
by Integration?
Did you know that... the propositions of
12
1
A. ~sin( 2x 2 + 7 )+C
Pappus was inspired by two fruits, namely ll. --(3!-4) +C
4

y apple and lemon! Pappus imagined a


circle cut by a line not passing though its
12
B. ~cos( 2x 2 + 7 )+C
center and rotate the area of the major ('>. 1
~(3t-1 )4 +C
4 ( sine ) +C
part about the said line and produced a C. 2
solid which looks like an apple while the 1 3 4 ( x +7 )
ll. ~(3t-1) +C
remaining minor area when rotated about 4 2
the same line forms what is known as a D. sin( 2x + 7 )+C
X lemon. (>.J:Z: ECE Board November 1:998
y I valuate the integral of dx I (x +2) from -6 &34: ME Board April :1995,
X (() 10
---------L--+-----------~----~ ~uote: ME Board April 1:997
0 "It is truth very certain, that when it is not in Integrate: (7x 3 + 4x2 ) dx.
1\ :> \/:'
our power to determine what is true, we
I\ IJ:>
ought to follow what is most probable." 7x 3 4x 2
'y = JdA·-(~Y
A
= fdA·y
(.
2 lrr :\ ----.+-+C
lx II lrr :•
3 2
2, - Rene D~scartes
~

368 l 00 1 Solved Problem~i.Il.~~gineering Math~matig~Jznct l~c!itio~!~~~ Rojas


Day 15-:. Integral Calculus 369
7x 4 4x 2 , 6o39: CE Board May 11.?9"1
B. -+-+C 6. e•in2x
4 5 C. y_+ sin2y +C
Evaluate fx(x-·5) dx
12
4 4 B. - -2 +C
7x 4 4x 3 5·'
C. -·+-·--·+C D. y + sin 2y + C C. _ e•in2x +c
4 3
P.. 0.456 D. e•in2x +C
4 4x 6441 ECE Board November 1998
D. 7x --+C 8. 0.556 Integrate the square root of ( 1 - cosx) dx.
2
C. 0.656 6491 ME Board April 1995
D. 0.756 . 1 ME Board October :1997
63SI CE Board No·veJuber 1!9'9~
5
What is the integral of sin x cos x dx 1f
3 A. -2J2 COS-X +C The integral of cos x with respect to x is
640: CE Boa!.'d November ll'\\"'9& 2
the lower limit is zero and the upper limit is 'I
xdx B. -2 .fi. COS X + C A. sin x + C
n/2? Eva Iuate
f ·----~---
o (>r+ i)8 C. 2.fi.cos.!X+c
B.
C.
secx+C
-sin x + C
A. 0.0203 2
D. cscx+C
B. 0.0307 D. -2 .fi. COS X + C
A. O.IJ11
C. 0.041"7
B. . 0.022 6501 EE Board April 1997
D. 0.0543
C. 0.033 6451 ME Board October 1997 Find the integra! of [ (e exp x - 1] divided
D. 0.044 Evaluate the integral of cos x dx limits from by [ e exp x + 1] dx
63&: CE Boar·d Noyember 1.'11'94
~to~
5
What is the integral of sin x dx if the
&41! ECE Board April 1'998 4 2 A. In ( e exp x - 1) square + x + C
lower limit is 0 and the upper limit is n/2? n/6 B. ln(eexpx+1)-x+C '
A. 0.233
Evaluate J(cos3J.\) dA 8
A. 0.423 C. In ( e exp x - 1) + x + C
0
B. 0.293 D. In ( e exp x + 1} square - x + C
B. 0.333
C. 0.433 C. 0.923
27n D. 0.329 6511 EE Board April :1997
D. 0.533 A.
~163
Evaluate the double integral of r sin u dr
6461 EE Board ApriJ :1997 du, the limits of r is 0 and cos u and the
637: CE Board May l'l99& a5n limits of u are 0 and pi.
Find the integral of ·i2 5
sin x cos x dx if
5 B.
768
Evaluate the integral of In x dx, the limits
lower limit = 0 and upper limit = 7[/2. are 1 and e.
_?:~~- A. 1
C. B. 1/2
A. 0.2 765 A: 0
B. ·1 C. 0
B. 0.3 12n
D C. 2 D. 1/3
C. 0.4 81
D. 0.5 D. 3
65:.1& CE Board November 1'996
6421 EE Board March 1998 Evaluate the integral of {3x2 + 9y2) dx dy if
&38: ECE Board Aprilll997 &471 EE Board October 1997
the interior limits has an upper limit of y
Evaluate the integral of sin
6
x dx from 0 to Integrate -1- w1th
. respect to x an d 10
21og10 edx
n/2. · 3x+4
evaluate the result from x = 0 and x = 2.
Evauae
I t
f- - - - -
1 X
and a lower limit of 0, and whose outer
limit has an upper limit of 2 and lower limit
ofO.
7t
A. - A. 0..278 A. 2.0
32 B. 0.336 J B. 49.7 A. 10

·1.-'~
2n C. 0.252 C. 3.0 B. 20
B. -
17 D. 0.305 . · D. 5.12 c. 30
D. 40
c.
3n
&43: ECE Boaa·d November l:9«JJ:
'II
t,, 6481 CE Board May :1995
32 2 What is the integral of cos 2x e 2
sin • dx? 6531 EE Board April :1996
Evaluate·the integral Jcos ydy x/21 2
5n
D.
32 sin2x Evaluate JJJzdz r dr2
sinu du .
y sin2y A e-·-+C 0 0 0
A. ·'-+-..--+C 2
2 4 A. 2/3
B. y+::>cosy+C B. 4/3
370 . lO<h SolveP, Problems in Engineering Mathematics (2"d Edition) by Tiong & Rojas Day 15- Integral Calculus 371

:. 1/3 659: ME Board April1.999 B. 9 955 670: Find the coordinateP. of the centroid
D. 5/3 Find the area bounded by the parabolas x?: c 5.955 of the plane area bounded by the parabola
= 4y andy =·4. D. 5.595 y = 4 - x? and the x-axis.
654: EE Board April1993
Find the area of the region bounded by / A. 21.33 665: ECE Board April1998 A. (0, 1)
c: Sx and y = 2x. · B. 33.21 Find the area (in sq. units) bounded by the B. (0,1.6)
C. 31.32 =
parabolas x? - 2y 0 and C. (0,2)
A. 1.22 sq. units D. 13.23 .; + 2y- 8 = 0. D. (1,0)
B. 1.33 sq. units
C. 1.44 sq. units 660: EE Board October 1997 A. 11.7 6711 Locate the centroid of the plane area
D. 1.55 sq. units Find the area bounded by the line x - 2y + B. 4.7 bounded by the equation y2 = 4x: x = 1 and
10 = 0, the x-axis, they-axis and C. 9.7 the x-axis on the first quadrant.
Ms: CE Board November %994 X::: 10.
D. 10.7
What is the area bounded by the curve x? A. ~.~
= -9y and the line y + 1 = 0? A. 75 666: ME Board April1998 B. ~.~
B. 50 What is the area between y = 0, y = 3x?, x C. ~~~~
A. 3 sq. units c. 100
= 0 and x = 2? D. (~5.V~
B. 4 sq. units D. 25
C. 5 sq. units A. 8 67::&1 Find the length pf arc of the parabola
D. 6 sq. units 661: CE Board May 1996 B. 24 x? =4y from x = -2 to x = 2.
What is the area (in square units) bounded C. 12
Mfn CE Board May 1995 by the curve y2 = 4x and .; = 4y? D. 6 A. 4.2 units
What is the area (in square units) bounded B. 4.6 units
by the curve y
2
=
x and the line A. 5.33 ",'li
6&7: CE Board May 1995 C. 4.9 units
X- 4 = 0?
B. 6.67 What is the area bounded by the curve y2 D. 5.2 units
c. 7.33 = x and the line x -4 =0?
A. 30/3 D. 8.67 673: Find the surface area (in square
B. 31/3 A. 11 units)·generated by rotating the parabola
C. 32/3 66::&: CE Board May 1997 B. 31/3 arc y = x? about the x-axis from x = 0 to x =
D. 29/3 Find the area enclosed by the curve x? + C. 10 1.
By+ 16 = 0, the x-axis, they-axis and the D. 32/3
657: EE Board October 1997 line x-4 = 0. A. 5.33
Find the area bounded by the curve y = x? 6&8: CE Board November 1.996 B. 4.98
+ 2, and the lines x = 0 and y = 0 and x = A. 7.67 sq. units CE Board November 1998 C. 5.73
4. B. 8.67 sq. units Find the area of the curve ~ = a 2 cos 2e. D. 4.73
C. 9.67 sq. units
A. 88/3 D. 10.67 sq. units A a 674: CE Board May 1997
B. 64/3 B. 2a The area enclosed by the ellipse
2
C. 54/3 663: ME Board October :1997 C. a x2 y2
D. 64/5 What is the area bounded by the curve y = D. a3 - +- =1 is revolved about the line x = 3.
9 4
x3 , the x-axis and the line x = - 2 and x = What is the volume generated?
M8: EE Board April1997 1? 6&9: Locate the centroid of the plane area
Find the area bounded by the parabolas y bounded by y = x2 and y = x.
= 6x - .; and y = x2 - 2x. Note: The A. 355.3
A. 4.25 -B. 360.1
parabolas intersect at points (0,0) and B. 2.45 A 0.4 from the x-axis and 0.5 from they-
axis C. 370.3
(4,8). ·c. 5.24 D. 365.1
D. 5.42 ll. 0.5 from the x-axis and 0.4 from they-
A. 44/3 square units axis
B. 64/3 square units t: O.~i from the x-axis and 0.5 from they- 675: CE Board May 1996
664: ME Board April 1999 The area in the second quadrant of the
C. 74/3 square units <~xis
Find the area in the first quadrant bounded circle X: + y2 = 36 is revolved about the
D. 54/3 square units by the parabola l = 4x, x = 1 & x = 3 l> 0 IJ from the x-axis and 0.4 from they-
il><b =
line y + 10 0. What is the volume
generated?
A. 9.555
372 100 l Solved Problems in Engineering Mathematics (2"d Edition) by Tiong & Rojas

A. 2218.33 680: CE Board November 1995


B. 2228.83 Find the moment ofinertia, with respect to
C. 2233.43 x-axis of the area bounded by the parabola
D. 2208.53 y2 = 4x and the line x = 1.

676: CE Board November 1995 A. 2.03 Topics


2
=
~tegral Calculus
12x 2.13
The area bounded by the curve y
and the line x = 3 is revolved about the
line x = 3. What is the volume generated?
B.
C.
D.
2.33
2.53 DMon · Definite and indefinite Integrals
Fundamental Theorem of Calculus
A.
B.
C.
179
181
183
DTue
Basic Integrals Formulas
- Exponential, Logarithmic,
D. 185

D ~
Trigonometric, Hyperbolic,
Inverse Trigonometric, By Parts,
677: CE Board November 1994
Given the area in the first quadrant Theory Wed Trigonometric Substitution,
= =
bounded by x2 8y, the line y -2 0 and

D D
Wallis Formula, etc.
the y-axis. What is the volume generated Applications
when the area is revolved about the line y
Problems Thu ··Area, Centroid, Arc Length
-2 =0?
A.
B.
28.41
27.32
D
Solutions Fri
Surface Area, Volume, Work
Moment of Inertia, etc.
Propositions of Pappus
C. 25.83
D. 26.81

678: Find the volume (in cubic units)


D D
Notes Sat
Hooke's Law
Multiple Integrals

generated by rotating a circle x 2 + l


+ 6x
=
+ 4y + 12 0 about they-axis.

A. 39.48 ANSWER KEY RATING


B. 47.23
C. 59.22
631. A
632,D
644.A 657.A
645. B 658. B
670. B
671. B 0 43-50 Topnotcher
D. 62.11 633.A
634.C
646. B 659.A
647.A 660.A
672. B
673,A 0 30-42 Pa·sser
679¥ CE Board May 1!.99$
Given the area in the first quadrant by i! =
635.C
636. D
648.A 661. A
649.A 662. D
674.A
675. B
0 25·-29 Conditional
8y, the line x = 4and the x-axis. What is
the volume generated by revolving this
637.A
638. D
650. D 663.A
651. D 664. D
676. B
677. D
0 0-24 Failed
area about the y-axis. 639.A 652. D 665. D 678.C If FAILED, repeat the test.
640. B 653.A 666.A 679. D
A. 53.26 641. B 654. B 667. D 680. B
B. 52.26 642. D 655. B 668. c
c. 51.26 643.A 656.C 669.A
D. 50.26
374 100.1 Solved Problems in Engineering Mathematics (2nd Edition) by Tiong & Rojas Day 15- Integral Calculus 375

IJI Using Wallis formula: m = 5; n = 3; and


a = 1, since both m & n are odd numbers
Using Wallis formula: m = 6; n = 0 and
a = rc/2, since m is an even number
1
6
J<x + 1f xdx = uv- Jvctu
'

I(3t -1) dt =
3
i I<3t 3
-1) (3dt)
0

I
n/2 nJ2
3 sin 6 x dx = 5( 3 )( 1) • (2:_)
= i( ~ Jc
(3t 1)4 + 0
5
sin xcos X dx
f
0
6(4)(2) 2
= Sn
32
= x[(X+1( ]- J(x+1( dx
-7 -7
= (5-1)(5-3)(3-1) •(1) _ -X 1 r, 7
= __2_,(3t -1)4 + c - 7(x + 1)7 + 7 J(x + 1f dx
12 • .
. 8(6)(4}(2}_
=0.0417 IIJ
+..![~+1r ]
6

rm
-10

I
--B
dx
X+2
•I
n/2

sin x dx
5
6
2
Jx(x-5f dx
5

Note: Judv = uv- Jvdu


= -x
7(X+1)
7

-X 1 [
7

11
-.---42 (x+1)--Bt
-6

Ju
du 0
-1 0
Note: =lnu Let: 7
7(1 + 1)7 - 7(0 + 1)
u= x; du = dx
-10 Using Wallis formula: m = 5; n = 0 and
l-1o 13

..:...(x_~-~-=-)- 4~[(t+1f6 -(0+1f6]


I
--B
-dx- = ln(x + 2) --B
X+2
a = 1 since m is an odd number

n/2
dv = (x-5)12dx; v = -

=0.022
= ln(-10 + 2)-ln(-6 + 2)
dx=~=0.533
= ln(-8) -ln(-4) J sin 5 x
5(3)(1)
6
2
0 Jx(x- 5f dx
BIB
ml
=In(~) =In 2

2

12
.rr/2

I
0
5
sin xcos x dx 5
5

x(x-5f3 _ __!_ r(x-5f3dx


13 13 J

=~(x-5f3 _ __!_
13 13
(x-5)
[
14
. 14]!:6
n/6

0
Jcos 3A dA 8

Let: u = 3A; du = 3dA, thus dA = du/3


Ixcos(2x + 7)dx 15

=5; n =5 and Change limits: when A = 0, u = 0;


2
Using Wallis formula: m =~(6-5)13 -~(5-5f3
= ± Jcos(2x + 7)( 4xdx) a = 1 since both m and n are odd 13 13 when A = rc/6, u = n/2

2
numbers
--1[(6-st -(s-st]·· Substitute:
=±sin(2x + 7)+C
I
nl2 182
5 5
12 sin xcos x dx = 0.456 n/2
J
n/2

ml
• J cos6 u -du =-
0 1 cos6 u du
=12( (5 -1)(5- 3)(5 -1)(5- 3))· (1) 3 3

I
0 0
3 + 4x 2 )dx =7x 4 4x 3 10(8)(6)(4)(2)
} (7x -+-+ C
4 3 =0.2 1 Using Wallis formula: n = 8 and a = rc/2,


xdx

1m J (X+ 1)6 = (x + 1)--B xdx since n is an even number

l J
0

Let:
.:!_
3
n/2
cose u du = ..!( 7(5 )(3 )(1) \
3 8(6}(4)(2))
(!:)
2
J(
n/2
5
sin x cos x) .dx 3
nJ2

I sin
6
x dx fi u x; du =dx
1)
0
1057t 357t
0 0 tl
·ii
dv (x t 1) 11 dx; v
(X I f =--=-
2304 768
371 .1 00 l Solved Problems in Engineering Mathematics (2nd Edition) by Tiong & Rojas


Day 15 -Inteqral Calculus 377

rm Note: Jeudu = eu + C
f du
=
s-du Ju-1
du
2

J -dx
-=- 1 -dx- J
2
1<12
u(u-1) -u-+
= -lnu+ln(u-1)-
0
3x+4 3 3x+4
0
J COSX 1"'2
. X x/<4
d X::: Sin Jcos2x e
51
n
2
"dx =~ Jes•n 2x (2cos2x)dx =-In( ex+ 1) +In( ex+ 1-1)
2 n/4
=-ln(3x+4)
1 1 = -~es•n2x + C = -ln(ex + 1) + ln(e•)
. 7t ( . 7t)
2 - Sin 4
3 0 =Sin 2 =-ln(e•+1)+x
1

=0.305
1
= rn[(3){2) + 4]- rn[(3)(0) + 4]
3 3
Note: TCI2
=0.293

=90" and TCI4 = 45" • Thus:

e• -1
f
--dx = ln(e• +1)


e• +1
Ill Jcosx dx = sinx + C


-[ -ln(e• +1)+x ]+c
2
Jcos y dy = §..!2- ~s 2
Y)dy
e
'Zi
= In( e~ + 1) +In( ex + 1)- X + c

=i J(1+COS 2y)dy
J'nx dx
J
=21n(ex +1)-X+C
= ln(e• +1)2 - x +C
fe• -1 e•dx dx


1

=i ± Jdy + Jcos 2y(2dy) let: u =In x; du = dx ; dv


X
=dx; v =x e• +1 dx =-;.--+1-

=In (ex +1)- - -


e• +1
dx
=j'_+ sin2y +C e
Je• +1
2 4 Jrn x dx =uv- Jvdu
e• + 1, thus e• = u -1; du = e'dx
"cosu . "rr lcosu
1 I et: u = J Jrsinudrdu = J2" sinu du
Ill = (lnx)(x)- Jx( dxx)
J du
0 0 0 0

J-b-cosxdx =xlnx-xj~
·I(,

f dx
e• +1 = e•(e• +1) = i "j( 2 2
cos u- o )sinu du
du 0
= e(lne) -e- [ 1(1n1) -1]
= f(u-1)(u:_1+1)
-i "
sin(~)=P-~sx
2
=1 du = Jcos u(-sinu)du
Note:

Ill
f
= u(u-1) 0
3
r;; X r;;---
1 A 8
=-..! cos ul"
Thus, v.<:sin-='.11-cosx 2 3 0
2 10 10
··-=-+-
u(u -1) u u -1
Substitute: J
1
21og e dx _
-=10
- - - 21og10 e -
X
Jdx
1
X
1=A(u-1)+Bu
1 =Au-A+Bu
=

1
-~(cos3 7t- cos3 o)
10 =--(-1-1)
f.J1-cosxdx = .J2 Jsin ~ = 21og10 e lnx1
1 1 quate constant:
6
=21og10 e (In 10 -In 1) 1
=-
=2fi ~sin ~)d2x =2 A 3

=2J2(-cos i)+c
~ ~2..[2 cos(i) +C
•Jcos2x e5in 2x dx = Je•in2x cos:tx dx
1\

I qu<Jte coefficients of u:

o
o
A11J
11 n

fy
2
2
Jc3x + 9y
2
) dx dy = r;
2 3 ly
+ 9y 2 x dy
let: u = sin2x; du =·cos 2x (2) II 1 0 0 0 0 .... ·

~··
378 1001 Solved Problems in Engineering Mathematics (2nd Edition) by Tiong & Rojas Day 15- Integral Calculus 379

2
Jx3 + 9y2xl: dy = Jv3 + 9y3) dy
0
2

0
ID rm ~
2 y2 =8x ~Eq.1 x2 =-9y ~ Eq. 1
= J1oy d~ 3 y =2X ~Eq.2 y + 1 =0
0 y = -1 ~Eq.2
Substitute Eq.2 in Eq. 1:
= 10y412 Substitute Eq.2 in Eq.1:
4 0 (2x)2 =8x
4
10(2)4 4x 2 =ax x2 = -9(-1)
=-- X=±3 A=2 Jvdx
4 X=2
0
=40 y = 2x = 2(2) =4 4

A=2 Jv'X dx
111 0
3
x/2 1 2 2 4

= JJJzdz r dr sinu du 2
3 3
"-A= 2(x)
~
=ix3'21
3 0
0 0 0
A= J-ydx = J--<YL- Yp)dx 2
x/21~2
JJ~ 2
r drsinu dl;J
-3 -3 A =i(4)3'2
3
0 0

JX
x/2 22 02
;
0

Jr drsinu du
2
2
=-J [-1-( ~J}x A = 32 square umts
3
.

R~
2
0 0 A= Jvdx Q Another Solution: Use the formula for
x/2 1 =- -1Jdx the area of a parabolic segment
0
= JJ2r drsinu du
2 -3

EE]
.
2
3
~ YL) dx
0 0 A= J<YP
= -[ (;r -x]' A= 3_bh

=2 Jr3 11 sinu du
x/2 3 0
2 1-3 A=li4X4)
3 · . h .==_4 · -r II

J(~ -2x)dx
=-(<~~3 -3]+((~3/-(-3)}
0 0 A= A_ 32 .. • · · .·· .o
0
-3 square units . ··. · ··
=2 J-
xl2[13-03J
-
3
sinu du
J8x2 2x 2
3
A = 4 square units
0 A=----
3 2
a
x/2

=~ J sinu du 2 1m
0 2J8x3/2 2 12
A -x
y 2 =X ~Eq.1
2
,;-(-cosu) 1"'2 3 0
X= 4 ·-t Eq.2
3 0
A= 2J8(2)a'2 (2)2 -[ 2J8~o)3'2 -(0)2 J
=~( -cos%+cosoo) Substitute Eq.2 in Eq.1:
A = 1.33 square units
2 / 4
3 4
y 12
-~~ A= Jydx
y· 0
380 J..OO I Solved Problems in Engineering Mathematics (2"d Edition) by Tiong & Rojas
Day IS- Integral Calcull.!s 381
4 4

A= J(x 2
+ 2) dx A= J( 8x- 2x )dx
2
Ill Substitute Eq.2 in Eq.1:

0 0
2 c- x2
\ 2y ~ 10 =0 vx - -
4
A=~ 3
· 2x
14 = 8xz- 2x314
when x = O: y = 5
x4
2 3 6 wheny=O:x=-10
0 4X=-
16
43 =4(4) 2 -~(4)3
A=( ) +2(3)
rx=-~~
3 64 = x3
3
88 ' 64 ' X=4
A=- square un1ts A = - square un1ts
3 3 y=214

rm
y=4

II y = 6x -x 2
Thus, the parabolas will intersect at (0,0)
Solving for the coordinates of the and (4,4).
x2 - 6x = -y intersection: 10
[X= 10J

(x-3f = -Y+9 A= Jydx


2
x 2 = 4y = 4(4)
(x-3) =-1(y-9) 0
X =±4

By inspection, the vertex is at (3,9) and = 1n 5+~}x


the parabola is facing down. 0

2110
y = x 2 -2x .~ 5x + 2~2) o
x 2 -2x = y
102
(x-1)2 = y+1 4 (0,0) =5(10)+-· 4 4
4
(x -1)2 = 1(Y+ 1) A=2 Jxdy A ·= 75 square units A= Jydx= J<Yupper-Yiower)dx
0 0 0
By inspection, the vertex is at (1,-1} and 4 4
u Another Solution: Use the formula for
the parabola is facing up. =2 f<J4Ydy = 4 Jy 112 dy the area of a trapezoid = x2JX- :)dx
0 0 0

4( ~:~~ J %(Y)3'2[
~c:J~ = 2;;~2 -4(:)[
1
A 2(a+b)h
= =

=~(4)3/2
A ~(5+10)(10) II
Ill
.0
4(4)3/2 (4)3
3 A 75 square units h = 10 =-3--12
A= 21.33 square units
A = 5.33 square units
·0 Another Solution: Use the formula for ml
the area of a parabolic segment
y' 4x
Ill
4 4

A= Jydx = J<Yupper- Yiower) dx A=~bh b=8


y 2/x ~ Eq.1 x2 =-By -16
0
4
0
3
2
A =-(8)(4) '\!17 4y x2 =-8(y+2)

= K( 2 2
6x - x ) - { x - 2x) dx J 3
A= 21.33 square units y
X

4
'lq ;> By inspection, the vertex is at (0,-2) and
0 the parabola is facing down.
382 l 001 Solved Problems in Engineering Mathematics (2nd Edition) by Tiong & Rojas Day 15- Integral Calculus 383

dx

=
1
A 2 = Jx dy
0

~[
3
By 1nspect1on the vertex is at (0,-4) and
the parabola 1s facing down.

x 2 = 2y ~ Eq.2

By inspection, the vertex is at (0,0) and



the parabola is facing up.
o4'_ (1)4
4 Equate Eq.1 to Eq.2:
A 2 = 0.25
2(y-4)=2y
Atatal = JA1j + JA2j dx
4 8 =4y
= 4 +0.25 2 2
A= J-ydx y=2
Atatal = 4.25 square units 2 A= Jydx = J3x dx
2
0 x = 2(2)
0 0
X =±2
Note:
x 2 + 8y+16 = 0
Ill rhus, the two parabolas intersect at
x3
=3-
3
points (2,2) and (-2,2).
-By= x2 +16 =x31:
x2
-y=-+2 (0,0) =(2)3
8
A = 8 square units
Substitute:

Iii
A= X~ +2}x
0
~____!_/ y2 =X ~Eq.1
~ 2 X =4 ~Eq.2
14 Y2 =4x
= 3~8) +2X 0
3
A= Jydx
y = 2../x ··2
Substitute Eq.2 in Eq.1:
2
= (4)3 +2(4)
24
3 3 y2 =4
= f(Yupper- Y1ower)dx
A= 10.67 square units A= Jydx = 2 J..fXdx y=±2

n ~
-2


1 1

= 2x3/2 = ~ x3'213 = 8 -2x2 - x: ] dx


3/2 3 1 -2
2
= ~[(3)3/2- (1)3/2 J
0

A1 = Jx dy
3 ~- J[ 8 - 2x
2
J dx
A = 5.595 square units 7
-2

~10
• ·J[
3
2
= ; [ax ; 4
-2 A= 2 Jydx
04 -{-2)4
=----
4
x 2 +2y-8=0 ::I [ B(2) ~ (2) 3
) l8(-2)- ~(-2) 3 )] a
4
A1 =-4 x2 =-2y+8 A I(} tili( •;qu;llf~ tlllll~l A= 2 J..fX dx
x2 = -2(y- 4) ~ Eq.1 0
_ _ _ _ _ _ _ _ _ _ _ _ _ _ _ _ _ _ _ __,D:::.a= 15- Integral Calculus. 38_5
384 i 001 Solved }>robJems in Engineering Mathematics (2"d Edition) by Tiong & Rojas

A= 2(x)
~
2
3
2
= ~x.3121
3
4

0

x
2
=y~Eq.1
y =X ~·Eq.2
A•x=

=
1

0
1
2
f(x-x )dx•x
1

Jydx•x= J(YL -yp)dx•x


0 •X
y = 4-· x2
2
=-y+4
A =~(4)312 ()
3 1
x2 =-1(y-4)
Substitute Eq.2 in Eq.1:
A =32
- square um'ts
3
co• f(x 2 -x 3 )dx By inspection, the vertex is at (0,4) and
x2 =x


0 the parabola is facing down.
X=1
Y=X =1
=-~3- :4[ at y = 0, x = ± 2

Thus the parabola intersects the x - axis


r 2 = a 2 cos2e 1- 13 14
Thus, the parabola and the line intersect ··-X=--- at points (-2,0) and.(2,0).
at points (0,0) and (1, 1). 6 3 4
e r
x=0.5 (0,4)
o· ±a
30" ± 0.707 a
45" 0
so· i
go· i
120" i ~­
.(2,0)
150" ± 0.707a
L1ao· ±a Solving for the area of the parabolic
1 1 segment:
45°
A= Jydx= J<YL-Yp)dx 1 1
. 2 2 32
0 0
1\•y= Jxdy•y= J(xp·-xddy•y · A=-bh=-(4)(4)=--
1 3 3 3
o• = J(x -x2 ) dx
0
1
0
1 1
0
= f{Jy -y)dy•y A•y=2 Jydx•~= Jy dx
2

A=4
[
i Jr
45° 2
de
l = x2
2 31o
12 13
=---
_tJ
=-
0
1

f( y3/2- y2)dy
0
=
1

J4-
0

x
2 2
) dx
0

0
2 3
~:~~- y:[
45° 2
=2 Jr 2
de A=-
1
6
= = jt16- 8x2 + x4 )dx
0 0
45° 1. (1)5/2 (1)3
= 2.(;) J 0
2
a cos 20(2d9) (j
yoc•------
5/2 3 8x 3 x5·
=16x-3-+5 o
1
2

y 0.4
2
=a sin2ej
45"

0
(332)(y )=16(2)--8(~)3 + (~5
I hus, the coordinates of the centroid is
=a 2 [ sin(2)145.)- sin(2)(0·)] "' (0 ~i. 0.4)
1 1 y=1.6
A=a 2
A•x= Jydx•x= J<YL -yp)dx•x
Thus, the centroid is at point (0, 1.6)
0 0
386 1001 Solved Problems in Engineering Mathematics (2nd Edition) by Tiong & Rojas Day 15 -Integral Calculus 387


l=4X
atx=1,y=±2
-r.
A•y=2. J(4x)dx
1
1

=~( 4;2 J[
.
Change the limits:

when, X::: 0; 8 = 0
X= 2; 8 = 45o dy
-=2X
dx
Y = x2
dy = 2xdx

Thus; the parabola and the line intersect s= JJ4


45° ,.------;:-
2
+ 4tan 8 (2sec 8 d8)2 dY) = 4x 2
( dx ·
at points (1,2) and (1 ,-2).
j(Y )=<1>2 0

J
45°
2 2 Substitute:
= J4(1 + tan 8) (2sec 8 d8)
- 3
Y=- 0 1
4 45° 2
A = 27t JJ1 + 4x dx • x
2 2
-y Thus, the center is at point (3/5, 3/4). = J J4(sec 8) (2sec 8 d8) 0
(0,0)


0 1
21t ~( 2)1/2
45°
2
= B J 1 + 4x 1 (8x)dx
= J 2sec8 (2sec 8 d8) 0
0 2)3n 1
45° = 7t ( 1+4x = 27t(1+4x2)3'21
3 4 3/2 12
Solving for the area: =4 J sec 8 d8) 0

=~[(1+4(1)f'
0 2 312
(0,0) 45°
-(1+4(0)) ]
1(2
A=-
2 3 3
1 4
-bh ) =-(4)(1)=-
3 4(~sec8tane +~ln(sec8 + tan8)l A = 5.33 square units

l
I
-c

2.
2
s=2 fv1+(:~) dx
1
0
-sec45°tan45° ml
~ln(sec45°
A•x= Jydx•x 4 1 x2 y2
0
X
2
=4y = [: + tan45°) Given equation: - +- = 1
1 9 4
2xdx =4dy s '4.6
= J<2.JX )dx • x


x2 y2
0 dy =~ Standard equation: 2 + 2 =1
1 dx 2 a b
312
=2 Jx dx .( dy)2 x2
By inspection: a = 3 and b =2
0 ldx =4 Y= x2
1
=2()(612) when x =1, y =1
Substitute:
5/2
0

j(x) = ~< 1 >s'2


S=2 2F?
J 21¥
J- x2
1+"4 dx=2
4+x2
-dx
4
- 3 0 0
X=-
5 2 2
Using the second proposition of Pappus:
=fJ4+x2 dx=J~(2)2 u2 dx A
1

Jds• 21td
0 0 V=A•21td
1 1 0

Jvdx"~=i Jldx
fJ--------2
= ( 7t8b). 27td
A•y= Let:
~~)
I
Q 0 x =2tane; dx == 2sec2 e de A 1I ( dx • 21tX =(~)(3)(2)(21t)(3)
II V = 355.3 cubic units
388 . 1001 Solved Problems in Engineering Mathematics (2nd Edition) by Tiong & Rojas


Day 1~- Integral Calculus 389
Substitute r = 6:
y-axis

~ V=(~nr2 )-2n(;: +10)


46
=..!.(n)(6)2 (2n)( ( ) +10) C(-3,-2)
4 . 3n (0,0)

Solving for y ·

V =; 222B.91B cubic units

x. b V
. . ~~!+:.3~x ~
Using the disk method:

V =n Jy
4

0
2
dx
, dx
By inspection, d = 3.

Using the second proposition of Pappur.:


'" '" V~y I y
4
2
Ay= JdA•y/2 ~ =n f(YL- YP ) dx V = A• 2nd
0
r · = nr2 • 2nd
=Jydx•y/2 41( 2]2 =n(1)2 (2n)(3)
=n gl2- ~ dx
0 V = 59.217 cubic units


r

=~ Jldx =n 4-
x2
+
x4
dx
c
r Using the shell method:
X 0
2 64 J
4
=~ J<r2 -x2)dx x3 xs x2 =By
= n( 4
x- 2(3) + 64(5) J
0
V·+•P3-x)y dx] 10 At X= 4, y = 2

=~(rx-~;)[ 3
(4)3
=n [ 4(4 )- 2(3) + 64(5)
(4)5 ]
Thus, the parabola <:~nd the line intersect
at point (4,2).
3 =4n J(3- x)( .fi2X) dx v = 26.B08 cubic units
r J


2 3
4nr
1 (-
Y) =21 ( r -3 0
3

=4n J(3.J12x 112 - 312

~x'?\(y )=; (! r'~)


.J12x ) dx
0
3 x 2 +y 2 +6x+4y+12=0
- 4r X3/2 XS/2
(0,0)
y=-
3n =4n[3J12--J12--
3/2 5/2 ]1 By completing square:
.Q

-~.Ji2(3)5 12 J
~
2 312
Given equation: x + y 2 = 36 = 4n[ (2.Ji2)(3) (x+3) 2 +(y+2) 2 =-12+(3) 2 +(2) 2
(x + 3) 2 + (y + 2) 2 =1
By inspection, r = 6 V = 1B0.955 cubic units
~dy
Using the second proposition of Pappus:

V=A•2nd • Standard equation: (x- h)2 + (y- k)2 =

By inspection, h = -3 and k = -2, thus the


conter is at (-3,-2). Also the radius of the
r2
Using the ring method:

I
x 2 =By 2

V =( ~nr )- 2n(y+ 10J


2 ctrclc is equal to 1.
V= nJ( XL2 - Xp
2
)dy
At =
y = 2, x ±4 0

~:
ll
·~·
390 100 1 Solved Problems in Engineering Mathematics (2nd Edition) by Tiong & Rojas

~ )dy
2
2
v =1t K(4)2 -( JSY)2J dy IX= 2 jy (1-
0
2

= 1t J(16-8y) dy =2Xy2- :4}y Topics


0

=1t(16y-
8
~
2
0

{-4-y3 y5
2

)0
D Man
Integral Calculus
Definite and Indefinite Integrals
)[ = 4(5) Fundamental Theorem of Calculus

= 1t[16(2)-4(2) ]
2
= 2((2)3 - (2)5)
4 20
1

D Tue
Basic Integrals Formulas
- Exponential, Logarithmic,
Trigonometric, Hyperbolic,

D [Q]
V = 50.265 cubic units
=2.13


lx Inverse Trigonometric, By Parts,
Theory Wed Trigonometric Substitution,
Wallis Formula, etc.

l=4X
D D
Problems Thu
Applications
-Area, Centroid, Arc Length

At X= 1, X=± 2 D D
Solutions Fri
Surface Area, Volume, Work
Moment of Inertia, etc.
Propositions of Pappus ·.
Thus, the parabola and the line intersect
at points (1 ,2) and (1 .~2).
Notes
D Sat
Hooke's Law
Multiple Integrals

.. ' ~ ~ ., ' " ~

lx
2

=2 JldA
0
eJ
tT
'

'

'

2 ll
2
= 2 Jy (xdy)
0
2 <·<·'> "'~''"'")'."•<,·~ ~·"' ·!?;1'<-~""''~h··~·-'"'"·.Cd'''

2
lx = 2 Jy (xL- Xp) dy "•··~«<'r~oi,.-. ~:<-#0".~:.'<>< ·~. ~,•f.f<~~1<'<'<<'<'>~~&~·'i" .... »OO<'i""ii<.''i"••"'"""""'<

0 '· " ,. ><-"'-< " * "+ .... .,. " " ~ ~ .- •. "'. "-·" ·<·
392 l GO 1 Solved Problems in Engineering Mathematics (2nd Edition) by Tiong & Rojas

.;h. •¥ .,.,.,.
Topics

<;-. « ~-
D
~l1on
Differential Equations
Types of Differential Equations
Orders of Differential Equations
D
Tue
Degrees of Differential Equations
Types of Solutions of Differential
Equations

Theory
D
VVed
Solutions to First-Order Differential
Equations
Applications to First--Order DE
D lQJ
'_.,; " ~ ..; ,. ·' " '
(Population Growth, Radioactive
,, ·' ....... ~ "' ~ ' ~ .:!" ,· ~ ,, <-. " ~ • '" ' < -'" ".(> ~ <,•
Thu
,<( '' y N

Problems Decay, Continuous Compound

<·-->"'·~~

·~«<·0<~'~"''""-
~*~ ~ ·' '"'''~,.

,.,~ ...
.....,.':'""' ......

..<.,..«.-~~-;,""~
D []
Solutions Fri
Interest, Flow, Cooling and Heating,
Newton's Second Law of Motion,
Geometric problems, Orthogonal
.. ~ / " ' ~"~-:%>>".*"<''<' ·~·.• .• :·t~~~~~

'· v • ,. \;<' ~ ~ .., 4>:15:


·D D
Notes Sat
Trajectory)

What is a Differential Equation?


Example:
1\ differential equation is an equation that
The differential equation dy = 2x has for
' ontains one or more terms involving dx
1lr !rivatives of one variable (the dependent 2
v.1riable, y) with respect to another its solution y = J2xdx = x + C , where Cis
v.u iable (the independent variable, x). an arbitrary constant.

I xamples: Whatl!_re the Types ~f Differential


Equations?
a. (2xy +x 2 ) dx + 2y dy = 0

(-<:!_~~J2- Jd2y2 J3 + x3 ~ 0 1. Ordinary Differential Equation- an


b. equation containing only one
dx
3
l dx independent variable, thus
having only ordinary derivatives in
c. (x - 1) y" + xy' + y =0 the equation.
(j
ilx ax
y-=
I 2 3
ily (fz Examples a, b and cat the left of this
page are examples of ordinary
llu· :;olul1ons of differential equations are differential equations
lttll<.llow; and not just numbers like the
dq' ·Ill ;tic: r" 111< 1l1oll' •
394 1001 Solved Problems in Engineering Mathematics (2nd Edition) by Tiong & Rojas Day 16- Differential Equations 395

2. Partial Differential Equation - an Example: x3 - 3y =e- Test for homogeneity of degree n: General solution:
equation containing two or more where: C =arbitrary constant
independent variables, thus having
partial derivatives in the equation. 2. Particular solution - the solution
Let: f(x,y) = M(x,y) dx + N(x,y) dy
f(rx, ry) = r" f(x,y)
. 1
y1-n -- ef(1-n)P(x)dx f<1-n) Q(X) ef(1-n)P(X)dx dx

that has no arbitrary constant.


·Exampled in the previous page is an 3. Exact typf! ·
example of partial differential Example: 2xy + i - 2=0 The Bernoulli Equation is named after the
equation. Standard form: brothers Jakob (1654 -1705) and Johann
Bernoulli (1667,... 1748).
Ill What Is an Order of a Differential What are the Solutions to First Order M(x,.y)dx + N(x, y}dy ""0
Equation? Differential Equations?
What are the Applications of First-Order
II The order of a DE is the order of the 1. Variable separable type Differential Equations?
General solution:
highest ordered derivative which
I appears in the equation. Standard form: F(x, y) = C 1. Population growth problems.
I Examples: P(x)dx +Q(y)dy =0
where: F(x,y) = jM(x, y)ax + k1 dP =kP
dt
First Order: dy = 2x2 + 5x + 3 F(x,y) = JN(x, y)ay + k 2
dx General solution: where:
dP =rate of change of the population
d2 . jP(x)dx + lQ(y)dy-~:c. Test for exactness: dt
Second Order: - { =5x 2 + 1Ox + 3 P =number of inhabitants at any time
dx
2. Homogeneous type OM oN t
ay""' ax k = constant of proportionality
Third Order: d3y = 3x2 + 6x + 2 Standard form:
dx 3 2. Radioactive decay problems
fVl(x,y)dx +N(x,y)dy =0 4. Linear type
What is a Degree of a Differential General solution: dQ""kQ
Equation? Standard form: dt

The degree of a differential equation


=
Substitute y = vx or x vy and dy + y P(x)"" Q(x)
the resulting DE becomes a dx where:
whose terms are polynomials in the dQ
variable separable type· - = rate of change of the substance
derivatives is defined as the highest
power of the highest order derivative. General solution: dt
Q =amount of the substance present
Note: First-order differential equations are at any timet
Examples:
not always separable. Homogeneous y(i.f.) = Ja<x)(Lf.) dx + c k = constant of proportionality
differential equations, however, may .be
3
transformed into separable equations by 3. Continuous compound interest
First Degree: 2 ..1...) =15
d2 + (d
x-Y
the substitution of a variable.
dx dx problems
where: .1. f. = .10tegra!"mg factor = eJP(x)dx
2 An expression is said to be homogeneous
3
Second Degree: d2
x ( dx; J + (dd~ ) = 15
if all terms have the same degree. The
term "homogeneous" is also used to
Note: Integrating factor is also known as
Euler's multiplier.
dP =rP
dt
indicate that the right-hand member of a where:
linear differential equation is 0 5. Bernoulli's equation type
What are the Types of Solutions of dP
- = rate of change of the account
Differential Equations? dt
Standard form:
P = money ·present in the account at
1. General solution- the solution has a~timer /
at least one arbitrary constant. dy 1 y P(x) y" Q(x) r =nominal rate of interest per year
dx
396 1001 Solved Problems in Engineering Mathematics (2"d Edition) by Tiong & Rojas

4. Flow problems

dQ
dt = rate of gatn
. - rate of loss (:~)! =-(:;)g
where: Topics
wnere:

~I dQ
- =
=
Q concentration of the mixture at
any timet

rate of change of the


( dy)
dx 1
=slope of the orthogonal
trajectory
D Mon
Differential Equations
Types of Differential Equations
Orders of 'oifferential Equations

~
dt
concentration of the mixture ( dy)
dx 9
=slope of the given family of
curves
D Tue
Degrees of Differential Equations
Types of Solutions of Differential
Equations
D D
II 5. Cooling and heating problems
( dx) 1 Solutions to First-Order
! dT =k(T -t s )
dy 9 = ( dy)
dx 9
Theory Wed Differential Equations

liJ ~ Application to First Order DE


dt l3r Cooling process
.(Population Growth, Radioactive
Proceed to the next page for your 16th
Problems Thu Decay, Continuous Compound
dT =k(t -T)
dt

where:
$
1:'/r Heating process test. Detach and use the answer sheet
provided at the last part of this book. Use
pencil number 2 in shading your answer.
'~i
il,
D D
Solutions Fri
Interest, Flow, Cooling and
Heating, Newton's Second Law
of Motion, Geometric Problems,
dT
- =
dt
=
rate of change of the body temp.

T temperature of the body at any


timet \lrribia:
GOOD LUCK!
D D
Notes Sat
Orthogonal Trajectory)

=
t. surrounding temperature Did you know that. .. the fame and
influence of Pythagoras spans for
centuries! Pythagoras even appears in t.8u Determine the order and degree of c. 2r dx == (J?- + 1> dy
Shakespear's Merchant of Venice, when 1111~ differential equation D. y dx + (2x- 3y) dy = 0
6. Newton's second law of motion

dV
reference is inade to the society's belief in
the transmigration of souls. 2x _r
d4 + 5x
dy
4
2 ld_1_ ) -
. dx
3
xy =0 . 684: ECE Board April1998
2
F=m- The equation y = ex is the general
dt
~uote: solution of:
1\ Fourth order, first degree
where: "The most painful thing about mathematics
II Third order, first degree
F =force
m = mass of the body
is how far away you are from being able to
use it after you have learned it."
<. First order, fourth degree A. y'= 72y
1, First order, third degree
~~ = rate of change of velocity - James Newman B. -y'= y2x
(,,.·zz Which of the following equations is
.111 oxacl DE?
7. Geometrical problems (Orthogonal
C. Y·-
- _1_
2x
2
trajectory) 1\ (x + 1) dx - xy dy = 0
A curve which intersects all curves of II x dy + (3x- 2y) dx = 0 X
a given family at the same angles is Jxy dx + (2 + J?-) dy = 0 D. y' = 2y
referred to as a trajectory; if the II x1 y dy - y dx = 0
intersection is at right angle, the curve
is called orthogonal trajectory. 685: EE Board Mareh 1998
1.1-1, Which of the following equations is a
Solve the differential equation: x (y - 1) dx
"· 111. rhiP ~~eparable DE?
+ (x + 1) IJy = 0. If y = 2 when x = 1, /
determine y when x = 2.
11 (x 1 x"y)dy=(2x+xy2)dx
II ( •. • y) d X ?y dy - 0
398 100 I·· Solved Problems in En Mathematics Day 16- Differential Equatl.ons 399
3
A. 1.80 5 696: Solve the linear equation: A. (y") - xy" + y' = 0
B. 1.48 A. y(k)=4-k B. y" - xyy' = 0
dy +'i.. = x2.
C. 1.55 C. xy" - (y') 3 - y' = 0
B. y(k)=20+5k dx x
D. 1.63 D. (y') 3 + (y") 2 + xy = 0
C. y(k) =C...: k , where Cis constant
68&: EE Board October 1997 x3
D. The solution is non-existent for real A. xy 2 =-+C 701: EE Board April1997
If dy = x 2dx; what is'the equation of yin values ofy 4 Radium decomposes at a rate proportional
terms of x if the curve passes through x4 to the amount at any instant. In 100 years,

~I 691& EE Board Aprll1995 B. xy= +c 100 mg of radium decomposes to 96 mg.


(1 '1 )? 4 How many mg will be left after 100years?
Solve {y-~x 2 +y 2 ) dx-xdy=O x4
x 2y = -+C
I~ x 2 - 3y + 3 = 0
A. C.
4 A. 88.60
B. x3 - 3y + 2 = 0 x3 B. 95.32
3 2 A. ~x2 +y2 +y =C D. y=-+C C. 92.16
Ill C. x + 3y + 2 = 0 4 D. 90.72
-~ D. 2y+X 3 +2=0 B. ~x2 +y2 +y =C
b971 CE Board May 199"/
C. ~x+y +Y =C Find the differential equations of the family
· 702: The population of a country doubles
687: ECE BoaJ'd November 1998 in 50 years. How many years will it be five
Find the equation of the curve at every D. ~x 2 -y +y =C of lines passing through the origin. times as much? Assume that the rate of
point of which the tangent line has 'a slope increase is proportional to the number of
of2x. A. ~x-~y=O inhabitants.
692& ECE Board November 1994
Find the differential equation whose B. ~y-~x=O
A. X=-y 2 +C general solution is y =.C1x + C 2ex. C. ~X+~y=O
A. 100 years
B. 116 years
B. y =-X 2 +C D. ~X+~y=O C. 120 years
A. (x- 1) y" - xy' + y=0
c. y = y2 +C B. (x + 1) y" - xy' + y=0 D. 98 years
2 C. (x- 1) y" + xy' + y =0 b98: CE Board May 199&
D. X= y +C
D. (x + 1) y" + xy' + y=0 What is the differential equation of the 70:J: Radium decomposes at a rate
family of parabolas having their vertices at proportional to the amount present. If half
688& ECE Board Aprll1995 the origin and their foci on the x-axis. of the original amount disappears after
69:J& EE Board October 1995
Solve (cos x cosy- cot x) dx- sin x sin'y
Find the general solution of y' = y sec x 1000 years, what is the percentc>ge lost in
dy = 0 A. 2xdx- ydy = 0 100 years?
A. y = C (sec x +tan x) B. xdy + ydx = 0
A. sin x cosy = In (c cos x) A. 6.70%
B. y = C (sec x- tan x)
B. sin x cos y = In (c sin x)
C. y = C sec x tan x c 2ydx- xdy = 0 B. 4.50%
C. · sin x cosy= -In (c sin x)
D. y = C (sec2 x tan x) dy C. 5.36%
D. sin x cos y = - In (c cos x) [) --X=O D. 4.30%
dx
694& EE Board Aprll1996
M9: EE Board Octeber 1997 Solve• xy' (2y- 1) = y (1 - x) 704: ECE Board November 1998.
Solve the differential equation dy -xdx = 0, C.99: CE Board November 1995
Determine the differential equation of the Find the equation of the family of
if the curve passes through (1,0)? orthogonal trajectories of the system of
A. In (xy) = 2 (x- y) + C family of lines passing through (h, k).
B. In (xy) = x - 2y + C parabolas y 2 = 2x + C .
A. 3x 2 +2y-3 =0 C. In (xy) = 2y - x + C
1\. (y-k)dx-(x-h)dy=O
B. 2y + x 2 -1 = 0 D. In (xy) = x + 2y + C
A. y = ce-•
. d
C. x 2 -2y-1 = 0 695& EE Board April 1996
II. (y-h)+(y-k)=_'j_
B. y = Ce 2 "
dx
D. 2x 2 +2y-2 =0 Solve (x + y) dy = (x- y) dx~
c (x-h)dx-(y-k)dy=O C. y = ce•
A. ~+f=C I) (x+h)dx-(y-k)dy=O D. y = ce-2•
690& ME Board April1996
What is the solution of the first order B. ~- + 2xy + f = C
=
differential equation y(k+1) y(k) + 5. C. ~- 2xy- i =C 7oor Determine the differential equation of
D, X: - 2xy + l =C llu: l;tmily of circles with center on the
y ~IXIS
400 l 001 Solved Problems in Engineering Mathematics (2nd Edition) by Tiong & Rojas

70S: According to Newton's law of A. 171.24 lbs


cooling, the rate at which a substance B. 124.111bs
cools in air is directly proportional to the C. 143.25 lbs
difference between the temperature of the D. 105.121bs
substance and that of air. If the
temperature of the air is 30• and the 709: ME Board April1998
substance cools from 1oo· to 70° in 15 If the nominal interest rate is 3%, how
minutes, how long will it take to cool 1oo• much is P5,000 worth in 10 years in a Topics

It
to so·?

A. 33.59 min.
continuously compounded account?

A. P5,750
DMon
Differential Equations
Types of Differential Equations

~ B.
C.
D.
43.50 min
35.39 min
45.30 min
B.
C.
D.
P6,750
P7,500
P6,350 D
Orders of Differential Equations
Degrees of Differential Equations
II Tue Types of Solutions of Differential

I 706: An object falls from rest in a medium


offering a resistance. The velocity of the
object before the object reaches the
71:0: ME Board Oetober 1997
A nominal interest of 3% compounded
continuously is given on the account.What D D
Equations
Solutions to First-Order
ground is given by the differential equation is the accumulated amount of P10,000 Theory Wed Differential Equations
dV/dt + V/10 = 32, ft/sec. What is the
velocity of the object one second after it
after 10 years?
D (g Application to First Order DE
(Population Growth, Radioactive
falls? A. P13,620.1 0 Problems Thu Decay, Continuous Compound
B. P13,500.10
A.
B.
C.
40.54
38.65
30.45
C.
D.
P13,650.20
P13,498.60 liJ D
Solutions Fri
Interest, Flow, Cooling and
Heating, Newton's Second Law
of Motion, Geometric Problems,
D. 34.12

707: In a tank are 100 liters ofbrine


containing 50 kg. total of dissolved salt.
D D
Notes Sat
Orthogonal Trajectory)

Pure water is allowed to run into the tank


at the rate of 3 liters a minute. Brine runs
out of the tank at the rate of 2 liters a
minute. The instantaneous concentration ANSWER KEY II I RATING
in the tank is kept uniform by stirring. How
much salt is in the tank at the end of one 681. A 691.A 701. c c::J 25-30 Topnotcher
hour? 682.C
683.C
692. A ·· 702. 8
693.A 703. A c:J 1&-24 Passer
A.
B.
15.45 kg.
19.53 kg.
684.C
685.C
694. D 704.A
695.C 705.A
c:J 15-17 Conditional
C.
D.
12.62 kg.
20.62 kg.
686.8
687.c
696.B 706.
697.B 707.8
c 0 0-:Lif failed
688.8 698.A 7.08.A If fAILED, repeat the test.
708. A tank initially holds 100 gallons of 689.C 699.A 709:8
salt solution in which 50 lbs of salt has 690. B 700. c 710. D
been dissolved. A pipe fills the tank with
brine at the rate of 3 gpm, containing 2 lbs
of dissolved salt per gallon. Assuming that
the mixture is kept uniform. by stirring, a
drain pipe draws out of the tank the
mixture at 2 gpm. Fi.nd the amount of salt

I in the tank at the end of 30 minutes.

'·L. ..
402 I 00 I" Solved Problems in Engineering Mathematics (2"d Edition) by Tiong & Rojas Day 16- Differential Equations 403

Ill a x(y - 1) dx + (x + 1) dy = 0
xdx +~:cO
dy
-=2X
.
J!!'-111 dy- xdx = 0
Fourth order, since the highest derivative
dx ~
X+1 y-1 dy = 2xdx
in the equation is 4th derivative. First dy = xdx
degree, since the exponent or power of 1 -) dx+--=0
1-- dy Integrating:
the 4th derivative is 1.
( X+1 y-1
Integrating: Jdy = Jxdx
Integrate: Jdy =2 Jxdx
ED f f~+ f~= fo
xz
I A DE whose equation isM( x,y )dx +
dx-
X+1 y-1
2x 2
y=-+C
2 at
Y=-+C
x = 1; y = 0
2

I N( x,y )dy = 0 is exact if aM= aN X - In (X + 1} + In (y - 1} = C y = x2 +C


o'=l.!t+c


ay Ox 2
I Note: From the choices, choice (c) is an
exact DE.
atx=1;y=2
-ln(1 +1) +ln(2-1)=C
1
C=--
2

'
2xydx + ( 2 + x
2
} dy = 0 (cos x cosy- cot x)dx- sin x sin y dy = 0
c = 0.307
M = 2xy N=2+~ M=cosxcosy-cotx
Thus the solution is,
a(2xy) a( 2 + xz) =2X at x = 2; y =? x2 1
il(cosxcos Y-cot x) = -cosxsin y y=---
--=2X 2- In (2 + 1) + In (y- 1) = 0.307 2 2
ay Ox ay
In ( y - 1) = - 0.594 · 2y = x2 -1
y _ 1 = e·0.594
mJ N = _sin x sin Y x2 2y -1 =0


-

y = 1.55 a( -sin X Sin Y) = -COSX sin Y


A DE is a variable separable if the ax
coefficient of dx shall be functions of x
only and the coefficient of dy shall be
1m dy=~dx
2 Assume the solution has the form: y(k) =
functions of y only. fdy = Jx dx Note: aM = aN , then the given DE is an
20 + 5k
ay ax
x3 ,,xact DE.
Note: From the choices, choice (c) is a y= -+C Solve y(k + 1) using the assumed
variable separable DE 3
solution:
atx=1;y=1
2y dx = (x2 + 1)dy 1 = (1)3 + c J(cosxcosy-cotx)Ox.+k 1 =C y(k + 1) = 20 + 5(k + 1)
2ydx (x 2 + 1)dy 3
2 y(k + 1) = 20 + 5k + 5
y(x + 1) = y(x 2 + 1) C=3. ·~1nxcosy-lnsinx+k 1 =C ~ Eq. 1
3 y(k+1)=(20+5k)+5
lxz2+ 1)dx = ( )dy ~ Thus the solution is,
J(-sinxsiny)8y+k 2 =C y(k+1) = y(k)+5
-sinx(-cosy)+k 2 = C
x3
2
Ill Y=-+-
3
3
3y = x + 2
3
sinxcosy+k 2 =C ~ Eq. 2 Since, the resulting equation is the same
to the given DE, then the assumed
equation is the solution to the DE.
y 2 =ex I ry comparing equations 1 and 1, k 1 = 0
x3 - 3y + 2 = 0 .u1d k2 = - In sin x
c= L
X
2

1ill IS, the solution is,


1m
Differentiate:
0 = x(2yy ')- yz
Ell sinxcosy -lnsinx = C
( y- ~x 2 + y2 } dx- xdy = 0
slope = dy = 2x
xz dx sinxcosy = lnsinx + C By inspection, this equation is a
y2 = 2~yy' homogeneous DE, since the coefficient of
2. Since tangent, the slope of the unknown the dx and dy are of the same degree
y' =-y-= y_ curve is equal to the slope the line of ''"''' c; 'Inc
(degree 1).
2xy 2x
·~IIIXCOSY Ill('"""')
Da~J:.)if!erential Equations 405
404 1O"o I Solved Problems in Engineering Mathematics (2nd Edition) by Tiong & Rojas

Let: y = vx; dy = vdx + xdv Substitute (3) in (2):


1m (x+y)dy=(x-.y)dx
where: P(x) = .! ; Q (x) = ~
y' = c1 + y" By inspection, this equation is a
X
Substitute: c1 = y'- y" -7 Eq. 4 .f _
homogeneous DE, since the coefficient of 1.. -eJP(x)dx
vx dx- ,J;.z + v 2 2
x dx- x(vdx + xdv) = 0 the dx and dy are of the same degree

vx dx- ,J1 + v 2 xdx- vxdx- x 2 dv =0 Substitute (3) in ( 1): (degree 1). i.f. = ef~
y =C1x + y" -7 Eq. 5 i.f. = elnx
Let: y = vx; dy = vdx + xd
-~dx-xdv=O =x
I Substitute (4) in (5):
y = (y'- y")x + y" = y'x- y"x. + y"
Substitute:
Substitute:
i.f.

I JX
dx+J~=O (x - 1)y" - xy' + y = 0 x(vdx + xdv) + vx(vdx + xdv) = xdx - vxdx


2
,j1+V 2 y(x) = Jx (x)dx + C
"' vxdx + x2dv + v 2 xdx + vx2dv = xdx - vxdx
~ Let: v = tan 9 ; dv = sec e de
2
y' = y sec x xdx (2v + 0 - 1) + ~ dv ( 1 + v) = 0 xy=
x4
4 +c

' Jdx +
X
f sec e de2

J1+ tan 2 e

f~ + fsec e de = o
0
dy = ysecx
dx

Jdy = Jsecxdx
y
In y = In (sec x +tan x) + c
dx+ (1+v)dv =O
X V +2V
2

dx + _! J(2 + 2v)dv = 0
J x 2 v 2 + 2v -1
-1

lnx + ln(sece +tan e)= C In Y =C
secx + tanx 1
In[ x(sece + tane)] = C In x + - In (0 + 2v - 1) = c
y 2
x(sece +tan e)= C sec x +tan x = C , where: ec =c m = 'f._; where: m =slope (constant)
X
Multiply 2 on both sides:
=C (sec x +tan x)


where: ec = C y Differentiating:
Jln x + In (0 + 2v - 1) = k, where: k = 2c xdy -ydx =
0
Refer to the triangle, substitute values of xz
sec e & tan e to the solution: In[~ (0 + 2v -'1)] = k
· xy'(2y -1) = y(1- x) x
2
(0 + 2v - 1) =C, where C =ek xdy- ydx = 0

IP!!IP.II

·b>:'
= y(1- X)
X dy (2y - 1)
';ubstitute v = 'j_ : liiliilll
dx
X
2(2y -1)dy = y(1- x)dx
2Y - 1dx - 1 - x dx =0
2
x ('i._ + Y -1J = C
2
2 (0,0)
x x
y X
l + 2xy - x 2 = C
2
x,J1 + v +v = C
2 Jdy - r-; - rxx + Jdx =c
x
2
- 2xy - y 2 = C

xJ1+(~r +~=C Note: C is an arbirary constant which can


1", placed in either side of the equal sign. Y2 = 4ax
2y -In y -lnx + x = C 2
~xz +Y2 + Y= C 2y + x + C =in x + In y 4a= L
[D ~~ +(~)y=xz X

ID ln(xy) = 2y + x + C

Note: C is an arbirary constant which c.an


Differentiating:
x(2Y9Y)- y dx =
2
0
l~olo This equation is a linear DE: xz
y = C1x + C2e• -7 Eq. 1
be placed in either side of the equal sign.
y' = C1 + C2·e· -?>Eq.2 v (• r ) Jo(x)(i.f.) dx + C -7 Gen. Eq. 2xydy - I dx = 0

y" =C2 e• -7 Eq. 3 2xdy-ydx =0


• •
406 · l 00 1 Solved Problems in Engineering Mathematics (2"d Edition) by Tiong & Rojas Day 16- Differential Equations 407

elny ::: e-X+C ::: e-XeC


dQ =kQ
dt


Let: Q = amount of substance present y::: ce-x' where: c =ec

dQ =kQ sdg = Jkdt


dt
In Q = kt + C ~ Gen. Eq.
J~ = fkdt when t = 0; Q = Oo
Let: T = temperature of the body
dT = k(T- 30•)

II In Q = kt + C

when t= 0; Q = 100 mg
~ Gen. Eq.
In Oo= k(O) + C
C =In Oo
dt
_dt_= fkdt

II y-k
m = - - ; , where: m =slope (constant)
x-h
In 100 = k(O) + C when t = 1000 years; Q = 0.5 Oo
J T -30•
In (T - 30•) = kt + C ~ Gen. Eq.

ll Differentiating:
(x- h)dy- (y- k)dx =
c = 4.60517

=100; Q =96 mg
In 0.5Qo = k(1000) +In Oo
when t = 0; T = 1oo•

If (x -hf
0 when t

In 96 = k(100) + 4.60517
k :::- 0.000693
when t = 100 years; Q =? In( 100° - 30°) = k(O) + C
(x- h)dy- (y- k)dx = 0 c = 4.24849
k =- 0.0004082 In Q = -0.000693(100) +In Oo
(y- k)dx- (x- h)dy = 0


elnQ = e-0,0693+1nClg
when t = 200; Q = ? Q = Oo e·0.0693
when t =15 min; T =70•
In Q = -0.0004082(200) + 4.60517
Q = 0.933 Oo In (70·- 30•) = k(15) + 4.24849
Q = 92.16 mg

C(O,k) •
Let: P = number of population
%lost= Oo - 0 ·93300 x100%

%lost= 6.7%
Oo
when t = ?; T = so•
k =- 0.0373

r:
dP =kP
dt

= Jkdt • y 2 = 2x+C
In (so• - 30•) = - o.0373(t) + 4.24849
t = 33.. 59 min.

>( + (y- kf

Differentiating:
::: r"

2x + 2(y- k)y' = 0
In P = kt' + C

when t = 0; P = Po

In Po
~ Gen. Eq.

=k(O) + C
C =In Po
Differentiating:
2y dy = 2

(:~l =~
dx
• dV
dt
+~=32
10
10dV + V = 320
dt

10f~= fdt
320-V
X +yy' - ky' ::: 0 when t = 50 years; P = 2Po
X Note: The slope of the orthogonal
k= -+y - 10 In (320- V) =t + C ~ Gen. Eq.
y' In 2Po = k(50) + In Po trajectories is given by:
k = 0.0138629 whent = 0; V = 0 (at rest)
dy 1 (dx)
Differentiating: dx=-(dy) =-dye
y'(1)-xy"+ '=O when t = ? Q = 5Po - 10 In (320- 0) = 0 + C
dx c
(y')2 y c =- 57.6832
In 5Po = 0.0138629(!) +In Po
· y'- xy" + (y') 3 = 0 t = 116 years :;ubstituting:
dy when t = 1 sec; V = ?
xy" - (yf - y' =0
ml dx = -y - 10 In (320- V) = 1-57.6832
In (320- V) = 5.66832
Let: Q = amount of radium present ry =- Jdx
y 320 _ v = es.sss32
In y =- x + c V = 30.45 ft/sec

409 l ~0 1 Solved Problems in Engineering Mathematics (2"d Edition) by Tiong & Rojas Day IS-Differential Equations 409

Note: This is a linear DE when t = 10 years; P =?

Let: Q =amount of salt in the mixture a(i.f.) = JA(t)(i.f.)dt+C 7 Formula for In P = 0.03(10) + 8.51719
100 + (3 -· 2)t = 100 + t =volume of p::; 6,749.30
the general

• r:
mixture at any solution of a
timet · linear DE
dP = 0.03P
_.9__ :::: concentratiQn of salt i.f. = efP<tld! dt
100+t
It 2
=0.03 Jdt
d~ = rate of gain - where: P(t) = - - · A(t) = 6

I dt
rate of loss
.f
I ..
_
- e
100 + t '
fP(I)dt .
when
In P = 0.03t + C

t =0; P =10,000
Ill ~9=0-2(-a-)
dt 100+t
'f
1. • = e
J-2-
too+t
In 10000 = 0.03(0) + C
i.f. =e21n(tOo+tl
fada= - f1oo+t
,!1 dt c = 9.21034
+C i.f. =eln(100+tl'
In a=·· 2 In (100 + t) + C when t = 10 years; P =?
i.f. =(1 00 + t)2
In a= -In (100 + t)2 + C 7 Gen. Eq. .· .i.· In P = 0.03(10) + 9.21034
Substituting: ·~
when t =0; a =50 kg. I~)~ p = 13,498.60
2
a(100 + t) = J6(100 + t) dt + 2
c i
In 50 = - In ( 100+ 1)2 + C ~
c:: 13.12236 a(100 + t)2 = 6 (100 + t)3 + c
3
when t = 60 min; a = ? Q(100 + t)2 = 2(100 + t) 3 + c

In Q =~In (100 + 60)2 + 13.12236 when t = 0; Q = 50


Ina= 2.972 ·
50(100) 2 = 2(100)3 + c
a= 19.53 kg.


C=-1.5x106

when t = 30 min; a =?
Let: a = amount of salt in the mixture 2
a(100 + 30) = 2(100 + 30)3 - 1.5 X 1()6
100 + (3- 2)t = 100 + t =volume of Q =171.241bs. ·~
l!i

• r:
mixture at any
timet
~
_a_ =concentration of salt dP =0.03P
dt
r
100+t
~,,,.
=0.03 Jdt l
~~
i~

= rate of gain - rate of loss In P =0.03t + C I


da (-a-) when t = 0; P = 5,000
l''
?cl
-=3,2)-2
dt 100+t

dQ
dt + Q
( 2 ) 6
100 + t =
In 5000 = 0.03(0) + C
c = 8.51719 l t
)
£.
412 1001 So1:ved Problems in Engineering Mathematics (2nd Edition) by Tiong & Rojas

. ' "' ~·,


,,,~' '"'~*'•'<-A V ~"'""'"' ~.,.,.,.,: .. ,... , •• ,~ ... :;...,,.,,

, ... ~~"'"'''""'"·~ "~'"""'"'0~'~>.~~,...,~ 0+• ··~'-l~,.,.., ..... .,~·f· Topics

D
"'"' 'o"«,. ~ * N. :,0 _. 'A~'( ) <",- ;. {' ;. <, ... ~ ~ '!<"'" ·'do ,: ~ "' ,,. "<> '< •'<> t ~ »· '< ·>. <- ~ ~ >' * -:< ~ < ,; ~ ~ •., ..I< <· " < ·, '
ll Advanced Engineering Math
I I
' ~·' ·•' ~ "

' . , ... .+
~ '

V A 0V-" ~ ' 0 ~ .. < "'' ~ "'~


~ .. ~.-~·.,~ r~··-.. ....... ,."
Mon Complex Numbers
Forms of Complex Number
1: D Operation of Complex Numbers

I Tue Matrices
Operation of Matrices
~II ~ .< ,, ~ » ·~ ~ ., " w " "' "'"' ·' ~ ~
D
Theory Wed
(Addition, Subtraction, and
Multiplication)

D D
"' ~ ., :t ~ .,
Transpose Matrix
Cofactor of Entry of a Matrix
"'"~ ....,.·.,.~,~..,:;. ~ -~~ •. ~, .... ,,._.>r~"'·"·"'~"""'"'~ '""••<:• .... ,.. •• ,.. ,.,.,
Problems Thu Cofactor Matrix

D ~
~ * ">!' ~ " If "'" ~ « ~ o· '"" '.• » « " ~ .,. ~ ¥. « ?- • ~ ' ~ +. .p ~ ·• '' ''i ~· ~ » ~ ,_. "' 1' -.,;"< '! .,.. • • ~ ~ ~ « "' o ,, ,
Inverse Matrix
'•'<o-<.<->1- ....'%*'!""~ ;io.<-'"'><,~'1<*!,-:i«<>·-:;•vP<•<~<o$':~,'':<« *'0'000. %<!>
Determinants
~~····~~· ~~, ............. ~ •. ::. .... ~ / • .;_ *' > -:; ·~ Solutions Fri Properties of Determinants
~<)-yo':,._~41/"«~~-lt~t<~~,P.

::.. ~.;;: .. ~ ~:i.-«·<)·,,.>··~·,s.»-lr-i«

.,: « ,.. » * "'., • . ., ~· 1~ .&. '" ~ ... '<- -~ ~ ~· ~


"''<-.jj~•1<

"' * «·*·'-"· ,.
~ >$( '1(0

;<- i:-
-'>-,j•I'"I!-,Y~O«··~'··R,i:t>:

~ '~««·~~%%-0'"'"-''"'""<>:~-~
.. ~ ..'-<<>" 0"

i:
D D
Notes Sat
Laplace Transforms
Laplace Transforms of Functions

~-~/,.,,;.,..,,,.~ "'~ '"*"* ~ ;; ,1< >" < >!• ~ ~ 'r ·'II' ... ~ 0 /~ ;·, ~ " > .-> ~ "!" < •< "< X X 'v • > '".). ~ "' ..~
What is a Complex Number? Su~cessive integral powers of i or j
.. +,·~"* ~'·~·· .;..,..,.*~''"'~ ¥'\o)~"~".<' '.~\< '**"" ~~ ;, '<1·.'<-'J<'~.y ',.~··· ': ~

•· '»~·• ~ >: ~-.,·~ <"~·C .o .<~ ·~·" ~.,.. .. ,.. : ; . , , . ··~ / ;-·,~ o.. ~'""Y. ~ ~ ~o "'""' >
By definition, a complex number is any j2 =- 1
number expressible in the standard form f = jl j2 ) =- j
a + bi or a + jb. = l) =
j4 j2( 1
j5 = j3( j2 ) = j
The value "i" or "j" is the imaginary unit or = =-
j6 j4( j2 ) 1
number. It is equal to the J=-1 . = f)=-
f i5 ( j
=
j8 j4 ( t) = 1
... etc
Example:
Find the value of x of the equation x? + .1 = What are the Different Forms of
0. Complex Numbers?
Simplifying we get x ::: ± J=-1
' ·' "'" ,, " 1> .• • "'" , ' " ·~ ~ .; ,.. ' ' ., .... '" .. '<- ~ ' ..... -~· " ..... v •• i< ;. ~ '
Thus the solutions are i and ..,. i. a. Rectangular form

''~'~-~ ;-;~ ~
z=a+jb
·:'«·i<"'ff•.x- '·(.""'''·'
=1

~~-1
where:
•.
a= real part
b = imagina;y part
414 1001 Solved Problems in Engineering Mathematics (2"d Edition) by Tiong & Rojas Day 17 -AdvancedEngineering Math 415

b. Trigonometric form b. Multiplication of complex numbers Arc sinh z =In (z± .[zz + 1)
sino= ei" -e-iB
z = r cos e + j r sin e z=rcise Rule: Use the polar form j2 Arc cosh z = In ( z ± ~)
(r1 L91)(r2 L9 2 ) = r1r2 L(6 1+ 62 ) coso= eio + e-iu
c. Polar form 2 1 (1+Z)
Arc tanh z =-In --
2 1-z
.
z= rLO
c. Division of complex numbers Note: tan 0 = sine
[ Rule: Use the polar form
case
cote= case
What is a Matrix?

Matrix is a rectangular array of real


f
where:
sine
r = modulus or absolute value numbers arranged in m rows and n
r1Le1 = 5_ L(91 -62)
6 = argument or amplitude in r2L6 2 sec e = _1 columns. The term "matrix" was introduced
I degrees
r2
case
esc e = _1
by the English mathematician James
Joseph Sylvester (1814- 1897) in 1850.

l Imaginary axis
d. Complex number raised to exponent
"n"
II sine The size of a matrix is determined by the
number of rows and columns. The
expression "m x n" is the dimension or
h. Hyperbolic functions order of the matrix. If the matrix has only
bl '/ Rule: Use the polar form one column, it is called a column matrix
and if it has only one row, it is called a row
0
matrix. The following is a 3 x 3 matrix or
a
Real axis (rL6)" =r" L (n6) sin h0 = .ee -2--·-

square matrix (i.e. 3 rows and 3
0 columns).
e. n1h root of a complex number cos h6 = e +e-o
.r""' -/a 2 +b 2 6= tan· 1 ~
2 6 5 41
a Rule: Use the polar form A= 2 1 4
[ 3 -2
~nh0

I
Note tan he= -2
(rLe )* "" rt L e + k(36oo) coshO
d. Exponential form n
cot he = cosh~ The first non-zero entry in a row of a
9
sin he matrix is known as the leading entry or
z = r ei where: the leading element. In the matrix above,
k = 0, first root or principal root sec he= _1_
where:
k = 1, second root i coshe 6 is the leading entry. The diagonal from
the upper left to the lower right is called
e = argument in radians k = 2, third root
k = n -1, n1h root ' esc he= . _1_
sinhe
the principal diagonal or main diagonal
and all entries in the said diagonal are
called as diagonal entries.
f; Logarithm of a complex number i. Inverse trigonometric functions of
What are the Mathematical O~erations
complex numbers If all entries in a matrix above the main
of Complex Numbers?
Rule: Use the trigonometric form diagonal are zero, then it is said to be a
a. Addition and Subtraction of Arc sin z = - j In (iz ± ,)1- z2 ) lower triangular matrix, and if all the
complex numbers In z = r efl = In r + In efl entries below the main diagonal are zero,
the matrix is referred to as the upper
In z =In r+ j e Arccos z =- j In (z±~) diagonal matrix.
(a1 + jb1) + (a2 + jb2) = (a1 + a2)
+ j (b, + b2)
Arctanz= -i 1n(1+jz)
~21
g. Exponential & trigonometric 6 0
2 1- jz
(a1 + jb1) - (a2 + jbz) = (a1 - a2) functions of a complex number A= 2 1
+j(b1-bz) [
j. Inverse hyperbolic functi6ns of 3 -2
ei 0 = cos e + j sin e complex numbers
Lower Triangular Matrix
e-ie =coso- j sino
Day 17 -·Advanced Engineering Malh 417
416 100 i Solved Problems in Engineering Mathematics (2"d Edition) by Tiong & Rojas
What is the Division of Matrices?
6 5 4]- Example: Find A - B.
A=+f1 2]
4 -3]~1
•=[~ ~]
B= 0
[0 01 -24 .)~~
''',l,
Division of matrices is done by multiplying
the numerator by the inverse matrix of the
L2 1
'
[
A=~ denominator.
The determinant of A is:
Upper Triangular Matrix Solution: Let: A and B are matrices

6 0 0] ~=AB- 1 ~~ l= (1)(1)- (2)(2) =- 3

A-B=[~ -3] [-4 -3]


det A:=
[: [
C= 0 1 0 -1 + -7 5
8
1
'
where 8' is called the 1nverse matrix Thus, the cofactor of6 is - 3:
0 0 -2 8 -8 -2 of matrix B
1: 0 -6] What is a Cofactor Matrix?
l Diagonal Matrix

Two matrices are said to be equal if and


[-5 6
A-B= -12 4 What is a Transpose Matrix?
A cofactor matrix is formed by replacing
,11 If matrix A is reflected in its main diagonal, each element in' the given matrix by its
only if there orders are equal and their
so that ali rows become columns and all cofactor.
corresponding elements are equal. What is the Product of two matrices? columns become rows without changing
their relative order of entries in the rows Example: Find cofactor matrix of A
Example: Matrices B and C are equal. Supposed that A is a matrix of order m x p and columns, the result is a transpose

2]
and B is a matrix of order p x n, then the
product of A and B, denoted by AB, is the
matrix, AT.
A=[! ~J
B =[3 2J
2 -1
C=[3
2

What is the Sum of two matrices?


-1 n x n matrix for which the element in the ith
row and the jth column is the sum of the
products formed by multiplying each
element in the ith row of A by the
corresponding element in the jth column of-
Example: ·

A_ [-4-1 21 53] AT= [


--4 -1
~ ~
l Cofactor matrix of A= [

What is an lnvers~ Matrix?


~ ··~]

If A and B are two matrices of the same B.

I
order, the sum of A and B, denoted by A + Steps required to find for the Inverse
B, is the matrix for which each of its Example: Find DC.
What ts a Cofactor of an Enta:y of a
Matrix (say. A):
.~atrix?
elements is the sum of the corresponding
elements of A and B. 2 -3]
~- ~1 c =[-4-1 1 3] A cofactor of an entry of a matrix is the a. Form the cofactor matrix of matrix A
Example: Find A + B.
D=
[ 2 5 -~
~.
same as the cofactor of the sam~ entry in
the determinant of the matrix and thus, is
b. Fom1 the transpose matrix of the
cofactor matrix A
~ defined only for square matrices. c. Evaluate the determinant of matrix A
A =[-15 -2 6 ]
8 -3
B =[4 7 1
7 0 4
J D is a 3 x 2 matrix and C is a 2 x 3 matrix.
The product DC can be obtained because
S1gn conventions:
d. Divide each element in the (matrix
cofactorf ·
the number of columns of D (2 columns) is
equal to the number of rows of C (2 rows).
+ - +] Example: Find the inverse matrix of A.
Solution:

A + B = [ 5 + 4 -2 + 7 6+1]
The product DC will be a 3 x 3 matrix_

DC=
[: :] - + -
[+ - + 2 0
A=i2 1 4
-1+3 8+0 -:-3-4 1 xample: Find the co-factor of 6 in the
2(-4)+(-3)(-1) 2(1)+(-3)(2) 2(3)+(-3)1 4 2 6
lollowing matrix.
4(-4)+(-1)(-1) 4(1)+(-1)(2) 4(3)+(-1)(
A+B=[9 5 7] [

A=[~-~~
2 8 -7 1(-4)+5(-1) 1(1)+5(2) 1(3)+5(5 -2 4 0
-5 -4 -9] Cofactor matrix A= 1-12 6'
-4 6
What Is the Difference of two matrices?
[
DC= -15
-9
2
11 28
7 8 -3

If A and B are matrices having the same I ht~ .:quivalent matrix is: -2 -12 0
order, then the difference of A and B,
denoted by A - B, is defined as
(Cofactor matrix A)T =I 4 6 6
A- B =A+ (-B) 0 6 --3
418 .1001 Solved Problems in Engineering Mathematics (2"d Edition) by Tiong & Rojas f Day 17- Advanced Engineering Math 419

6. The value of the determinant is not


~- .•; -2
I5
-5 7

A=~;; -~
changed if a column is replaced by the
I 1 2 0 .• :.
det A= 0 -5 I=
I2
'
column plus a multiple of another .
.·'. . . 1·

det A = 1 4 I =6 column. Similarly for rows. -'~ ',' 3 11 -6


''"c
4 2 6 . f -2 -5 7 -2 -5
Determinant of a 2 x 2 matrix:
5 0 -5 5 0
-12 0 2 for 2"d row
1-2 3 11 -6 3 11 1 for 151 column
Thus , A- 1 = _.!_
6 4 6 6
det A= 1: :1
[: 0 6 -3
~,;
det A= 2(0)(-6) + (-5)(-5)(3) + 7(5)(11)-
[3(0)(7) + (11)(-5)(-2) + (-6)(5)(-5))
[ What is a Determinant?
detA =ad- be
·t detA=(5)~-5 51(-1)2•1
det A= 200 11 -3
;.'i

l Determinant is a square matrix (i.e. the


number of rows= the number of column).
Determinant of a 3 x 3 matrix: _,,_,_,
= 5 [(-5)(-3)- (11 )(5)) (-1)

~II
Every deierminant can be associated with b. Using Modification method: det A= 200
a real number. a b c
1,.·1

~
det A =I d e -4 3 -1
What are the Properties of What is a Laplace Transform?
Determinants?
g h
A=
1
2 -4
@0
-1 The Laplace transform of a function f(t)
1. If the rows of one determinant.are the a b c a b 5 1 4 denoted by l,. [ f(t) ) is defined as a function
same as the columns of another, and det A =1 d e f d e of a variable "s" by the integral:
in the same order, the two g h i g h Set the encircled numbers to ·zero by·
determinants are equal.
det A= (aei + bfg + cdh)- (gee+ hfa +idb)
Multiplying column 2 by 1 and add it ="'
F(s) = L [ f(t) 1 Jf(t) e-"1dt
2. If two columns (or rows) of a to column 1 0
determinant are interchanged, the Determinant of a 4 x 4 matrix:
Multiplying column 2 by 2 and add it
value of the resulting determinant is

I
to column 3 where: t > 0 and s is any number (real or
equal to the negative of the value of a. Using Pivotal Element method:
Multiplying column 2 by -2 and add it complex)
the given determinant. to column 4
Example: Find det A.
laplace transform of some elementary
3. If two columns (or rows) of a i The new matrix becomes,
2 -4 3 -1 functions:
determinant are identical, the value of
~1 CD~2 · · · 2
the determinant is zero.

4. 'If the elements of a column (or row) of


a determinant are multiplied by k, the
A=
I 3
-2
?
5
-4
1
-1
4
A=~~~ cb~: · ~· · ·
5 f 0
.
-5 1.
f(t)

1
F(s)

1
-
value of the determinant is multiplied 3 $ 11 -6 s
by k. detA = 1
2. t
2- (-4)(-1) 3- (-4)(-2) -1-(-4)(2)1
-2 -5 7 52
5. If the elements of the jth column of a 3- (2)(-1) -4- (2)(-2) -1- (2)(2) (1)::
2

determinant 0 are the sum aij + bij, then


1-2- (5)(-1)
1- (5)( -2) 4- (5)(2) ......... A=(1)j 5 0@ i(-1)2• 2 3. t"
n!
sn+t
0 is the sum of the determinants 0' .... ····•········•• ... 3 11 -6
and 0" in which all the columns of 0, 0' 4. e±at
and 0: are the same except the jth;
furthermore, the jth column of 0' is aii, i
r----·;--;~·;--;~~--;~: and Set the encircled number to zero by
s=t:a
j 2 for 2"d column multiplying column 1 by 1 and add it t" e±at n!
= 1, 2, 3, ... , n, and the jth column of 0" 5. -·- )"+1
is bij. I = 1, 2, 3, ... , n. Similarly for rows. to column 3 (s+ a
e±at,sin kt k
6.
(s +a) 2 + k2
4ZG l 001 Solved Problems in Engineering Mathematics (2nd Edition) by Tiong & Roj!!

e±at cos kt
s=t=a
~uote:
7
.I
I
2 2
(s+a) +k "Obvious is the most dangerous word in
a mathematics."
8. sin at
s2 +~ - Eric Temple Bell \~;,;
x: Topics
9. cos at
s
']
I''
r;,~-----~_,_, __
l~
s2 + a2
[II 10. sinh at
a
s2 -a2
}
·N

·;:%:, tvlon 1
Adva11ced Engineering Math
Complex Numbers
II ·t Forrr1 of Complex Numbers
[ 11. cosh at
s
~2 -a2
i:'

0Tue
Operation of Complex Numbers
Matrices
l 12. t sin at
2as
(s2 + a2 )2
0 Operation of Matrices
(Addition, Subtraction, and
sz- a2
,111 13. t cos at Theory \Ved , Multiplication)
(s2 + az )2
I
14. sin (wt + 8)
s sin8+kcos8
~~ !~
, Problems
0Thu
Transpose Matrix
Cofactor of Entry of a Matrix
Cofactor Matrix

~
s sin8-kcose
15. cos (wt + 8) --52 +k2

Prm:eed to the next page for your 17th


0
Solutions Fri
Inverse Matrix
Determinants
Properties of Determinants
test Detach and use the answer sheet
prCNlded at the last part of this book. Use
pencil number 2 in shading your answer.
0Notes
0Sat
j Laplace Transforms
Laplace Transforms of Functions

GOOD LUCK!

I h
'
7U: ECE Board April 1999
: ;1mplify the expression i1997 + i1999 , where i
11; an imaginary number.
C.
0,
2i -1
1+ i

714: CE Board May 1:994


'O.Critiia: The expression 3 + 4i is a complex
Did you know that. .. the theory of 1\ 0 number. Compute its absolute value.
determinants dates back to the ancient ll -i
Chinese who use barnboo rods in c 1+i A 4
representing the coefficient of unknown I> 1- i
B. 5
quantities, and gain acceptance ·when C. 6
introduced in Japan by the 1th century 711:: EE Board April1997 D. 7
greatest Japanese mathematician Seki : >unplify: i29 + i21 + i
Kowa (1642- 1708)! Even if German 715: EE Boal!'d October 1993
Mathematician Gottfred Wilhelm von /\ 3i Write the polar form of the vector 3 + j4.
Leigniz (1646 -1716) and Swiss ll 1- i
mathematician Gabriel Cramer (1704- I. 1+ i A 6L:53.1°
1752) gave their valuable contribution to IJ :Ji
B, 10L:53.1 °
the subjeGt, it was Alexandre-Theopl1ile
'/II J: u:~
C, 5L:53.1°
Vandermonde (1735 -- 1796) ir1e one Boat•d April 1'997
3217
regarded as ti">e formal founder of W1 111 · 111 tlw form a + bi the expression i D. 8L:53.1°
1·\.'i. I 1111
determinant theory.

i\ II I

1'- I I I
422 . 1oo·i Solved Problems in Engineering Mathematics (2nd Edition) by Tiong & Rojas Day 17 -Advanced Engineering Math 423

71ft: ME Board April1997


Evaluate the value of r-10 x ..J-7
1:t:t1 EE Board October 1997
Find the principal 5th root of (50( cos 150• +
jsin 150°) ].
.1:~·
;I
''I
A.

B.
221-91i
169
21+52i
731.: EE Board April 1:999
Evaluate cosh ( j~ ).
4
A: i A. 1.9 + j1.1 'i! 13
A. 0.707
B. -JiO B. 3.26- j2.1 :~ C.
-7 + 17i B. 1.41+j0.866
C. JjO C. 2.87 + j2.1 13 C. 0.5 + j0.707
Ill D. 2.25-j1.2 -90+220i D. j0.707
D. Jfi D.
[
II!
7:£31 ECE Board April1999 169
737.1 EE Board Aprill.999
[ 71.'7: EE Board April 199ft
2
Simplify (3- i) - 7(3- i) + 10
What is the quotient when 4 + 8i is divided
byi 3 ?
77.7: EE Board Aprill.99&
What is the simplified expression of the
Evaluate tanh ( j~ ).
3
l A.
B.
- (3 + i)
3+i
A. 8-4i
I
6+j2.5
complex number--- ?
A. 0.5+j1.732

l~
B. 8+4i 1111
3+j4
C. 3-i
C. -8 +4i B. j0.866
D. -(3-i)
D. -8-4i
C. j1.732
A. - 0.32 + j 0.66
D. 0.5 + j0.86S
718: EE Board April. 199ft
B. 1.12-j0.66
1200 7:t4: EE Board October 1997 C. 0.32 -.j0.66
If A= 40 ei , B = 20L- 40•, C = 26.46 + 7331 EE Board April1999
jO, solve for A + B + C. If A = -2 -3i, and B ::;: 3 + 4i, what is ~? D. -1.75+j1.03
Evaluate In (2 + j3).
77.8: EE Board Aprilt.997
A 27.7L45• A. 1.34 + j0.32
18-i Perform the operation: 4 (cos 60• + i sin
B. 2.54 + j0.866
B. 35.1 L45° A. 60°) divided by 2 (cos 30° + i sin 30°) in
25 C. 2.23 + j0.21
C. 30.8L45° rectangular coordinates.
-18-i D. 1.28 + j 0.98
D. 33.4L45° B.
25 A. square root of 3 - 2i
734: EE Board October 1997
-18+i B. square root of 3 - i
71.9: EE Board October 1997 Evaluate the terms of a Fourier series 2
C. C. square root of 3 + i
What is 4i cube times 2i square? 25 ei10nt + 2 e-i1oxt at t = 1.
D. square root of 3 + 2i
18+i
A. -8i D.
25 77.91 EE Board June 1:990 A. 2 +j
B. 8i B. 2
C. -8 F.1nd t h e quot'1ent of -
50 -
+ j35
- C. 4
7:£51 EE Board October 1997 8+ j5
D. -8f D. 2 + j2
. . 4+3i
Rat1ona11ze ..:i . A. 6.47 L3°
7:tO: EE Board April1997 2 7351 EE Board March 1998
B. 4.47 L3•
What is the simplified complex expression Given the following series:
A. 1 + 2i C. 7.47 L3Q•
of (4.33 + j2.5} square? . x3 xs
11 + 10i D. 2.47 L53° s1n x = x- + + ......
A. 12.5 + j21.65
B. --
5
31 51
B. 20 + j20 730: EE Board March 1998 x2 x4
5+2i Three vectors A, B and C are related as COS X= 1--+-+ ......
c. 15 + j20 C. --
5
2! 4!
D. 21.65 + j12.5
D. 2 + 2i follows: !2B = 2 at 180°, A+ C =- 5 + j15, x2 x3
ex = 1 + X + 2f + 3! + ......
7:t1: ECE Board November 1998 C = conjugate of B. Find A.
What relation can you draw from these
Find the value of (1 + i) 5 , where i is an 7:t&: EE Board October 1.997 series?
imaginary number. A. 5- j5
. l'fy (2 + 3i)(5- i) u -10+j10
S Imp I (3- 2i)2 .
A. 1- i c 10-j10 A. e,x = cosx + sinx
B. -4(1+4) D 1b+j15 B. eix =cosx+isinx
C. 1 +I
D. 4(1 + i)
C. e ix · ·
=I COS X+ Sin X
424 . 100 1'Solved Problems in Engineering Mathematics (2nd Edition) by Tiong & Rojas Day 17 - Advanc~cj F;ngineering Math 425

D. iex =icosx+isinx

'736: EE Board October 1997


740: CE Board November :1996
Compute the value of x by determinant.
4 -1 2 3 1 :'il
D. I~ ;I 746: EE Board April 199ft

If matrix
1
2
-1 2]
1 3 is multiplied by
[
One term of a Fourier series in cosir)e form
is 10 cos 40nt. Write it in exponential form.
X=lI 2 0 2 . 7441 EE Board October 1997 0 -1 1
10 3 0 3 1 2
14 2 4 5 If A= 1-2 -1 0 I, what is the [ ~ ] ;; eqoal to mo, then ~trix [ ~ ] ;,
r:
A. 5 6 i40nt
0 2 -1
B. 5 ei407tt + 5 e-i407tt A. -32
cofactor with the first row, second column
B. -28
C. 10 e-i 40lt! o
[: D. 10 6 i40rd
c.
D.
16
52
element? A.
B.
c. 0
3
1

l, 737: EE Board April1997 741: EE Board Aprll1997


Given the equations:
A.
-1~ -~1 D. -2
Evaluate the determinant:
,1111 1 2 3
x+y+z=2
3x "'- y- 2z = 4
B. ~-~ -~, 747: CE Board November 1997
Given the matrix equation, solve for x and
-2 -1 -2 5x - 2y + 3z = -7 · y.
3 1 4 Solve for y by determinants. C.
I~ -~1 [ ~ ~ J[;J=[ ~ J
A.
B.
4
2
A.
B.
1
-2 D. _,-20 -1
01
A. --4,6
c. 5 C. 3
B. -4,2
D. 0 D. 0
C. ~4,-2
745; EE Board October 1997
If a 3 x 3 matrix and its inverse are D. -4,-6
738: ECE Board November 199:1 '7421 EE Board Aprlll997
Solve the equations by Cramer's Rule: multiplied together, write the product.
Evaluate the determinant: 7481 EE Board A.pril1996
1 6 0
4 2 7
2x-y+3z=-3
3x + 3y- z 10 =
.,...
A.
1 0 0
0 1 0
If matrix [ ~ ~ }s multiplied by [~ }s
0 5 3 -x-y +z=-4

A. (2, 1, -1)
0"0 1
equal to zero, then matrix [; is J
A. '110 0 0 0
B. (2, -1 , -1 ) I
B. -101
c. (1, 2, -1)
B. 0 0 q
c. 101 A. 8
D. {-1,-2,1) 0 0 0 B. 1
D. -110
0 0 1 C. -4
743: EE Board October 1997. D. 0
739: EE Board April1997 C. 0 1 0
Evaluate the determinant: 2 3 1 '
0 0
2 14 3 1 If A= 1-1 2 41 ,what is the cofactor of 749: EE Board October 1997
450 100
5 -1 3 ' 0 5 7
-2 2 -3 the second row, third column element?
D. If A =
16 7 3 and B 0 1 what = oj ,
1 2 5 0 0 1
3 -4 -3 -4
is A times B equal to?
A. 120 531
A. 489
B. 389 4 0 0
c. 326
D. 452
B. -1~ ~I A. lo
0, 0 5
1 o

c ··I~ ~I
I
426 l 00 i Solved Problems in Engineering Mathematics (2nd Edition) by Tiong & Rojas t Day 17 -Advanced Engineering Math 427

~I
0 0 0 75::&: EE Board October :1997 B. w I [ (s square)+ (w square)] 1
w I (s + w) C. -e 1 cosht
B. io 7 o 3
Transpose the matrixl-2
1
-1
2
0
C.
D. s I (s + w)
4
1 0 0 ..!e21 cosht
0 2 -1 r 756: EE Board April :1997
D.
2
6 7 0 't"

c. is 9 4
A.
-1
I0 -1
2 0
-2
Find the laplace transform of [ 21(s +1) ]- [
41(s + 3) 1.
2 3 5
[
~'
2 1 3 A. [ 2 e( exp -t ) - 4 e( exp - 3! ) 1
4 5 0
~'
3 2 0 .£:, B. [ e( exp -2! ) + e( exp - 3! ) ]
D. 16 7 3
[, 2 5 B. 11
2
-1
0
2
-1
.~!
C.
D.
[ e( exp -2t ) - e( exp - 3t ) 1
[ 2 e( exp -t) ][ 1-2 e( exp- 3t) 1

l 750: EE Board April :1997 3 1 2


~1
~~~
757: EE Board Marc:h :1998

'~
Matnx. I-12 31 I + 2 Matnx. 1-11 21 I= C. 11
-2
2
-1
-1
0 f
·.:. iI
·';'~
Determine the inverse laplace transform of
l(s) =
s2 -
200
50s + 10625
3 2
4 = 2e-251 sin100t
A. Matrix 1-2 D. 1-1 -2 0 A. l(s)
2 2 2 2 -1 B. l(s) = 2t e- 251 sin100t
2 C. l(s) = 2e-251 cos100t
B. Matrix 1-1
1 1 753: Determine the inverse matrix of D. l(s) = 2t e-251 cos 1OOt

c. Matrix ~-~ 3
G~) 758: EE Board April :1997
The inverse laplace transform of s I [ (s
square) + (w square) 1is
D. Matrix I~ 5
5 A. (-~ -~ ) A. sin wt

75:1: CE Board Mzy :1996

Elements of matrix B = [
1 2
]
B.
(~ ~) f
:~
B.
C.
D.
w
(e exponent wt)
cos wt
0 -5

Elements of matrix C = [! ~J
.C.
(~ ~) 759: Find the inverse laplace transform of
2s- 18 .
- - as a funct1on of x.

c~ -1 J
5 s 2 +9
D.
Find the elements of the product of the two
matrices, matrix BC. A. 2 cos x - sin 3x
B. 2 cos 3x- 6 sin 3x
754: EE Board April :1997
C. 3 cos 2x- 2 sin 6x
[ 11 8] k divided by [ (s square)+ (k square)] is
A. D. 6 cos x- 3 sin 2x
-20 -5 the Inverse laplace transform of:
760: Determine the inverse laplace
B. [ -11
19 :] A.
B.
C.
cos kt
sin kt
( e exponent kt)
f
transormo, ~.·~.
t 1
4s'l 8s
C. [ -10 9] . D. 1.0
-19 6 1 I .
755: EE Board April :1996, A ·-·e s1nht
D. [-11
-20 _:] EE Board April :1997
The. laplace transform of cos wt is II
4
1 e·''I sinh!
7
A s I I (s squam) t· (w square) ]
:I
·~ Day 17 -Advanced EMi.nE!~il:lg Math 429

jl ~

J
.. ::\
• i2= -1
i i
i4 =1
·····················i······················
. 3
J i =-I
.
Since
i426::: -

j427:::-

16 is exactly divisible by 4
·1
i

Topics i16= 1
--
[Ill i2 = -1
I~ ,
Note:

D
j17::: i
Advanced Engineering Math i3 =- i
i4 = 1 i16=- 1
Complex Numbers
~II
,,,··l;
[I Mon
Form of Complex Numbers Substituting:
,,
D Operation of Complex Numbers
If the exponent of" i " is exactly divisible
by 4, then the simplified equivalent of the + i18 = i - (- i) + (- 1) = 2i - 1


j3217 - i427

l Tue Matrices
Operation of Matrices
IIi
',
,'
·'
t
imaginary number is equal to 1.

,1111 D D
Theory Wed
(Addition, Subtraction, and
Multiplication)
'
''
,;,_'
:t
' Since 1996 is exactly divisible by 4,
f1996 = 1,
Let r = absolute value of the complex no.
(a + bi)

D D
Transpose Matrix =i
Cofactor of Entry of a Matrix
i1997
r= -/a 2
+ b2
i1998 =- 1
Problems Thu Cofactor Matrix Substituting:
=- i
~
11999
Inverse Matrix r= j{3)2 +(4)2
Determinants Substituting: .1997 + i1999 = i + (- i) =0 r =5
Solutions Fri Properties of Determinants
I

D D
Notes Sat
Laplace Transforms
Laplace Transforms of Functions
a
Since 20 is exactly divisible tiy 4,
Ill
The polar form of the complex number, •a

711. A
ANSWER KEY

724. B 737.C 750. D c:J


RATING
43-50 Topnotcher
I i20

i21

Since 28 is exactly divisible by 4,


= 1,
=i
+ jb" is given by: z = r L6

where: r= -la 2
+ b 2 and e = tan-1 E.
a
712.A
713.C
725.A
726.C
738. B
739.C
751. A
752. B c:J 30-42 Passer i28 = 1, Substituting:
r
~
='1/32 + 4-
714. B
715.C
727. B
728.C
740. B
741. c
753. A
754. B
c:J 25-29 Conditional i29= i
r=5
716. B 729. A 742.C 755. A c:J 0-24 Failed substituting: i29 + i21 + i ::: i + i + i = 3i


717. D 730. B 743. B 756. A
718.C 731. A 744. D 757.A If FAILED, repeat the test e = tan-1 .i
719. B 732.C 745.A 758. D 3
720.A 733. D 746.C 759. B
Since 32'16 is exa(.ily divisible by 4,
e =53.1°
721. B 734.C 747.A 760.A
722.A 735. B 748. D i3216:::: 1, Thus, the complex number is 5 L53.1 o
723. c 736. B 749. D
I i3217 =i
a
·;nu;e 424 is exactly divisible by 4
..J-io =JfO .[:1
i424 == 1'
'..J-10 = .Jffii
i4?5 "' i
430 I 00 I Solved Problems in Engineering Mathematics (2"d Edition) by Tiong & Rojas Day 17 -Advanced Engineering Math 431

Substituting:
H=.fi~
"--1 =.fii • 2
( 4.33 + j2.5) = 4.33 2 + 2( 4.33)(j2.5)

+ l (2.5)
2
,,,
4+8i- 4i+8(-1)
i3 -
4 +8i
--.3-
I
-(-1)
= -8 + 4i
2
(3-2i) =9-12i+4i2
(3-2il =5-12i

(2+3i)(5-i) 13+13i
2
(3-2i)2 = 5-12i
../-10 R X = (JiO i) {.fi i) ( 4.33 + j2.5) = 18.749 + j21.65
+(-1)(6.25)
I
li liJI
../-10 X R .fi0i 2 ltr.
IIH = Rationalizing:
2 -2-3i
( 4.33 + j2.5) = 12.5 + j21.65
[
../-10 x.fl = ..fi0(-1) 3+4i


~Ill
(2+3i)(5-i) = 13+13i(5+12i)
[
1~11

../-10 X R = -.fiO (3-2i) 2 5-12i 5+12i


Rationalizing:

l,
Iii

a Note: (r.Le)" = r" .Lne


-2-3i = -2-3i(3-4i)
3 + 4i 3 + 4i 3 - 4i
(2 + 3i)(5- i) _ 6s + 156i + 65i + 156i2
(3-2i)2 - 25+60i-60i-14M

'~~
(3- i)
2
- 7 (3- i) + 10 = 9- 6i + i2 - 21 + 7i . 1 + i = 1.4142 .L45° -2- 3i -6 + 8i - 9i + 12i2 (2+3i)(5-i)- 65+156i+65i+156(-1)
+10 3 + 4i = 9 - 12i + 12i - 16i2 (3-2i) 2 - 25+60i-60i-144(-1)
2 (1 + i)5 = (1.4142 .L45° )5 -2-3i -6+8i-9i+12(-1) (2 + 3i)(5- i)- -91 + 221i
(3-i) -7(3-i)+10'=19-6i+(-1)-21+7i
(1 + i) 5 = (1.4142) 5 .L5(45°) 3+4i = 9-12i+12i.-16i(-") (3- 2i)2 - 169
2
(3-i) -7(3-i)+10=-3+i (1 + i) 5 = 5.656 .L225° -2-3i -18-i (2 + 3i)(5- i) -7 + 17i
2 (1 + i)5 = - 4- 4i --=--
(3-i) -7(3-i)+10=-(3-i) 3 +4i 25 (3-2i) 2 =-1-3-


(1 + i)5 =- 4(1 + i)

a
Note: Convert all the complex number in
rectang:.~lar
form •
50 (cos 150° + j sin 150°) = 50L150°
~
liD
4+3i
2-i
6 + j2.5 = 6.5.L22.619°
3+ j4 5.L53.13°
A= 40 ei 120'
A= 40 .L120°
A = - 20 + j 34.64

B = 20Z-40°
~50L15oo = (50)115 .L150°(1t5)
~50.L150° =2.1867.L30°
~5QL150° = 1.893 + j1.093
I Rationalizing:

4+3i_4+3i(2+i)
~----

2-i
--
2-i 2+i
4 + 3i 8 + 4i + 6i + 3i 2
6 + i2 ·5 = 1.3.L- 30.5°
3+ j4
6 +i 2 ·5 =1.12-"0.66
3+ j4 J

B = 15.32- j12.855 ~50.L150° = 1.9 + j1.1


2=1= 4+2i-2i--,i 2 liD
c = 26.46 4 + 3i 8 + 4i + 6i + 3( -1)

A + B + C =- 20 + j 34.64 + 15.32
tm 2=1 = 4 + 2i- 2i-(-1)
4(cos60° + i sin60°) = 4L60°
2(cos30° + isin30°) 2L30°
4 + 3i 5 + 10i 4(cos60° + i sin60°) = .L o
-i.....
- j12.855 + 26.46 4 + 8i 4 + 8i · since i3 = - i --=-- 2 30
~i3--
-~
2-i 5 2(cos30° + isin30°)
= 21.78 + j 21.785 4+3i 1 2" 4(cos60°+i sin60°) = 1.732 +i
- -=+ I
= 30.8L45o Rationalizing: Multiply the denominator 2 -i
2(cos30° + isin30°)


with its conjugate.
liD 4(cos60° + i sin60°) =
2(cos30° + isin30°)
J3 + i
4+3 8i = 4+ 8i(i) (2 + 3i)(5 ~~) = 10- 2i + 15i- 3i2
Note: i2 = -1 i -i i
i3 =- i 4 +8i 4i+8i2 (2 + 3i)(5- i) = 13 + 13i
-.3-=--.2-
1 -1
(4i 3)(2i 2)·= (4)(- i)(2)(-1)
( 4i 3 )(2i 2 ) = 8i
43Z l 00 1Solved Problems in Engineering Mathematics (Z"d Edition) by Tiong & Rojas 7_Advanced Eng~ering_!\iath__.43;'!,_
---------------·-::::D~a;:.!_..:1..:.7_

• 50+ j35
8 + j5
61.03L35°
= 9:43L32°
eJ•., + e- f.f• = 2cos (1t x-;-

e1~ +e·i! =1.4142


Therefore,

cos h( 14
.• ) =~-
1.4142
4
180") i•'
1./


Let: x = 2 el10.t + 2 e·i10.t
X =1.28 + i0.98

10cos(40nt) =10 [ ~-~e


j40.t·
_J'l
-j40l<l

50+ !35 =6.47 L3" 2 Substitute: t = 1 10cos(40nt) = 5 ei~Om + 5 6 -i40.t


8+]5 x = 2 ei10n(1) + 2 e-i1D•!11
IIIII! cosh(H) =0.707
x =2 (ei1o~ + e-i10•)
[
Hklllli
l:l o Ill 121
[ Iill ~ == 2L180" tanhx = e• -e·•
e• +e·• ;
Note:. eJ9 + e-fo =2 cos 0 7 Euler's .2 3 1 2
B
l. ~=-2
B
tanh(.7t)
J- =
3
e~- e -~•
=2 (ei10x .+
equation D=l-2 -1 -2
3 1 4
-2 -1
3 1
A=-28
e'~ +e-~. X e·i10•)
D = [(1)(-1)(4) + (2)(··2)(3) + (3)(-2)(1)
'llllllj
:/,~ ( 180")] - (3)(-1)(3)- (1)(-2)(1)
Let: B = a + jb; C = a - jb Note: efo + e·j(l =2 cos a } Euler'~ x =~ [ 2cosl107t x -n-
efo _ e·iO = j 2 sin 0 equat1ons - (4)(-2)(2)]
x""4


A+C = -5+ j15 D ::: - 4 - 12- 6 + 9 + 2 + 16
·-28 + c =-5+ j15 ., +e-r,., = 2cos [1t x-7t-
180°] 0:::5


er,
.,-2 (a + jb) + (a - jb) =-5 + j15 3
Let x = ix
-2a-2jb+a- jb = -5'+ j15 eii +e-ig = 2cos60°
-a- jb =-5 + j15 2 3 4 5
e~+~-~=1 e1
,
=•1+. I X(ix+ ) (ix )
--+-+--+--
(ix ) (ix ) 1 6 0 6
By inspection: e~ + e-~ =j2sin60" 2!
(ix)6
3! 4! 5! 0:.1 4 2 7 4 2

e~ +e-~ =j1.732
+6! ........ .. 0 5 3 0 5
a=5
-3b = 15 ,. . - x2 .:...ix3 x4 ix5 D = [(1)(2)(3) + (6)(7){0) + (0)(4)(5)
Therefore, e =1+1X+-+-+-+-
b=-5 2! 3! 4! 5!
.1t)_j1.732 - (0)(2)(0) -- (5)(7)(1)
a=5 t an h( J- - - - -')1(6
3 1 +6! .......... . - (3)(4){6)]
................ ., .. .., ........................................
Therefore, ,, · x 2 X4 X 6 .i 0=6+0+0-0-35·-72
A =-2(5- j5) tanh(ii) = j1.732 e =:1--~+---·+ ... .
21
~~ 1~)(····· . . ··~3 ···.···~~·················),! D =-101

•D=le
A=-10+j10
111
•coshx =e• +e·•
2
jg
cosn(H) =e • + e·r.
'I!
Convert (2 +j3) to polar form, then to ·
exponential form:
2 + j3 =3.6L56.3"
2 + j3 = 3.6 ejS6.J•x,:.
COS X
v
.. + 11 x-3!+5!+ ..... !
l

v
i sin x
)

I
I
1
3
1

-2
-4
: -~ ~- ·
2
-3
-3
-4
!li
2 + j3 = 3.6ei0· 98 I hus, eix = cos x + i sin x

Note: ejll + e·jll =2cos0 7 Euler's


equation let: x = In (2 + j3) rm , Using Pivotal element method: (Use the
second row, tirst column element as the

~~~· 2~osn
pivot number).
1 e 1' -= 7 Euler's equation
x =In (3.6 eious)
x -ln3.6 + lne1ooo
434. 1001'-Solved Problems in Engineering Mathematics (2nd Edition) by Tiong & Rojas ~ -~ _____ . ____ J:>ay 17 -Advanced Engineering Math 435

Dy = [12 ... 20-.21 • 20- 14 -18] Dy = [(2)(10)(1) + (-3)(-1)(-1) + (3)(3)(-4)


-2 -1
ll
14 _ 2i5) 3 _ 2(-1) 1- 2(3) I
+' -4 31
D = (1) -2-1(5) 2-1(-1) -3-1(3) (-1)2
X=l
0 0 oG) Dy = • 81 - (-1 )(10)(3)- (-4)(-1 )(2)
1--4- 3(5) :..3- 3(-1) -4- 3(3)
8 3 -2 1 - (1 )(3)(-3)]
4 5 -5 4 5 4 2 -6 5 I .;, Y= Dy Dy =[20 - 3 - 36 + 30 - 8 + 9] =12
D = (-1) l-7 3 ..,.6-7 3 -2 -1 -4 ·..~ D
X=(1)1 8 3 -2 ( -1)2+4 ,Sf
y=-
-81 y = Dy
-19 0 -13 -19 0 'jg D
Ill! -27
4 2 -6 I~
12
y=3 y=-


[
~lilt D = (-1) [(4)(3}(-13) + (5)(-6)(-19) 6
l.li
-2 -2

I
+ (-5)(-7)(0)- (-19)(3)(-5) -1 -4 -1 y=2
[
1'111
X= I 8 3 -2 8 3

t - (0)(-6)(4)- (-13)(-7)(5)]
4 2 ..,.6 4 2 ., 2x--y+3;t=-3
3x+3y-z= 10
Dz=l
2
3
-1
3
-3
10
2
3
-1
3

'~~
-x-y+z:=-4
D= (-1) [-156 + 570 + 0-285 + 0-455] -1 -1 -4 I -1 -1
D = (-1)(-326) X= [(-2)(3)(-6) + (-1 )(-2)(4)
+ (-4)(8)(2)- (4)(3)(-4) I! I oJ ~
-1
3
3
-1
I 32. -1
3 Dz = [(2)(3)(-4) + (-1)(10)(-1) + (-3)(3)(-1)
D = 326

a - (2)(-2)(-2)- (-6)(8)(-1)] ~1 -1 1 ·I -1 .- 1 - (-1)(3)(-3)- (-1)(10)(2)

.
- (-4)(3)(-1)]
X = [36 + 8 - 64 + 48 - 8 - 48]
D = ((2)(3)(1) + (-1)(-1)H) + (3)(3)(-1)
X=- 28
Dz = [- 24 + 10 + 9-9 + 20 -· 12]
4 -1 2 3 - (-1 )(3)(3)- (·1 )(-1 )(2)- (1 )(3)(-1 )]
2 Dz= -6
2 0
X=l
Ill D =[6 - 1 - 9 + 9- 2 + 3) =6 Dz
10 3 0 ' Z=o

l
14 2 4 5 x+y+z=2 -3 -1 3 -3 -1 -6
3x-y -2z =4
Dx=l 10 3 ~1 10 3 Z=-
Multiply column 4 by -2 and add it to 5x - 2y + 3z = - 7 6
column 1:
3(- 2) + 4 =- 2 By Cramer's rule; y = ci (.

-4 -1 -4 -1 z = -1

1(-2)+2=0 1 1 1 1 i Dx = ((-3)(3)(1) + (·1)(-1)(-4) + (3)(10)(·1) Therefore, the answer is ( 1, 2 - 1).

3~
1(- 2) + 10 = 8 D =I 3 -1 -2 3 -1 :f - (-4)(3)(3)- (-1)(-1)(-3)
5(- 2) + 14 = 4 5 -2 3 5 -2 'I~
l
,r
"(1)(10)(•1)] • 2
ox= (- 9- 4 .. 30 + 36 + 3 + 1OJ= 6 A=l-12 4
Multiply column 4 by -2 and add it to D = [(1)(-1)(3) + (1)(-2)(5) + (1)(3)(-2) 0 5 7
column 3: - (5)(-1)(1)- (-2)(-2)(1)- (3)(3)(1)] Ox
3(- 2) + 2 =- 4 x=o Let: X.'= cofactor matrix of A
D = [- 3-10-6 +5-4- 9]
1(-2)+2=0 6
X=l~ ~~(-1)2+ 3
D = -27 x=s
1(-2)+0=-2
1 2 1 1 2 X:: 1
5(- 2) + 4 =- 6

The new matrix becomes,


Dy =I 3
5
4
'--7
-2
3
3
5
4
-7
X=-\~ ~I
2-3 312-3

Dy = [(1 )(4)(3) + (2)(-2)(5) + (1 )(3)(-7)


Dy=:
-1
3 10
-4
~1

-1
3 10
-4
3 CD 2
- (5)(4)(1)- (-7)(-2)(1 )- (3)(3)(2)] Ill A =1-2
0
-1
2
0
-1
436 1001 Solved Problems in Engineering Mathematics (2"d Edition) by Tiong & Rojas Day 17-Advanced Engineering Math 437

Let: x = cofactor matrix of A

X= 1
-2Q -1o I
(-1)1+2
6- 3y + 2y = 0
y=6
x=2-6
X=- 4 ''[

!!f

Note: T~e transpose of a given matrix is
formed by interchanging the rows and
columns.

Note: e-•t sinkt =

20_0~-
(s+a) 2 +k 2
k -7 Formula

X=-1-20 0
-1
I El 'I'
"!~~-
3 1 2
Th ....,.__
us, s2 -50s+ 10625

• ~ -2(
ll'lil jl
~-
[ _; ][;] = 0 A =j-2 -1 0 100 )
[
~~11 .li 0 2 -1 - (s+25)2 +(100)2

[
1~1

Let: A = 3 x 3 matrix By inspection, since the resulting product 3 - 2 0 = 2e-251 sin100t ·


A" 1 = inverse of matrix A


is zero, x = y = 0.


'!ill
A transpose = j1 -1 2

lll A (A" 1
) =A ( ~)
A
2 0 -1
s
~~~~ Ill
1
A (A ) = 1(unity or identity matrix) - --
2 2
= cos wt 7 Formula
Since matrix B is a unity matrix, then A x S +W


Note: A unity matrix is a matrix whose B is equal to matrix A.
Solving the determinant of the given
elements in the main diagonal are
4 5 0 1 0 0 4 5 0 matrix:
all number 1.
1 0 0 6 7 31X 0 1 0 6 7 3
O=l; ~I 2s-18 2s 18
Unity matrix = 0 I 1 0 2 5 0 0 1 2 5
s2 + 9 = S2 + 9 - s2 + 9


0=9-10
1 o o
0 =-1 2s-18 _ 2 [ - s J- 6 [-3-J
52 + 9 - s2 + 9 s2 + 32
Note: For a 2 x 2 matrix, say,
El 1-12 ; I +2
1
-1
1
21=12
1 -1
11
3 I I. . .. Note: cos at = - 2- -
s +a 2
}
s

l
. A = ac bd , 1ts 1nverse 1s g1ven by:
rnatnx
. a Formulas!
~-~ ~I sm at=-2- -

[ 1 -1 2] [ z
s +a 2
2 1 3 X
y =0
+
·1-.l
·,
A-1 =_!ld
0 -c
-b
a
I 2s -18 .
0 -1 1 Thus, - 2 - = 2 cos 3x-6sm 3x
lit s +9


2-2 1+41 ·~

By inspection, since the resulting product


is zero then, x = y = z = 0.
I
= -1 +2 3+2
rhus,
A_,= _1_19 -5


(-1) -2 ' 1
5j By completing the square of the
=I 0 51 A-1= 1-92 5 I denominator:
-1 4s2 - 8s = 4 ( s 2 - 2s)

[ ~ ; J[;]=[ ~ J
1(x) + 1(y) = 2
x= 2-y -7 Eq. 1

[~ -52] [! ~J
ml
1m
. kt
s1n = - 2-k -2

cos wt = - 2- -2
s +k

-7 Formula
s
-7 Formula
_4s2 - 8s = 4 ( s 2

4s 2 -8s=4[(s-1)

1
--=4 (s-1) -1
1[
- 2s + 1) - 4
2

12
-1]
]
1(3) + 2(4} 1(6) + 2(1) J S +W

3(x) + 2(y) =0 -7 Eq.2 = [ 0(3) + (-5)(4) 0(13) + (-5)(1 )_ Note:


1m Note: e at =-1-
s-1 a
7 Formula
e"1 sinh kt = k2
(s-a) -k 2
+Formula

. =[ _;~
Substitute Eq. 1 in Eq 2:
··58 J 1 2 4 1
"('h us ------ 1 , h\
=- eI s1n
3(2 - y) + 2y = 0 I hw;, ;> e 1\ n "''
s I I <; I :~ ' 4s~ 8s 4
~ 1001 Solved Problems in Eng!n~ring MatheJU&tiCS (2"4 Edition)by Tiong & Rojas

.. ·'- .; ,.·,.. _,. ~ ("


~ 'f '!o· ~ z. ,, a ..- (t 'i< ' ~ "' .s- ~ .;_ ? '< " ~ ( <· ~- ~ "' ~ ~ "' ,. •· 6 ¥ -.. ·,· " « ·4; ~ ~ ., " ~

·~"' ......... .;, ... <• ... " .v .; ""' 0 ~- ,v < ' ~ .,_"' • ,:.,. < ~ ~ ~ A V 'o'• • 0 o ' 'X '>: " # V .,.. \o !• A' ~ < "' ~ v, ' ,
}~.
,:;g
t; i< 11 "' " • -~ ' *"" ...... ~ .. < ~ ~ • .;,. ,, "~ > < ." " ~ •• "lli f ~ ... " ~ "' ., ·~ •••• ,. ' ~ " ~ ., ,, « ~ ~ " .. >· ~ ,. v •. > + ,. p" •\
Topics
ltl~li~l ';~
0
f>"i'lt•""'"' I><"~" 1·{·~~..,~~··,,_,.~ «.<.><~<-~~'-\">·t»/•'>~·1'Wox.~~"< <11>_,..,,.,,.
··~~- ,<~-~~ ..<~t-~.}·~-~
.~~. Physics
111111111!11 ·r ~· ~ ,. { '"' " .• • ~ .< ' ~ ' '11• &,.'~ ,. /A.,,.
;l Vector and Scalar Quantities
r~' Mon
i,t~:
l",'<)',•~·.!tfc~;....,.,t:Yt"-<1} {lj· Y.·."/l·'i! ;!-·l!<+.~~.'>e,o>·~~ ~":<_'*:1>.~ H<t "'<~''<·~.{'<···~ < --~~>!<»
·~ \o,..., "',,.'<II"'-~
Classifications of Vectors
~-~~I

L~,
* ~ W\.4

;,, ~".-~~".'!)< ".r,


1' _..,. 'I "'/ {·, '<• :• :'>

k >l" f: ~· '<
<':": ''

f<'"' 'f J" »"<" ..,_. f- •' #


~· •'i':1' ;f4 '<' "- ~ 'll \!

II {' ·~ <>
'$ ~~ j( J ·•. f,) 4

"·t "• (· !"-"" 1-" '' ·~: 9' ·~ -#- 4 ~


~~ X •: ~ i' 1· .~ '>. ~ ~ ~ 't· ~

.,_ f..\o; ":1.


»

!"".'*'·~ '-* ,. _,.,. y·~ ·."-It"- .if" ,<•


.1' .§. '<!- 4, "<' ·~ ~,S. ~ ~ .<~ ·\' i<c <" 4.' ~ ,"> •I·

< ;o ..o,· • .~· i-> '-<" '-o l' .~ ..... 'J it--~ '•·
,f

"'> >'

·.''
0Tue
Speed and Velocity
Distance and Displacem~nt

'~
~·'f «,*''to>;: "I',. *: ~-"> '=" 0<. it- !t·•·cf.;. -~ » ~ *' "'· ~.,."" :->=:->;~ ~·1· :.O..<o."' .,_._,.·.-:~·"' ~ ~ ~ :"~; ··~.,'< Acceleration
,'?,.If.; !I' Pit

,,,.-i-·-';~..;,

\<- ~·~ Jt.., ,."~.;, ~


iJ.'«'li,

f;"«:·<i-1' t-·~ >~-"'-""'":o><'l·}

-..:-:;·:> _;; •~ "l' » ~ ., ~...,.<of •;".0


*-i'-*#-~·,.·;,.1<! +"~J·,.,···"t-.,41.~

-t•f. 'II"'<! -i< >{,''<to";..;..:,..,


« oY ~1i·~;<u··~~'"'"'

4. :j.,':.<~ "">;< <>


,J
..o »- <·,

1o-<r..i1: 1<
•O:"f'..j. ,..,•.._

~·<t !.1·""" ""':¥.~ ·"'·.~ ~ *'!'.~


"'"' • ,. .•

~,.'"'!'(i;.f'j,·:t.1t'\""'·"'.!>.~!!P.;S·.f..s.'oJt:O '~

.<-
'» yx,"..,

r(i.• <'.'~,j,. •. .<- '"".~ • ~·


j.
•,

'#
' ~'1
Theory
0Wed
Laws of Motion
Forces
• ;,. <),

·~<I<
l<'.-'. fi- "'*"} 'f :<l1r1J·~

<I'*.*"'' >10 -t~ ~,~:.,. ~


1;- <1: v

"?;" =·i <'4< .o:t


~ <\:.~<t ·Si'

<£ ,..,..-.,:..
~ !·• ~ ~

;.!
«·.,;," .<.

~\·~ 'P- ,..,.


~·· ·:J>·~ 'ft. t>.~~-lt- ,o::"..,_l.!;,l•.,. o1 ~:<~'< l!"4.l!·~· <fi -"'.+ •.'.{~~

'!I :i'\~.,;.·.,,.,.11!-"*"'!" <l-',!<"'<' .~".?I*"''~:.<,...~ .,..t,-<i.<J'<>t


!( /i'-$'

<:t-'<>".f!-A',.. j.>
"":"''""'<I' ,.··/.of""-"' {.•""

"·"'·~. :! , ,1--·>. ;;~· .... io 0


Problems
0Thu
Law of Universal Gravitation
Work, Energy and Power
Law of Conservation of Energy
1'"'" .t ""

0 0
« ~ .'-, ~ 'v" '· ~ ~ '( ~< ~ ><. 't ... ~! =t/>f ;<t /• ·~,;. 'i " "!' .. :/<'.,/,~ ,.. ," ~ \P ., ,.. ~ ~ ~ 0~ :" '\< :< ~ ·~ 1 'f' ~ I'

Momentum and Impulse


1f

~· • ·), .< ~ i • "' ~ ,. .. "'.., ~"' ~, '· .;; .,.. ~ .• ,~ ~~· t '\"' ., •, " J .,.; il< ~ , ·_,.,. * 4, .... ~ ~- & r"~~ "")<,.<,.,..l.,,
Law of Conservation of Momentum
·y:~·l,!--..1, i ¥.Jib-~~· i '..,.';:..,.·...," ~· f.",~'t i\'.,.·,,., "ff< ~ ~ .'• <1 ~ .. ,) -~,· « .. ,...,.__ :,_,..,. ~·ol·<'t 1l-..;. ,:,~-"' ~ v. i+ ;-·~1 ~ -~* 1<-:<1!., ... ~ ' " .,. Solutions Fri Gas Laws
~.~f..,~,.,·;.. =~~«>~·.~t;.'>+·.~

•t·;~-r.~~
~.>'~.,. i>"<"·i·~t·r-~<".'<~o.r':(,.,

»<'·ic~f<.·,., -;,, ... ·~.,~~~·..~.,~.,"'-.'·'~"';.<>.,


" *"* 1>, " .., ~- ;. "'f" .f.:~ ,. "'·"" ~ .• ,,, .., " ~ ,.. -<... '{· .. ~, ... i

.!<·l~<r<• ·'~li<'"~···
'<-'""

0 [g Properties of Fluids
Archimedes Principle
Nctes Sa!
,_.;.~.'k/•,, ·~..) < ~I '.~, V~ > ~ ''~.4·~' Y<'""• .. .'•l~><>,•'"~ •>~1 "i'>"'.''-l<f,:<- •

I
..,.,_. :• o' "V <1-f< e ''•"

~ v • ,. * .<· ~-.,_,: -:> 'r 0 ;o ~ ,o, < • ., ~.>.<...d.,~-)! 41 ,, "'''i '\> -~ .......

~-"··~'>t.ff<~jo."''·~~-~""''"' r--. ··~f<•yo ........ ~ ~ ... -..., .• .,.~ ......i> ... <«"<«~."" .. ,~·-~·""<!<·"'1"""·~f"•'1
What are Vector and Scalar Quantities? Yl/hat are the_Ciassifications 91
·~· Vectors?
~ *' •( '" ;..,. ,, Y,
' ·"' ,, ~ "·" ' ~ .... ~ " ... \-. .., " ' ~ ~- ,., . ' " "~ ,;, ..:· ~ "' .., 1 ~ ,.:,. ....... '~ .; " " .... ~ • • ~ .. ·_, " ... , ;-."' -~· .. }! ... .;. ,. ••~
Vector quanities are quantities whose
"""~·.~"'* "'''"" '.i r ~F·.,.-" l ~ ,-~·" :>-:<..< ·" o·' ,..,.. e ... ._.~'->~-"- • «'< !'>) -..'",.; '1.·.<·~"'."" ,·..... ..,..~-"} *·*·~··;.~~ < f" measurement is specified by magnitude 1. Free vector- is one whose action is
;md direction. The following are e~amples not confined to or
·!· .... ~·!•"!-.:'"'"'"""""""'\'""'''".'"'~'·<1",<"~"<". '""~"-->'·,•J-0.'. .•~ ,,.,;;.•,...;,,:0,~*':'-~.'-·'~~., ..... ,,»;~ . .<"C'>"'i"'~ .....
of vector quantities: Weight, momentum, associated with a
'"""'~,.. ....... -'"'*f+·~·,o:> ..... ·.. , \ .............. ;t-·....... .,_:.:<1. ...... ·.... ~--.:;. • .., .. "" """ ~:<-"'-<-·i ' · ' .. ~' ~ '"·~ ;.;. ~ .......... ~*+of*:~·"' torque, velocity, displacement, unique line in space.
<~cceleration, electric field intensity, etc. 2. Sliding vector- is one for which a
~...,. ~~~~''"' '~.:....,.'".~ ''-:""~;':.;i"l~ .....\.ll-.<·:·~·~'"·""''""'"""*"'"'
unique 1ine in space
~ . . '* *'' <· ~ "¥"":'~. o>r» ,; ,_,,t.,\"'-•"·'-'"·\~.*l$-<t<:\·~~·~le11'>~><,.f<,"YI'!'*•.-t-io<1!->:).o~\'";t
Scalar quantities are those quantities
''>' '-"lf.'(-1<¥'<:
must be maintained
,;,. ,.,. '~" ~ >t~· "' .... "O«!" " .f!! ... ,.. ;>. 'i' 1 .,.. ., • ., ".;· ~ *·,. .~.., " ""+'<'!'l'lf,~1><l1.1i>01A-<:·~·... ,..~~~-·"*'':'
which have only magnitudes. The following along which the
.~,,~examples of scalar quantities: Speed: quantity acts.
'« ..... -~ .,...,,..,. "'~ ~·~f" ..,'to,..,." ~:,. ... A-~ •> ~ ~ « * l"'> 0
' * "' ... '- , .... +. ; ~ "'" ~ $< .j ·"' ~ ' " ~ ~ ' ! ."' ~ if- ~ , '• 11\ass, volume, energy, length,
..,. ..... +·f.~··~ .;ci><.,~ ~ ~ ~<·•,"" «,<>'.. ) ~ ., '".,; (' ~"'."' .. <(!> .:,..~ ..... *'·t- ., *''" <; $ ~ <!"« ~ '. ~~ ~'t ,, ...,:~ .. ,,. .,.. >:"1'>,·~ ~'
I• "nperature, pressure, voltage, time etc. 3. Fixed vector- is one for which a
1he term "scalar'' comes from Latin "scala" unique point of
~~-~ .. ~·-;:,{.~1""' ·~·>'' ;:"~·.).. <.><"•' ~ ,.,., ~ ~·' :<~
f '· "'·' ~ ·~· ., ·,.""), ~~l· ·> ..., ' ... ~ ... .., ""'<_,
wl11ch means "a ladder". application is specified
·" '~ ·> ', < ~ 1",. ' ' ~
·' ,. '. ~, '>" and therefore the
v.~ctor 1s the line whose length indicates vector occupies a
' ~ ., ,;. "'*" «
''' ·~•;aft' fhP maqnitude of the vector
, ,.
particular position in
'l'l<~rJtrty and whose direction indicates the space.
"" ''"'"" nl ltw quant1ty The term "vector"
'"'"'"·""'II l.ol111 "vt·h•·r"" wl11chrw~ans
'It' I ;lily''
44Z. 1001 Solved Problems in Engineering Mathematics (2"0 Edition) by Tiong & Rojas Dayl8- Physics 443

What is the difference between a Speed Newton's Second Law: Slug is the mass to which a force of one
and Velocity? (The law of acceleration) pound will give an acceleration of one foot
per second per second.
"Whenever a net (resultant) force acts on

~
Speed is defined as the distance per unit
time. Speed is a scalar quantity. a body, it produces an acceleration in the Gram force is one-thousandth the pull of
direction of the resultant force that is the earth upon a standard kilogram at a
place where g has a value of 980.665
t'lllllb~
Velocity is defined as the displacement
per unit tim~. Velocity is a vector quantity.
directly proportional to the resultant force
and inversely proportional to the mass of cm/s 2 .......
~

the body." Centrifugal


lt1m1~~~~ What is the difference between a What is a Frictional Force? Force
t"" Distance and Displacement? F=ma
a-lii~ It Frictional force a force acting on the body
whenever it moves while in contact with
l.,, Distance is a length from one point to
another usually measured in a straight
line. It is a scalar quantity.
Newton's Third Law:
(The law of reaction)
another body. This force always opposes
the direction of the motion. The frictional

'~~
force is proportional to the normal force Let Fe = centripetal force.
Dis!)lacement is the change in position, • For every force that acts on one body and is directed parallel to the surface.
specified by a length and a direction. there is a second force equal in magnitude For Sl System:
Displacement is a vector quantity. but opposite in direction that acts upon F=J..i.N
another body." 2
What is an Acceleration? Fe"' tnVr
-r-. Newtons
What is a Force? where: J..l. = coefficient of friction
Acceleration is the change of velocity per
where.
unit time. Force is a push or a pull that one body Coefficient of kinetic friction is the ratio m =mass in kg
exerts on another. This includes of the frictional force to the perpendicular
gravitational, electrostatic, magnetic and
Vr = tangential velocity in m/s
Instantaneous acceleration is the time Ioree pressing the two surfaces together
.r = radius of curvature in m
rate of change of velocity. contact influences.
F For English System:
Uniformly accelerated motion is defined Constant forces are forces that do not
vary with time. External forces are those
J..l.k "''N
as the motion in a straight line in which the
2
direction is always the same and the actions of other bodies on a rigid body J:n'Vr... lbf
Fe=~
speed changes at a constant rate. while those forces that hold together parts Coefficient of static friction is the ratio of 9c.
of a rigid body are' called internal forces. the limiting frictional force to the normal
What are the Laws of Motion? Ioree. where:
Weight (of a body) is the resultant m = mass in Ibm
There are three Laws of Motion which are
commonly called as "Newton's Laws of
gravitational force acting on the body due
to all other bodies in spac.e. It is always a J..I.,=N
Fmax =
Vr tangential velocity in ft/sec
r = radius of curvature in ft
Motion". vertical force acting downward.
gc = 32.2 Ibm- ft
Newton's First Law: Newton (N) is the force that will give to a ~· sec 2 -lbf
What is a Centripetal Force? ·
(The law of inertia) mass of one kilogram an acceleration of ~:
one meter per second per second. Normally, the centrifugal force is equal in

·~
Centripetal force is the force (real force)
• There is no change in the motion of a on the body towards the center of rotation magnitude with the centripetal force.
~' '
Dyne (dyn) is the force that will give to a ......I·
body unless an unbalanced external force when the body is moving around a curved
is acting upon it." mass of one gram an acceleration of one I path.
centimeter per second per second. What is the Law of Universal
Inertia is the property of the body by virtue Centrifugal force is the force (apparent Gravitation?
of which a resultant force is required to Poundal is the force that will give to a lor cc) on the body directed away from the
change its motion. mass of one pound an acceleration of one ':• ~~~ll~r of rotation when the body is moving The Law of Universal Gravitation is also
foot per second per second. uorrrHI a curve path. known as Newton's Universal Law of
Gravitation is stated as follows:
i·!

Day 18- Physics 445


444 100 i Solved Problems in Engineering Mathematics (2nd Edition) tiy Tiong & R~
What is the Law of Conservation of where:
"Every particle in the universe attracts Energy is also defined as the ability to do Energy? m = mass of the body
every other particle with a force that is work. Energy is a scalar quantity. v = velocity of the body
directly proportional to the product of the The Law of Conservation of Energy is
masses of the two particles and inversely The typical units for energy are joules, stated as follows: What is the Law of Conservation of
proportional to the square of the distance calories and BTU (British Thermal Unit). Momentum?
between their centers of mass." The unit BTU and calorie are used for "Energy can neither be created nor
thermal energy. destroyed; it merely changes from one The law of conservation of momentum is
11.111111
form to another." stated as follows:
( Calorie is defined as the amount of heat
lllillltlllf
required to raise the temperature of one Transformation of Kinetic & Potential "If there is no net external force acting
(
1~1~111~
gram of water 1°C. Energy: upon a system of bodies, the momentum
l'lllklli F of the system does not change."

l~~~~
British Thermal Unit (BTU) is defined as Potential Energy = Kinetic Energy
the amount of heat required to raise the What is Impulse?
m1

'~~
mz temperature of one pound of water 1°1=.
fngh :=; ~tnv
2
or v = J2gh Impulse is the product of the force and the
Gm 1mz What are Two Types of-Energy? . time during which it acts. Impulse is equal
F=~ to the change in momentum.
Energy is classified either a potential
energy or a kinetic energy. Transformation of Work & Kinetic
where: G = gravitational constant Energy: Impulse= FAt.
G = 6.673 x 1o· 11 N. m 2fkg 2 Potential energy is also known as the Impulse·= p 2 - P1
G = 6.673 X 1o·B cm 3 /~. s 2 energy of position or configuration or Work = Kinetic Energy
Impulse= mvrmat -mvinitiat
G = 3.436 X 10'8 lbf-ft /slug 2 gravitational energy. This type of energy is
G = 3.320 x 10'11 lbf-ft2/lbm 2 decreases as the elevation of the body
2
G = 3.436 X 1o·B ft 4/lbf-sec4 decreases and increases as the elevation Fs=imv where:
of the body increases. Normally, the lost in F =force
What is Work? potential energy is converted into heat or At = change in time
kinetic energy. \11 What is Power? P2 = final momentum
Work refers to the process of changing the '· P1 = initial momentum
energy of a particle, body or system. Work EP =Wh Power is the time rate of doing work or the
is a scalar quantity. The typical units of
work are joules, foot-pound and inch-
EP = mgh ,,
;~~
amount of work done per unit time. Power
is a scalar quantity. What are the Two Types of Collisions?
pound.
where: Elastic collision is a collision of two
The unit "joule" is equivalent to the units of W = weight of body P=Y!_ bodies in which kinetic energy as well as
N • m and kg • m 2/s 2 . This unit was named m = mass of body t momentum is conserved.
in honor of the English Physicist, James g = gravitational acceleration
Prescott Joule (1835- 1889). g = 9.81 m/s 2 The typical units for power are watts, ft- Inelastic collision is a collision of two
h = height of body lbf/sec and horsepower. bodies in which only the momentum is
Mathematically, work is defined as the conserve but not the kinetic energy.
product of force and the displacement in Kinetic ener~:IY is the energy in motion. 1 watt = 1 joule per second
the direction of the force. 1 hp = 746 watts
1 What is the Coefficient of Restitution?
W = Force x distance Ek =-mv 2 What is Mom~ntum?
2
Coefficient of restitution is the negative
where: Momentum is the product of the mass and ratio of the relative velocity after collision
What is Energy? m = mass of body velocity of a body. Momentum is a vector to the relative velocity before collision.
v = velocity of the body quantity. ·
Energy is the property of the body or e'= __vz,..- '~~2s _ ~ -v2-
system of bodies by virtue of which work p c~ nw
v1A- '~~1e '~~1a /vtA
can be done.
446 100 1 Solved Problems in Engineering Mathematics (2"d Edition) by Tiong & Rojas Day 18- Physics 447

If e = 1, the collision is perfectly elastic What are the Properties of Fluids?


while if e = 0, the collision is completely Fluid •· \!rtibia:
Inelastic. Density is the mass per unit volume. This w displaced Did you know that. .. the most difficult
is expressed mathematically as problem in mathematics is the "Fermat's
What are the Gas Laws? Last Theorem! The search for the proof of
m this theorem begun right after Fermat's
Boyle's Liiw: P=v 'i~~i death in 1665 and remained an unsolved
IIIII
"If the temperature remains constant, the theorem through centuries of hopeless
product of the pressure and volume is Weight density is the weight per unit search. A British-born professor in
(
1111001111 constant." This was named after l~ish volume. The relationship between density mathematics at Princeton University,
1~11.!11
[
physicist Robert Boyle (1627- H)91): and weight density is expressed in the Andrew Wiles brought an end to the
following equation. · search of the proof in 1995, i.e. 330 years
:11\l'~
.PV.:;:k later. Because of this, Fermat's Last
l.,,,
,,, Charles' Laws:
"If the volume of a confined gas is
constant, the pressure is directly
D= W
v D=pg

Specific gravity is the density of the BF


Theorem was regarded as the Mount
Everest of Mathematics.

~uote:
proportional to the absolute temperature." substance relative to that ofwater. This is "With me everything turns into
also known as relative density. Water is mathematics."
BF=pV.
considered the standard substance which - Rene Descartes
.Pl ~fz. has a maximum density at 4•c.
T1 -:-r2
where: Vs = volume submerged
. . density .,
"If the pressure of a confined gas is <i\sp~cific
, '.I 9f'aVi~
.· "' . density ofwater 1 p = density of the fluid
constant, the volume is directly
propl)rtional to the absolute temperature."
The values for the density of water are as
IIJ;
II
Also by equilibrium, the buoyant force is
equal to the weight of the body, thus
v: ····~.
.....!=~
follows: j,

BF"'W
1'1 T'z 62.4 pounds I ft3
1000 kg I m 3 Note: The buoyant force is always acting
9.81 kNim 3 at the centroid (center of gravity) of
General Gas Law: 9810 Nlrn 3 the submerged volume.
The combination of the Boyle's Law and 1 gram/cc
the Charles' Uiws may be regarded as the
general gas law. This is expressed · The specific gravity of water at densed Proceed to .the next page for your18th
mathematically as condition (4"C) is exactly 1.0 test. Detach and use the answer sheet
provided at the last part of this book. Use
Pi\'1 P.Na What is the Archimedes Principle? pencil number 2 in shading your answer.
PV::::nRT or Tt = Tt
The Archimedes' Principle is stated as GOOD LUCK I
follows:
m
where: n= w "When a body is immersed (partially or
n = mass .of the gas
atomic mass of the gas
R = universal gas constant
wholly) in a fluid, it is subjected to an
upward force (buoyant force) which is
equal to the weight of the fluid displaced,~
This was discovered by Greek
mathematician and inventor, Archim~des ·
,
:s_!
it!
~:t,¥
"i.!
(287- 212 B.C.).
Day 18- Physics 449.

7651 ME Board October 1997, C. 0.6 mph


•I ME Board April t998 D. 0.4 mph

Topics
II
•'\J

;~
100 g of water is mixed to 150 g of alcohol
(p = 790 kgfm 3 ). Calculate the specific
vol.ume of the solution, assuming that it is
mixed completely.
7701 EE Board October 1996
A 10 g block slides with a velocity of 20
r;m/s on a smooth level surface and makes
a collision with a 30 g block moving in the
.( A. 3
0.82 cm /g opposite direction with a velocity of 10

(
~IIIWi~ll

lli>'i~lijl
lii!IMI
0Mon
Physics
Vector and Scalar Quantities
Classifications of Vectors
B.
C.
D.
0.88 cm 3/g
3
063 cm /g
3
1.20 cm /g
cm/s. If the collision is perfectly elastic,
what is the velocity of the 30 g block after
the collision?

[
~llllil~l []
Tue
Speed and Velocity
Distance and Displacement
766: ME Board October 1997
100 g of water is mixed to 150 g of alcohol
A.
B.
15 cm/s
10 cm/s
LOlli Acceleration =
(p 790 kg/m 3 ). Calculate the specific C. 25 cm/s

0 0
I
gravity of the total mixture. D. 5cm/s

'""
Laws of Motion .. I
Theory Wed Forces A. 1.862 '771t ME Board April 1.997
A 60 ton rail car moving at 1 mile/hr is
[Q] 0 Thu
Law of Universal Gravitation
Work, Energy and Power
Law of Conservation of Energy
B.
c.
D.
0.963
0.286
0.862
coupled to a second stationary rail car. If
the velocity of the two cars after coupling
Problems is lftls {in the original direction of motion)

0 0 Momentum and Impulse 767a ME Board ApriiJ.996 and the coupling is completed in 0.5
Law of Conservation of Momentum The specific gravity of mercury relative to second, what is the average impulsive
Solutions Fri water is -13.55. What is the specific weight force on the 60 ton rail car?
Gas Law
of mercu~y? The specific weight of water is
[J ~
Notes Sat
Properties of Fluids
Archimedes Principle
'
62.4 lbf/ft .
3
. A.
B.
C.
50!bf
3500 !bf
1200 lbf
A. 102.3 kN/m

I
B. 132.9 kN/m
3 D. 60 lbf
3
C. 150.9 kN/m
761: ME Board October 1994 7&3: ME Board October 1994, D. 82.2 kN/m
3 7721 ME Board April :1:997
The weight of a mass of 10 kg at a location MEBoardApril1998 What momentum does a 40 Ibm projectile
where the acceleration of gravity is 9.77 The mass of air in a room 3 m x 5 m x 20 7&81 ECE Board November 1998 posses if the projectile is moving at 420
m/s 2 is m is known to be 350 kg. Find its density. ~
.: A 16 gram mass is moving at 30 em/sec mph?
,'.r
while a 4 gram mass is moving in an
A. 79.7 N A. 1.167 opposite direction at 50 em/sec. They A. 24,640 lbf-sec
B. 77.9 N B. 1.176 collide head on and stick together. Their B. 16,860 lbf-sec
C. 97."7 N C. 1.617 velocity after collision is C. 765 lbf-sec
D. 9T7 N D. 1.716 D. 523.6 lbf-sec
A. 0.14 m/s
7&:&: ME Board AprU :1998 7&4: ME Board April1996 B. 0.21 m/s 773; ME Board Aprii199S,
How much does a 30 Ibm weigh on the An iron block weighs 5 N and has a C. 0.07 m/s ME Board April1998
moon? Gravitational accelerati.on in the volume of 200 cubic centimeters. What is D. 0.28 m/s A 10-kg block is raised vertically 3 meters. :
2 What is the change in potential energy?
moon is 5.47 ft/s 2 and in earth is 32.2 ftls . the density of the block?
7691 ME Board October 1996 Answer in Sl units closest to:
A. 2.0 lbf A. 988 kg/m 3 A 60 ton rail car moving at 1 mile/hr is
B. 3.21bf B. 1255 kg/m 3 instantaneously coupled to a stationary 40 A. 320 J
c. 3.41bf C. 2550 kg/m
3
ton rail car. What is the speed of the B. 350 kg-m 2/s2
D. 5.0961bf D. BOO kg/m 3 .~~ coupled cars? C. 294 J
D. 350 N~m
I,, I\ 0 88 mph
ll 1 mph
,\\)\
4\50 · l 00 l Solved Problems in Engineering Mathematics (2"d Edition) by Tiong & Rojas
Day 18- Physics 451
"174: ECE Board AptriiD 1.997 779: ME Board October l'.991J C. 2.49 atm 788: ME Board Octol>er 1.997
An aircraft engine develops a forward What is the water pressure if manometer is D. 9.24 atm A 10 Ibm object is acted upon by a 4.4 lbf
thrust of i 5,000 N. If the gross mass of the 0.6 m Hg? Mercury is 13.6 times heavier force. What acceleration in ft!s 2 does the
aircraft is 100 tons, what horsepower does than water. 784: ME Board April :1996 object possess?
the engine develop if it is flying at 1000 'il A volume of 400 cc of air is measured at a
'l'li\'
l\ph? A. 27.4 kPa
~:~
pressure of 740 mm Hg abs and a A. 12.4
B. 47.2 kPa temperature of 18°C. What will be the B. 10.0
A. 150,000 C. 79.97 kPa
'~. volume at 760 mm Hg and 0°C? C. 14.2
(
1111~1»1
B. 5585 D. 72.4 kPa D. 13.0
~~ll!illlll c. 5400 -'{'
A. 376 cc
lif;illlll
D. 3108 780: EE Board October 199& B. 326 cc 789: A 50 kN truck traveling with a speed
( A mercury barometer at the base of Mt. C. 356 cc of 50 kph hits a lamp post and is brought
11~11~11
'1"1§: EE B«~J;u•d October :1994 Makiling reads 654 mm and at the same D. 366 cc to rest in 0.1 s. What is the average force

l~'""
if a i 0 kg piece of copper falls 100 m, how time another barometer at the top of the of the truck?
much heat might be produced? mountain reads 480 mm. Assuming 785: EE Board October :1995

'
specific weight of air to be constant at 12 The pressure of the nitrogen gas A. -408 kN
3
,111 A. 3.81 kCal N/m , what is the approximate height of thermometer is 76 em at 0°C. What is the B. -508 kN
B. 32 BTU Mt. Makiling? temperature of a liquid in which the bulb of C. -608 kN
C. 2300 Cal the thermometer is immersed when the D. -708 kN
D. 9.41 kJ A. 1,934.5 m. pressure is seen to be 87.7 em?
B. 3,508.4 m. 790: A tennis ball moving horizontally to
7'1&: ME Board October 1997 . C. 4,168:2 m. 1\. 34°C the left at 40 m/s hits a racket and
A rocket is moving through a vacuum. It D. 2,647.7 m. 11 45°C rebounds horizontally to the right at 30
changes its velocity from 9020 fUsee to c. 60°C m/s. If the mass of the ball is 100 grams,
5100 ft!sec in 48 seconds. How much 78:1: EE. Board October 1994 IJ 90°C find the impulse of the force (in kg-m/s)
power is required to accomplished this if Assuming the barometer reads 760 mm exerted on the ball by the racket.
the rocket's mass is 13,000 slugs? Hg, what is the absolute pressure for 900 78&: ME Board April1998
mm Hg gauge? A transportation company specializes in A 1

I
7
A. '1.63 X 10 hp tile shipment of pressurized gaseous B. -1
B. 3.16 X 107 hp A. 74.213kPa rnaterials. An order is received for 100 C. 7
C. 3.61 X 107 hp B. 221.24 kPa l1ters of a particular gas at STP (32oF and D. 12
D. 1.36x107 hp C. 48 kPa I atm). What minimum volume tank is
D. 358 kPa ru:cessary to transport the gas at 80°F and 7911 Two steel balls of masses 500 kg
"Jt'li/1 ME Board Apnrifl lt998
A force of 200 lbf acts on a block at an
~ .r maximum pressure of 8 atm? and 50 kg, respectively are placed with
78:&: CE Board May 1.994 \~ their centers 0.5 m apart. The two balls

~
angle of 28° with respect to horizontal. The A barometer reads 760 mm Hg and a A. 16 liters attract with a force of
block is pushed 2 feet horizontally. What is pressure gage attached to a tank reads ll 141iters
the work done by this force? 850 cmof oil (sp.gr. 0.80). What is the "'i
<: 10 liters . A. 6.67 X 10"10 N
absolute pressure in the tank in kPa? ll. 12 liters B. 6.6'7 X 10"7 N
A. 320 J C. 6.67 X 10-6 N
B. 480 J A 168.1 kPa '187x EE Board April1996 D. 6.67 X 10"3 N
C. 540 J B. 186.1 kPa !\ 20 liter sample of gas exerts a pressure
D. 215 J C. 118.6kPa ',f 1 atm at 25°C. If it is expanded into a 40 19:&: EE Board October 1.995
D. 161.8kPa 111"' vessel that is held at 100°C, what will A 50 g mass hangs at the end of the
7"1&1z What average force is necessary to 1" • rts final pressure? spring. When 20 grams more are added to
stop a .45 caliber bullet of mass of 15 783: EE Board April 199& the end of the spring, it stretches 7 em
grams and speed of 300 m/s as it A sealed tank contains oxygen at 27°C at 1\ 0.50 atm more. Find the spring constant.
penetrates a block to a distance of 5 em? a pressure of 2 atm. If the temperature II 1 0 atrn
increases to 100°C, what will be the o :\15 atm A. 2.8
A. 12.5 kN pressure inside the tank? ll 0 ti:~ atrn B. 2.9
B. 13.0 kN C. 4.:3
C. 13.5 kN A. 4.92 atm D. 2.5
D. 12.0 kN B. 4.29 atm
452 1001 Solved Problems in Engineering Mathematics (2nd Edition) by Tiong & Rojas ,
!l
::1
A. 3min.
793: EE Board April1996
',,
~t: I
Determine the submerged depth of a cube B. 1 min.
of steel 0.3 m on each side floating in C. 4min.
mercury. The SJJecific gravities of steel and D. 2min.
mercury are 7.8 and 13.6 respectively.
798: ME Board April199& Topi4:S
A. 0.155 m. What is the power required to transfer

[
101!11

IIMilliil~ll
B.
C.
D.
0.165 m.
0.134 m.
0.172m.
97,000 couiombs of charge through a
potential rise of 50 volts in one hour? 0Mon
Physics
Vector and Scalar Quantities
Classifications of Vectors
(
'~1~1111

~~~~Iii 7941 EE Board October 1995


A block of wood floats in water with 5 em
A.
B.
C.
0.5kW
1.3kW
0.9kW
0Tue
Speed and Velocity
Distance and Displacement
L.,,l,ll projecting above the water surface. When D. 2.8 kW .,:· Acceleration
placed in glycerine of specific gravity of
0 0
' 1.35, the block projects 7.5 em above the 799: EE Board October 199& Laws of Motion
liquid. Determine its specific gravity. How much oil at 200°C must be added to ·~ Wed Forces
,1111 Theory
50 grams of the same oil at 20°C to heat it
A.
B.
C.
0.514
0.704
0.836
to 70°C?

A. 12.39 grams
0
Problems
0Thu
Law of Universal Gravitation
Work, Energy and Power
Law of Conservation of Energy
D. 0.658

7951 EE Board October 1996


B.
C.
D.
29.12grams
19.23 grams
23.91 grams Solutions
0 Fri
Momentum and Impulse
Law of Conservation of Momentum
Gas Law

~
A solid cube material is 0."75 em on each
side. If it floats in oil of density 800 kg/m
with one-third ofthe block out of the oil.
What is the density of the material of the
3 soo: EE Board October 19'96
The temperature of three different liquids
are maintained at 15°C, 2o•c and 25•c
0Notes Sat
Properties of Fluids
Archimedes Principle

cube? · respectively. When equal masses of the ,[1.


first two liquids are mixed, the final ··r: 1

A. 533 kg/m
3 temperature is 18·c and when equal .· ·
3 masses of the last two are mixed, the final ': ANSWER KEY RATING
B. 523 kg/m
C.
D.
513 kg/m
543 kg/m
3
3
temperature is 24°C. What temperature
will be achieved by mixing equal masses 761.c 771. B 781. B 791. c c:J 34-40 Topnotcher

79&1 CE Board November 1993


of the first and the last liquid? 762. D
763.A
772. c
773.c
782. A
783. c
792.A
793. D c:J 26-33 Passer

A hollow cyJinder 1 m in diameter and 2. m A.


B.
15.87·c 764.C
765. D
774. B 784. D
775.C 785.A
794. D
795. A.
c:J 20-25 Conditional
high weighs 2825 N. How many kN of lead
weighing 110 kN/m 3 must be fastened to C.
D.
10.3o•c
8.65°C
23.5TC
766. D
767. B
776. D 786: B
777. B 787. D
796.A
797.C
0 0-19 Failed
the outside bottom of the cylinder to make
it float with 1.5 m submerged in water?
768.A 778. c 788. c 798. B If FAILED, repeat the test.
769.C 779. c 789. D 799:C
770. D 780. A .,790. C 800.D
A. 8.5 kN
B. 6.5 kN
C. 10.5 kN
D. 9.5 kN

7971 ME Board October 1995t


ME Board Oetober 1996
How long must a current of 5 amperes
pass through a 10-ohm resistor until a
charge of 1200 coulombs passes through?
454 1001 Solved Problenis in Engineering Mathematics (2nd Edition) by Tiong & Rojas
" " IIDal

Day 18- Physics 455
~

rm W=mg mt = m1 + mz Initial momentum = Final momentum

W = (10 kg)(9.77 m/s 2 ) mt = 100 + 150 Note: Specific weight of water is 62.4 m1V1 + m2V2 = mtV1 + m2V2
mt =250 g
l·r.J.i I lbs/ft3 or 9.81 kN/m
3
10(20) + 30(-10) =10(-V1') + 30W
W= 97.7 N (j)


sp.gr. ::. __
m_
Vt= Vt +V2 (!)water
- 100 =- iOV1' + 30V2' 7 Eq. 1
Vt = mt.+ mz COm= 13.55 (9.81)
[
1111~~"~1

Mass in earth = Mass in moon Pt P2 Note: For a perfectly elastic collision,


COm= 132.9 kN/m 3
coefficient of cestitution (e) is equal

•-
ljlil(l'~*ijl
3
w Note: Density of water (Pt) is 1000 kg/m
(
li;li~llll
30 to 1.
v 0.100 0.150
'l,~!~llill 32.2 5.47
t = 1000 + 790 . v2 ·-v1·

C°~m J
w = 5.096 lbf. e=--
LOIIIIII
Vt = 2.8987 x 10" m x
4 3 0 ·~.·
I
.. .. Vt v2
V1-V2

'
1 ... 1 = v2 ,_ v1,
''i~ ' -+-----
~IIIII Vt = 289.87 cm 3 Vt-V2
v1 - = v2 ,_ v1,
v2
Volume of air is the same as the volume
of the room · vt 289.87 mt ,,,2
v=--=-- 20- (-10) = V2·- (-v1 ')
m1 250 -v
v = 3(5)(20) V1 • = 30- V2 -> Eq. 1
= 1.2 cm 3/g


v
V = 300 rn 3 Substitute Eq. 2 in Eq. 1
m1 +m2
m
p=-
v mt = m1 + m2 Initial momentum'= Final momentum
-1oo = -10(3o..:.v2 ')+30V2 •

mt = 0.100 + 0.150 -100 = -300 + 10V2 '+ 30V2 '


- 350 kg mN1 + m2V2 = (mt + m2)
P- 300 rn3 mt = 0.250 kg. v2' = 5 cm/s
16(0.3) + 4(-0.5) = (16 + 4)V
p = 1. 167 kg/m 3 Vt == Vt + V2 V = 0.14 m/s
Vt::: mt +!':lz..
Pt P2 V =~X~X5280 ft
1
Initial momentum= Final momentum hr 3600 s 1mi
3
W=mg Note: Density of water (pt) is 1000 kg/m v1 = 1.4667 ft/s
v = 0.100 + 0.150
mN1 + mN2 = (mt + m2)V
5 = m(9.81)
t 1000 '790 60(1) + 40(0) = (60 + 40)V
m = 0.5096 kg. F(t) =m(V1 - V1 ')
Vt = 2.8987 X 10"4 m 3 V = 0.6 mph
F(0.5) = _?0( 2 000) (1.4667 -1)
(~)
3
3
V = 200 cm x m 32.2
100 em p=- Note: Since the second car is stationary,
v its velocity (Vz) is zero.
F := 3478.509 lbf
V = 0.0002 m3
- 0.250 kg
m

-- -
P- 2.8987x10c4 m3 Note: From the choices, the nearest
p=- • Vt
v p = 862.45 kg/m 3
v2 answer is 3500 lbf.
0.5096 kg
p=----
0.0002 m 3
sp. gr. = ·_e__
p = 2548 kg/m 3
Pwater m1 m2 V = 420 mix~x5280 ft
sp. gr. = 862.45 hr 3600 s imi
Note: From the choices, the nearest
answer is 2550 kg/m
3 1000 V1' ~--·--· ____,_ V2' V=616ft/s
=
sp. gr. 0.862
taeJ t
~).Q't.t.r.I)Nl l\'l'!>l'l'!l~
I --II
I' }

IV ,
~

:
Momentum =mV
I'
!lf..,

Dayl8- Phyf:!iCs 457

• •
456 lOO 1 Solved Problems in Engineering Mathematics (2nd Edition) by Tiong & Rojas

Momentum= 40(616)
Momentum

24640
=24640 lbm-ft/s

.
Momentum=-- = 765 lbf-sec
p = b.K_§_
~t
1,,
l!r
Jl,

v-
8
F-
Pabs = Pgage + Palm
Pabs = 0.8(9.81)(8.5)+13.6{9.81)(0.76)
Pabs =168.1 kPa


32.2

• 101!111
"-+
s
[
b.KE = ~m(V0 2 - V 2 ) Work done by retarding force = initial
llll!(!:iljljil
kinetic energy of the bullet
[
l.ili~~:~l
PE =mgh t.KE = ~(13000)[(9020} 2
-(5100n
J P1V1 = P2 V2
T1 T2
'11~1~1~11
PE = 10(9.81)(3) !t/
=3.597x1 011 lb-ft .:~ F(s) =!mv2 Note: V, = V2

1
t.KE
PE =294 J 2

' •
L""'l'
~H1111
V = 1000 kmx~x!OOO m
p = t.KE
at
P = 3.597x10
11
~

·~·.
;I;,
)i
... '. F(0.05) = ~(0.015)(300)2
F=13500N
F=13.5 kN
Ill
2
27 - 273
P2 = 2.49 atm
p2
= 100 + 273


hr 3600s 1 km
48
V = 277.778 m/s PtVt P2V2
P = 7 .49x1 0 9 lb-ft/s
~=--,=;
P=FV Pressure (P) =Specific weight (ro) x 740(400) 760V2
p =15000(277.778) 1hp Height (h)
P = 7 .49x1 09 lb-ftls x 550 lb-ftls 18 + 273 = 0 + 273
P = 4166670 watt V2 =365.38 cc
P=1.36x10 7 hp P = {wH9 }h

1hp
p = 4166670 watt ( 746 watts

p = 5585 hp
)

• p =(Sp.gr.Hg )( (I)Waler )h
p = 13.6{9.8)(0.6)
Note: From the choices, the nearest
answer is 366 cc.



P = 79.97kPa

PE ::::mgh
PE =10(9.81)(100)
PE = 9810 J w =rcose(s)

Let: h = height of Mt. Makiling

Paouom =Prop + roAi,h


Note: V1 =V2
78
PtV1 = P2V2
~

87.7
T2

roHghb = roHgh, + ro Airh 0-273 =--,=;


let: Q =heat W = 200cos28° (2)
(sp.gr.)Hg(I)Walerhb = ( Sp.gr.)Hg (I)Waterhl + roAirh T2 =306.95° K
w =353.1791b- ft
Ass~:~ming no losses, the total heat equals 13.6(9810)(0.654) = 13.6(9810)(0.48) + 12h T2 = 306.95-273
the initial PE of the copper.


h = 1934.5m T2 = 34°C

m
cal . 1kg 1m
0=9810 J x-- W = 3.53.179 lb-ft X 2.202 lb x --
3.281 ft
4.2 J
Q = 2335.714 cal x9.81 ft/s P1Vt P2V2
W = 479.55 N-m I 'ub• pgngo + palm ~=--,=;
' 1 BTU w~480J I ',~>·• 13.6(9.81)(0.9) + 13.6(9.81)(0.76)
Q =2335 714 calx - - · 1(100) 8V2
· 252 cal """' 221.47 kPa 32+460 = 80+460.
Q =9.268 BTU
V2 =13.7 liters
Ntltu I rom the choices. the nearest
Note: From the choices, the nearest iiii'IW<'I I'> /74 /4 kPa v2 "'141iters
answer is 2300 cal
458 .1 001 Solved Problems in Engineering Mathematics (2"d Edition) by Tiong & Rojas --···----------------------------------~ !8- Physic~ 459

• P1V1
T1
___!t?_QL =
= P2V2
T2
P2 (40)
Ill
Momentum = mass x velocity

P1 =mV1
j
II
~:~ where
F = ks -7 Hooke's Law

I" = tension in the spring due to


the load attached to it
A

5 em
p1 = (0.1)(-40) k = spring constant
25 + 273 1()0 + 273
P2 = 0.63 atm P1 = -A kg-m/s =
s elongation of the spring due

(
~1111~~~~
to the load attached to it
I!'P.9!tl
•lllll!limll lliiia P2 = mVz
50= ks
(
111!1~111~1
p2 = (0.1)(30)
'll~li~~ll
F = ma ~ Newton's second law of
P2 = 3 kg-m/s S=50
- -~ Eq. 1 W=BF
motion k
L_IIIMI ( sp.gr. )\11/ood Pwater VTotal = Pwater VDisplaced

(__!Q_}a Impulse = change in momentum 'i F = k(s + 7)

~"""
4.4 = ~- ( sp.gr)wood (Ah) = A(h- 5)
32.2 50+ 20 = k(s + 7)
Impulse = P2 - P1 :r h-5
2
=h-


a= 14.2 fUs 70 = ks + 7k -) Eq. 2 (sp.gr.)Wood --+ Eq. 1
Impulse= 3- ( -4)
Impulse= 7 kg-m/s Substitute Eq. ·1 in Eq. 2:


m1 F

s
m2 k=28
(50)
7() cckj-- + 7k
\ k

i!ll
A

7 em

Glycerine
sp.gr. = 1.35
Using the formula for universal
gravitation:
Using the relationship between impulse
and momentum:
.F = Gm1m2. W=BF
Impulse = change in momentum sz (sp.gr. )VII:Jod rwater VT Pwater VD
where : F = force of attraction in N.
(spgr.),i\I'A:d (Atl) = (1.35)A(h-7)
F6t =mV1 - mV0 m1 and m2 = respective masses BF
of two particles in "i:'
2fv"" 0 = !.:_35(h- 7.5) -~ Eq. 2
kg. h
50000 N s = distance between the centers· W=BF
m = ----=~-
9.81 of the two particles in m. ( sp.gr. )Steel Pwater \/Total =(sp.gr. )Hg Equations 1 and 2:
m = 5096.84 kg G = gravitational constant
3 2
2 7 81(9 81)(0 3) = l3.6(9.81)(0.3) d
G = 6.67 x 10'11 N-m -~=-_!?_ = ~~~-~5(h - 7.5)
kg2 d=0.172m
V. = 50 km
0
x -~x 1000 m h h
hr 3600 s km 11- 5 = 1.35h -- '10.125
V0 = 13.89 m/s Substituting: 1'1 = 14.64 em
i .ci I\ ::':.: base area
Substituting: (6.67x10- 11 )(500)(50) h ·:-:· hf~ioht the V·Jood Substitute in 1:
F = . 2
(0.5)
F(0.1) =5096.84(0 -13.89) 14.64-5
F = 6.67 x 10-6 N sp.gr.vvoor! = -·-14'.64-
F = -707951 N
. • .
:·!S.l.LJi.,!l/ood cc 0.658
F = -707.951 kN
_.60 100 1 Solved Problems in Engineering. Mathematics (2nd Edition) by Tiong & Rojas Day 18- Physics 461

• w
BF2 = Pwat.,;vLead
BF2 = 9.81Vtaad
I
=
Heat gained hE!at loss
50c(70- 20) = mc(200- 70)
m = 19.23 grams

[
•01111~1

1!1:!~1~11
wlead = 11 ovlead

BF1 + BF2 = Wcy,inder + Wtoad


11.56 + 9.81Vlead = 3.825 + 11 ovlead
vlead = 0.0772 fT13
• Heat gained = heat loss
mc1(18 -15) = mc 2 (20 -18)
3c1 = 2c 2 ~ Eq. 1
(
illll~lll~il

11~:~11~11
wlead = 110(0.0772) Heat gained = heat loss

L" "~"·' W=BF WLaad = 8.5 kN mc 2 (24- 20) = mc3 (25- 24)

l".,
Pcube Vrotal = POit Voisplaced

Pcube(x
3
) = 800(~x ){x
Peube(0.0075) = 8oo( ~ }o.oo75)
3
2
)

3
• Q =It
1200 = 5t
t = 240 sec
c 2 = 0.25c 3 ~ Eq. 2

Substitute Eq. 2 in Eq. 1:

3c1 =2(0.25c3 )
c 1 =0.1667c3
Peube = 533.33 kg/m3 · 1 min


t = 240 sec x 60 sec Heat gained = heat loss
We t=4 min mc1(t -15) = mc3 (25- t)
0.1667c3 (t -15) = c 3 {25- t)
Note: ampere = coulomb/sec.
0.1667t- 2.5 =25 -t

Ill t =23.57°C

Q::lt
97000 = 1(3600)
I =26.944 A

P=EI
p = 50(26.944)
p = 1347.2 watts
· 1 kW
BF1 = Pwater Vo;splaced p = 1347.2 watts x 1000 watts
2
* P=1.3472 kW


BF1 = Pwatar( )d Y

BF1 = 9.81( )<1) (1.5)


BF1 =11.56 kN
* 2

Heat = mc(at)

where: m = mass
c = specific heat
=
at change in temperature

I
~
fl
484 ·lOOI Solved Py>blems in Engineering Mathematics (aM. Edition) by Tiong & Roj..-

~~~<S..t:>tt·!.,:~i.ot~.Jif"t: ... ~.t1f.~·~·Jll-.~"<--~1>'>:"->- !.,..• ., . . . ,. .... ~ o;·~:J'"*'.,~"l$fo.it ''""" ,..<-+··"<! ........... ,, ~·-·..o.~:j! ~-~"" .:-
~I
' \'

'"""'; '!')• "' ... ,.,..lt:>ll. . . ..,_.:~#."'-..:~ CO<"J-,., .. *!'""'"'"*·.. ..,"""'l"'":...="l' ........-......, -"-"· '"'*""' ~ ~ ~·· 0" .... "- " , . , ........ ,.. ,,:,."" .. , . . . , , . :..·~<! #>>! '(·.

·~.:.·»-!1:,»·• ., .,.._,.,'-'.• 't'~"' •.'If ... ,.·-~~. *... :$'+~~. » ~.+r:\.'V. ~ '"<tt-'i. 't--1' 'I'" of< J>l"f<? '!- ... ~·-)f.. Jl.4f.-f- .o·~" ....... 'l jf.$. .-~_. . . . +..< Topics
• -t ~- 1>· t ~ -~- ~-- ... ;1: t;,;. 1""" .. ~ ,. .,, '* ~ ... ,:#' t <y .. " "' ~ ~ *"" ,~ .. -~ "·" { « :"" * ,<· ,;. ••: ~ .<. ~ "' f -~ '>I ~ ~ " •• < ...: >I ' -~ <. .;,. ; ;: ., il' ·"'·
-~ Engineering Mechanics
'~'-"'1''l> r• l'~..,,~ "'""'"'~ ~f '1< ~., ,_.,. + "'~ .,·_,.,._., .. _~_.., ~~ _.-~ t ;,. ., ~-"!'~ "< ~ -" ' ~ 0• ~ ~ t ., -~ • ' '!" ., 4 ~ ,.. ., ~ ~ .. '0 -~ -s.
--.....1
Mon Branches of Mechanic..s
~ ~ ~" ~ f!- 4< "\' ''\ /.o. to ~- 4".";::4'. >" PJ; #' i • ;. • ~ l· '<- "" <Y P ~ ~ -t: 1>- ~ • t « '{ o " '", <> < ~ 'J• ~ i< .~ ·~· ~ v, \ .. -(< "< ..< ~ <-• 1 .-c ;' 4 ·_,. :I, "·A'- v .,.. '
Conditions for Equilibrium
X "<1. }II 1>. t:

~ .<> ,. ·!I , ~" !f 111- <!I··~..,~~,~.""'·.,.:..._ i tf v" ! ·,. '~""' ~ ,oc'~. ,~ t -so: ~>- ~ ·.' ~ r .·; ..! >~ ~ ~ * ;. ~ :t< "<'. ~ ~> ;<; ~· ~ ;.. ~•. ~ • 6 • ., ~ .., 4 'f. ., ·,. ~ "'· • •• 'Yo~ o. • ,,
.~
.~
[] Friction

L,,,,~,"
..,.'$' .... ., ».><>.; .t,il- ·~·'!')> ... « """' ~¢,ij.'fk ...... 4·":., t"' .J ........ "'~"'<I"'"',< t·-' ,; "'/'i'-ot.'ili<t 4-'.!i- .;;¥1 ~· ... ~'t'"'" .... ,. "'".:"' .~ ..';~. ., ~·
~-~ Tue Parabolic Cable
:s Catenary
0
~ *)P> .4<,..,. •, ~·~}! t.·~jliti'.~.*·¥.:J>'l!>.~.-. Jff><-·"':·t·~ ~<~.• "': "'<11 _.,,.~;,•.""$., ..:,.'-~>::,·~ 0..'1!' '*' ""·~ ~.'«~)<'·*.><-* i;t·,,:., #:•.~-* >t-.,~,. ot-~ ><::
4
~ffi,'¥-~·<j'> ~<t:t'..,. <h*-*>,»-1.} '* ~ ~.'41•'•'f-'-"""''4rlii. oii #. u. ~t ~~~i >lt :~if io! ~ ,1' •. •'f4%.'*:~. """*"f..., <t. 1-* ~ ~ ~ •4• ..... ~ ~·- j *· ~ .... k jl:o , ,

r ~-- Wed
Centrolda! Moment of Inertia
Moment of Inertia with respect to
\'""" :;_.,~~,.,:,..i'•-'ll"V>If.~...-:.~-;:*-~>(.,..,.<ir"'·""'~l%>'...,~.>t< ,...,. ~"'·>~"'~~'Ill',.~ :i\c1 ,r '!I' 'i<:~~~'*_...,.·.,.<!< .., ~ i<.'or. 'fi ,._..........,_ ..~,.,.,... Theory

• '\'!
*
~:• tf~ ~-~.~ ... t~)l'ti ••'*"'!i!',;!l;,..,.,11'.4.'~·\f'-4-~ <'f·~-~~:•*:}'1-·.'*'"'i'k-~lil "'~.:41-'«'~ ..... 'i ·*· ~·'? !j.-;,: ·~···.;'~·~·...·'* 'l ~ ~i. •.~ 4<

i-,,i.,-~ ... ,....,..,.,.,,.,.._.,>!!! !f'*=*_Wolt-'1'# *''11!···""·~~·:~¥·..,.,,,:~--?< ~·~· ·····¥~~.~~ .. ~~~ t: ~· .... ·<!<·*~. ·~ ;..· . . ~~~· ~··~.:~.,.,:it
[J []
Proi}lr,ms Thu
the base
Transfer Formula
Mass Moment of Inertia
*·11'.¥:,•· , •."~>."' ~.f"#'# ~~.,..,._!it . . :~:....·,. •• ,. if t;"::f. ~"" >ll'11>.fl!. ~~-:"='if~ ... ·~ :o!-.'1-i<'f'~- ~'11-t;j>'~ . . . ""',. '~';;'" ll' ~.!J."t ,.,..,"'"*'If-~'»""~

[_] ~-=]
11.<

iic.:>t ¥~!~!-.• .!·~-~.~;- .....:".'\!-~ ~ ._,.,ifl!./0 .. W' ~·.:; ~-· '*:t,li'loi 't<'i ~.fie-if*-~ t·.,:..-.·.(·t ~ i4 """·""·~ j> "l ... ~· ~=%~ ** ot .~<'··f,;. ... ,.;.,·••;':4.,: ,•..,...

"!*-~. ~ .•Jt~··:~,t!~··~~·~:"t il"+'·(l!t:ilt>ft·...:f'+*"'··~·~···>< 1<·~ 1,~···--·*''" .!J'~;..>,S.,~·.t... ··•·• ~ih~'* ot ...·. •*:'H ~:···:·lol.4-,;·i.·~··.:~, Solutions Fri
<t.~.;jrt:4J•.!'}~ -+:*'·~:'t'~ ~..~·$~..,,;.:. ".~.~,~:j, ~·~·* •.<>.+ ~·~··(_>:-'!' ~·*'*~· $<~ ~"t~"1'.'"' ~.ll'y'f'-~\0: ",."" v.".o.,.. ~.,..

.• .•.. ~··'*·

·.W:i!.:'-'""11-llt ..i<
~· 4, 11:~ ~ l!t il,>'!<•• ::..k.4\''l!.~'.fl\o <i '!>*,!.:~·\.flo ik·'\1'· ~ +.M, ',l>,~t'l!' "*~.-;

~·-i~;C'ti·."f:fl>l/.w ... ..::'!<:·• .• ~ ,fc.<fo!·~ •* ,..-t- ~ t ,;, i¢;.·¥1,'.,:' 4:\:.


:rr.olo <4 ,.,$<-~{.~.IIi ,AI~~ '8-~ .~ ill'~;.., J< f; ~·• to' ¥o -t ·~ i

~"' ill")<::·;,·»'*..,, <I ~,."'}''*"it. i< ~·.,i>. ~


K "'"*:*

;, '< «
,:~-; ~ * ;.·" ..;.,~'<

~ ...,
1>11

~ ~ * ·~. ,.,·~,
·~ ,
LJ
Notes
[=]
Sal.

... A\. ,;~.it·4·:> .<1!;:.0·,;,.. ....*- k.4o .•;:""''". ""~·::t«-'ii-P.. .. 4: :-:. ~ t ..l!'~'~.~ ....i,."' "'>f . ..J,.. .., ·~ '!- :< ....... li< .. ~ <4-·~.~.i '~ <. :' "·~· ~ "·~"' -~>.+ .... .~
#;

,'f< <!:'r,. <e:t .~ ~ ":~<',. ·~~~~ < "''*, :t- if, ~ .:..ts '-."" " ~ t :., 4 .. '" •., ~ ·: .~ ~~~ ::- ~ ... 4! .. ~ ·~· "'"' '.,._ ~ <.< .,.. '!' #, A' ,., 'I * "''* >~< ~ ~ , • .< ~ .., .,. 'o: .. . ., ~ •."> ~hat .!iL~!!_eerin.!l Mec~anics? C. Kinematics. It refers to the study of
. ' ' ' . .~ . . motion without reference to the forces
1>.!("'!"........ 11' ... ,.,.,.""'1'· .
~,,.., .t;'<f""':'.""'·"'·"~"': "~"". ,.-,.,~·~.~ ".~ "f~,~)" ......\.,..., ..... 1-,..~., .. ~"':'"'""·" .1; "')'"'" ,, ~ .... ,.,-~ ¥ .. '"'."~'"
Mechanics is the branch of physics that which caus1~s the motion,
,.;., '!:~·~. ~ ,.,.~ "":t ~t;'l' p,. ~"" "'~-,. ~ '*' l·'f'"''-"'· .. ·~ ·~ . ~ 1.<: ~.~·.'-'·, ·*'·~.~~·i ~-"f.,.~.; .If.~"""·...~ ..:.~ 4o.fl '11 ... +<~ ,... ~ .. '!".""'"' ": *.1'>'~, ... ! considers the action of forces on bodies or
fluids that are both at rest and in motion. What .11re the CQ~s fo!
~.,..~'~~~~· ....·;to,., "'·"'·~.r.••"t.*'i~• ... -1'*."!', :it • 'l>~_,....,l~.,,. .'11..... » "'+;..;,»_,., ~-'¥. 4· ~¥'> •.,,,. ~ , n.p '# '4·~ ~.t • ~ ·~-~."': _._ =<~ .~ .i ~ ·-;-.~ ~ ~ , ....
~guilibr_!!!m1
1..- •:..;.~ .•·;.:~~"··\ ......IIO;fl·.i. ~~,,... ·Y:~,.i'~·.:.. .... _~<~.A-·~~,1' ~ ~ ,_ >Ji,~, ..o: si.,'#.· .... ~u:?><"'.:f:":o$',.. "~ ~ ~ *."' :.: .. .:c: .. ~·: ~ "- '*· ":· ...... -"'·"' ~ -. •.,. '*"" Engineering Mechanics is the branch of
·~!t'-.*<ii,.»\!f! =.;,.t .;..,,. ,-;·w:i>.lo ~·, • .,:*"~•:~·!" *"*·k~':(-;f,ii,.jj·~#+·f·'* ·"':.~,,.,;~i.*:rilt':,.-~ ·~ !.' *.*t ll.:or. 1!1:,.,_ ~·,. *~ ~·i',., ..\ ')<;~ ..."~· ·~ngineering that applies the principles of A body is in equilibrium if it satisfies the
mechanics to any design that must take following conditions.
'!<·~ !ti "·*'. lt}~>'*!~:t (<i<t-f. .. ~ ~·.• ~.~ &:~.~~ ...·.~:$' ~.~.~t..t:,.t --6'. i'!ji *·· *'·,... .~ * .¥' .s. ~ i ;j,~ ':ri:·.li: ~: )': • .j:·~.'* ~ :a.• ~-'II •••,.;,...:'~ 4 ii. ~ i*..~ * 111to account the effect of forces.
'.'~·~·~"*'"' ~-.·'4·1t<,<lll~,.. •.,~jt:.>l'.): j!.'l;'.,t.li'.+~~ ... ,fl.'.t ~~ ~:;~.# *'··~ "'.""""~"'··~~:.. ·?~,.:,:,.it ~~ . ~lto;ft-0!' -;,.,i-tf: *~~.*·~·?.1-.~ ""~if. 1. G1·aphical condition: Under this
Wh_~t are the Branches of Mechanics? condition, the forces or vectors are
; "';JI> lio'~····~·.)l..;t~~* ............... .<t 4: ........... "'!;.'l",if.*'~'*·'*''*.~.i'* ~'*if~·~:,., 'f'" *·-j>'*"'»"' ... "'.";. ... ..:t'lt.t..-t· .... :o;··•*''*"'·!t~tift'll:;t-it transformed into a force polygon. For
: 'lloi..:"'~ J. ···~ ~.!14v;....,;••t10<e)£.f!>.••11·~-·:t:...ji,.. ~ <11'':1,"" ~ lri'~'t,!l,IJ'~'II!·~':'\> *'4'.:..... ~-.,.:~·-(~~:*·····.;:~·"' :;.. .-.~ .. >~,'Wit:':it-:'f.•t-:.
I he branches of mechanics are the equilibrium, the force polygon must
l••llnwing: close.
il:f.,.·~<'\' ~'*·'*·:~·If illi1*;. ... ·,.."~~' .,.:.~ "'>";ll''i·~· t~ 4: F1
·.r:); ,;\'~,"' ~ ~-\1. ¥·~ ;!< ,. ... '* ~.., ~ •·:.-.·~ i)- •• ,<;;, !0\ ;,"" ~.,;vo. ~ ~· ~·t ,."" ~-"'~ .. . , lfl:1t 1 ,*."' *J:·~ .,..,.......~~:-• ._. ·>ll.:i.~""'r.~ ~ i ~·; ~ .. -·.~·. I\ Statics. It deals with bodies in the
;;tate of rest.
.,,,,:..;,.*'""""" ~ .~> ."~',"' ,.~ ~· .. ~-.;.;, ~ ~ ~ :,.,;:~....:~:.. ~'"'"*i>!... ·n.,:'* ~-·., ~.:+ ..
1

,, F2
.4: '<+·<- ,, ("''"'"' .•·' ,, .; "<;<;i<·· "'·"'"'""""?<,i:>;V·.'>''>V i>

; .• ,. ·• ·'""""f.<~- "'~;-.';"·-<-:..o..o ,._,.,.,.,., .6o·i> "'"' ~ ·~-"'"" ~ ~"' '· ,:, .. " ~ ... -~ "''"" • "'~ "~·*-·••·"'·.,."' ,.,.,~"' "'"'-"':"-·~· ';,"".~'... ~~a.,~., Dy11arnir.s or Kinetics. It deals with
l11><ilc'·; 111 rnotion under the action of
..t "' "'~ • ·' l': "' ~ ... ., ~ ..; .f, " ~ i .. ""'· .. ~·"" "' .~ "' v, ~ , ·~ ~ ~ ~ ,.... " ~ :' ~ 1< ... _,. .... .,. ""<I< "" • ~·!''I· '· ·' , ~ • "' '" .., ~ ·"' .~....... <I<· ,, " ; '( ... '
(!11<'1 •;'
?,. ........ ~.,.~ .... ..r.•~<#oif"'''·~.~.,q..~ ~:.;''~"'·.l+ '·~--~ . , .. ,. •, I < " ' .. • ~ ,' f•''•' _,. •

\ F.l
466 · 1001 S0lved Problems in Engineering Mathematics (2nd Edition) by Tiong & Rojas Day 19- Engineering Mec;:hanics (Statics) 467

N~
F=~N c) Length of parabolic cable, S: C. Minimum Tension, H:

6 F2
Force Polygon
F
tan~= f.1 S =L + 8d2 32d4
3L - 5L3
Like a parabolic cable the minimum
tension in a catenary occurs at the
lowest point

where: N = normal force H=wc


F = frictional force
2. Directional condition: If three or R = resultant force where: S = length of parabolic cable D. Span, L
more non-parallel fqrces or vectors ll = coefficient of friction d =sag
are in equilibrium, then they must be L = span or distance between L=2x
cp = angle of friction
concurrent supports
F1 w = unit weight or load per unit
·X= c.ln S+ Y
What is a Parabolic Cable? length
c
T = maximum tension (usually at
Fz A cable is analyzed as a parabolic cable the support)
when the loading is uniformly distributed H = minimum tension (usually at E. Length, S
horizontally throughout the cable. the lowest point of the cable)
111 1 ~~~
length= 2S
What is a Catenary?
F3

3. Analytical condition: If forces or


vectors are in equilibrium, then 'it must
w JTIITffiUTl.!!J
· S =length
A cable is analyzed as a catenary when
the loading is distributed along and
throughout the cable. The word "catenary"
S =L + 8d2 32d4
3L - 5L3

satisfy the three static equations; comes from the latin word which means
namely

- -
"chain". It is a graph of the equation y:::: If the cable has uneven supports, the
L =span cosh x. formulas to be used are the same, only
}:F, =0 that all unequal dimensions will now
y-axis contain subscripts 1 and 2. For example,
Analyzing half of the cabl~;:

LFy=O i = l the distance from the origin to the left


support is now taken as x1 rather than x,
and xz for its distance to the right support,
LMx =0 0=0 T
y r and so on.

What are the Centroidal Moment of


_..,.._ ltU'Jio H Inertia of Common Figures?
What is Friction?
!...... ---
X X x-axis
U2 Moment of inertia is also called the
Friction is defined as the limited amount second moment of area.
of resistance to sliding between the a) Tension at the lowest point, H: A. Distance from the x-axis to a point on
surfaces of two bodies in contact the catenary, y: Centroidal moment of inertia is the
moment of inertia of the figure with respect
w H= (j)t: to an axis passing through the centroid.
p Y2 = 8 2 +c2
ad
A. Rectangle y
b) Tension at the support, T:
II Maximum Tension, T:
Like a parabolic cable, the maximum Centroid
tension in a catenary occurs at the
support hll 01

T=-my
N R ~
b
-
t
~I
1~

468. 100l'Solved Problems in Engineering Mathematics (2nd Edition) by Tiong & Rojas
'·~
~.
Day 19- Engineering Mechanics (Statics) 469

What are the Moment of Inertia of where: m = mass of sphere


i <muote:
.
ilx=f2
i)h3·
, . b~h .
...........
•. Y -.1'2·.
Common Figures with respect to the
Base?
j
~
I

lit B.
r = radius of sphere

Spherical shell
"How happy the lot of the mathematician.
He is judged solely by his peers, and the
·.• standard is so high that no colleague or
~
A. Rectangle rival can ever win a reputation he does not
B. Triangle deserve."
-W. H. Auden

' JD.
llii!IWI

1~??:~
1=_g_mr 2
[
*I Will~~~ 3

[
i,llllllllil~l

l~!~lli~l~i~l
.. b ·-,. .-b-aXIS
. ,:':"

L." ~'" '


....._:
where: m = mass of sptrere

t
~--.,
r = radius of sphere
·. bh3 t=bh$
\,,,.,., ).

TOaA
'~ .. 36 . 3 . C. Cylinder

.I
C. Circle B. rt(O·.·
............
... ....

1::: 1 mr·
.
2
'' ••,::/1-·.}~A > ~ a.,,:,

--~----~---1---- X
b-axis
b where: m = mass of sphere
r = radius of sphere

Proceed to the next page for your 19th


•'X "'·'')' 1tr•
1rt)"
Jx··.=-.::~ test. Detach and use the answer sheet
4· 64 For composite figures and for axis not at provided at the last part of this book. Use
the centroid nor at the base, moment of pencil number 2 in shading your answer.
where: r = radius of circle inertia may be calculated using the
D = diameter of circle transfer formula, which is as follows; GOOD LUCK I

D. Ellipse y
L=l8 +Ad2
mrtbia:
where: d = distance from the centroid
to the axis
Did you know that ... the integral sign J
I .... • \
~ X
A = area of the figure and elongated S denoting sum (Latin for
"summa"), was introduced by Gottfried
What are the M~ss Moment of Inertia of Wilhelm Leibniz, who named integral
Common Solids? calculus "calculus summatorious" !
... the definite integral which is defined as
A. Sphere the integral between two values of an
,rab3
I'· _'lrt!~b
,1 independent variables is also known as
lte '4··· r--;r .. "Riemann Integral" after the German
mathematician Georg Friedrich Bernhard
where: a = length of semi-major axis ,l:~:~~yf Riemann (1826- 1866)!
b = length of semi-minor axis

l
Day 19- Engineering Mechanics (Statics) 471

:i A. 19.47° downstream 810: A beam rests on a fuicrum, 1.2 m

~~I B. 19.47" upstream from one end. A weight of 350 kg is


C. 18.43° downstream suspended from this end causing the
D. 18.43° upstream beam to balance. If the weight of 350 kg is
suspended on the opposite end of the
Topics 80&: EE Board October 1997 beam, it is necessary to suspend a 1000
A 100 kg weight rest on a 30° incline kg weight on the first end in order to effect
Engineering Mechanics plane. Neglecting friction, how much pull an even balance. Find the length of the
[
~~~~II

must one exert to bring the weight up the beam. ·


tlln~~~~~
Mon Branches of Mechanics plane?

[
,.,qi<~il
Conditions for Equilibrium A. 2.48 m
14!~1~~~~
DTue
Friction
Parabolic Cable
A.
B.
C.
88.67 kg
100 kg
70.71 kg
B.
C.
D.
3.24
3.43
4.21
m
m
m
Catenary
L.""""''
l:. , D D
Theory Wed
Centroidal Moment of Inertia
Moment of Inertia with respect to
I I
I
D. 50 kg

807: ECE Board November 1998


A block weighing 500 kN rest on a ramp
8U: EE Board October 1:991
A simply supported beam is 5 meters in
length. It carries a uniformly distributed

~ D the base
inclined at 25° with the horizontaL The load including its own weight of 200 N/m
Transfer Formula force tending to move the block down the and a concentrated load of 100 N, 2
Thu

o· o
Problems Mass Moment of Inertia ramp is _ _ _ . meters from the left end. Find the
reactions if reaction A is at the left end and
A. 121 kN reaction B at the right end.
Solutions Fri B. 265 kN
C. 211kN A. RA = 81 0 N & Rs = 700 N

D D
Notes Sat
D. 450 kN

808: CE Board November 1:994


B.
C.
D.
RA = 700 N &
RA =810 N &
RA = 700 N &
Rs
Rs
Rs
= 800
= 780
= 810
N
N
N
A 200 kg crate impends to slide down a
ramp inclined at an angle of 19.29° with on: A beam of span "x" meters with
80J:r Three forces, 20 N, 30 N and 40 N the horizontaL What is the frictional uniform loading of "w" kilograms per meter
A. 332.5 N
are in equilibrium. Find the largest angle resistance? is supported at one end (A) and a distance
B. 323.5 N
they make with each other. of 2m from the other end (B). Find the
C. 313.5 N
A. 612.38 N reaction at support A.
D. 233.5 N
A. 104.48• B. 628.38 N
B. 105.58• C. 648.16N wx 2
8041 ECE Board November :1998 A.
c. 106.69° D. 654.12 N 2(x-2) kg
A load of 100 lb is hung from the middle of
D. 107.96° a rope, which is stretched between two
809: EE Board October 1:99:! wx(x-4)
rigid walls 30 ft. apart Due to the load, the B.
80:&1 ME Board October 1:996 A man can exert a maximum pull of 1000 2(x-2) kg
rope sags 4 feet in the middle. Determine
Two forces of 20 units and 30 units act at N but wishes to lift a new stone door for
the tension in the rope. wx(x-2)
right a!)gle. What is the magnitude of the his cave weighing 20,000 N. If he uses a C.
lever, how much closer must the fulcrum 2(x-2) kg
resultant force? A. 1651bs
be to the stone than to his hand? wx
B. 1731bs D.
A. 36 C. 1941bs 2(x-2) kg
B. 42 A. 10 times nearer
D. 1491bs
c. 40 B. 20 times farther
C. 10 times farther
D. 44 sos: A boat moving at 12 kph is crossing D. 20 times nearer
a river 500 m wide in which a current is
so:sz What is the magnitude of the flowing at 4 kph. In what direction should
resultant force of the two forces 200 N at the boat head ifit is to reach a point on tho
20° and 400 N at 144"? other side of the river directly opposite its
starting point?
47_? 100 r Solved Problems in Engine~~ing Mathematic_;U?n<~.~Tiong & Rojas Day 19- Engineering Mechanics (Statics) 473

Sll!U When one boy is sitting 12 m from tU7: ME Board March 1998 821: EE Board October 1991. 825: EE Board October 1993
the center of a see-saw, another boy rnust Assume the three force vectors intersect at A certain cable is suspended between two A copper cable is suspended between two
to sit on the other side 1.5 m from the a single point. I supports at the same elevation and 500 ft supports on the same level, spaced 600 m
center to maintain an even balance. F1 ::: 4i + 2j + 5k apart, the load is 500 lbs per horizontal apart. The cable hangs under the influence
However, when the first boy carries a(1 F2 = -2i + 7j -3k foot including the weight of the cable. The of its own weight only. Under these
II additional weight of 14 kg and sit 1.8 m F3 = 2i- j + 6k sag of the cable is 30 ft. Calculate the total
length of the cable.
conditions, it is desired to calculate the
maximum sag (at the center of the span)
from the c,enter, the second boy must What is the magnitude of the resultant
move to 3 m from the center to balance. force vector, R? when the maximum stress in the material

(
oi<llll~l 2
Neglecting the weight of the see-saw, find A. 503.76 ft. is 1000 kg/cm . The cross-section of the
"ffi iilll~~l the weight of the heavier boy. A. "14 B. 502.76 ft =
cable is 1. 77 sq. em. Weight of cable 1.6

(
~~~.~ 11~~1JJI B. 12 C. 504.76 ft kg/m. Use parabolic equation.
A. 30 kg C. 13 D. 501.76ft
.. ~~~~~1:!11
B. 42 kg D. 15 A. 42.26 m

L. .,lllMII
C.
D.
34 kg
45 kg 1111:111: EE Board March :l998
822: EE Board April1994
A ·cable supported at two points of same
B.
C.
43.26m
44.26m

~~"'
Given the 3-dimensional vectors: level has a unit weight , ro of 0.02 kg per D. 45.26 m
8:14: CE Board November 199ft A = i (xy) + j (2yz) + k (3zx) meter of horizontal distance. The allowed
A 40 kg block is resting on an inclined B = i (yz) + j (2zx) + k (3xy) sag is 0.02 m and a maximum tension at 826s A cable weighing 0.4 kg/m and 800
plane making an angle of 20° from the Determine the magnitude of the vector the lowest point of 1200 kg and a factor of m long is to be suspended with a sag of 80
horizontal. If the coefficient of friction is
0.60, determine the force parallel to the
sum lA + Bl at coordinates (3,2, 1) safety of 2. Calculate the allowable
spacing of the poles assuming a parabolic
· m. Determine the maximum tension.

incline that must be applied to cause cable. A. 414 kg


impending motion down the plane. A. 32.92 B. 420 kg
B. 29.92 A. 64.02 m c. 416 kg
A. 82 C. 27.20 B. 66.37 m D. 400 kg
B. 77 D. 24.73 C. 67.76 m
C. 87 D. 69.28m 827: A cable weighing 60 N/m is
D. 72 819: At what angle does the force F = suspended between two supports on the
6.23i- 2.38j + 4.92 k N makes with the x- 82:J: A cable carries a horizontal load of same level at 300 m apart. The sag is 60
815: EE Board Oc:tober :1997 axis? 20 kg/m. Neglecting its own weight, find m. Compute the distance of the lowest
A 250 ib block is initially at rest on a tlat the maximum tension on the cable if the point of the cable from the ground level.
surface that js inclined at 30°. If the A. :39.2° distance between the supports is 100m
coefficient of kinetic friction 0.30 and the B. 40.2° and the sag is 5 m. A. 205.5m
coefficient of static friction is 0.40, find the C. 41.3° B. 196.8m
force required to start the block moving up D. 42.2° A. 5099 kg C. 200.5m
the plane. B. 50591<g D, 188.2 m
s:zo: ME Board Oc::tober ll99& C. 5199 kg
A. 1901b Assume the three force vectors intersect at D. 5215 kg 8:18: Find the location of the centroid of
B. 2121b a single point. the composite area consisting of a 10-inch
C. 125 lb F1 = i + 3j + 4k 824: CE Board May 1993 square surmounted by a semi-circle. The
D. 751b =
F2 2i + 7j- k Determine the sag of a flexible wire cable centroid of a semicircle is located 4r/37t
=
F3 -i + 4j + 2k weighing 60 N/m over two frictionless above the base (diameter) of the semi-
8161 A 600 N block rests in a surface What is the magnitude of the resultant pulleys 100 m apart and carrying one 10 circle of radius r.
inclined at 30°. Determine the horizontal force vector, R? kN weight at each end. Assume the weight
force P required to prevent the block from of the cable to be uniformly distributed A. 6.0 inches from the bottom
sliding down. Angle of friction between the A. 15 horizontally. The cable extends 5 m B. 6.2 inches from the bottom
block and the inclined plane is 15°. B. 13.23 beyond each pulley to the point they are C. 6.4 inches from the bottom
C. 14.73 attached to the weights. D. 7.0 inches from the bottom
A. 160.75 N D. 16.16
B. 198.55 N 1\ 7.2 rn
C. 164.60 N II /Am
D. 190.45 N c I G rn
ll r 11 rn
47 4. 1001 ·solved Problems in Engineering Mathematics (2"d Edition) by Tiong & Rojas
4
8%9: EE Board March 1998 D. 2304 in
~ Electrical loads are arranged on horizontal
'Ill x, y axes as follows: 833: EE Board March 1998
il An isosceles triangle has a 10 em base
Load X- y- Kilowatt and a 10 em altitude. Determine the
~I coordinate coordinate load moment of inertia of the triangular area Topics
1 0 2 100 relative to a line parallel to the base and

(
d~ijj!I~I[IJ 2
3
1
1
1
3
180
200
through the upper vertex in cm 4 .
[QJ Engineering Mechanics
Branches of Mechanics
' ltl~i~~l!ll A. 2750 tvlon
2
(
1111'~11~~~11 4 0 120 Conditions for Equilibrium
0
B. 3025
111~11111~~111
5 2 4 150 C. 2500 Friction
6 3 1 200 D. 2273 Parabolic Cable
Tue
L.,,,l""'" Catenary

~"'"""' --
7
8
3
4
3
2
180
100
834: ECE Board Apri11999
What is the moment of inertia of a cylinder
of radius 5 m and mass of 5 Kg?
0Theory
0
Wed
Centroidal Moment of Inertia
' ' '

Moment of Inertia with respect to


A.
B.
C.
D.
X= 2.000, y = 2.049
x=2.163,y=2.195
X= 1.854, y = 2.211
X= 2.146, y = 1.902
A.
B.
C.
120 kg-m 2
80 kg-m 2
62.5 kg-m 2
0
Problems.
0Thu
the base
Transfer Formula
Mass Moment of Inertia

0
. 2
D. 72.5 kg-m
830: A rectangle has a base of 3 em and
a height of 6 em. What is its second 835: ECE Board April :1998 Solutions Fri
moment of area (in cm 4 ) about an axis What is the inertia of a bowling ball (mass
through the center of gravity and parallel to
the base?
= 0.5 kg) of radius 15 em rotating at an
angular speed of 10 rpm for 6 seconds? 0Notes
0Sat
2
A. 64 A. 0.001 kg-m
B. 34 B. 0.002 kg-m 2
C. 44 C. 0.005 kg-m 2 ANSWER KEY RATING
D. 54 D. 0.0045 kg-m 2
801. A 811. A 821. c 831. B c:J 30-35 Topnotcher
831: EE Board March 1998
A circle has a diameter of 20 ern.
802.A
803.A
812. B
813. B
822. D
823.A
832. D
833.c c:J 21-29 Passer
Determine the moment of inertia of the
circular area relative to the axis
804.c
805. B
814.c
815. B
824.
825. A
c 834.c
835. D
c:J 17-20 Conditional
perpendicular to the area through the
center of the circle in cm 4 .
806. D
807.c
816. A
817. B
826.
827. B
c 0 0-16 Failed
808.c 818. B 828.D If FAILED, repeat the test.
A. 14,280 809. D 819.c 829. A
B. 15,708 810.C 820.A 830. D
C. 17,279
D. 19,007

83Z: ME Boa,.d October 1993


The moment of inertia of a section 2" wide
x 2' 0" high about an axis 1'0" above the
bottom edge of the section is:

A. 1834 in 4
B. 384 in 4
C. 9214 in 4
iI

476 · 100 1 Solved Problems in Engineering Mathematics (2nd Edition) by Tiong & Rojas Day 19- Engineering Mechanics (Statics) 477
:E Finclined = 0
~,I
'II
il
II
For equilibrium, the force polygon must
close.
T L - ..
30ft

l
w
P = Wsine
P = 100sin30°
p =50 kg
F=P
F = 200(9.81)sin19.29°
P =648.15 N

I Ill 1m
(
II!IUI~~llill

tl'~ ~~~k4~11
("l~lli11*1141il 40N 1001bs
By cosine law: :EFv = 0
a..~~~~~lli~~M 2Tcos9=100
2 2 2
( 40) = (20) + (30) - 2(20)(30)cose
T=~
L""'."'" =
e 104.48 2cose

\.,. ,,, 1!1 T= 100


2cos75.068° .-..-,

-~0 •
T=1941bs p -25°! 500 kg

.G .

' 30
4 kph
t)
........· . ·.
. LMFulcrum = 0
20000(x 2 ) = 1000(x 1 )
_20_0_0_0x~2
The component, P along the plane of the x1 = 1000
force 500 kg is the force that tends to x1 = 20x 2
R = J(20)2 + (30)2 move the block down the ramp.
R = 36 units Thus, the fulcrum must be placed 20
4 :E Finclined =0 times nearer his hand than the stone
Ill sinB=-
12 P=Wsine door.
.e = 19.47" P = 500sin25°
P=211 kN a
II 100 kg
II 350 kg We

F1=200N X

By cosine law: Let: We= weight of beam

R~ = 200 2 + 400 2 - 2(200)(400)cos56° LMFulcrum =0


R=332.5N W8 (0.5x -1 .2) =350(1.2)
· 3o•....--:
1OOsm \ 30"!1 00 kg Let: F = frictional resistance
We = 420 7 Eq. 1

~
Ill n .p
P = component of the weight of
crate along the inclined
O.Sx-1,2
"15
tane=-
. Lf~ndlnod =0
4 .
..... -············ \
9=75.068°

1
~

I
11

478 · 100 l·Sclved Problems in Engineermg Mathematics (2"a Edition) by Tiong & Ro~. Day 19- Engineering Mechanics (Statics) 479
X w = 40 kg.
'I
·~
ll
1000 kg

,
W8

0.5x
x-1.2
350 kg

~ H: 2- 2
fl
,t
e = 20°
I
2:Fy = 0
N=Wcose
N = 250cos30°

U:U~ !1.
1 i
---····,· wx

~ A?:!
1 N = 216.506 lb
I
M: =0 : Substitute the value of N in Eq. 1:

[
diM~~~

411111~!1~~
l,llil.l'lllll
L Fwenm
350(x-; 1.2) + W8 (0.5x -1.2) = 1000(1.2)
l-
RA i
t x-2
Ra
2 ··
y-axis
p = 125 + 0.4(216.506)
P=212!b
!
(
4 ~~li~~jl~
We= 1200-350(x-1.2) 7 Eq. 2 0.6N =40(9.81)sin20° + P 1m W= 600 N
L,,,.,.,., 0.5x -1.2 RA (x- 2) = WX (X ; 4) P=0.6N-134.208 ~Eq.1
e =~oo
I

~'""'
Equattng equations 1 and 2:
wx(x- 4) kg
RA = 2(x-2
LFY =0
~= 1200-350(x-1.2) N=Wcos20°
~
0.5x -1.2

350x =1200
0.5x -1.2
420 = 1200 - 350x + 440
- w,
!12m i 1.5m
I
We

::J
N = 40(9.81)cos20°
N = 368.735

x=3.43 m t: ::::::;; ::; ·


:: :~;; ~:;;
t
: Substituting the value of N in Eq. 1: y-axis
.3 2:Fx = 0
EDI L~=O
F p = 0.6(368.735)-134.208
P = 87 Newtons
Pcos6+F=Wsin8
Pcos6+f.1N=WsinEl
"'{8 (1.5)- WA (1.2) =0
WB =0.8WA
a W=250 lbs.
Pcos30° + tan15° (N) = 600sin30°

A f . :: ::5 ~::;:::::•:: :~ 8 WA +14 We


e =30°
N=
300-0.866P
0.268
~Eq1

RA Re 11.8 m
t;;;;:~;;;:·~~7~-;;;-:;w~d
3.0m ! L;Fy = 0
2:MA =0
N = Wcos8+Psine
100(2) + 1500(2.5)- R8 (5) = 0 F
N = 600cos30°+ Psin30°
R8 =700 N LMF=O y-axis N = 519.6 + 0.5P ~ Eq.2
2:M8 =0 W8 (3)- (WA + 14)(1.8) = 0 '-'F
L~ X = 0
RA(5) -100(3) -1500(2.5) = 0 (0.8WA)(3)-(WA +14)(1.8)=0 Equating equations 1 and 2:
RA =810 N 2.4WA -1.8WA -25.2 = 0 P =Wsin6+F
P = WsinEl+f.1N 300-0.866P =519.6+0.5P
WA =42kg
Ill P = 250sin30° +0.4N
0.268
P = 160.75 N!3wtons
LMs=O Ill Pcc125+0.4N ~ Eq.1

RA(x -2) = wx(~-2) 2:Fx =0 Note: Since in the condition of the


problem, the block is to start moving, then
F = WsinEl+P
''~":tile cooftrcient of static friction.
f.1N=WsinEl+P
480 100 l Solved Problems in Engineering Mathematics (2"d Edition) by Tiong & Rojas ----------------------------~D~a~y~i~9~-~E=n~gtn~·~e~e~nn~·~M~~tat~~

11
'Iii'
,111

II
I~
a
R=F1 +F2 +F3
R = (4i+2j+5k)+(-2i+ 7j -3k)+(2i- j+6k)
R =4i+8j+8k
r = ../6.232 +2.382 + 4.922
r=8.287

d = J2.38 2 + 4.922
d-5.465
Ell
L=69.28 m

roe
H=--··
8d
2
• S=A
F=SA
F

. 1000 kg
F =--err?
x 1.77 cm 2
H= 20(100)
IRI=F~r~(Aj)
2 2 8(5) F = 1'770 i<g
(
~fUtlll~lllili~
+(Ak) By cosine law:
H=5000 kg
~ lU~~ili~l\
IRI =J42 +82 +62 Note: This force is equivalent to the
(
lt:II~~Wil!l~l!
d2 =r 2 + x 2 - 2rxcose tension (i) at each support.
T=~Lr +~
IRI = 12 units
1-il~l ~~~il~ I
5.465 2
·= 8.287 2
+ 6.23 2

L. "~'" " 1!11!11


liMill
- 2(8.287)(6.23)cose
cose = 0.7517339
J( 20(~00) J
T2 =( ~!: +H2
l2)
y
l"~'"'
Substitute the values of x,y and z to the T= + 50002 . 12


(:) = 41.3°
two vectors: ii! 17702"" [.1.6(:00lj +H 2


T=5099 kg
A= i(xy)+ j(2yz)+k(3zx) H =1703.67 kg
A= K3)(2)+ j(2)(2)(1) + k(3)(1)(3) R=F1 +F2 +F3
ro=60N/m
A=6i+4j+9k R =(i+3j+4k)+(2i+ 7j-k)+(-i+4j+2k) 11 ,..roe
R=2i+14j+5k 8d
B = i(yz) + j(2zx) + k(3xy)
1703.67 "' .~~~00)
2

~s~
B = K2)(1) + j(2)(1)(3) + k(3)(3)(2) 2 2 8d-
B=2i+6j+18k IRI= J(A;) +(AJ) +(AS d"' 42.26 m
IRI = J22 +142 +52
A +B = (6+2)i +{4 +6)j+(9+18)k
IRI = 15 units


A+B =8i+10j+'27k

IA+BI =J(A;t +(Ai +(AS

lA +BI = Ja 2 2
+10 +27
2
Let: S = length of the cable

2 4
T '= 10000 + !30N (5m)
T =10300 N
m
~···tao
8d . 32d
lA + Bl =29.88 units S = L + - - -3- c
3L 5L 2

Ell s = 500 + 8(30)2 - 32(30)


4
T2 = (mL)
~ +H2
3(~00) 5(500) 3 \ 2
2
s = 504.76 ft


103002 =[60(100)]
--2-. +H2 T=roy
4.92
H=9853.42 N T = 0.4(130 +c) ~ Eq. 1
J
11
'J·········~~axis roL
H=-
2
H= ool2
8d 2S =800
8d

e =H[~J
S =400 m
~ ~0(100)
2
9853.42
8d- y2= 8 2 +c2
L2 =' 1200[8(0.02)]· d= 7.6 m
2 0.02 (80 + c)2 = 400 2 + c2
L2 = 4800 6400 +'2G + c 2 "'400 2 + c 2
Co,960 m
48~ 100 l Solved Problems in Engineering Mathen1!ticsJ.?..::=..~iticm) by 'f!ong & ~5?1!!. ------- Day 19- Engineering Mechanics (Statics) 4~3

1
-~~

I
il
Substitute the value c in Eq. 1:

T = 0.4(80 + 960)
T=416 kg
Ar =

Ar = 10

Ar = 139.27 in 2
2 (
+ Asomi-cil'"...im
·1 )( n: \1
+l:Z l4)10)
2
bh 3
1x.. =--bhd2
4

Substitute values:
• 1 2
1=-mr
2
I= _!(5)(5) 2

. x=150
.
• x=150 .
ArY =A1Y1 + ·7 Eq. 1 ix., = 3{6)3
3 - 3(6)(3)2
2
I= 62.5 kg-m 2
1Xo =54m 4
'---~ )>,

60
wher-e:

'J2==10+
5 in.

3n
rm
Y2:::: 12.12 in. I= 3.mr2
let: J = polar moment of inertia 5
Substitute Y1 and Y2 in Eq. 1: nd 4 I= 3_(0.5)(0.15)2
J=-
c 32 5
139.27y = 100(!)) + J"' n(20)
4
I= 0.0045 kg-m 2
-~~ri!)

y = 7 inches from the bottom 32


~)li-~~--JWA&~~ft!!t.,$1 L , $
J = 15708 cm 4

S+y
X=Cin--
C
X = _..!.._..!____.--"._e______, __ , _________.!!._.!.!.
S + (60 +C) bh 3
150=cln-~--- ->Eq. ·1 Ll +L2 + lx=--
c 12
x~~O):_!OO(~+;;m:1)+1ax2'J+'~!~+;m;3)"':100:3~~~
1C0+100+2Xl+ 1:tfJ+ ito+ilJD+1lD+'!CO lx =~(24[
Yz = 8 2 +c2 X=2 12
(60+c) 2 =S 2 +c 2 lx =2304 in 4
3600 + 120c + c 2 = S 2 + c 2 y =- '-p 1 :~.::..~~-~~---~-:.:;:.L~.:_:~~·-:.:.:._r_:._-:~:_~l
L1 + L2 + L 3 ......L,
s == FGoo .; 120c ---'*' Eq. 2
- 1~) + 10C(1l::.?Q:5~+E()\0)+ ~4)+20Q£1l_+'l80(3) +.!_C0:2)
Y- ®+®+2Xl+~+~+2Xl+®+~ ....~,.·································:Jr.····················~.········ X1
Substitute Eq. 2 in Eq. 1:
y =2.049
2
= cln ../3600+120c + (60 +c} d =-h
150 h = 10 em 3
c
By trial and error: c = 196.8 m
l'l!'R
~ h=6
b = '10 em
)(
4r !Xl =co ivn + Ari

Using transfer-axis moment formula:


,
L-1'·~-~--+-bh
br1 3 . (2
-hi
,=,
r. ~lb 2 3 )

':'_!'_
10 in.
J': lx =i Xo +
oh
4
3
.L\.cf \ 1"-,
_..

2:' ;uul:lil'
-1-
1
2(10)(10) 3(10) (4 )l

j
r ~

486 ·1001 Solved Problems in Engineering Mathematics (2nd Edition) by Tiong:_ & Rojas

I
'~I

I I' Topics

0
Mon
Rectilinear Translation
- Horizontal Translation
-Vertical Translation
~
Tue
-Free Falling Body
Curvilinear Translation

''"~~~~~~
Theory
0
Wed
- Projectile
-Rotation
D'Aiembert's Principle
Centrifugal Force
Banking of Highway Curve
Probien1s

Soluttons
[]
Fri

Notes
[]
Sat

i :cctil!near translation is a motion in a


!llane or one dimension.
r Not this path

ca::::·.::·.-..:. . . . . ,....,:::::::}:::::::·····. .+::::~.·.


This path
i l1.e are l:h<" of rectilinear (straight line)
11 ;m::,lalion:
Top View
1. Hori2:ontGI transiation
2. Vertical translation Vo a=O v
.............)!»-
3. C;Jiviline;;;,r ~

11 ., u nnl;1l tr·;;n~;laiion (motion) is a straight


~~l~•illl)l :llui'ifJ ~~ hcni.;-:ont;]j ~;\a;J;;;_
s
w1,.,,., dllH• ~·· ···j,·,· {, ri: rr:~5·'- :1nc,~tv
·i iliG [ :,,, l·y' H 'f '! Elevation View
r
·"
t •jj '9jd;l'l/) 1 1\111

I
..
,,
'ii

488 1001 ·solved Problems in Enginee~~g Mathematics (2nd E@;ion) by Tiong & Rojas Day 20- Engineering Mechanics (Dynamics) 489

1J V=Vr:;
as it goes down and decreases as it goes
up.
• t Vo=O
Since the ho'rizontal component of the
velocity is constant throughout the
II,, projectile, then the Value Of X iS calculated

S=Vt ·-~Vo ~g
as follows:

I y
x =V0 coset
where: Vo = initial velocity (time = 0)
V = final velocity (time = t) : a
'f
(
••IIWI~~II1

S ::: distance traveled after y So the time of flight, t is:

-~v
....
IIIIIN~~~~~~~~~
,~,
elapsed time = t

[ llll~ij/·~11!

111~~111~1~
If acceleration is constant, the initial
velocity, Vo is not equal to the final
velocity, V. ·-~v ' 1
'/=2gt2
t=~
V0 cos6

~'"'""
The following formulas will be used: In the figure, the value y is obtained using
Vo a v the formula:
............. -~
~~~lil'
,~
What Is a Curvilinear Translation?

Curvilinear translation (motion) is a motion


v ~J • ""';.; 1:;~
·, ""'':VIfSII'I:<>•'""-·l;l~· ,
. . ' ,2
V""Vo:t:'ati along a curve path either on the vertical
____:__J plane or on horizontal.plane. This type of
motion includes a trajectory or projectile, Substituting the value of the time of flight
s which is a curve path on the vertical plane "t" in the above equation:
The following formulas will be used: '""~~" and rotation on horizontal plane.
2

= "v,0 Sin
. O -X- +-
1 --
V 7 =V0 ~ ;:J:2a$ where: Vo =initial velocity (time= 0)
V = final velocity (time = t)
A projectile is a body which after being
given an initial velocity with an initial angle y (V cose )-· 2 (V cose J
0
9
X-
0
S = distance traveled after of release is allowed to travel under the
V = V0 :tat elapsed time = t action of gravity only. A projectile is a Simplifying the above equation will
a = acceleration trajectory which is a graph of a parabola. produce the "General Equation of
Projectile".
1
S ""V0 t±:o-at2 Note: Use + if going down and - if going v ....................
' .2 up. (O,O).x.···· "··-.. ' (x,y)

where: Vo = initial velocity (time = 0) If the acceleration of the body is not given, . . . . .• X j;·,'·., ..····.....
V = final velocity (time = t) then it is presumed that the acceleration to ""'!!!lll!lllll!llii!I-.,4~W.flll\'!i~Y 1 J.!ii!i:t'#/Z;;;;pi @ ~¥k ~
S = distance traveled after be used is the acceleration due to gravity Since the variable "x" is in second degree
elapsed time = t (gravity on earth). while the other variable "y" is in the first
a = acceleration Let: v.
and Vy be the horizontal component degree, then the graph is a parabola.
g = 9.81 m/s2 and the vertical component, respectively of
Note: Use + if accelerating and - if g = 32.2 ftls2 the initial velocity, Vo. Note: The vertical component of the
decelerating. velocity decreases as it goes up and is
zero at maximum point of the projectile
What Is a Free Falllnp Body?
kVo
v. = V 0 cose and increases as it goes down, while the
yvhat Is a Vertical Translation? horizontal component is constant.

Vertical translation (motion) is a straight


motion along a vertical plane.
A body is said to be free falling if the initial
vertical velocity is zero. The acceleration
of a free falling body is due to gravity
tJilfrv. Vv =V0 sine If the initial point of the projectile is on the
same elevation as the point of impact, the
which is equal to 9.81 m/s2 or 32.2 ftls 2 . following formulas may be used:
In this type of motion, acceleration cannot
be equal to zero since the velocity of a
body rnoving in vertical motion, increases
490 ·1 Ob l Solved Problems in Engineering Mathematics (2nd Edition) by Tiong & Rojas Day 20- Engineering Mechanics (Dynamics) 491

11
I
I(
Highest Point

...····················•·······················...
Wh;;it is a Rotation?

Rotation is a motion along a circular path.


i
"1"'
,'~
'{7
force mentioned in D'Aiembert's
nrinciple is known as the reverse
,. ;r:fe:<r;tiv® force.
where: W =weight of body
V = tangential velocity
r = radius of curvature
g = gravitational acceleration

r; Vo . •••
/'\e
Ymax •·••••
···... -....
. ...... ;,.. a

F".J,p.~~'<'Wb~!\'t~v«;'"'1J,'i~'"i11,-W('"&!~ '!:f,-"?'ry..'<'"""!-:;,.">~''7~"-"J1<_'-,,Y~ ""-:oW>->"' 'T'~~ -"'"(~": -'>4\'


..
Fe~-
Initial Point Impact Point ~REF
(
diM1 1 I!~IIII

.'\ "(
f•jllii!III\WI
Xmax =Range = R
rJ1i~~~~lfil!l s reverse effective force is equal to the F
f... ,~lll~~~~~lll Maximum height, Ymax: ,,,nduGt of mass and accelertion.

L. , . ., ., ''· !,\i.;;.-~themat.~cally, Elevation view


"· If there is no force other than the force due

~ ....: Range, Xmox or R:


Let: S
V
=linear distance in meters
= linear velocity in m/s
i- REF ==ma or
REF::: W a
~~
to friction between the tires of the car and
the pavement of the road to resist the
centrifugal force, then:
a ::: linear acceleration in m/s2
R"' Vo 9in2El
2 e = angular distance in radians Fe =F
m = angular distance in rad/s
g
a ::: angular acceleration in rad/s2 ·-
g ::-c acceleration where: F = force due to friction
If the initial point of the projectile is not on
the same elevation as the point of impact, s,.. wf!
the following formulas for ran!=)e, R may be Why is Banking of Hl!it~l_"!\' go. !D'_~
used: f:C'lrC~0
is the force thRt will necessary?
Highest Point v,,no 1''1 movinq in a circular o.8th
fmm U1e center. In order for the weight of the carlo help
...···············:··""········ Impact Point the frictional force resist l.he
~"'=ret force, banking of highway curve is
Vo ·\e necessary. Once banked, a car can move
._,L. _t.lt,. .,.,_,.,
Initial Point
For variable anqulat aceeleration, the
following formulas must be used.
at higher speed due to bigger resistance of
the centrifugal force.

ui "'w02 ±2a6
R _ 2V0 2 sinecos(e + ~) "I
- gcos2 p ro"" ro 0 ±at

Highest Point 1
9 ""ro0 t ±

~Q~eJ§.mbe!f~ rap view ot"


c1rcular track
D'Aiembert's principle is stated as follows: The ideal angle of banking, e may be
calculated l!Sing:
"When the body is subjected to an I ·II' centr if11gal force
2
acceleration, there exists a force opposite
tane"' v
R "'~V,/ si~eoos(e-~) the direction of !he mo!ion and equal to lhA
I,
1/V\11
gt
gcos2 13 product of mass and acceler~:.~tim;" 'JI
492 100 l Solved Problems in Engineering Mathematics (2"d Edition) by Tiorig & Rojas
For greater velocity without skidding,
1\,
I, tan(e + cl>), V
2

~
gr
Topics
=ll
(
dillllllll~lm
1

Utliljl11ill\lllll
tancjl

where: cjl = angle of friction


DMon
Rectilinear Translation
- Horizontal Translation
ll = coefficient of friction -Vertical Translation
[QJ
,111 ..~1~~111

I - Free Falling Body


I
(
<4 ~~~~~1111 L'l

Proceed to the next page for your 20th I Tue Curvilinear Translation
L.,.. -Projectile
D D
IIIIIIIIO!I test. Detach and use the answer sheet

~ •.,.,1
provided at the last part of this book. Use -Rotation
pencil number 2 in shading your answer. Wed D'Aiembert's Principle
Theory
GOOD LUCK I
~D
Problems Thu
Centrifugal Force
Banking of Highway Curve

'QI;ribia:
Did you know that ... about 500 B.C., th~
Pythagorean Brotherhood was originally
D D
Solutions Fri
aware of the four regular polyhedrons and
considered them to represent the four
basic elements namely tetrahedron- "fire",
octah3dron- "air", hexahedron- "earth",
D D
Notes Sat
icosahedron - "water"! When the
Pythagoreans learned the existence of the 838: CE Board November J.99fl
fifth regular polyhedron, dodecagon, they s:s«t: ME Board April 1.996 A train passing point A at a speed of 72 '

considered it to represent the fifth element What is the acceleration of a body that kph accelerates at 0. 75 m/s 2 for one
- "universe". 111creases in velocity from 20 m/s tp 40 m/s minute along a straight path then
111 3 seconds? decelerates at 1.0 m/s2 . How far in km
from point A will it be 2 minutes after
~uote: 1\. 5.00 m/s2 passing point A?
"He who loves practice without theory is ll. 6.67 m/s2
like the sailor who boards ship without a (' ·1.00 m/s2 A 3.60 km
rudder and compass and never knows ll. 8.00 m/s2 B. 4.65 km
where he may be cast" C. 6.49km
•H7: ECE November 1.998 D. 7.30 km
- Leonardo da Vinci I low far does an automobile move while its
·.peed increases uniformly from 15 kph to 839: CE Board May J.99fl
•1', kph in 20 seconds? From a speed of 75 kph, a car decelerates
at the rate of 500 m/min 2 along a straight
I\ 185m path. How far in meters will it travel in 45
II 167 rn seconds?
I. 200m
ll 1l2m A. 790.293 m
B. 791.357 m
C. 793.238 m
D. 796.875 m
494 I 00 I Solved Pmblems in Engineering Mathe~ti~ (2"' Edition) by Tiong & -~ Day 20 - Engineering Mechanics (Dynamics) 495

840: CE Board November 1997 B. 19.8 m/s 8491 lEE ha~rd October 199& 8§3: ME ik,lard April :1995,
,~, A train starting at initial velocity of 30 kph C. 21.2 m/s A ball is dropped from a height of 60 ME Board October 199&
travels a distance of 21 km in 18 minutes. D. 22.4 m/s meters above the ground. How long does A shot is fired at an angle of 45° with the
'I, Determine the acceleration of the train at
•.ii
it take to hit the ground? honzontal and a velocity of 300 fps.
1(' this instant. 845: ECE Board April 1999 Calculate, to the nearest value, the range
«; A. 0.0043 m/s 2
·A ball is dropped from a building 100 m
high. If the mass of the ball is 10 gm after
'J~
'
'il'
A.
B.
2.1
3.5
sec
sec
of the projectile.

B. 0.0206 m/s 2 what time will the ball strike the earth? C 5.5 sec A. 932 yards
C. 0.0865 m/s 2 D. 1.3 sec B. 1200 yards
D. 0.3820 m/s 2 A. 4.52 s C. 3500 yards
B. 4.42 s ilil:!OO: ECE Board April 1998 D. 4000 yards
841: EE Board October 1996 C. 5.61 s A baseball is thrown from a horizontal
An automobile moving at a constant D. 2.45s plane following a parabolic path with an 854: CE Board May :1.995
velocity of a 15 m/sec passes a gasoline initial velocity of 100 m/s at an angle of 30° A projectile leaves a velocity of 50 m/s at
station. Two seconds later, another 84&: ME Board AprU :1995 above the horizontal. How far from the an angle of 30° with the horizontal. Find
automobile leaves the gasoline station and A ball is dropped fro, n the roof of a throwing point will the ball attain its original the maximum height that it could reach.
accelerates at a constant rate of 2 m/sec2 . building 40 meters tall will hit the-ground level?
-~'itJII~ How soon will the second automobile with a velocity of: A. 31.86m
overtake the first? A. 890m B. 31.28m
A. 50 m/sec ll 883m C. 30.63 m
A. 15.3 sec B. 28m/sec c 880 m D. 30.12 m
B. 16.8 sec C. 19.8 m/sec n. 875m
C. 13.5sec D. 30m/sec 1!155: ME Board October 1997
D. 18.6 sec ~'ill!ME Board April :11:991 A shot is fired with an angle of.45o with the
847: ME Board April J:'JC):! /\ plane dropped a bomb at an elevation of horizontal with a velocity of 300 ft/s. Find
842: EE Board October :1996 Using a powerful air gun, a steel ball is 1000 meters from the ground intended to the maximum height and range that the
If a particle position is given by the shot vertically upward with a velocity of 80 hit the target at an elevation of 200 meters projectile can cover, respectively.
expression x(t) = 3.4e- 5.4! meters, what meters per second, followed by another from the ground. If the plane was flying at
is the acceleration of the particle after t 5= shot after 5 seconds. Find the initial :1 velocity of 300 km/hr, at hat distance A 800 ft, 1600 ft
seconds? velocity of the second ball in order to meet Irom the target must the bomb be dropped B. 923 ft, 3500 ft
the first ball 150 meters from the ground. lo hit the target Wind velocity and C. 700 ft, 2800 ft
A. 1.02 m/s 2 o~tmospheric pressure to be disregarded. D. 1800 ft, 3000 ft
B . rn/s 2 A 65.3 m/sec
C. 3.4 m/s 2 B. 45. 1 m/sec .'\ 1024.2 rn S$6: CE Board November :1.996>
D. 18.1 m/s 2 C. 56.2 m/sec 11. 1055.6 m A ball is thrown from a tower 30 m high
D_. 61.3m/sec (; 1075.5 m above the ground with a velocity of 300
84~: ME Board October 1!995 u "1064.2 m m/s directed at 20° from the horizontal.
The distance a body travels is a function of 848: EE Board O<.rto~r 1!99:1> How long will the ball tlit the ground?
time and is given by x(t) = 18! + 9f. Find A ball is thrown vertically upward from the 1.1',-;::e,: ME &11!1rd: O~"l:ooor :ll'Jil'\ll1
its velocity at t = 2. ground and a student gazing out of the ll!o muzzle velocity of a projectile is 1500 A. 2'1.2s
window sees it moving upward pass him <Jt Ips and the distance of the target is 10 . B. 22.2 s
J.\. 36 5 m/sec. The window is 'I 0 n1 above the ''Illes. The angle of elevation of ihe gun C. 23.2 s
B. 54 ground. How high does the oa!l go above ""1st be: D. 24.2 s
C. 24 the ground?
D. 20 /\ 21"59' s:n: In the last 2 seconds of NBA finals
2:~ tl i'
0

A. 15.25 m II featunng Chicago Bulls VS Utah Jazz, with


844: CE Board May 1998 B. 14.87 rn ?!lo:r:r the latter ahead by 2 points with the former
Determine the velocity of progress with the C. 9.97 m Jl /h"Hl' at 94-92 count Bulls Michaei.Jordan
D. 11.30 m decides to snoot from a certain point on
given equation: D = 20t + ~ when t = 4 the rainbow territory wt1ich counts 3 points
t+1 if converted. During the play, i(Jorcjan
seconds. releases the ball at 7 m from tne basket
a11d 2. i 5 m above the ground and an
A. 18.6-m/s
I
Day 20 - EngiJl~(!ring Mechanics (Dynamics) 497
496 1001, Solved Problems in Engineering Mathematics (2nd Edition) by Tiong & Rojas

inclination of 40" with the horizontal and ~W1


866: ME Board April1991 8'701 ME Board April 1998
86!.1 ME Board October :199:1.
A boy tied a 80 grams stone to a string Traffic travels at 65 mi/hr around a banked
~II assuming no block was made by the
opponents, at what velocity will the ball be
The flywheel of a puncher is to be brought
to a complete stop in 8 seconds from a
:~1
;!) which he rotated to form a circular motion highway curve with a radius of 3000 feet. ·
I. given to cast the winning basket? The speed of 60 revolutions per minute.
with a diameter of 1000 mm. Compute for What banking angle is necessary such that

~
the pull exerted on the string by the stone friction will not be required to resist the
basket is 10 feet from the ground. Compute the number of turns the flywheel
if it got loose :eaving at a velocity of 25 centrifugal force?
will still make if its deceleration is uniform.
A. 8.57 m/s m/sec.
B. 8.86 m/s
A. 3.2•
A. 5 turns
A. 120 N B. 2.5°
(
*MIIV\~1111~
C. 9.03 m/s B. 3 turns
D. 9.27 m/s C. 4 turns
B. 100 N c. 5.4°
r•ii. ifi~IIII~Wl·

~UIIIIill
8581 CE Board May 1995
D. 6 turns
C.
D.
150 N
135 N
D. 1Po

~ .. ,.ur~l~~l.il~ A projectile is fired with a muzzle velocity 8621 ECE Board April :1998
8711 ECE Board April1999
867: EE Board April :1997
L. ..,,,IJ of 300 m/s from a gun aimed upward at an
angle of 20" with the horizontal, from the
What is the speeq .of a synchronous
earth's satellite-situated 4.5 x 10 7 m from
A man keeps a 1kg toy airplane flying
horizontally in acircle by holding onto a.
Determine the arigle of the super elevation
for a 200 m hi-way curve so that there will
be no side thrust at a speed of 90 kph.

l ..,.:! top of a building 30 m high above a level


ground. With what velocity will it hit the
ground in m/s?
"the earth?

A.
B.
11,070.0 kph
12,000.0 kph
1.5 m long string attached to its wing tip.
The string is always in the plane of the
circular path. If the plane flies at 10 m/sec,
A.
B.
19.17"
17.67•
find the tension in the string. C. 18.32•
A. 298 m/s C. 11,777.4 kph
B. 299 m/s D. 20.11"
D. 12,070.2 kph
A. 28N
C. 300 m/s
B. 15N 8'721 ECE Board April :1998
D. 301 m/s 86~1 ECE Board November :1998
C. 67N The inclination of ascend of a road having
A rotating wheel has a radius of 2 feet and a 8.25% grade is _ _ _.
D. 18 N
8591 CE Board May :1.995 6 inches. A point on the rim of the wheel
A stone is thrown upward at an angle of moves 30 feet in 2 sec. Find the angular
30" with the horizontal. It lands 60 m velocity ofthe wheel.
868t ME Board October :1.996 A. 4.72°
measured horizontally and 2 m below
An automobile travels on a perfectly B. 4.27°
measured vertically from its point of A. 2 rad/s
horizontal, unbanked circular track of c. 5.12"
radius R. The coefficient of friction D. 1.86°
release. Determine the initial velocity of B. 4 rad/s
between the tires and track is 0.3. If the
the stone in m/s. .C. 5 rad/s
car's velocity is ·15 m/s, what is the 8731 ME Board April :1.996
D. 6 rad/s
A. 22.35 m/s smallest radius it may travel without =
A cyclist on a circular track of radius r
skidding? 800 feet is traveling at 27 fps. His speed in
B. 23.35 m/s 8641 CE Bqard Novembeio :1997
C. 24.35 m/s the tangential direction increases at the
A turbine started from rest to 180 rpm in 6 2
A. 68.4 m rate of 3 fps • What is the cyclist's total
D. 25.35 m/s minutes at a constant acceleration. Find
B. 69.4 m acceleration?
the number of revolutions that it makes
8601 CE Board November :1.99Z within the elapsed time.
c. 71.6 m
A wooden block having a weight of 50 N is
D. 7-6.5 m ·A. 2.8 fps2
B. 3.1 fps2
placed at a distance of 1.5 m from the A. 500
8691 CE Board November :1998 C. 3.8 fps2
center of a circular platform rotating at a B. 540
A hi-way curve has a super elevation of D. 4.2 fps 2
speed of 2 radians per second. Determine C. 550
7degrees. What is the radius of the curve
the minimum coefficient of friction of the D. 630
such that there will be no lateral pressure 8'74r ME Board October 1997
blocks so that it will not slide. Radius of the
between the tires and the roadway at a A concrete hi--way curve with a radius of
circular platform is 3 m. 8651 A flywheel is 15 em in diameter
speed of 40 mph? 500 ft is bar;~ked to give lateral pressure
accelerates uniformly from rest to 500 rpm
A. 0.55 equivalent to f = 0.15. For what coefficient
in 20 seconds. What is its angular
A. 265.71 m of friction will skidding impend for a speed
B. 0.58 acceleration?
B. 438.34 m of60mph? ·
c. 0.61
C. 345.34 m
D. 0.65 A. 2.62 rad/s2
D. 330.78 m. A 1-1. ~
0.360
B. 3.45 rad/s2
C. 3.95 rad/s2 .
B. Jl < 0.310
D. 4.42 rad/s2 c. Jl > 0.310
498_ 1001 Solved Problems in Engineering Mathematics (2nd Edition) by Tiong & Rojas · ~.
•...

,,, D. ll < 0.360 A.


B.
19.63 feet
19.33 feet (
',
•,
87'>: EE Board April1993 C. 18.33 feet '.1·1
~~II What force is· necessary to accelerate a D. 19.99 feet
,(' . 30,000 pounds railway electric car a.t the
~~ rate of 1.25 fUsec2 , if the force required to
overcome frictional resistance is 400
880: ME Board April1997
A pick-up truck is traveling forward at 25 Topics

A.
B.
pounds?

1565 pounds
1585 pounds
m/s. The bed is loaded with boxes whose
coefficient of friction with the bed is 0.4.
What is the shortest time that the truck can
be brought to a stop such that the boxes
D
Mon
Rectilinear Translation
- Horizontal Translation
-Vertical Translation
C.
D
1595 pounds
1575 pounds
do not shift?
~ -Free Falling Body
Curvilinear Translation
,.~.,,.J A.
B.
2.35
4.75
s
s
Tue
-Projectile
D D
87&: ME Board October 1995
A car weighing 40 tons is switched to a 2 C. 5.45 s -Rotation
',! percent upgrade with a velocity of 30 mph. D. 6.37 s
lllllllllliii!M~
If the train resistance is 10 lb/ton, how far Theory Wed D'Aiembert's Principle

D D
up the grade will it go? Centrifugal Force
Banking of Highway Curve
A. 1124 ft on slope Problems Thu
B. 2014 ft on slope
C.
D.
1204 ft
1402 ft
on
on
slope
slope [I] D
Solutions Fri
877: EE Board April199&
A car moving at 70 km/hr has a mass of
1700 kg. What force is necessa~ to
decelerate it at a rate of 40 cm/s ?
D D
Notes Sat

A. 4250 N
B. 0.68 N ANSWER KEY RATING
C.
D.
680 N
42.5 N 836.B 848.D 860.C 872. A c:J 3!-45 Topnotcher
878: ME Board April1998
837.B
838.B
849. B
850. B
861.c
862.C
873.8
874.B
c:J 27-37 Passer
An elevator weighing 2,000 lb attains an 839. D 851. D 863.D 875.A c:J.22-2b Conditional
840.B 852.C 864.B 876.c
upward velocity of 16 fps in 4 seconds with
uniform acceleration. What is the tension
in the supporting cables?
841. B
842.B
853.A
854. A
865.A
866. B
877.c
878.D
0 0-21 failed

843.B 855.C 867.c 879.B If FAILED, repeat the test.


A. 2,150 lb 844. B 856.A 868. D 880.D
B. 2,4951b 845. A 857.C 869.A
C. 1,950 lb 846.B 858.D 870.C
D. 2,250 lb 847. D 859. D 871. B

8791 ME Board April1998


A body w.eighing 40 lb starts from rest and
slides down a plane at an angle of 30• with
the horizontal for which the coefficient of
friction ll = 0.30.' How far will it move
during the third second?
'l
500 .100 LSolved Problems in Engineering Mathematics (2"d Edition) by Tiong & Rojas
Day 20- Engineering Mechanics (Dynamics) 501

,,,

~
1~1
Ill V = V0 +at
40 = 20 +a(3)
a= 6.67 m/s2
s, = (20)(60) + i<0.75)(60f
S1 = 2550 m
S1 = 2.55 km
• t,= t2+2
-v,=15
a,= 0
Overtaking point ~! v = dx = 3(3.4)e- 5.4
dt
v = 1o.2e- 5.4
dV = 20.4t

""''"""~!
rm V = V0 + at -7 Eq. 1
V1 = V0 +at1
v, = 20 + 0. 75(60) t2
;,

s,
:l!!~Z.,X ; ;;.; i dt

dV
a=·--= 20.4t
I"'I!Wf!il~(l V1 = 65 m/s ~ -v2=0 ! dt

:ul;.;w{~n';; .m.tMittiMM¥.: =;;~. 2 a= 20.4(5)


""'""1 V. = 15 km x 1000 m x~ .ALNAA<J."' sw1
lj•~lllll:~ 0
hr 1 km 3600 s 1 2
s2 i a= 102 m/s2
S2 = Vot2 + 2at2

~->Gx
V0 = 4.167 m/s
L.MIIIIIIJ s, = V.t11 '
s2 = 65(60) + i(1)(60)2

l. .~~
s, = 15(t2 +2)2
V= 45 km X 1000m X~
s ~~~ =1st+ ge
S2 = 2100 m 1 ;::: 15t + 30____.----
hr 1 km 3600 s 2 . -7 Eq. 1
S 2 = 2.1 km dx
V = 12.5 m/s • v = - = 18+18t
dt
d
.
S2
1 2
=V2 t 2 .+ -at l,,i
Substituting values to Eq. 1: Solving for total distance: 2 2 At timet= 2:
2
V = V0 +at S=S 1 +S2 si = o(t 2) + i<2)(t2)
V=18+18(2)
12.5 = 4.167 + a(20) s = 2.55+2.1
S =4.65 km
s2 = t/ -7 Eq. 2 V =54 m/s
a = 0.41665 m/s2

s = V0 t+-~ae Ill
Equating equations 1. and 2:
1m
2 s, =S2
5
s = 4.167(20) + ~(0.416665)(20) 2 V. = 75 km x 1000m x~ 15t2 +30 = t/ D=20t+-
t+1
2 0
hr 1 km 60 min t2 2 - 15t2 - 30 = 0 dO -5
S =167m V0 = 1250 m/min V=-=20+--
dt (t + 1)2
l!!l!tl By quadratic formula:
.5
liiiiiJI 1 2 V=20---
S = V0 t+ --at 2
2 t - 15±J15 -4(1)(-30)
2 ( 4 + 1)
Vo-
~(500)(
45
)
2
v, - s = 1250(
45
)+
2
- 2(1) V = 19.8 m/s


60 2 60 t - 15±18.574
S =796.875 m 2- 2
t 1 =1 min.
s,
=1 min .
~-----------;·~~------~

V. = 72 km x 1000 m x ~
0
hr 1 km
tz

3600 s
s2
• V. = 30 km X 1000 m X _!.!!!:
0
hr
V0 = 8.333 m/s
.1
1 km 3600 sec
Taking + sign:

t2 = 15+18.574

t 2 = 16.8 seconds
2
Note: Since the ball was dropped, initial
velocity of the ball is zero.

1 2
h=V0 t+-gt
2
V0 = 20 m/s s = V t+-ae
0
2

1. 2 21000 =8.333(18){60) +ia[(18)(60)]


2 1m 1oo = o + i<9.81)e
t = 4.52 seconds
S1 = V0 t 1 + at1 = 3.4t 3
2 a =0.0206 m/s 2 X - 5.4t

.l
Day 20- Engineering Mechanics (Dynamics) 503


502 I 001 Solved Problems in Engineering Mathematics (2"d Edition) by Tiong & Rojas

-IIi I

I.!I
S:
V 2 = V0 2 +2gh
V 2
o
= + 2(9.81)(40)
v3 = 61.3 m/s
1
h3 = V3t3 - 2gt3
2

150 = V3 (9.155)-~(9.81)(9.155)2
il
.•1
Let: R = range of the projectile

R = V/sin2e
g
1002 sin[ 2(30°) J
• R = V/sin2e

2
g
J

••
.liM V = 28m/s 300 sin [ 2( 45°)


R= . R= -----:"~-'""'
9.81 32.2
R =833 m R = 2795 ft

•··4"':.
V=O
V1 1= o R = 2795 ft x 1 yard

.:v1. = 5 ...·i
;0·;: 3 feet
···········;;0,;··· Vo = 83.33
.: .
''
; -
R = 932 yards

.. _ .
: •,; t 2 h 1
. h2
6 .1 BD
t
m •••••••••

.
h1
t1 ~
. : · · · · ...·•
T 10
l H
Yl= 8001
v, =50/ /. • r~
t3 .~ ..
h3
'Vo=? ~~=30o . L lt i#~
!Vo=80 t' Let: H = maximum height
V3=?
. ;;

V2 = V/ :...2gh
~·~"'!'!'l'~~~~

v
2
H=-"-o_
2
sin_ a·
2g
0 =52 - 2(9.81)h 1000
V/ = V0 2 -2gh
h=1.3 m
V0 = 300 X
3600
2
50 (sin30
H=-~---'--
t
0 =802 - 2(9.81)h V0 = 83.33 m/s 2(9.81)
h1 =326.2 m H=31.86m


H=10+h
gx2
H=10+1.3
V1 = V0 -gt1 Y = xtane- 2Vo2cosze
H =11.3 m
0 = 80-9.81t,
t, =8.1555 s

h2 = v,t2 + 2gt2
1 2
• 1 2
h = V0 t +-gt
2
60 = O(t) + ~(9.81)t 2 Ill
X= 1064.2
(9.81)X 2
-800 = xtanoo - 2{83.33)2 cos2 oo
2
sin 2 e

2
v
H=-"-o- -
2g

300 (sin 45°


H = ---'-----''-
2(32.2)
t
326/2-150 = O(t2) + ~(9.81)t/
H = 699 feet


2
t = 3.5 seconds
t2 =6 seconds R = Vo sin2e
g
R = V0 sin2e
2

g
2
t3=~+t2-5
Vo =100 ...
... -.. 10(5280) = 1500 sin[2e]
32.2
300 sin [ 2( 45°)
R=---
2
J
t3 =8.155+6-5 ·, 32.2
e = 24.540
t3 =9.155 s ~-
9=3~
·. ·,
~ 0 = 24032.4'
R = 2795 feet

.. Am .£1. I 1&.¥4 iii. liiJ$$ q;;; ~ Note: l'he closest answer from the
R I choices is 700 feet and 2800 ft.

504 .1 00·1 Solved· Problems in Engineering Mathematics (2nd Edition) by Tiong & Rojas Day ZO- Engineering Mechanics (Dynamics) 505

V=O Let: t = total time (t) of the flight


1,1 ·.r 83.54 -1.206V0 2 + 0.004"
4

S:
1.11
t1
.-·
H
lf- = V0 coset
7 =V0 cos4o•t
=0.004V04 -0.
83.54 -1.206V02 = -0.1 E .. . . . ¥-~~-~. :. ~. :. ~. ~->:::
l Vo=?

•.;··:.,
1:··'vo ...,.
.l.
·~ij 2
V0 =.9.03

~
h t = 9.14 v. ~
E 1
q.
.....():-. .~. = 2~· 0
... 1111111~11 ..
· liiiiilll v1.=o
( ·, '*' ... :·· ..
r:::::1 30
V = VoY -gt1
V = V0 sin 0 - gt1 . .•·
_.. T .·,
t1/ . ·, i X
~·-~~111U~ 0 = V0 sin40°- 9.81t1 ..·.. !
Vay / H
L~,J ,.·v -2 = 60tan30o _ (9.81)(60)2

l~. :~
v =vov -gt 1
t 1 = 0.066V0 ~ Eq. 2
. !<~.=-~~:. . . .. ,12
·,
·. .
.
Vo = 25.35 m/s
2(Vo )2 cos2 30•
"


o = 300sin2o• - 9.81t1 ...
t 1 = 10.46 seconds H= o
v. 2
sin 2
e- 30
·,
...
2g
V2 = v.y -2gH 300 sin 40" 2 2
2 H=-----
0=(300sin20") -2(9.81)H 2(9.81)
Fe
H= 536.595 m H = 0.021V0 v2y
V/ = V0 / -2gH
1 2

~i
H = --gt 2 0 = (300sin2o•f - 2(9.81)1-t
2 1 2
h = -gt2
2 H =536.59 m N
30 + 536.595 =-i(9.81)t2 2

h = -i(9.81)t/ !
t 2 = 10.747 seconds V2 y2 = V/- 2g(30 +H) V=rro
Total time = 11 + 12 t2 = 0.45v'h v
V2 y2 = 0- 2(9.81)(30 + 536.59) = 1.5(2)
Total time= 10.46 + 10.747 V=3m/s
V2y = 105.43 m/s


Total time= 21.2 sec h = H-0.9
h = 0.021V0 2 - 0.9 wv2
V2x =Vox Fe=-
IJ

V=O 1 = 0.45J0.021V 2 -0.9 ~ Eq. 3 · gr


....................... ;,;·.·t-tj·.·:.;··
2 0 v2x = 300cos20° 'II
,· .·

v2x = 281.9 m/s Fe= 50(3)2 II
. t2 t = t1 + 12 9.81(1.5)
t1
' •.
-9.14 = 0.066V0 + 0.45.j0.021V
. 02 -
0...9
v2 = ~(v2S + (v2S
Fe= 30.58 N :II
H Vo
~(281.9) 2 + (v2S
ti/v'
(}\e
o
= 40"
. 2
9.14 = 0.066V0 + 0.45v0.021V~ - 0.9
~2
V2 =
v2 =301 m/s
LFtt.=O I :1
'I
!
F -Fe =0

:-----
2.15m

x= 7 m
3.05lm =10ft.

~i
9.14- 0.066V02

Squaring both sides:

83.54 -1.206V/ + 0.004V0


l
=0.45v0.021V 2
0 -0.9

4
• P- 30.58 = 0
F=30.58 N

F 30.58
Jl=-=--
- N 50
:.II
I

il
2
= 0.2025V (0.021V 2
- 0.9) J.1=0.61
0 0
'I
j
~
.
,'·f


506 l 00 1 ·Solved Problems in Engineering Mathematics (2nd Edition) by Tiong & Rojas Day 20- Engine4!rin9 Mechanics (Dynamics) . 507 .

Ill V=rm l Ill


!l,.:ll 15= 2.5m :f I V= 40mi x.5280ft X~ X~
1~111 o> = 60 rev x 1 min m =6 rad/s
Note: The force exerted by the string is
equal to the centrifugal force
,(',j 0
min 60 sec hr mile 3.218 ft 3600 s
.,ill,~ V =17.88 m/s
.ll!llll
rev
m0 = 1 -
sec Ill Fe=--
wv2
gr '2
tan~=~
"""'"""''Ill m =m0 -at m =m0 -at Fe= mV2 gr
( lllllii~J~
lill ..
0 = 1-a(8) 180=0-a(6) r 2
tan7° = 17.88
····"''"I
1<1o•1Ujll!ll'l~~l a=0.125 re:s rev
a=30 - . -
2
Fe = 1(10)2
1.5
9.61r

....
mtn r =265.41 m
=66.67 N
,.~.,. ",1
Fe "' 67 N

I
m2 = mo 2 -2a9
2
m2 = mo2 +2a9
2
1802 = (1) + 2(30)()
Ill
0 = (1) - 2(0.125)9 llii1l
"Uii·M~II!
e =540 rev w V = 65 mi X 5260 ft X ~


9 = 4 rev or 4 turns


hr mile 3600 s
V = 95.33 ft/s

V=rm 500 rev 1 min 2n rad


m=---x--x--
min 60s rev
1 rev
m=--X--X--
2n rad hr =
m 52.36 r~d/s
24 hr ., 1 rev 3600 s N
m =m0 -at
-5 rad
m=7.27 x 10 - 9= 5.40
s 52.36 =0- a(20) l:FH :.Q
a =2.62 re:s Ill

V = (4.5 X 107 )(7.27 X 10'5 ) Fe= F = tJN
V= 90'k"' X 1000m X~
V =3271 m/s hr km 3600 s
Fe l:Fv =0
V=25 m/s
V=3271m x ~ X 3600 S N=W
s 1000 m -;,:- V = 25 m/s v2
tane=g;
V =1.1777.4 kph Fe= tJW
r =0.5"m - wv2
F.e---- (25)2
Ell gr tan9 = 9.6:1(200)

f.LW= wv2 9 = 17.67,0


s
V=-
gr

V=30
t
0.3 = 152
9.81r
Ell
2 Fe= WV2
r = 76.5 m
V = 15 ft/s ~r
'i
i
I
r=. 2 +-
6 I
12
r = 2.5 ft
I
'

\
508 ·100.1 Solved Problems in Engineering Mathematics (2nd Edition) by Tiong & Rojas Day 20- Engineering ~-~cha~tics (Dynamics) 509

slope= tan9 2 I V. = 50 mi X 5280 ft X ....!...!!!:_ V = V0 +at


1"11 tan9 =0.0825 8.53° + cjl =tan·, , {88}
I
0
_ hr 1 mi 3600 s 16 = 0 +a(4)
~~~ e = 4.71°
32.2(500) V0 = 44 ft/s a= 4 ft/s 2

~
cjl = 17.160

Ill J.l = tancjl


L !'=inclined =0 LFv =0
J.l = tan17.16°
T=W+REF
J.l = 0.309 REF =;:·F + Wsin9
T=W+ Wa
Wa =F+Wsine g
The coefficient of friction is less than g
0.310 T = 2000 + 2000{4)


a at 40(2000) a= 10(40) + 40(2000)sin1.146° . 32.2
..•.,.,,I T = 2248.41b
w a = 0.8049 ftls2

I' kz .............,..+a V2 =V02 -2aS


Note: The nearest value from the choices
is 2250 lbs.


Fe= wv2 i&k&JL i.JL
gr 'WJ-REF 2
0 = (44) ..,. 2(0,08049)8
Fe= mV2
r
Fe =ma
LFH =0
-
F = ~00 lbs.
s = 1203 ft
Note: The nearest value from the choices
is 1,204 ft.
+a
v2 ............
a=-
' r
p = REF+F
Wa F·
Ill .............,..-a
27 2 p = ---+
a=----- g
' 800
a, =0.911 ft/s

2
30000 ( 1.25) + 100
p = 32.2
p = 1556Jbs
\
N
a =a/ +a/
a2 = (0.911)2 + (3) 2 Note: REF means revers~ effective force LFH =0 '\kf_Position of the
2 by d'Aiemberts principle.
a= 3.1 f/s body after 3 s


F=REF LFinclined =0
F=ma
Ell F = 1700(0.4)
Wsin9 =REF+ F

Wsin9 = Wa + J.LN
-a F=680N g
V = 60 mi x 5280 ft x _h_r_ .......... V=O
hr
V =88 ft/s
mi 3600 s
Ill. T
Wsine = Wa + J.1(Wcos9)
g
a sin30° = ~ + 0.3cos30°
tane = 0.25 32.2
e"' 8.53° a= 7.734 ft/s 2

2 Let: V =velocity after 2 seconds


tan (9 + $)::;: V S =distance traveled iii the third
Qf
tane = 0.02
w
!
~EF
secol)c:Ywith reference from
the position at the end of 2

1
9=1.146° seconds.
,I!

~
.
, I~
jl.'
i
510 1001 Solved Probtema in Engineering Mathematics (2'"1 Edition) by Tiong & Rojas
'[
·:
V = V0 +at 'l
''"~l,,l v = 0 + 7.734(2) ',';: j

v = 15.468 ftls
~I
.11~11 1 2
S =. Vt+-at
2 •

[ s = 15.468(1)+~(7~734)(11
r····'"1 s = 19.33 ft
~ ~~~~~u~
....

l.~,.,,l
l . . :!

s
w

REF
F

LFH=O
I!
F=REF

J1N=-a
w
g

J1W=-a
w
g
R,~•·'<·~~O'~' (~'' "<<•' •(.'·O~+ -'•'~<'(••X~~'

a= J19
I
a = 0.4(9.81) ~10,<.,.0•:<'.~<~·-••«<t> ~<~ ~.~x.

I
a = 3.924 m/s2
~·•11"~:~''•><;v•;<;' <'<•(><<~·~x~v;,.,.,.;~<

' ~ ~ "' * !< < x. "'·, .... ~ '4"' •. l « "* ,, -'t * ,•... «" y ;, < ••• - ' I'
V=V0 -at ~····~«'~<~-~.,-.,.~,. ... ·,._.,,,,,;.., .... «_'(!'~("'''·+~<<'<-'.~'<-~,..-~ <-•~ ~4~~"'' +<

0 = 25- 3.924t •<•''> .;,'to.< >~• <v•>•~<~<O<-> <•f'• ~«~•O<C<c> ... ><'('X''<~<<'>'>'<"««<,<«•>'"<''"»«><<-»

1: 1
t =6.37 s '_,,, .,,:~~.,..,.,,...... ,.,._,, .. ,..;,;y-..~"'~"'·~\0'~'\•+'<'·*· ~'""~~. ·~""""<?~- <>0<-.·~· ~ ........ .

"" " ' < ,. Jt •, < < < X ~ * < .... _<." 0 _,,(;;<,.·<A .<·' < .-' ·< * oi:., ~ < ~ , « '> < < .,- _,_ ·",. ~ ~ <'! '< < , • <)' '-. <t- ~ ·~
0 ' \ ...

• " • ' ~ ' ~ ' ..., ~ ..... ~ ' ,.. "" ... • • .. ... "' <· •• «

' \ .~ > ... ,< ' A ' y ' "< ~ ~·.,. > 'j- ,.. o
512 :1001 SolVeclProblems in ~S Mathematica(a"'i Edition) by Tions & Roiaa
'
,,,~

1,1 ., ~- ... + ~ !< :c * ~- 1<"' ~.. "* ,. ~- ~ 0 ~ " ' 'f. ... .0 ~- • < ~ "" ~ "" .;,. <t :~ "'."' ~ .. ~ -< "" ,, II- >!< ~ 1 -~- ;. "' ., ~ ~ , • " * 1o '¢ <!- ·" ~ ~ ,.· ;. ~ ~ ~ ~ • ~ ~ ~

(' :~ ~ ~-,. .. .,_ >' ..- « "' "' .. ~ .,. .,. "' " ... ~ v ~ ... < ,. ""' "' ... "' ... " ., ~ ~ >t ...... " " ........ "· ... ~- ~ "' •

.s
"'" < J< "' .,. "' "<II t< ,. .,. 11< "' ,... ,. "' 0 • ,. " "' ·> ol! .. '

' ., .,.. ,.. * ~ • "'(' -~ !<!" ' t:< *' "'· '"'. "' <t ., < ., ,. ~ ·"" ""·" .., ..,. . . . , -1' "' -t"·,. .. , ' -' ~ "' " ~ :• "' « " <- " ·'< > ~ ... ' •• l< ·,... " :
Topics
*....
0
,. • .0, )<." 0 ~ ~ ~ .0: 'I,""' ~ " ~ :r. "'·" 'f< V 1' ~"' A h • >1- It,., -1< ~ ... f "' .... ~ 0~0 .. ~ ~ " .. ;..•'< <• ~ " ., A \- 0 ' ~ <_..a ,. '> J!. 'f ,}

Strength of Materials
'""'~' <' ,_ •.;, v < "<f » 4',. • o>, 'I! .. "'"(< .. ;,>,."' t <' <• <- ~ '-' ~" « <' ~ <- :0 ->'h .,. 0< ,f_ .0 10."'" ~ ·• ¥ • • .. "< <fo ¢'.., .. A .. "''< X'>'~~'<! 't ,._ > M ... <"

•1111~111~11~ Mon Simple Stress


·, e ~ .,. "' 'i' ., ,. ; ., ~ t .J ._. o!. " -~ ,. ~-:c. "' ., •• t tt " <t ,. c~ ~ ~· • ~ ~ ~ ,. ~ « ., .. .,. .. <- t ., ~. ""."' "''t ."' ~ .$ .,.. ~ " < .., .,. .... ~ ., .. ~ ~- .., ~ .. ~
Types of Normal Stress
'""'"'I
HIIII'
'I ..
"' :"> ll' X.

>f $. !J' ,>,


< .; ,;-. > :1;- ~ ¥:" ~ !'

><- ® "> ~~ ill: ";_lit;.::..<':' ,. "':


•& ..... l1c <0 >'

.j). ~' ;,.;,


~. ~ ~- 't> ~ ~ ·~ ~ -~ ~

"(:< .. y I"< '" .,.,- <$o ;< <If


::C. '!·~ ~ ·~

>~ ~ 'P '0


(: :.;

'< ;,.
~ ~

)I
"' ·t

'f ·" ~."' -" '{'


J:< ~ ~ ¥

X
~

>1 'l'
¥ ~ '"

J_ '.,i "< ~
~ <; <> " 1f <'1 ; .. ', 4 J :1 ·~ •.

;1- .>< ~ ~ ,. ~ lr-~ ij "' .,.·,. / ...


"4 ~ "

,o/< ,><- ,•
4 "

~
t' ,.

'
~

"', (.
,_J
~--·,

Tue
Simple Strain
Hooke's Law
..,.,,I $tress .. strain Diagram
'I'
·-~ *•.>'<.""-.$' 1<<i<1o

-~~ ~,.,~:-:.:.;/' ~ <'\


i ji<<t ~ <~- ,.·,.

~<· ~4 ~ ..;.>'i'·4<-} lli


$! «. ii<#'f.WIIIi.t< ~;.,:~·I!>~«"" :t II{ .if.

.1tl ,.,,·:.·-~lit~ f<':' of!.~~:,.,~·.,;~~ ·~·'*:·lt_-,;·.,. •. ¥-···+}- '@<.,.;:-><...,~'tis


lo.._o(.f< ~ fl ot ~ ~·4'·,.·,; ,..·,. ».;. ,._·,. ... '* ·.,.~ "'* ,; _.. .t ». ,.·x- 11.11 .... 'if"'.""-¥.
:3- f' ,to';-·~·«' ~4'·., ·*'.·)!> i·;.,.., :(< [QJ Wed
Thermal Stress
Thin-Walled Cylinders
..,m.,. *"'*~· * •~!'.*~~' ~ ~•·:~~oo• ~ .~ i<J.·-..4·$. ""·~ .0 ~#"i'+ .;...,.·\". ..,_., ·~·w. ~)< -p~ ~~--:~·· ~:_4<,. ~ Theory

LJ L--l
......~Mflllill 1<J :<- •+'>t"ot.ti. )0; <* 1!!.11< '!''II' <I >ol; _f: ,
Torsion
~:,., ljl ')<.·!fl.·~ . . "" ,j\'\"'.t '(<';t' ~~II:!'*"''*, ~ .
.,...oli- +;: 1'<& ~ .., ~ ~·<! ~ ·-~~···~·'!!~ ~~ .. -*' *'** ~··*'·"" -·~ -··· (-.. .....·,_-..:::.;J"~':,: ~Ill 'f'<ti :f.
Helical Spring
.""~< .,·•. , ",.. f,.,... "''"'"' ~·,o;·,. .,. ~ .,.~... 1! .... "' '<- <t,. "'i>JI! ""'"' ~ .,.·,. .ii ~f-.~·~ :t ~ ~·.~ .... & .. "'"'···~·""""I!"!;:., ,~,.,. .. ·>9 .i<f! ~ "'"' 1{
Problems Thu
..-

<i-
~ ......... ~ .,\0!

*' ~~·1<>-.·~·i<d"'
of',_,.._,.~·,.,...

~ 1, •'il0
.. _,..,,.._,..,_.,. ............. "" ... ~~-"'.fl:' . . ,.

Jii·1r'*f< ..-b'<'·'\0"-t-';;- ":.,·~ <t'lo 1< ~


"'""' *''*·'!:•,. ···"~".""'*-.«'"' ...... ,.~ "'." ... ,.,..~,.,._~·It·"' ... ,.,. .... ., ... ~
1<

«."' "'~ lf-t-¥ "'<·~ '" ~ ~ tj'1f <i" ~~':~ -tY •~!! 1< ·~ _,...,..,. . . . ,., •• _,..·.._·':¥ [] 0
Solutions Fri
~ ;,. ~ « ~- .;- :~ ~ 4< ,-( "' ~· ~' :t '!" :w.. •· '<.' {! ~ JR "' -:o- ·~ ~ ¥. '* <.:; -r ; ·~ ·_, ~·"'' ~ 1< '11 ~ ,·~ "'. "' ~

0 [-]
"' "' .;> ' :,q. 'f 1< ·>- '!I <.: '" ,. lO •>f 10 <-· , "" '-. · ·x'Jo ., ·•·If ·"' lf!' 'II

.,. ""'. "' -!<.f' .. ~ ., < I!'$ ~"" + ~ <\-." " ~ .... ~ ... ~. "" ~
)!< ' • t."' 1!' .. ,. '* .. ~ .... ». * « .~ ~ ¥o"' ........
1(: "< " t ·~>I! .... ~ ~ ~ j' ., *'h. o. ~ ~ * *·~.. ~

·~ <· '( ....... ., ~ "! <!'" ~ 1 ,. : " ,. "-. ~ ""= '> .. ,. '+'"' ~ ~)<I<" ( ' ..... •$.-It .... ~ ~ • ,.,.... ~ ... * . . ~ "'"'~ . . .,..,., ~ ~ ~ + ": .. " ~ I" ...... ..;,.,"' ... · .. ~..:. -\.·. Notes S<lt
~ , ~ . . . . . . -.< " ,... ~ ••• "' .: ..,. ... ~ ;_. : ~ .~« ~
.> > 'l' • ~ l- ~ " 4 4-1,. ~ ~ >' >. ··~ ~.;. ~ ~···· «
'~ 1 > >0 X.<! 1<4 ~.

.., <. ,.<&,,,·~ \,>'<' >'-!.~ < ·~·"·) ~ ... o' >"' O</ ~ N' ....
perpendicular to the area resisting the
"" ' ~· " "' i ,. ' 1 'i! " -A " • I ' ,. ..., ~ • '' > "' ? " Of ·, .~. 1>"-» '•
Wl1at is_ Stre!]!Lth of Materi~l!!.1. forces. Normal stress is also called as
. .
bearing stress. Shear stress, r, is the
·' ,. >. "' , .... "" " • ~ ~ ., ........ "" ~~ •• ~ ... ~ ~- < ~ ~ ~ « ~ ~
:;trength of Ma'terials is the branch of type of stress which is caused by forces
""-"~~rN'<''>--:=:."';"'"< :;cience that deals with the elastic behavior acting along or parallel to the area
of load bearing engineering materials. resisting the forces. Shear stress is a!so
,· ... ., ~ ..., .. " ... •'
:;tren9th of Materials is also known as the called as tangential stress.
_,.. >1 >< )<' >< )1. « • ~ ~ ,. ,. o0< .o< .4 .« ~ .< o<- Y l- M ,_ •• oO< X "< » • M ...... ~ X ,_,· ~ « « -k ..... ><- ... ~ > -... -1'. .. • -6- ~ v > 0." ~ 0' 0 ~ X >.-;. 'o. ~ .-o. • X " ..: « +. <- im1Ghanics of materials.
":~1-"' ~ ~-~"''~""''<~~_,;.<.~~·"'**'"'' •• .,~ o:~»x-x<:'"'~~"""•·>• • ;·~''''·"'- <·~:o>o,... ~;i>~'o

4-- -$ ~ ;• '<• ~~ _, ~ ~ .~. "4; ~ "" <llo 'I> -«. ~ l< A" "' "f ' -"•"' ~ ">"" « ~ .< ·• ~ ,_ ~· <;; ,._ '>,"* .-> ~· "> '~!. '<' .. -t '> , •• $ ,. ~· <!- ")- .. ~ ~· -4 'IF.~ 11-.::. -"' "' " t * If·"~ t- 11\[~t is Simple Stress?
·> ><"~-¥-"."'.,.~ "'"' ""'J';'(.<t"~>l-'f»:>;«-»-~ "'""""'~X·~ ,_,,,..,. ...; "t .... "-t .... r. 1>. .,. ,. ».,. ,. ,; ... ~ ~

~>imple stress is defined as the force per


<. ,<-. \0 : ~ It Jo; ~ •-;1· :,>. J!' 1" -1! -:> .<, .;,-: -'j• ,~ " ~ ~ :"' .f!l o+, 'll' 00: ,. .,. ¥ ') '> <0: » ?; \t X ~ f!j 'f. & ~ ~ "' '< « ~ ... ~ 3> '< :,1- <f .. -f • ~ j'>.,;.· ... ~·-% 4' if ~ ~~ 'f,. ~ .ji" · • 1111it area. The typical units of stress are Area
2
MPa, lbf/in and ksi (thousand of pounds
~ .. .,. ~ ~ ~ " '-! ~ "", •• ... :'!(: $.~ "'· .. "'' ~ "''JII "':i ~ ,., ~. <k ~ 10 <. :(<- ... " , <"< ,. 4 ,. ~- 10 I< 'It ... f w .... li" i.. 'li ~ ."' .. ,. .. r. ~ ;: -i' '« ~ .. ,. It .•; • _f..,."'
per square inch). The MPa is equivalent to
'., i<..::-'l>il<'il·"** ,.,,. ~..--~"4>~ 0! t<l.,.,._.- . . . r.' ~ '< """' •<!. II<&~ #,.'11'.1<:., ..... ~ <! ... 1.o:10 «:,. \1' ...... •<!-$-4. -t ·~·~·~ ......·'to"., .. ~,.~ ... • MN/m?. or N/mm 2
' '" ' ' : ~ ~ ~ .. ;; "' ~ ~ ~ ~, ., .... ~ ~. 1> l ' ... "'i 'f ~ .. !:. ~ .... ~ ~ ~ )1..~ ~ of! ol! . . . .~- l .. ' "*': \! ,. ~ <""•·
1he two basic types of stress depending
... ..- ... " ,. ~ " .... ~·. " ~ ~ ~ " ~ "'·..-. > ... ~ ' ; • ~· ... •• y •• ,. :,·_I, • ~
'lllthe orientation of the loaded area are
•<~11ll:1l :,;iTess and shear stress .
.. ,. .. ~ .,,: ~. ~ ~ • ' ii' ·» ........ ~- "' "' ,. l ~ • ~ "<' ' "0 <.* ~ • <(' ·>. ~ ~ ~ ~. « .~ ~ ~ 'f .,- ~ '"." ~ ........ ~ ' ., 1 - 't· ~ :' •.

p
-<- ;... • < ~ , ,..~ ....... "". "" ~ <, ~ ~ '• "' .; " . ~ :..1 .. ", .. » ~ o ~ 1> "' • .~u ~·- ': ~ 11 ,. ~· Nonnal stress
;,.:" .. :l1:t."O ¥ ... v· 4 "! · ·•
Nonnal ~;ilxl.'s~. o, IS the type of stres::; fl""A

\ \
~,~.._ ... ,,. .... ¥;t. ... ~ ...,,..,.. • .,,... ~~·'-• ... ··~'¥'''"'•' ,, .............. ~ "' ' ...... y'l.. ~ II. • ',, ,, •• •• wh1< .II 1:. "·" l';~:d I lV forct"S actin<)

-~
"I'
J
514 1001 Solved Problems in Engineering Mathematics (2nd Edition) by Tiong & Rojas -------------------D~ay~21~txengthof~~
the materia!' even without any
,,,1~1 corresponding increase of load.

1.11
to:;~p
I
},
Ultimate stress (or ultimate strength) I'
i
(' ~ refers to the highest ordinate in the stress-
ilm;: strain diagram.
I
·!~1111 p I

Rapture strength is sometimes known as

(-"'...."1"l
JIWI<I'AI1UI
p
+
where: G
1:i=GS

= shear modulus
the stress ·at failure.

Working str·ess is the actual stress of the


Shear stress 't"""A material when loaded.
-··~1111,1
. .,,I,!, What are the Two Types of Normal Strain, E=l
0
What Is Stress-Strain Diagram? Allowable stress is the maximum safe
stress which the material can carry. 1:,.
II

Stress? A diagram that shows the relationship ]•

between the stress and the strain is known Factor of safety is the ratio of the ultimate
,.lol!ll!ll~ll stress to allowable stress. ! I
The two types of normal stress are the as the stress-strain diagram. The
tensile stress (forces away from the area) What is Hooke's Law? diagram below is the stress-strain
diagram. Shearing strain is the angular change
and the compressive stress (forces
Hooke's Law formulated by Robert Hooke between two perpendicular faces of a
towards the area). I,
in 1678 is stated as follows: differential element..
Stress
"Within elastic limit, the stress is F Modulus of rigidity (G) -refers to the I
I
modulus of elasticity in shear.
proportional to strain."
'I D •••e ..••·"'
Mathematically,
Area What .is..Tit~!!~~
0" OCE
E
Thermal stress is the stress on the
~ material caused by the intern<;~! forces due
cr=EE to change in temperature. The change in
temperature can cause a change in length,
area and volume of the materia!. The
The constant of proportionality, E is the temperature deformation (linear) may be
modulus of elasticity of the material. It is Strain calculated using
Tensile Stress Compressive Stress also known as the Young's modulus.

cr:::.~ ~.:.·t.•~mp. change


A -7 proportionality limit
The Young's modulus was named after
What is a Simple Strain?

Strain refers to the elongation of the


Thomas Young who introduced this
constant of proportionality in 1807.
B -7 elastic limit
C -7 yield point
D -7 ultimate strength
.•. ·,:::=J, S. ubje.·c.· ed.
...
-·······
.,.r..·---~--
.t.

.E

material with respect to its original length E -7 rapture strength ~--------~~


li
Also,
when subjected to a load. The typical units
P Eo
F -7 actual rapture strength . L o1t
of strain are mm/mm, inch/inch or no unit I

at all. Strain may be expressed as a


A ""'L Elastic limit refers to the stress beyond
'•1!,

which the material will not return to its


Br == aLt6.1')
percentage of the original length.
original shape when the load is removed.
When the material is subjected to a where:
fhe permanent deformation caused by
shearing stress, as shown in the figure on a = coefficient of linear expansion
excessive stress is called permanent set.
the next page, ·
Ylold point refers to the point where there
Ill an appreciable elongation or yielding of
:1~ :
I'
1111'

I
~~~~
1,•

516 : 100 1 Solved Problems in Engineering Mathematics (2nd Edition) by Tiong & Rojas Day 21 -Strength of Materials 517
'Miat is a Thin-Walled Cylinder? What is Torsion? where: T = torque applied
'"1~1 A thin-walled cylinder is a cylinder under Torsion refers to the twisting of solid or I
L =length
J = polar moment of inertia of
Proceed to the next page for your 21st
test. Detach and use the answer sheet
1.11 hollow circular shafts. I
1~ of its
I
cross-section provided at the last part of this book. Use
~
pressure and has a thickness, t :::;
G = modulus of rigidity pencil number 2 in shading your answer.
•:~1 inner radius. A. Shearing stress:
r t E. Transmit power, P GOODLUCKI
~~
-r:p
(-""'~1 '{ "'j
P =27tf T
:::::·,,
where: T = torque applied
where: P =power in watts ·~tibia:
,. ..1111111
p = radial distance from the T = torque in N-m Did you know that... QED, the
center of cross-section f = frequency or' speed rps abbreviation of Quod Erat
.,.,.,J,, J = polar moment of inertia of Demonstrandum, Latin for "which was to
the cross-section be demonstrated" was commonly used by
',In'
What is a Helical Spring? mathematicians to indicate that a ·
'I!IJI!II"Ii!lfll'lll
B. Maximum shearing stress: conclusion has been reached, was first
A spring which forms a helix is called a introduced by Euclid using its Greek
helical spring. equivalent in the 3'd century B. C. !.
The thin-walled cylinders will experience
Max.f::;~$f·.
'J
tWI> different stresses, namely, tangential A. Maximum shearing stress:
stress and longitudinal stress. ~uote:
where: r = radius of the cross-section "Mathematics is often defined as the
A. Tangential Stress: T = torque applied t"" 16PR
3
(1 +..!!_) science of space and number ... not until
J = polar moment of inerti~ of rcd 4R the recent resonance of computers and
pD the cross-section mathematics that a more apt definition
CJT :;:2t became fully evident: mathematics is the
C. Maximum shearing stress of: or science of patterns!"
p: 16PR(4m-1 + 0.615)
where: p = pressure in N/m3 rcd 3 4m-4 m
A. Solid shaft
D = inside diameter in mm
- Lynn Arthur Steen
t = thickness in !Tim
Maxit.~ 1&1·· where: P = axial load
B. Longitudinal Stress: ·. . · ·.:-:;;r
/c •>''"'\' ;_.'.~·'"
R = mean radius of helical
spring
d = diameter of rod/wire of
pO B. Hollow shaft spring
O'L=M m = ratio of the mean
diameter of the spring to
il,l!
=
where: p pressure in N/m
3 Max. t·= . ~~:;;P '"'i .• t
#''
the mean diameter of the
spring rod or wire
D = inside diameter in mm
t = thickness in mm 2R D
m= - = -
d d
where: d = inner diameter of shaft '!I',II
Another term for tangential stress is
D = outer diameter of shaft
ciR:umferential stress, or hoop stress or B. Spring deformation: 1111111'111111

girth stress.
D. Angular deformation, 9:
3
Note that the longitudinal stress is one-half 0 = 64PR4 n
the value of th~ tangential stress. . e....;r:~· Gd
-Ji·
where: n = number of turns
G = modulus of rigidity
Day 21 -Strength of Materials 519

·~ 885: ME Board April 199ft C. 4.09 in~hes


,,,a ~, If the ultimate shear strength of a steel
plate is 42,000 psi, what force is
D. 3.96 inches

1.1 necessary to punch a 0.75-inch diameter 890: EE Board Apri11996


(' It hole in a 0.625 inch thick plate? A cylindrical water tank is 8 m in diameter

'st Topics A. 63,000


and 12 m high. If the tank is to be
completely filled, determine the minimum

·-'' '1 0Mon


Strength of Materials
Simple Stress
B.
C.
D.
68,080
61,850
66,800
thickness of the tank plating if the stress is
limited to .40 MPa.

:::::·1, Types of Normal Stress. A. 11.77 mm

""'!iilllli I 0 Simple Strain


Hooke's Law
88&: ME Board October 1995
What force is required to punch a 1/2-inch
B.
C.
13.18 mm
10.25 mm

. . .I, Tue
Stress-Strain Diagram
hole on a 3/8 thick plate if the ultimate D. ·12.6 mm 1!:

~
shearing strength of the plate is 42,000

,f' 0
Theory Wed
Thermal Stress
Thin-Walled Cylinders
psi? 891: EE Board October I990
A water reservoir of 24m high and 12m in
I

-·""" A. 24.~0 diameter is to be completely filled with

Problems
0Thu
Torsion
He.lical Spring
B.
C.
D.
24,620
24,960
24,740
water. Find the minimum thickness of the
reservoir plating if the stress is limited to
50 MPa.

0
Solutions·
0 Fri
887: A single bolt is used to lap joint two
steel bars together. Tensile force on the
bar is 20,000 N. Determine the diameter of
A
B.
C.
24.5 mm
28 mm
21 mm

0
Notes
0 Sat
the bolt required if the allowable shearing
stress is 70 MPa?
D. 26mm

89:t: EE Board April 1995


The stress in a 90-cm diameter pipe
!\. 17 mm
ll. 18 mm having a wall thickness of 9.5 em and
ill:
c. 19 mm under a static head of 70 m of water is
88lU IECE Board November 1998 883: ME Board April 199ft ll 20mm
An iron column of annular cross-section A steel tie rod on bridge must be made to A 325 kPa
has an outer diameter of 200 mm and is withstand a pull of 5000 lbs. Find the 888: EE Board October 199& B. 32.5 kPa
subjected to a force of 74 kN. Find the diameter of the rod assuming a factor of
thickness of the wall if the allowable safety of 5 and ultimate stress of 64,000
What is the stress in a thin-walled C. 32.5 MPa II
:;pherical shell of diameter D and a wall D. 3.25 MPa
compressive stress is 10 MPa. psi. thickness t when subjected to internal [,
pressure p? . 893: ME Board October 1994
A. 12.75 mm A. 0.75 A cylindrical tank with 10 inches inside I
B. 12.. 57 mm B. 0.71
!\ S = D/pt diameter contains oxygen gas at 2,500 psi. II
C. 17.75 rnm C. 0.84
D. 15.75 mm D. 0.79
II. s = 4D/pt Calculate the required thickness in mm
c: S = pD/4t under a stress o.f 28,000 psi.

88:&: ME Board April1998 884: EE Board October 1996


() S = pD/t
A. 11.44
A force of 10 N is applied to one end of a Determine the outside diameter of a hollow B. 11.34
889: ME Board April I998
10 inches diameter circular rod. Calculate steel tube that will carry a tensile load of C. 10.60
c ornpute the safe wall thickness of a 76.2
the stress. 500 kN at a stress of 140 MPa. Assume D. 10.30
•Ill diameter steel tank. The tank is
the wall thickness to be one-tenth of the
·.cci>wr.ted to 7.33 MPa pressure and the
A. 0.20 kPa outside diameter. 894: ME Board April 1:995
·.1•·•:1 cn<Jterial has a yield stress of 215.4
B. 0. '15 I<Pa A solid shaft 48.2 em long is used for a
Ml 'd !he factor of safety to use is 3.
C. 0.051{Pa A. 111.3 mm transmi,ssion of mechanical power at a rate
D. 0.10 kPa B. 109.7 mm of 37 kW running at 1760 rpm. The stress
!\ I I 1/ inches
C. '113.7mm is 8.13 MPa. Calculate the diameter.
II I W) 111dws
D. 112.4 mm
'I~!
ll;

520 :100 l Solved Problems in Engineering Mathematics ~2nd Editio~:l_!~fl~q & Roj~ Day 21 -Strength of Materials 521

A. 30mm A. 1.512 x 10-s m4 A. 200 GPa 909: A simply supported beam, 10 m long
l~~l B. 35mm B. 1.215x10-am4 B. 180.32 GPa carries a uniformly distributed load of 20
IJil, C. 40mm C. 1.152x10-em4 C. 148.9 GPa kN/m. What is the value of the maximum

g D. 50mm

895: ME Board October 1995


What is the modulus of elasticity if the
D. 1.125 x 1o-e m4

9001 ME Beard April 1:997


D. 106.48 GPa

90S: EE Board Aprii19C)6


shear of the beam due to this load?

A. 250 kN
What power would a spindle 55 mm in A cylinder of diameter 1.0 em at 30°C is to B. 100 kN
stress is 44,000 psi and unit strain of diameter transmit at 480 rpm. Stress be slid into a hole on a steel plate. The C. 1000 kN

.....'"'J allowed for short shaft is 59 N/mm 2


0.00105? hole has a diameter of 0.99970 em at D. 500 kN

" . '1
UHJIIill
A.
B.
C.
41.905 X 106
42.300 x 106
41.202x106
A.
B.
42.12 kW
50.61 kW
30°C. To what temperature the plate must
be heated? Coefficient of linear expansion
for steel is 1.2 X 10" 5 em/" C.
910: A simply supported beam, 10 m long
carries a uniformly distributed load of 20
C. 96.88 kW kN/m. What is the value of the maximum
....,,J, D. 43.101 X "106 D. 39.21 kW A. 62°C moment of the beam due to this load?

r
---
8961 ME Board October :1995
A 2-inch solid shaft is driven by a 36-inch
gear and transmits power at i 20 rpm. If
90•: A 30-m long aluminum bar is
subjected to a tensile stress of 172 MPa.
Find the elongation if E = 69,116 MPa?
B.
C.
D.
65°C
48°C
55°C
A.
B:
C.
10,000 kN-m
5,000 kN-m
2,000 kN-m
the allowable shearing stress is 12 ksi, 906: EE Board April :t99S D. 250 kN-m
what horsepower can be transmitted? A. 0.746 m An iron steam pipe is 200 ft long at 0°C.
B. 0.007 m What wi.ll its increase in length when
A. 29.89 C. 6.270 mm heated to 100°C? Coefficient of linear
8. 35.89 D. 7.46 em expansion is 10 X 1o-s ft/"C.
C. 38.89
D. 34.89 90:&: EE Board October 1'996 A. 0.18 ft
A steel wire is 4.0 m long and 2 mm in B. Q. 12ft
sen: ME BOard October :199/i diameter. How much is it elongated by a c. 0.28 ft
A hollow shaft has an inner diameter of suspended body of mass 20 kg? Young's D. 0.20 ft
0.0:-15 m and an outer diameter" of 0.06 m. modulus for~steel is 196,000 MPa.
Compute for the torque in N-m, if the 9071 ECE Board November 1996
stress is not to exceed "120 MPa. A. 1.123 mm A simple beam 10 m long carries a
B. 1.385 mm concentrated load of 200 kN at the
A. 4500 C. 1.374 mm midspan. What is the maximum moment of
B. 4100 D. 1.274 mm the beam?
C. 4300
D. 4150 903: A steel wire is t} m long, hanging A. 250 kN-m
vertically supports a load of 2000 N. B. 500 kN-m
8<~JI!I$ ME Board October 1996 Neglecting the weight of the wire, C. 400 kN-m
Compute the nominal shear stress at the determine the required diameter if the D. 100 kN-m
surface in MPa for a 40-mm diameter shalt stress is not to exceed 140 MPa and the
that transmits 750 kW at 1500 rpm. Axial total elongation is not to exceed 4 mm. E = 908: ME Board Oc:tober 1993
and bending loads are assumed negligible. 200,000 MPa. A beam supported at both ends and
carrying a uniformly distributed load:
A. 218 A. 3.4 mm
B. 312 B. 4. 4 mm A. has its maximum bending moment at
c. 232 C. 4.26 mm the supports
0. 380 D. 5.4 mm B. has its maximum shear at the center
of the beam
899: ME Boa!l'd O~tobe~ 1lHS 904: A copper rolled wire 10 m long and C. has its maximum shear at the
A hollow shaft has an inner diameter of 1.5 mm diameter when supporting ,,. supports
0.035 mandan oute.r diameter of 0.06 rn. weight of 350 N elongates 18.6 rnrn. D. has uniform shear throughout the
Determine the polar moment of inertia of Compute the value of the Youn!ir:·. length of the beam
the hollow shaft. .modulus of this wire.

J
11
'll".
r,\:

....
.
.
Day 21 -Strength of Materials 523 f:
1!!!111
liiilll D = 0.2 m liiiiiill
•.•
l~~ll
,.
I p
cr =k-
A

H Topics
where: k = factor of safety

D
64000 = (5)5000
Strength of Materials
·-"""
,......w~ll'~~
...111l"
lvlon
Simple Stress A= 0.3906 in2
A

Types of Normal Stress Uli :·


.,.,.,1116

_.,,J,,
I
DTue
Simple Strain
Hooke's Law
cr=~
A A=~d
4
2
··!1

=75000
r D (g Stress-Strain Diagram 10 X 106
A 0.3906 =~d 2

....
Thermal Stress · 4
_ ...Ill A= 0.0075 m2
Wed Thin-Walled Cylinders d = 0.71 in
Theory

D D
Torsion
A= ~0 2 -~d 2
Helical Spring 4 4 liiiill D
Problems Thu
0.0075 = ~(0.2) 2
- ~d 2

D
Solutions Fri
4
d=0.1745 m
4

Solving fort:

D D
Notes Sat
D = d+2t
0.2=0.1745+2t
t = 0.01275 m

ANSWER KEY

881. A 891. 6 901. D


RATING

[:} 25-30 Topnotcher


.....
liiilill
t = 12.75 mm cr=~
A
140 X 106 = 500000
A
882. A 892. D 902.0
883. 6 893. 6 903. 6
[:} 18-24 Passer . 1ft X -1m- . . A =0.00357 m2
d= 10 x 12 in 3.281 ft
884. D
885.C
894. D
895.A
904. D
905. D
0 15-17 Conditiona.l d=0.254
In

m Note: t = 0.1 D
886. D 896.6 906. D [:} 0-14 Failed
p
887.c 897. A 907.6
cr =A 7t 2 7t (
A= -:0 - - D- 2t )2
888.c 898. D 908.C If FAILED, repeat the test. 4 4
889. 6 899. D 909. 6 p
cr= - A= -D 2 - -
7t 7t(
D·-2(0.10) )2
890.A 900.C 910. D 4 4
~d2
4 A =·0.282702
10 0.'00357 = 0.2-8270 2
cr=---
~(0.254)2 D = 0.1124 m
4
D;, 112.4 mm
cr = 197.35 Pa "' 0.2 kPa

I
J
lill, 1'\11'
1

524 100 l Solved Problems in Engineering Mathematics (2nd Edition) by Tiong & Rojas Day 21 - Strength of Materials SZS 111' 1.:"
]I' :'

CJ=k-
p 11.1 pD
Ill 1111 1::

'•••"•'
~,
A
215.4 X 106 = 3(7.33 X 10 )(0.762)
6
CJr =21
E=~
&
II; I:

2t Note: The biggest pressure occurs at the


'~~-:
E = 44000
t = 0.0389 m bottom 0.00105

Cl:
-~
t = 3.89 mm
p = yh
p = 9810(70)
..,. E=41.905x106 Pa

·-·1''
p
CJ = -
A
1m p = 686700 Pa
liiiill
•t~~JOII·II Ill cr, = pD 16T
p 2t t=-
.....,,,, 42000 = n(0.75)(0.625) cr = pD
nd3
16
Note: The biggest pressure occurs at the I 2t 12000 = T
.rill p = 61850 lb
bottom of the tank 686700(0.9} n(2)
3
I!
l!tiB cr, = 2(0.095} T = 18849.551b-in
--~~~~~~- liiiiilll p = yh
p
CJ=- cr1 = 3252789.474 Pa
A p = 9810(12}
p
cr1 = 3.25 MPa p = 2nfT.
p = 117720 Pa 33000(12)
42000 = n(0.5)(0.375)
p = 24740 lb t = pD Ill p = _2n~(1_2-'0)'-'--(1_8_84_9_.55.....!._)
33000(12)
2cr1


J!IP.II!W pD p = 35.89 hp
Uilll t = _1,.-17_72_0_,_(8-::'-;) CJr = -
6
2t
p 2(40 X .10 )
CJ = - 28800- 2500(10)
A 16TD
t = 0.01177 m 2t t-
p n(D4 - d")
CJ=-- t = 11.77 mm t = 0.4464 in x 2.54. em X10
· ·mm
--
~(d2) ln em 120 x 106 = 16T(0.06)
7t[(0.06)4 - (0.035)4 J
where d = diame~er of the bolt Ill t = 11.34 em
T =4500 N-m
pD
70 X 106 = 20000
CJr = 2t 1m 1!9!!11
~(d2) p = 2nfT liii6ia
Note: The biggest pressure occurs at the 60
d=0.019m bottom of the tank
p = 27tfT
37000 = 2n(1760)T 60
d=-19mm 60
p = yh 750000 27t(1500)T
T = 200.75 N-m . 60
~ p = 9810(24)
liiiiilll T =4774.648 N-m
p = 235440 Pa
pD 16T
crL = - 7 Formula t=-
4t 7td3 16T
t = pD t=-
8.13 X 10& = 16(200.75) 7td3
lf!I!R 2cr1 7td3 16(4774.648)
liiiiilll t = _23,...-54_4_0(,_12_,_} d=0.050 m t= 3
7t(0.04)
crt=kpD 6
2(.50 X 10 ) d=50 mm
. 2t t=380 MPa
t = 0.028 m
where: k = factor of safety t= 28 mm
I
I
~
!,:Ill!'
SZ& :100 1 Solved Problems in Engineering Mathematics (2"d Edition) by Tiong & Roip
..


~~ Day 21 Strength of Materials 527

Ill A=~d 2
ST = 1- 0.99970

'"'" J = _!t_(D4 - d4) Maximum shear of a symmetrically·

'"
H
32
4
J= ;2[(0.06) -(0,035}
4
4
A = *(0.002)
2
81 = 0.0003 em
ioaded system is equal to the reaction at
the supports.
,I'!"1


)
81 =aL(t2 - t 1) 'I

J=1.125x10-6 m4 A=3.1416x10.s m2
0.0003 = 1.2 X 10'5 (1)(t 2 -30°)


]I!
20(10)=200 kN
t 2 = 55°C
~
.

I)= 196.2(4) ill
:':ll
fi~J:P!Jj
3.1416 X 10--6(196,000 X 108 )
16T
t=- 15 = 1.274 x 10-3m
7td3
Or= aL(t 2 t 1) R1 10m R2
HI:

15 = 1.274 mm -
59=~3 01 = 10 X 10-6(200)(100° -0°)
n(55)
81 = 0.20 ft
L:Fv =0
T =1927391.637 N-mm Considering limitations of stress: R1 + R2 = 200
T = 1927.39N- m
y=-
p
A
Ill 200kN
2R1 = 200
R1=100kN
p =~ltfr
140 106 = 2000 ~ Maximum shear= 100 kN
. ;r~~ ; :=;;~;t
60 X

p = 21t(480)(1927.39)

P=96.88 kW
60
!:d2
4
d =0.00429 m
t'~:::.
a
Refer to solution in Problem # 909:
d=4.26mm Note: Since the load is at midspan, then
the reaction at both supports must be
ID Considering limitations of elongation: equal.
LFv =0
Consider the half of the beam:

20(5)=100 kN I
l5 = PL -+ Eq. 1.
15 = PL

~
AE 1:11111
p AE R1 + R:;i = 200 .,

aS=- ~ Eq.2 2R1 = 200


A 0.004 = 2000(6)
R1=100kN
~d2 (200000 X 108) Iiiii
Substitute Eq. 2 in Eq. 1: Note: Maximum'tnoment.of a
d = 0.0044 m R1=100 kN
15 = aL symmetrically loaded system is at
E d= 4.4 mm midspan. max. moment= LMc
8
. I)= (172 X 10 )(30)
Note: To be safe for both stress and Consider the half of the beam: max. moment= R1(2.5)
69,116x106
elongation, use d = 4.4 mm. max. r:noment = (100)(2.5}


8 =0.0746 m 200kN
max. moment = 250 kN-m
&:7.46 m
~::::::::.·: :;- ::fc
a 15 = PL
l>= PL
AE
350(10) 1
~(0.0015)2
AE .....-- max. moment= LMc
0.0186 = E
where: P = weight of the body max. moment= R1(5)
4
= =
P mg 20(9.81)
E = 1.0648 x 1011
=
P 196.2 N Pa max. moment== {100)(5)
E = 106.48 MPa
I max. moment 500 kN-m
l

I
11
1

J
'f··'l

530 100 f Solved Problems in Engineering Mathematics (2nd Edition) by Tiong & Rojas

' "'.'"'
~I
q·,l:: ' ~ ' '!" ~ ' .,. '* < '

···'·""
~~~:~11',11

Topics
' ~ ~ ., ~ '
Engineering Economics
,A<,.<.:·>.< ~ ' ~ < " <- ' < > " <. ' ,.... "',' ,>
Consurner & Producer Goods
r-1 and Services
!i
II
L_J Necessity and Luxury
~ ,, ""'• <\ < >' ;-! '< N"" ~.... ·~ "<' !" '0 Tue
--, Different Market Situations
.< <f "'""" '*' ,..... •, < .> " •

.
~ ,, y .., ,, ·: ""~ ,... " •' ~ ••••• ;.. • ·'< ., '<: '> ~

·O.~.j.oi·~t*
' < ::> ~ '·•

"'"'".'-~ ''':~'
~ >: ... ~ ,; "'

~ [__J Demand·
Supply
law of ~upply and Demand
~. ~ ~-'".~t·~><'r'ol•c.~~ '''·''

Theory Wed
~ .....« ~ "1:""' :«.,. <)-_.., .• ,.,_ ' 1, "" ~ " .. ·~ i 't .• " ~ ,.. .. ,« -1
Simple
;< "' .., "['<;' .,
~· ~ ~··>"' "'.~ "~ ·.,:.. 1< ·~:>

'< ,~ •r, ~ >' <'i- -~ ~ It- ~ "" • "'.,.. # 4 ·.~.:¥·~ I("" ·,<:<>- ~ '='. >\' ~ ·1· 'I' -t' :~ .,. -<· oi' ,; [ ___J -Ordinary Simple interest
..... -~ ... ,.,.. Problems hu - Exact Sirnple Interest IIi

[-·-]
""'"'«.».'!"•,.,~-~.,..~ ,..,..,.,.,..,.,,.,.."!!~'~'~·· ¢

.<: ;;.- •'i' .. ' '< ~ '!' ';-.,. • ~.

,. ...... ·~ ;-;' :<·


1 ~"' ... "",1 ~ ~ ~ ... A-,, . . , ~ *' ,.. ~ • .,. 1'1 ~ • t ,. ;< '!

;._.,
~ ~ ~ """" « :"·"' ~- ,., "' • ~ >t "!'""' «

Solutiom;
[=]·
Fri
Compound Interest
Continuous Cornpounding
~ ' ,...-._. :!<- ,: •
'< ,. .....

•• 'l ,. ~~ 4 ~ ;
~ ,, ,. < .,... i. ,; • ,.. ~ <o < ........ ;, :,. :::. ~ ,, .,; ,.

[-] l~] Nominal and Effective Rate of


Interest
~ ·~ • ':;' ... II' ~ .• ••· • ~ Jo ~ :::. • •
Notes Sat
, 0 < It. ··-~ <1 ... "

.w.!•at is E_@.!lOmics? Producer goods and services are those


~ "' !II ........ .,,.,_,.,; ~ ' • ~ • ,; '"' .;. ,,. ' '
that are used to produce the consumer
E~:onomics is a sc1ence which deals with goods and services.
!he attainment of the maximum fulfilment
~ ~ if; -4 '> ,• of society's unlimited demands for goods What.l§.l.b!L.Q1fi~I!LIJCe betwemJ.
.. "":- ._
and service. · !'!feces*illY...¥!nr!_Lu~.!Y1
'
<: "" l· ' _>. -~ *" ~ ~ ,. "' ~ ,, ) •( f. < "y•->' I < -~ ... ~ -~ ~

~···<v·<-.~·~··•o<.·<'<-•«~~.<-_,.·_r.,{, «o-~:>"<<'<".,'f,o:_• ~"""<•jo.»h·<<i~:O..,.;.w«


V¥.1:!J!1Js Engineering Economics? Goods and services are divided into two
~%._ ... -~·1,.,,,..-,_.,.-,/«<-";:<jo..'!'~~~ .;..·~~-,,-~,.;->: ..,~~·-··
types, namely necessity and luxury.
Engineering Economics is the branch of
,. ~ f ~ ·~ ~ ., + ,, -.; <". '" ,.. "' '• .... ~ ~ ' .. >i ~ 1;- ~- •i .. I>.;-_... < '- ,, .\ " ' ' ;., '!'
., ' .<.'d < " t( ,_ • •--.
t!conomics which deals with the Necessity refers to the goods and
.1pplication of economics laws and theories services that are required to support
IIJVolving engineering and technical human life. needs and activities.
1< ~ ~ ., "\>
r 11 ojects or equiprnents.

IIVIJ<l(i!_~_G_on~umer and Producea:s


! :.cJ_ot!~!!!l.ci_Services(
Necessity product or staple product is
defined as any product that has an
income-elasticity of demand less than one.
I,
This means that as income rises, ' 'I~ "

consumer goods and services refer to proportionately less income is 'spent on I


~ ,. '·, _.
1/w 1>1oducts or services that are directly such products. Examples qf necessity
11·:.·d l1y pt~opl" to satisfy their wants. products include basicfoodstuffs like
I ·.. JIIlpl, .. ; :lit' food. dotltlrH), shc!ter or bread and rice, clothing, etc.
lit liiH' l"IJ ;I If!

'rT
Day_2_g- Engtneering Economics (Simple and_ Compoun<i_lnterest) 533
532 l 00 i Solved Problems in Engineering Mathematics (2nd Edition) by Tiong & Rojas
E. Absence of all economic friction: A. Few seller and many buyers: The
The price of any commodity or product will
l,illt,,, Luxuries are those goods and services
depend largely on the market situation.
There is a total absence of economic bulk of market supply is in the hands
that are desired by human and will be friction including transport cost from of a relatively few sellers who sell to
1.11.,1111 acquired only after all the necessities have The following is a tabulation of the different
one part of the market to another. many small buyers.

'
~.,,',, been satisfied. market situations:
jljll 1~11 This market situation provides an B. Homogeneous or differentiated
Luxury product is defined as any product assurance of complete freedom on the products: The products offered by
that has an income-elasticity of demand part of both the vendors and the buyers the suppliers may be identical or more
greater than one. This means that as though the latter benefits more from the commonly, differentiated from each
income rises, proportionately more income reduced prices brought about by the other in one or more respects. These
is spent on such products. Examples of competition while more and better services differences may be of a physical
luxury products includes consumer are afforded by the vendors or players in nature, involving functional features,
durables like electric appliances, the industry . or may be purely "imaginary" in the
expensive cars, holidays and sense that artificial differences are
r,,.
..... 1111111 entertainment, etc. Monopoly is the opposite of perfect
competition. This market situation is
created through advertising ahd sales
promotion.
Necessities and luxuries are relative terms
_.,.J because there are some goods and
characterized by the following:
C. Difficult market entry: High barriers
services which may be considered by one Perfect competition (also known as A One seller and many buyers: A of entry which make it difficult for new
person as necessity but luxury to another atomistic competition) refers to the market comprised of a single supplier sellers to enter the market.
person. market situation in which any given selling to a multitude of small,
product is supplied by a very large number independently-acting buyers. What is a Demand?
For example, a man living in the· of vendors and there is no restriction
Metropolis finds a car as an absolute against additional vendors from entering B. Lack of substitute products: There Demand is the need, want or desire for a
necessity for him to be able to go to his the market. are no close substitutes for the product backed by the money to purchase
workplace and back to his home. If the monopolist's product. it. In economic analysis, demand is always
same person lived and worked in another Perfect competition is a type of market
based on "willingness and ability to pay"
city, less populated with adequate means situation characterized by the following: C. Blockaded entry: Barriers to entry for a product, not merely want or need for
of public transportation available, then a are so severe that it is impossible for the product.
car will become a luxury for him. A. Many sellers and many buyers:
other sellers to enter the market.
Since there is a large number of
The demand for a product is inversely
What are the Different Market. selleos and a large number of buyers, There exists a perfect monopoly if the proportional to its selling price, i.e. as the
Situations? each seller and buyer will become single vendor can prevent the entry of all selling price is increased, there will be less
sufficiently small to be unable to other vendors into the market. The demand for the product; and as the selling
The term "market" refers to the exchange influence the price of the product monopolist is in the position to set the price is decreased, the demand will
mechanism that brings together the sellers transacted. market price. increases. Figure 1 below illustrates the
and the buyers of a product, factor of relationship between price and demand.
production or financial security. It may also B. Homogeneous prodm:ts: The
A natural monopoly is a market situation
refer to the place or area in which buyers products offered by the competing
where economies of scale are so Price
and sellers exchange a weli-defined sellers are identical not only in
significant that costs are only minimized
commodity. physical attributes but are aiso
when the entire output of an industry is
regarded as identical by the buyers - supplied by a single producer so that
Buyer or consumer is defined as the who have no preference between the
supply costs are lower under monopoly
basic consuming or demanding unit of a products of various producers.
than under perfect competition and
commodity. It may be an individual

. .:
oligopoly.
purchaser of a good or service, a · C. Free ma1·ket-entry and exit: There
household (a group of individuals who are no barriers to entry or Oligopoly exists when there are so few
impediments to the exit of the existing jp1 2
make joint purchasing decisions), or a suppliers of a product or service that the ........................ !

L
·········~·· ·······6~

government. sellers. act1on of one will inevitably result in a

Seller is defined as an entity which makes D. Perfect information: All buye!s am!
~•imilar action by the other suppliers. This ! !P2
tyrt~ of mark0t sJtuation is characterized by
product, good or service available to buyer ail sellers have complete inforrn''il.ion j ! Demand
till' loiiOWIII<J
or consumer in exchange of monetary · on the prices being asked and otlemd Figure 1
consideration. in all other parts of the mmlwt ·
I':,I
'-~

::f
§.!1..Jgo·-I Solved Problems in .Engineeri;l!~athematics (2r.d Editionl.PY 'I'iong & ~jas Day 22 - Engineering_J::<.:O_I\OIDi<:§._@J!lj)l_e and Compound Interest) 535

Assuming a linear function for the "Under conditions of perfect competition, When there is additional demand without Ordinary simple interest is based on one

'.,,,,,,
"~ ' "
relationship between price and demand, it the price at which any given product will be an additional supply, a new and higher banker's year. One banker's year is
shows that at point 1, the selling price, P, supplied and purchased is the price that price is established as shown in Figure 5. equivalent to 12 months of 30 days each.
is high, thus there is less demand for. the will result in the supply and !he demand Also, 1 banker's year = 360 days.
!l.,l·,
.lljil rwh
product as compared to point 2, where 'the
demand, Dz is great because of lower
. being equal." Price
The value of n is:
selling price, Pz. The relationship between price-supply- + d
demand may be illustrated by combinirlfj n=-
What is a Supply? figures 1 and 2. For general application, 360

Supply is the amount of a product made


available for sale.
the curves for supply and demand are no
longer represented in linear function. ~~'!.I ..... ···•····...
The simple interest is: !lil 1

Price I •·····... Previous ',·_p·· d 'I

-
If the selling price for a product is high, - 1-
Price Demand 360 I
more producers will be willing to work
.iII
harder and risk more capital in order to
reap more profit. However if the seiling
price for a product declines, capitalists will i ~ !'""' Figure 5
Units
Exact simple interest is based on the
exact number of days in a given year. A
' II
I I

,il

r,t··/···············~
not produce as much because of the normal year has 365 days while a leap
smaller profit they can obtain for their labor What is an Interest? year (which occurs once every 4 years) :ji
]':

has 366 days. Unlike the ordinary simple :i:!


and risk. :1111~11111· 1

Suppose a debtor loans money from a interest where each month has 30 days, in
Therefore, the relationship between price I I ! • creditor. The debtor must pay the creditor the type of simple interest, the number of II;,
the original amount loaned plus an ,,
and supply is that they ,are directly
proportional, i.e. the bigger the selli1·1g
price., the more the supply; and the smaller
tl....---;::-------
···· ······ - . ,.,·! ----·--·-
l "' Demand additional sum called interest. For the
debtor, interest is the payment for the use
days in a month is based'on the actual
number of days each month contains in
our Gregorian calendar.
,.1
']''
:;
li
the selling press, the less is the supply. uUOOIV =Demand Units of the borrowed capital while for the ''I
Figure 2 below illustrates the price-supply
relationship. Figure 3
creditor, it is the income from invested
capital.
To determine the year whether leap year
or not, one has to divide the year by 4. If it
il
is exactly divisible by 4, the year·is said to 'II
Price What is Simple Interest? !I
Figure 3 illustrates the price-supply- be leap year otherwise it will be ,,
demand relationship, showing equal considered just a normal year with 365
supply and demand at a given price. Simple interest (I) is defined as the days. However, if the year is a century
. . . . . . . . . . . . . .Sz···························f/
.. / l interest on a loan or principal that is based year (ending with two zeros, e.g. 1700, :'1'

1 When there is additional supply without an


additional demand, a new and lower price
only on the original amount of the loan or
principal. This means that the interest
charges grow in a linear function 'bver a
1800 ... ), the ye;3r must be divided by 400
instead of 4 to determine the year whether
or not a leap year. Hence, year 1600 and
I

/ is established as shown in Figure 4.


period of time. It can be calculated using year 2000 are leap years.
the formula

[
·······~·1··· p2 Price Previous
Supply Under this method of computation of
/ I= Pin interest, it must be noted that under
!pj New normal year, the month of February has 28
Supply
~Supply
days while during leap years it has 29
_ _ j where: P = principal days. Again, the values of n to be used in I
.II

~~JJ
i = interest per period the preceding formulas are as follows:

._ _ _ _ ,.
Figure 2 =
n number of interest period
For ordinary or normal years:
What is the Law of S!ill.P_tl and There are two types of simple interest,
D_emand? t---- · -------·- Units namely ordinary simple interest and exact d
~;irnple interest. n = 365
The law of supply and demand is stated a~
follows: Figure 4 II I ~

"li
,,
,,,

:,y.·
,,
,)

536 , 100 l Solved Problems i~ E!!,qineering Mathematics (2nd Edition). by Tiong &: Rojas Day 22- Engineering_~~nomics (Simple and Compound·Interest) 537

For leap years: What is Compound Interest~ P(1 + i)2 What is Continuous Compounding?
r=--
I P(1 + i)
Compound interest is defined as the The concept of continuous compounding rs
'"'"' " d
interest of loan or principal which is based r = 1+i based on the assumption that cash

'.......' t'l
"'~:
n= 366
"""
lllllll~li'l
not only on the original amount of the loan
or principal but the amount of loan or Using the formula for nth term of a G.P.
payments occur once per year but
compounding is continuous throughout the
What is a Discount? principal plus the previous accumulated an= atfn-1 year.
interest. This means that aside from the an = P(1 + i)(1 + i)n-1
Consider the following case:; wnere principal, the interest now earns interest as The basic equatioh for future worth of
:, discount is involved: well. Thus, the interest charges grow an =P(1+i)n compound interest is

·-·1""'
i'ltiiiiH l~i CASE 1:
exponentially over a period of time.
A. FUTURE AMOUNT, F:. F = P(1 +·i)"

I... .
Compound interest is frequently used in
...J.J A person has a bond or financial security
that is not due yet but payable in some
commercial practice than simple interest,
more especially if it is a longer period 0 1 2 3 n
but for !Tl periods per year
future date, desires to exchange it into an which spans for more than a year. NR)mN
_.,.rJ'' immediate cash. In the process, he will
F=P ( 1+m
accept an amount in cash smaller that the The future amount of the principal may be
face value of the bond. The difference derived by the following tabulation: let x =..!!!..
NR
between the amount he receives in cash t:;i~,:~:J .......................................» ~'1f};~:
(present worth) and the face value of the ,,,,, ''. Cash Flow ..,,, ,, , ''~

bond or financial security (future worth) is


known as discount. The process of
Period

1
Principal

p
Interest

Pi
Total
Amount
P+ Pi=
j.. "
F =P(1+it
F= p( +
1 .;rNR)N I
tt,l''''
,,
converting a claim on a future amount of Pl1 + i) I
money in the present is called -2 P(1 + i) P(1 + i)i P(1 + i)(1 F=P[(1+.;JrR)N
discounting. + i) = p where: P = Principal
(1 + i)2 i = interest per period (in
CASE 2: 3 P(1 + i) 2 P(1 + i)2 i P(1 + decimal)

In a bank loan, a person borrows P 100 for


i)2 (1 + i)
=P(} +
n = number of interest periods
(1 + i)n = single payment compound
but
x~oo
Lim (1 + .!)x
x
=e
1 year with an interest of 10%. The interest J2 amount factor
as computed is P 10. The bank deducts n P(1~
therefore,
the interest, which is P10 from P 100 and
gives the borrower only P 90. The B. PRESENT WORTH, P

.... I
The tabulation above shows that the future
borrower then agreed to repay P100 at the amount (total amount} is just the value P(1
. F ,.,p~<"~
:~;~;/,f
end of the year. The P1 0 that was + i) with an exponent which is numerically 0 1 2 3 n
deducted represent the interest paid in
'advance. In this case, discount also
equal to the period. where: P =principal
represent the difference between the
e =2.71828 ...
present worth (i.e. P90} and the future
It is also observed that compound interest NR =nominal rate
is based on the principles of geometric N= number of years
worth (i.e. P 100}. progression and using such method, the p .......................................... : f:t:r e<NRJN = ccintinuous compounding
total amount after each period are as
compound amount factor
Discount "" Future worth~ present worth follows:
First term, a,= P(1 + i) P- F
' =
Second term, a2 P(1 + i)
2
- (1 + i)n
The present worth of continuous
The rate of discount is the discount on Third term, a3 =
P(1 + i)
3 compounding is
one unit of principal per unit time.
and so on. w~ere: ( : i)" = single payment present F
Let d = rate of discount 1
P'iii~~~(Nl
Solving for the common ratio, worth tactor
1 . d a ,

j
d=1-- ·thus
1+ i '
'1=:::-'- r = -1. il'll
1-:-d a,
ill'
l'"li''!
!wtl
4:"1 ·"' ,.,\1::
1
:j

538 · 1001 Solved Problems ii). Engineering Mathematics (2nd Edition) by Tiong & Rojas Iii'·

,,,,,,,,,
What is the Difference Between nominal rate are equal if the mode of ~.·;,
.··.,
..
...
'',,,
Nominal and Effective Rate of Interest?. compounding is per annum or annually.
I ·.. i
Rate of interest is the cost of borrowing
'~
"' "'
,,,

,,
money. It also refers to the amount -earned. Proceed to the next page for your 22nd
'Ill''"' ' test. Detach and use the answer sheet !1111
1

~II 1rn:1 by a unit principal per unit time.


provided at the last pa.rt of this book. Use
Topics ,l•li

'
~II
There are two types of rates of interest,
namely the nominal rate of interest and the
pencil number 2 in shading your answer.
0 I Engineering Econ~mics II
,,._~1~~~ effective rate of interest. GOOD LUCK I Mon Consumer & Producer Goods .1il
and Services
-·,~.~~~~.'
·-~

_JJ
1~1
Nominal rate of interest is defined as the
basic annual rate of interest while
effective rate of interest is defined as the
0Tue I
Necessity and Luxury
Different Market Situations
'I'
Ii
'Urribia: Demand

_fj
actual or the exact rate of interest earned
on the principal during a one-year period. Did you know that ... the Gregorean
Calendar we are using was named after a
former teacher of law at the University of
I 0
Theory
0Wed
Supply
Law of Supply and Demand
'I
II!l
For example: A principal is invested at 5%

~
compounded quarterly, Bologna, Ugo Buoncompagni who became Simple Interest
Pope Gregory XIII in 1572! In February 24, ,_ - Ordinary Simple Interest
In this statement, the nominal rate is 5% 1582, he issued a Papal edict directing the Problems Thu -Exact Simple Interest

0 I
while the effective is greater than 5% former Julian Calendar be allowed to catch
because of the compounding which occurs
four times a year. The following formula is
used to determine the e.ffective rate of
up with the Lord's Time and that aside
from leap year every four years, leap year
be once in every four centennial years, i.e.
0
Solutions Fri
1
Discount
Compound Interest
Continuous Compounding
interest:

ER = [1+it-1
every 400 years.

~uote:
0Notes
0 Sat
. Nominal and Effective Rate of
I Interest
"Mathematicians are like Frenchmen;
where: m = number of interest period per whatever you say to them they translate it i
i

year into their own language and forthwith, it is C}U: ME Board April 1.995 913: ME Board Aprill993
i = interest per period something entirely different." P 4,000 is borrowed for 75 days at 16% Agnes Abanilla was granted a loan of P
per annum simple interest How much will 20,000 by her employer CPM Industrial
. NR
1=- -Goethe be due at the end of 75 days? Fabricator and Construction Corporation
m With an interest of 6 % for 180 days on the
A. P4,133.33 principal collected in advance. The
or B. P 4,333.33 corporation would accept a promissory
ER=[1+NmRr-1 c p 4,166.67 note for P 20,000 non-interest for 180
II

D. P 4,150.00 days. If discounted at once, find the


proceeds of the note.
· 9I::t: CE Board May 1.997
Note: i = NR if the mode of fl deposit of P 110,000 was made for 31 A. P 18,600 IIIII
compounding is annually days. The net interest after deducting 20% B. P 18,800
withholding tax is P 890.36. Find the rate C. P 19,000
Substituting the values of m and i:

ER =[1+
0 5
·~ J -1
of return <:mnually. D. 19.200
,,,:I
fl. 11.95% 91.4: ECE Board November 1.998
U. 12.75% What will be the.future worth of money 11:
ER =0.0509 c 11.75 o;,, after 12 months, if the sum of P 25,000 is
!

ER =5.09% IJ. 1/.?5 'X, invested today at simple interest rate of


1% per month?
So, the actual interest rate is not just 5%
but 5.09%. However the effective rate and· A. p 30,000
540 100 1 Solved Problems in Engineering Mathematics (2nd Edition) by Tiong & Rojas ··~
','?
•. .. '·
'.,' '.·.•.

d~
Day 22- EnginE!E!ring Economics (Simple and Compound Interest) 541
1111

,,,,,,./ B. P 29,000 money was borrowed and the loan is ~.~~~:,. 923: EE Board October 1993
1
inheritance as of the boy's 6 h birthday, if
C. P 28,000 payable at the end of one year. How much A bank pays one percent interest on the interest is compounded annually?
. ;li.
D. P 27,859 is the actual rate of interest. savings accounts four times a year. The =
Assume i 4%.

"'~"' "I
·I
II''"'
•~lrJ41{
9S5: ECE Board November S999
If you borrowed money from your
A.
B.
12%
14%
effective annual interest rate is

A. 4.06%
A.
B.
P 6,500
P 8,600

'
I friend with simple interest of 12%, find the
present worth of P 50,000, which is due at
C.
D.
10%
19%
B.
c.
1.00%
2.04%
C.
D.
P 5,500
P 7,500
!II:
I,
the end of 7 months. D. 3.36% I
919: ME Board April1998 929: ECE Board April!.999
A. P46,200 A bank charges 12 % simple interest on a 924: ECE Board November 1998 The amount of P 50,000 was deposited in
B. P 44,893 P 300.00 loan: How much will be repaid if The effective rate of 14% compounded the bank earning at interest of 7.5% per
C. P 46,730 the loan is paid back in one lump sum after semi-annually is annum. Determine the total amount at the 'I'll
D. P 45,789 three years? end of 5 years, if. the principal and interest
I
A. 14.49% were not withdrawn during the period?
9Sft: Annie buys a television set from a A. p 408.00 B. 12.36%
merchant who ask P 1,250 at the end of B. P415.00 C. 14.94% A. P 71,781.47 illli
,'I
60 days. Annie wishes to pay immediately C. P 551.00 D. 14.88% B. P 72,475.23
and the merchant offers to compute the D. P 450.00 C. P 70,374.90 II'
'I
cash price on the assumption that money 925: ME Board October 199& D. P 78,536.34
is worth 8% simple interest. What is the 920: EE Board October 1997 An interest rate is quoted as being 7.5%
,·l'
cash price? A man borrowed P 100,000 at the interest
rate of 12% per annum, compounded
compounded quarterly. What is the
effective annu!'ll interest rate?
930: ME Board April199~
Alexander Michael owes P 25,000.00 due
};
A. P 1,233.55 quarterly. What is the effective rate? in 1 year and P 75,000 due in 4 years. He ''il
B. P 1,244.66 A. 7.71% agrees to pay P 50,000.00 today and the il
c. p 1,323.66 A. 3% B. 7.22°/: balance it) 2 years. How much must he
D. P 1,392.67 B. 13.2% c. 15.7a %, pay at th'ti"end of two years if money is
C. 12% D. 21.81 %. worth 5% compounded semi-annually?
917: ME Board April1998 D. 12.55%
It is the practice of almost all banks in the Proble~ 92&: ECE Board April1998 A. P 38,025.28
Philippines that when they grant a loan, 921: ECE Board April!.999 The amount of P 12,800 in 4 years at B. P 35,021.25
the interest for one year is automatically What is the corresponding effective rate of 5 % compounded quarterly is C. P 30,500.55
deducted from the principal amount upon 18% compounded semi-quarterly? D. P 39,021.28
release of money to a borrower. Let us A. P 14,785.34
therefore assume that you applied for a A. 19.25% B. P 15,614.59 93S: ECE Board November 1998
loan with a bank and the P 80,000 was B. 19.48% C. P16,311.26 At an interest rate of 10% compoundec:l
approved at an interest rate of 14 % of c. 18.46% D. P 15,847.33 annually, how much will a deposit of P
which P 11 ,200 was deducted and you D. 18.95% 1,500 be in 15 years? ·
were given a check of P 68,800. Since you 927: ECE Board April1999
have to pay the amount of P 80,000 one 922:ME Board October 1995, Find the present worth of a future payment A. P6,100.0U
year after, what then will be the effective EE Board October 1997 of P 100,000 tobe made in 10 years with B. P 6,234.09
interest rate? Mandarin Bank advertises 9.5 %account an interest of 1.2% compounded quarterly. C. P 6,265.87
that yields 9.84 % annually. Find how D. P 6,437.90
A. 15.90% often the, interest is compounded. A P 30,444.44
B. 16.28% B. p 33,000.00 932: CE Board May !.995
c. 16.30% A. Daily C. P 30,655.68 How long (in year's) will it take money to
D. 16.20% ·B. Monthly D. P 30,546.01 quadruple if it earns 7 % compounded
C. Bi-monthly semi-annually?
918: EE Board April 199ft D. Quarterly 928: EE Board June 1990
A man borrowed P 20,000 from a local On his Gu' birthday a boy is left an A. 20.15
commercial bank which has a simple 11\herit.mce The inheritance will be paid in B. 26.30
II
interest of 1So/o but the interest is to be ;t lump swn of P 10,000 on his 21
51
c. 33.15
deducted from the loan at the time that the lllrlhd<~y Wh<tl r~; tlu~ present value of the D. 40.30 I

~J
t,a,,,,.
,,,,,j
542 l 00 1 Solved Problems in Engineering Mathematics {2"d Edition) by Tkmg & Rojas

9331 ECE Board April :1999


In how many ye.ars is required for P 2,000
A.
B.
P 1,925.00
P 1,860.00
l
[
Day 22 - Engin~~~!l9. Economics (Simple artd Compound Interest) 543

942: ME Boa&-d April :1998


A sum of P 1,000 is invested now and left
946: CE Board November 1994
P 500,000 was deposited 20.15 years ago
to increase by P 3,000 if interest at 12% C. P 2,345.00 for eight years, at which time the principal at an interest rate of 7% compounded
semi-annually. How much is the sum now?

K
l~ compounded semi-annually? D.. P 2,160.00 is withdrawn. The interest has accrued is
t1 'I left for another eight years. If the effective
I ~~.:1 A. 8 annual interest rate is 5 %, what will be the A. p 2,000,000
938: CE Board May 2:996
I B.
c.
9
10
P 200,000 was deposited on January 1,
1988 at an interest rate of 24 %
withdrawal amount at the end of the 16 h
year?
1
B.
C.
P 2,000,150
P 2,000,300
D. 7 compounded semi-annually. How much D. P 2,000,500
would the sum be on January 1, 1993? A. p 706.00
934& ME Board April :1.996 B. P 500.00 947: ME Board October 1995
Consider a deposit of P 600.00 to be paid A. P 401,170 C. P 774.00 In year zero, you invest P 10,000.00 in a
back in one year by P 700:00. What are B. p 421,170 D. P 799.00 15% security for 5 years: During thattime,
the conditions on the rate of interest, i% C. P 521,170 the average annual inflation is.6 %. How
per year cOmpounded annually such that D. P 621,170 943: ME Board April :1998 much, in terms of year zero pesos will be
the. net present worth of the investment is P 1,500.00 was deposited in a bank in the account at maturity?
positive? Assume i 2 0. 939: CE Board November :1996 account, 20 years ago. Today it is worth P
If P 500,000 is deposited at a rate of 11.25 3,000.00. Interest is paid semi-anr1Ually. A. P 12,020
A. 0 :=; i < 14.3 % % compounded monthly, determine the Determine the interest rate paid on this B. P 13,030
B. 0 :=; i < 16.7% compounded interest after 7 years and 9 account. C. P 14,040
C. 12.5% :=; i < 14.3% months. D. P 15,030
D. 16.7% :=; i < 100% A. 3%
A. P 660,550 B. 2.9% 948: ECE Board April1998
B. P 670,650 C. 3.5% By the condition of a will, the sum of P
9351 ME Board October :1995
A company invests P 10,000 today to be C. P 680,750 D. 4% 20,000 is left to a girl to be held in trust
D. P 690,850 fund,by her guardian until it amounts toP
repaid iq 5 years in one lump sum at 12 %
944: ME Board April :1998 50,000. When will the girl receive the
compounded annually. How much profit in
present day pesos is realized? 9401 ME Beard October :1996 A merchant puts in his P 2,000.00 to a money if the fund is invested at 8 % '
Fifteen years ago P 1,000.00 was small business for a period of six years. compounded quarterly?
A. p 7,632 deposited in a bank account, and today it With a given interest rate on the
B. P7,236 is worth P 2,370.00. The bank pays investment of 15 % per year, compounded A. 7.98 years
C. P 7,326 interest semi-anf1ually. What was the annually, how much will he collect at the B. 10.34 years
D. P 7,362 interest rate paid in this account? end of the sixth year? C. 11.57 years
D. 10.45 years
936a ME Board April :1996 A. 3.8% A. P 4,400.00
A firm borrows P 2,000 for 6 years at 8 %. B. 4.9% B. P 4,390.15 949: ME ~ard October 1996
At the end of 6 years, it renews the loan C. 5.0% C. P 4,200.00 You borrow P 3,500.00 for one year from a
for the amount due plus P 2,000 more for D. 5.8% D. P 4,626.00 friend at an interest rate of 1.5 % per
2 years at 8 %. What is the lump sum month instead of taking a !oan from a bank
due? 94:11 ME Board April :1998 945: ECE November 1998 at a rate of 18% per year. Compare how
If P 5,000.00 shall accumulate for 10 years . A man expects to receive P 25,000 in 8 much money you will save or lose on the
A. p 5,355.00 at 8 % compounded quarterly, find the years. How much is that money worth now transaction.
B. P 5,892.00 compounded interest at the end of 10 considering interest at 8% compounded
C. P 6,035.00 years. quarterly? A. You will pay P 155.00 more if you
D. P 6,135.00 borrowed .from the bank.
A. P 6,005.30 A. p 13,859.12 B. You will save P 55.00 by borrowing
9371 ME Board October :1996 B. P 6,000.00 B. P 13,958.33 from your friend.
A deposit of P 1, 000 is made in a bank C. P 6,040.20 c. p 13,675.23 C. You will pay P 85.00 more if you
account that pays 8 % interest D. P 6,010.20 D. P 13,265.83 borrowed from the bank.
D. You will pay P 55.00 less if you
compounded annually. Approximately how
boHowed from the bank
much money wilL be in the account after 10
years?
"

544 lOQl,Solved Problems in Engineering Mathematics (2nd Edition) by Tiong & Rojas

950: ME Board April %996

'•"' '~
What is the present worth of two P 100
payments at the end of the third year and
fourth year? The annual interest rate is
'K""''
'
"~
' "II
l,·,!h
8%.
'1"opics
I A. P 153

.,.,..
'til~
llh'll~ll
B.
C.
D.
P 160
P 162
P 127
0Mon
Engineertng ·Economics
Consumer & Producer Goods

-I'""~
and Services
~.-t iiUMf L.:]
Tue
Necessity and Luxury
Different Market Situations
(,,,!
0 [_j
Demand

-~J
Supply
Theory VIed Law of Supply and Demand

D lQ ~01 Simple Interest


·· Ordinary Simple Interest

.,
Problems Thu - Exact SimplfJ Interest

.~ [] Dist::ount
1 Compound Interest
Solutions Fri j Continuous Compounding
l__; D
r~otes Sat
!
Nominal and Effective Rate of
Interest

ANSWER KEY RATING

91 ·1. A 921. 8 93'1. c 941. c 0 34·-40 Topnotcher

c:J
·i't

912. c 922. D 932.A 942.A


26-33 Passer
913. B 923. B 933.A 943. c
914.
915.
c
c
924.A
925.A
934.8
935.A
944. D
945. D
w
0 20-25 Conditional
916.
917.
A
8
926. 8
927. c
936.·C
937.0
946. B
947. D
<I)~
0 0··19 Failed
\11
918. D 928. c 938. D 948.C il If FAILED, repeat the test.
919. A 929.A 939. 0 949. D
920. D 930. D 940. D 950. A
~'Uk~4&~A«d<t.ii~'
546 i 001 Solved Problems in Engineering: Mathematics ~znct Edition~ by Tiong: & Rojas

• •
Da! 22 -Engineering Economics @imEle and ComEound Interest} 547

1m I By trial and error:


\.;,,.! F~P(1+in) I~ Pin
,,,,,, !

F ~ 4,000[1+0.16(;6~)]
11,200 ~ 68,800(i)(1) N=4 0 6 21
~·:~ i ~ 0.1628
l

:. the mode of interest is quarterly
t ."' F ~ 4,133.33 i ~ 16.28%
I
I [
11
• F1

...,till I a a ER = (1+it -1
p!
F

ER=(1+ 0 ~ 1 J -1
·-r·
=~ ~!dij 0.801 ~ 890.36 I~ 0.16(20,000)
F =P(1 + i)"
I= 1,112.95 I= 3,200 21
10,000 = P(1 + 0.04)
~-· ~~~~~ ER =0.01
'p = 4,388.336
~-l.,,! I= Pin I=Pin ER =1.00%

1,112.95 = 110,000(i{3~~)
3,200 = (20,000- 3,200)(i)(1)
1m
-~J
F, = P(1+ i)"
i = 0.19
i=0.1175 F1 = 4,388.336(1 + o.o4t
i=19% ER = (1 + i)m -1

a
i = 11.75%
a F=P(1+in)
ER = (1+
0 4
·~ J -1
F1 = 5,552.645

Note: From the choices, the nearest


Interest= 0.06(20,000) ER =0.1449 answer is 5,500
F = 300[ 1+ 0.12(3)]
Interest= 1,200 ER = 14.49%
F = 408

a
Proceeds= 20,000-1,200
Proceeds= 18,800

F = P(1 +in)
• ER = (1+it -1

ER = (1+~r -1
• ER = (1+ir -1

ER = ( 1+ 0.~75 r -1
a
F = P(1+i)"
F =50,000(1 + 0.075)
F = 71,781.47
5
i~llllll i

F = 25,000[ 1+ 0.01(12)] ER = 0.1255


ER =o:on1


ER= 12.55% 50,000 p
F = 28,000 ER =7.71%
a t t
F=P(1+in)
7
50,000 = p[ 1+ 0.12( 12)]
ER = (1+it -1

ER = (1+iJ -1
ER = 0.1948
• F = P(1 +i)"

F = 12,800 1+( 0
·~ r4)
5
.
0
1

l
25,000
2
3

Solving for the effective rate per year:


4

1
75,000

p = 46,728.97 F = 15,614.59

a
Note: From the choices, the
nearest answer is 46,730

F = P(1+in)
mJ
ER = 19.48%

ER =(1+ it ·- 1
• F = P(1 + i)"

100,000 =p ( 1+
0
·~ 2 rO)
ER = (1+it -1

'ER=(1+
ER = 0.050625
0
-~~J -1 1 1

ll 1: II. ! I
I,,,
11111 . I
1.2so = P[1 + o.os( °6
)] ER=(1+~r-1 p =30,655.68
360
F = 1,233.55 ( 0 0951''
1.0984 = 1+-·-····j
' n . I lli'llil'll·
I
IIli
I
4

!,lllll\\r'
I]

548 · 1001 Solved Problems in Engineering Mathematics (2"d Edition) by Tiong & Rojas Day 22_- Engineering Economics (Simple and Compound Interest) 549

·~t.,l
p1
-,<-······························,
~ :
m1 1m Interest = F- P
n = 2(1993 -1988) Interest= 11,040.20-5,000
,,,,,I
i F = P(1 + i)"
n =10 Interest= 6,040.20


solooo 2

M
·~ 0 12) "
·'+.''1
~II

I
t t 1 3 4
2, 000 + 3, 000 = 2, 000 ( 1+ ~

2.5 = (1.06)
2
F = P(1 + i)"
F = P(1 + i)"
i' l l F = 200,000(1 +0.12t
2 "

F = 621,170 F = 1,000{1 + 0.05) 8


~p 25,000 75,000 Take log on both sides:
F = 1,477.455
i 2
r··············;
~
:

................................................................:
log2.5 = log(1.06t
ID After the principal is withdrawn:
. p3 log2.5 = 2nlog(1.06) n = 12(7)+9
n =93 Money left= 1,477.455-1000
n = 7.86 years
Money left= 477.455
n"" 8 years
50,000 + p1 = p2 + p3 F = P(1 + i)"

50 OOO + _P_
' (1+i)
2
= 25,000 + 75,000
(1+if (1+i)
4 rm F = P(1 +i)"
F =50 ooo(1 + 0.1125)93
' 12
Let: F1s = total amount after the end of
161h year

&l 000 +- p = 25,000 700 = 600(1 + i)'


F = 1,190,848.73 F16 = P(1 + i)8
2 1
' (1 + 0.050625) (1 + 0.050625) i = 0.1667 8
F16 = 477.455{1 + 0.05)
+ 75,000 i = 16.67% Interest= F- P F16 = 705.42
(1 + 0.050625t Interest= 1, 190,848.73-500,000
p = 39,201.28 ID F = P(1 + i)"
Interest = 690,848.73 Note: From the choices, the nearest
answer is 706

DJI F = P(1 + i)"


F = 3,5oo(1 + o.1
F = 6,265.87
ot
F = 10,000(1 + 0.12}
F = 17,623.42

Profit= F -P
Profit= 17,623.42-10,000
5


Note: From the choices, the nearest
answer is 690,850

• 3,000
F = P(1 + i)"

= 1,500 ( 1 + 4
NR)20(2)


F = P(1 + i)"
40
Profit= 7,623.42 NR)15(2l 2 = (1 + 0.5NR)
2,370 = 1,000 ( 1 + 2
F = P(1 + i)" 1m 6 2 2.37 = (1 + 0.5NR)
30
NR = 0.035
NR =3.5%
0.~7r
F=P{1+i) +P(1+i)
4P = 3.5oo(1+ 8
F = 2,000(1.08) + 2,000(1.08)
2 NR = 0.058
NR = 5.8%
ml F= P(1 + i)"
F = 6,034.66


4 = (1.035t 6
Note: NR = nominal rate F = 2,'000(1+0.15)


Take log on both sides: F = 4,626

log4 = log(1.035)
log4 =2nlog1.035
n = 20.15 years
2
"
F = P(1 + i)"
F = 1,000(1 + 0.08)
F = 2,158.92
Note: From the choices, the
nearest answer is 2, 160.
10

.F = P(1 + i)"

F = 5,000( 1 +
0 8
·~ r
• 25,000
F = P(1 + i)"
0
= p ( 1 + ·~
8
)
8(4)

F=11,~0 p = 13,265.83

,J
ii


550 ··1001 Solved Problems in Engineering Mathematics (2"d Edition) by Tiong & Rojas lijl,

"''"'~
••..,.,t

' ~
·:~1
.,:'1\1
F = P(1+i)"

F = 500,000(1+

F = 200,166
0~7r.tsc2J
• Computing for the amount due after
one year.

a. Borrow money from a friend


;I
I

~ Topics
r..-lill F =P(1 + i)"
0
1
Note: From the choices, tt)e nearest
Engineering Economics
lr!~'~lllij


answer is 200,150 12
F = 3,500(1+ 0.015) Consumer & Producer Goods
.:J:: F =4.185
Mon

0
and Services
Necessity and Luxury
-l,,l let: F =value of the account after 5
years considering there was
b. Borrow money from a bank
Tue Different Market Situations

-~
Demand
P' =
no inflation.
value of the account in
today's peso due to inflation
F =P(1 +i)"
F =3,500(1 + 0.018)
1 0
Theory
0
Wed
Supply
Law of Supply and Demand.
F =4,130
F = P(1 + i)"
5 0 (QJ Simple Interest
- Ordinary Simple Interest
F =10,000(1 + 0.15) Therefore, you will pay P55 less by Problems Thu -Exact Simple Interest
borrowing the money from the bank.
F = 20,113.57
0 0 l;)iscount

F= P'(1 +i)"
20,113.57 = P'(1 + 0.06)
p• = 15,030
5
• P =P1 +P2

p "'_5_ + _li_4
Solutions

Notes
0
Fri

Sat
Compound Interest
Continuous Compounding
Nominal and Effective Rate of
Interest

a F =P(1 + i)" p
(1+it
100
(1+i)
+ ---.!QQ
f1JJ

I
il ~ II i

3
(1+0.08) (1+o.oat iU',III1',]'f:

50,000 = 20,ooo(1+ o~ar P=153 '

2.5 =(1.04t"
.l
'II

I
Take log on both sides: ,,.'~'''*' ' ' " ' " ' '"'"'; ••! ,,

:'II
log2.5 = log(1.o2t" 1111

log2.5 = 4nlog1.02
". '
1[1
n = 11.57 years I
fill
II,
I I
·, ~ '
111·1(1
~ i ~· "" <" ~

.,..
'

;~._.$<;,\,,,,~~~·fO'··~"·,~'<c~'>'"'"' <:.~

,.,. y " "' • "" 'I; « ~ .. .,., ... " -. .<· " " <:o ""· ...... ~ ~ ~- ~ ., • ,, ... i!- ? y .,_ «· ' ' 0 .,. ' < ~ • ' • ••• , ~ • .. < .. ' ..., " '

ili
SSZ 100 l Solved Proble!lls in Engineering Mathe~d Edition) by Tiong & Rojas

~"'I
c.n"'
'a:1
Topics.
l
IIIII

.,:_,,~-r~-· ..,;;;·~-~ ·~· --~-, .... «;:;;<~


D Mon
Annuity
- Ordinary Annuity
- Annuity Due
<I "< <" '• ~ >'- '" ... _.; )> <' ., ·· ·,• .,., .-, ' ~ .~ <. < <: 1 ~ ·<
'• ~- *' ' " ' " ~ ~ ' "* ; ····"
'""·'''''"'""·'! '"
D Tue
- Deferred Annuity
- Perpetuity
Capitalized Cost

.,,,.,~ , , . , _ .. .,..
~., ... _~--o·~-~~-*~
-r_i_...

""•""'"

, ..,.._,_,,,<;.JI<W< ... ~.>~"'1'«,..-'<'~{,·<-<..tX<>'><~"-"


_,O)_.,,..~,,._~,. ·~"'·~.{+~

·~
,,..,

~ ~· }, ~ • ;< 'v•,,
I~
Theory
D VVed
Annual Cost
Bond

' ( y' ·~ ~· " ~ } > l _;t •


"'""

"" ( "" ' " /.


~!l.-\-<f.Y"*1<<t1<.·~~·~J<>~-,.,

;' N (
~·~*·~ ••·' ·~
[] D
Problems Thu
Depreciation
··Straight Line Method
-Sinking Fund Method

~ ~. ~ ~ ' ",;,· ., '· ,.


"' ,. ., ~ ' ' ,, <· ·"' ' ., ·~ ,

\)
"' ~

I'.'>~~~,!-.'«~<
'< < ... ·' -~ .,.. ~ "' • ~ ....• ~ .,. ': ~ ·, •

t•f'<-.<'''< o~-.,
~ ., "" ,. "

• l '1- ' ~-" ~ D @d


Solutions Fri
-Declining Balance Method
- Sum-of--Years Digit Method
Break-Even Analysis

>.~.'t'
0 ~
••••

' ' '~


v

O, h ' ~
\--c;·\·1·,

A ,i' ) \
D D
Notes Sat
Legal Forms of Business
Organization

What is Annuity? Let A be the periodic or uniform


paynent and assuming only four
' " h ' ,_""' ~
Annuity is· defined as a series of equal payments:
payments occurring at equal interval of

t-rll
time. When an annuity has a fixed time 0 1 2 3 4
•. •,<'· ' ·' ~ "•, ' '• .-· ,, ' « .• ,, ....... ' '·..: ., .. < ~ ~ ~ •• '
span, it is known as annuity certain.
,;"..;_,,.;._,,,
,. ' ~ .. " ,. >. ,,.,· ' ' •

!J Types of annuity:
.; e. ~,

·~ 1. Ordinary Annuity
.. ~-,, ' ~ "
·~·
!~ 2. Annuity Due
A A A A
,a 3. Deferred Annuity
. ~ ........,... A(1+ i)
4. Perpetuity
'j"'-"'.'.,..
: ................... -> A(1 + i)2
.' ~ ~ .. 1. Ordinary annuity is a type of annuity ~ ............. ,. ...............,._ A(1+ i) 3
where the payments are made at the
end of each period beginning from the
first period. Let: a1 =A
a2 = A(1+i)
Derivation of formula for the sum of a3 = A(1+il
ordinary annuity: a4 = A{1+i) 3
~'

1
F

554 .1001 Solved Problems in Engineering Mathematics (2"d Edition) by Tio~g & Rojas Day 23 -Engineering Economics (Annuity, Depreciation, Bonds, etc.) 555

ta,,U
,,.,,,,.,w

M. :~l
Annuity is based on the principles of
compound interest. Hence computation of
the sum of annuity may be done using the
formulas for geometric progression.
PRESENT WORTH OF ORDINARY
ANNUITY:

0 2 3 .. . A'
. . .I I I I
0 1 2 3 4 5--·n interest of the first cost and the annual
operating and maintenance costs, or

Annual cost =ann1.1al depreciation cost

.lIII
'"'~I + interest of first cost
JH~
Solving for common ratio: A A A A + annuaLoperating cost

• r = a2
a, A A A A
Cash Flow of Deterred Annuity
+ maintenance cost

What is a Bond?
r = A(1+i) 4. When an annuity does not have a fixed
..........................~ F time span but continues indefinitely,
A Bond is a financial security note issued by
then it is referred to as a perpetuity. businesses or corporation and by the ·
r =1+i p -«····· ...............................: The sum of a perpetuity is an infinite government as·a means of borrowing long-
value. term fund. It may also be defined as a
Solving for the sum:
Using compound interest formula: long-term note issued by the lender by the
PRESENT WORTH OF PERPETUITY: borrower stipulating the terms of
S=a,(r"-1) P=-F-
(1 +i)" repayment and other conditions.
r-1
0 1' 2.3 4···0CJ Bonds do not represent ownership of a
S= A[(1+it -1]

!Tlll
A[(1+i)" -1] business or corporation and therefore not
1+i-1 But: F = ---=-----""" entitled to share of the profits. The
4 i bondholder has no voice in the affair of the
S= A[(1+i) -1] Substituting ~he value ofF: business. However the bondholder has a
i A A A A A more stable and secured investment than
does holder of common or preferred stock
SUM OF ORDINARY ANNUITY: p = A[(1+i)" -1] p «!•• .. •················· .. •• ................... ; of the business or corporation.
i(1+ i)"
0 2 3 ... n P=~ Bonds are issued in certain amounts

.IIII A A A A
where:
[<1 + i)" -1]
=-----""" = uniform series present
i(1 + i)"
worth factor
where: i = interest per period
A = uniform payment
i known as the face value or par value of
the bond. When the face value has been
repaid, normally a~ maturity, the bond is
said to be redeemed or retired. Bond
rate is the interest rate quoted in the bond.
I

What is a Capitalized Cost? The following illustrates the normal life


··························~ f'' 2. Annuity due is a type of annuity cycle of a bond.
where the payments are made at the Capitalized cost of any structure or
beginning of each period starting from property is the sum of its first cost and the
the first period. present worth of all costs for replacement,
. . Af(1+.it,.,-1].
F,;:;:. l . . operation, and maintenance for a long time
i 0 1 2 3···0·1 n or forever, or '

where: i = interest per period


n = number of periods
A = uniform payment
l Il II.
A A A A A
Capitalized cost =First cost +
Cost of perpetual
maintenance Jtc!>~~
Lender Borrower
Cash Flow of Annuity Due What is an Annual Cost?
[<1+i)"-1]
-.- - = uniform series compound
I 3. Deferred annuity is a type of annuity Annual cost of any structure or property is
amount factor where the first payment does not the sum of the annual dFmr<>,..,i::~tion cost,
begin until some later date in the cash
flow. i
)II
' '
II
Day 23- Engineering Economics (Annuity, Depreciation, Bonds, etc.) 557
· 100 1 Solved Problems in Engineering Mathematics (2nd Edition) by Tiong &Rojas
What is Depreciation? 2. SINKING FUND METHOD
Vn
la . l Depreciation is the reduction of fall in the
In this method of computing
depreciation, it is assumed that a
.....,1 value of an asset or physical property sinking fund is established in which l !i fl

~.
'Iii

0~
during the course of its working life and funds will accumulate for replacement
·'~I \, 2 3 4 ··· n due to the passage of time. purposes.
0
! l ! l l
1111

f~
• Lender Borrower
Zr Zr Zr Zr Zr
Types of Depreciation:

A. Physical Depreciation is' due to the


reduction of the physical ability of an
Annual depreciation charge, d

:~ ~-·: : : : : :.·.·:.·.·:·.~·: ·.~:·:~·-~: : : : : ~


equipment or asset to produce
results.

B. Functional Depreciation is due to the where: Co = first cost

(!)~
reduction in the demand for the Cn = cost after "n" years
function that the equipment or asset (salvage/scrap value)

f ~ 1~oll1
was designed to render. This type of n = life of the property
Let V n = value of bond n years before depreciation is often called
(!) redemption obsolescence. Book value at the end of "m" years of
Lender C!J Borrower using, Cm !
i'
Vn = p1 + p2 7 Eq. 1 Methods of Computing Depreciation:
Cm =Co-Om~ '

Using formula for present worth of annuity: 1. STRAIGHT LINE METHOD


4. Borrower redeems bond after n years,
p~ys principal& gets back
In this method of computing where: Dm = total depreciation after
,,..,..,..i,fi,...-.+,., p _ A[(1+i)" -1] depreciation, it .is assumed that the loss "m" years
1
i(1+i)" in value is directly proportional to the

f4=&~
-

zr[(1+i)" -1]
age of the equipment or asset.
Dm = d[(1+it -1]
lend« 0 =*' l§,[\ {:J, \ ~ ;;;==-
Borrower
P1 = i(1+ i)"
Annual depreciation charge, d
3. DECLINING BALANCE METHOD
i

Using the formula for present worth of In this method of computing


compound interest: ·d- Cu ~en
---~ depreciation, it is assumed that the
Value of a bond is the present worth of all n annual cost of depreciation is a fixed
p =-F-
the amounts the bondholder will receive 2 (1 + i)" percentage of the book value at the
through his possession of the bond. The beginning of t~e year. This method is
two payments that the bondholder will p =--
c where: Co = first cost sometimes known as constant
2 (1 + i)" Cn = cost after "n" years percentage method or the Matheson
receive are the following:
(salvage/scrap value) Formula.
A. Periodic payments as interest of the Substituting in Eq. 1 n = life of the property
bond until it is redeemed. Matheson Formula:
Book value at the end of "m" years of
B.. Single payment upon maturity of _ zr[(1+i)" -1]+~ using, Cm
bond. This payment is usually equal
to the par value of the bond.
Vn - i(1+ i)n (1 + 1)1'1
Crn=C0 -Dm
k=1--t£:.
VCo'
or
vc;
.k=1--&.m ·

Derivation of the formula for the value of a where: Z = par value of the bond
bond: r = rate of interest on the bond The value k is the constant percentage.
per period where: Dm = total depreciation after Hence k must be decimal and a value less
C = redemption price of bond "m" years than 1. In this method, the salvage or
i = interest rate per period Dm d(m)= scrap value must not be zero.
n = nuf11ber of years before
redemption

v!.~
1 ~r
,; I
558 .1001 Solved Problems in Engineering .Mathematics (2"d Edition) by Tiong & Roj~ Day 23 -Engineering Economics (Annuity, Depreciation, Bonds, etc.) 559 1
I. I

4. SUM-oF-YEARS' DIGIT (SYD) Proceed to the next page for your last test.
~.,.1 METHOD Detach and use the answer sheet provided
...... Respective depreciation charges:
revenue at the last part of this book. Use pencil
number 2 in shading your answer.
'.lll
K: lg,
~,1, First year: ·.. .•.· ...
.· o1 .~{C0 ~C"}'!~e~rs
,.'n GOOD LUCK!

I '' ; .,,."· ''·''

'arrtbta:
Did you know that. .. there are three
n-1 problems in Geometry, which attracted the
Second year:
~~ ~(Co .,. Cn) ~)ears interest of mathematicians in the ancient
times, considered as "impossible
problems". They are the following:
n-2
. daf"T(Co -C")kY~iilrs
Third year:
production 1. Duplication of a cube -: to construct a
cube whose volume shall be twice that
.and so on ... of a given cube .
What are the Legal Forms of Business
Book value at the end of ".tJ:~ .. years of Organization? 2. Squaring a circle - to construct a
using, Cm ·square whose area shall be equal to
The legal forms of business organizations the area of a given circle.

¢ 11, =C0 .i(d1 +d2 ·-i-,,.:~t.d"') . are the following:


3. Trisection of an arbitrary angle- to
1. Sole proprietorship.- considered as construct an angle that is exactly one-
third of a given angle.
Sum of year's digit, 2: years t~e simplest type of business
organization wherein the firm is
owned and controlled by a single
.~yea($ =!i(n + 1) person. <!auote:
2 "No more fiction; we calculate; but that we
2. Partnership- is a firm owned and
may calculate, we had to make fiction
controlled by two or more persons
first."
who are bind to a partnership
What is a Break-Even?
agreement.
-Nietzsche
Break-even refers to the situation where
3. Corporation - is a firm owned by a
the sales generated (income) is just
group of ordinary shareholders and
enough to cover the fixed and variable
the capital of which is divided up to
cost (expenses). The level of production
the number of shares. It is also
where the total income is equal to the total
defined as a distinct legal entity
expenses is known as break-even point.
separate from the individuals who
owns it and can engage in any
Break-even chart is a diagram which
business transaction which a real
shows relationship between volume and
person could do. This is sometimes
fixed costs, variable costs, and income.
known as joint-stock company or a
The following is an example of a break-
cooperative.
even chart.

I
.I,] ,,

'I
i!
I 'j I; .
t
.' - "
______Day 23- Eng-ineering Economics (Jinnuity, Deprecia!~!~.:?o~:!E~,~~l~~~
D. P 40,544.29 9S9s ME Board O~t(llbell' 1!:.9'94

""'a:a'tn'"' 9551 ME Board April :1998


How much must be deposited at 6% each
If you obtain a loan of P 1M at the rate of
12% compounded anrually in order to
build a house, how much must you p.ay

I
_,,.,_ .... _ Topics
year beginning on January 1, year 1, in
order to accumulate P 5,000 on the date of
the last deposit, January 1, year 6?
monthly t\) amortize the loan within a
period of ten years?

D Man
Annuity
-Ordinary Annuity A.
B.
P 751.00
P 717.00
A.
8.
G.
P13,994.f7
p
P 15,855.45
- Annuity Due
D Tue
- Deferred Annuity
- Perpetuity
c.
D.
p 715.00
P 725.00
D.

9~0~
P ·t2,900.25

Ecm: ha1·d Ap~·!i~ l[9'91i


Capitalized Cost 956: ECE Board November 1.~98 How rnuch mljst you invest today in order

[] D
Theory Wed
Annual Cost
Bond
A debt of P 10,000 with 10 % interest
compounded semi-annually is to be
amortized by semi-annual payment over
to withdraw P 2,000 annually for 10 years
if the interest rate is 9°/c.?

D
Problems Thu
Depreciation
-Straight Line Method
- Sinking Fund Method
the next 5 years. The first due in 6 months.
Determine the semi-annual payment

A. P 1,200.00
A
B.
C.
o.
P 12,853.~12
P12,f581.37
P 12,::>65.32
P 12,Ba5.32

D ~
Solutions Fri
- Declining Balance Method
-Sum-of-Years Digit Method
Break-Even Analysis
B.
C.
D.
P ·t,295.05
P1,193.90
P 1,400.45
9fl>:K.f ECE Bom!rd ApwH fi."~
A person buys a piece of !ot for P 'IOC,OOO

D []
dow.1payment and ·i 0 deferred s~~mi­
Legal Forms of Business 9571 EE Board October •997 annual payments of P 8,000 each, startmg
Organization A young engineer borrowed P 10,000 at three years from now. What is the 1-m~sent
Notes Sat 12% interest and paid P 2,000 per annum value of the investment if the rate of
for the last 4 years. What does he have to interest is 12% compounded semi-
pay at the end of the fifth year in order to
951: EE Board October 1997 953: ME Board Oetober :1.996 pay off his loan?
A man purchased on monthly installment a You need P 4,000 per year for four years {1, p 134,866.80
P 100,000 worth of land. The interest rate to go to college. Your father invested P A. P 6,919.28 B. P 143,999.08
is 12 % nominal and payable in 20 years. 5,000 in 7 % account for your education B. P 5,674.00 c. p 154,696.80
What is the monthly amortization? when you were born. If you withdraw P C. P 6,074.00 D. P 164,969.80
4,000 at the end of your 1ir., 181h, 191h and D. P 3,296.00
A. P 1,101.08 201h birthday, how much will be left in the 9<&:2: ClE B~ilard l!llay :!998
B. P 1,121.01 account at the end of the 21st year? 9S8s EE Board April 199'7 A man loans P '187,400 from a bank with
C. P 1,152.15 Mr. Cruz plans to deposit for the education interest at 5% compounded annually. He
D. P 1,128.12 A. P 1,700 of his 5 years old son, P 500 a.t the end of agrees to pay his obligations by paying 8
B. P 2,500 each month for 10 years at 12% annual equal annual rayments, the first being due
95Z: ECE Board April :1998 C. P 3,400 interest compounded monthly. The amount at the end of 10 years. Find the annu<:~i
Money Qorrowed today is to be paid in 6 D. P 4,000 that wili be available in two years is payments. ·
equal payments at the end of 6 quarters. If
the interest is 12% compounded quarterly. 954: ECE Board November 1998 A. P 13,000 A. P 44,982.04
How much was initially borrowed if What is the accumulated amount of five B. p 14,500 B. P5G,143.03
quarterly payment is P 2000.00? year annuity paying P 6,000 at the end of C. P 13,500 C. P 62,334.62
each year, with interest at 15 % D. P 14,000 D. P 38,236.04
A. P 10,834.38 compounded annually?
B. P 10,382.90
C. P 10,586.99 A. P 40,454.29 .
D. P 10,200 . 56 B. P41,114.29
C. P 41,454.29
,.
562, 100 l Solved Problems in Engineering Mathematic:; (2"d Edition) by Tiong & Rojas Day 23- EngineeringEconomics (Annuity, Depreciation, Bond, etc.) 563

~ A housewife bought a brand new 967: CE Board November 1996 971: EE Board April :1997 97Sz A fund donated by a wealthy person

""'K'•·i
lP
"'~I
•·I'
washing machine costing P 12,000 if paid
in cash. However, she can purchase it on
installment bas1s to be paid within 5 years.
If money is worth 8% compounded
annually, what is ner yearly amortization if
A man inherited a regular endownment of
P 100,000 every end of 3 months for 10
years. However, he may choose to get a
single lump sum payment at the end of 4
A small machine has an initial cost of P
20,000, a salvage value of P 2,000 and a
life of 10 years. If your cost of operation
per year is P 3,500 and your revenues per
year is P 9,000, what is the approximate
to I lEE to provide annual scholarships to
deserving EE students. The fund will grant
P 5,000 for each of the first five years, P
8,000 for the next 5 years and p 10,000 for
each year thereafter. The scholarship will
years. How much is this lump sum if the
u all payments are to be made at the
beginning of each year?
cost of money is 14% compounded
quarterly?
rate of return (ROR) on the investment? start one year after the fund is established.
If the fund earns 8% interest, what is the
A. 25.0% amount of the donation?
A. p 2,782.85 A P 3,802,862 B. 22.5%
B. P 2,872.58 B. P 3,702,939 C. 23.9% A. P 101,605.71
C. P 2,400.00 C. P 3,502,546 D. 24.8% B. P 10i,505.21
D. P 2,827.58 D. P 3,602,431 C. P 100,506.21
97:2.: CE Board November 1996 D. P 99,601.71
964: ME Board October 1996 968: ME Board April 1998 A man paid 10% down payment of P
Mr. Ayala borrows P 100,000 at 10% A parent on the day the child is born 200,000 for a house and lot and agreed to 976: ME Board April 1998
effective annual interest. He must pay wishes to determine what lump sum would pay the balance on monthly installments A company issued 50 bonds of P 1,000.00
back the loan over 30 years with uniform have to be paid into an account bearing for "x" years at an interest rate of 15% face value each, redeemable at par at the
monthly payments due on the first day of interest at 5 % compounded annually, in compounded monthly. If the monthly enq of 15 years to accumulate the funds
each month. What does Mr. Ayala pay order to withdraw P 20,000 each on the installment wasP 42,821.87, find the value required for redemption. The firm
eoch month? child's 18th, 19th. 20th and 21•t birthdays. of)(? established a sinking fund consisting of
How much is the lump sum amount? annual deposits, the interest rate of the
A. P 870.00 A. 11 fund being 4 %. What was the principal in
B. P 846.00 A. P35,941.73 B. 9 the fund at the eno of the 1'2th year?
C. P 878.00 B. P33,941.73 C. 5
D. P 839.00 C. P 30,941.73 D. 7 A. p 35,983.00
D. P 25,941.73 B. P 38,378.00
965: ME Board April 1998 97~: ME Board April 1998 C. P 41,453.00
A house and lot can be acquired by a 969: ME Board April :1998 A manufacturing firm wishes to give each D. P 37,519.00
downpayment of P 500,000 and a yearly An instructor plans to retire in exactly one 80 employees a holiday bonus. How much
payment of P 100,000 at the end of each year and want an account that will pay him is needed to invest monthly for a year at 977: ME Board Aprii199:Z
year for a period of 10 years, starting at P 25,000 a year for the next 15 years. 12% nominal interest rate compounded A unit of welding machine cost P 45,000
the end of 5 years from the date of Assuming a 6 % annual effective interest monthly, so that each employee will with an estimated life of 5 years. Its
purchase. If money is worth 14% rate, what is the amount he would need to receive a P 2,000 bonus? salvage value is P 2,500. Find its
compounded annually, what is the cash deposit now? (The fund will be depleted depreciation rate by straight-line method.
price of the property? after 15 years). A. P 12,608
B. P 12,610 A. 17.75%
A. P810,100 A. p 249,000 C. P 12,600 B. 19.88%.
B. P 808,811 B. P 242,806 D. P 12,300 C. 18.89%
C. P 801,900 C. P 248,500 D. 15.56%
D P 805,902 D. P 250,400 974: CE Board November 1995
Find the present value in pesos, of a 978: EE Board A.pril :1997
'!i!Wo: ME Board April 1998 970: EE Board October :1997 perpetuity of P15,000 payable semi- A machine has an initial cost of P 50,000
A piece of machinery can be bought for P An investment of P 350,000 is made to be annually if money is worth 8% and a salvage value of P 10,000 after 10
10,000 cash or for P 2,000 down and followed by payments of P 200,000 each compounded quarterly. years. Find the book value after 5 years
payments of P 750 per year for 15 years. year for 3 years. What is the annual rate of using straight-line depreciation.
What is the annual interest rate for the return on investment for the project? A. P 372,537
time payments? B P 374,977 A. p 12,500
A. 41.7% C. P 373,767 B. P 30,000
A. 4.61% B 32.7% D. P 371,287 C. P 16,400
B. 3.81% C. 51.1% D. P 22,300
C. 5.71% D. 15%
D. 11 0 '%
'
'•l.
')

564 1001 Solved Problems in Engineering Mathematics (2nd Edition) by Tiong & Rojas Day 23 - Engineering Economics (Annuity, Depreciation, Bond, etc.) · 565

979: ME Board October 1992 98~: An equipment costs P 10,000 with a 987: ME Board April :1998 991.: CE BOard November :1996
~ . 1.
,,,,,, The initial cost of a paint sand mill,
including its installation, is P 800,000. The
salvage value of P 500 at the end of 10
years. Calculate the annual depreciation I
• I
A company purchases an asset for P
10,000.00 and plans to keep it for 20
At 6%, find the capitalized cost of a bridge
whose cost is P 250M and life is 20 years,
BIR approved life of this machine is 10 cost by sinking fund method.at 4% years. If the salvage value is zero at the if the bridge must be partially rebuilt at a

K:'ll'
'
~~.1
1
111 years for depreciation. The estimat~d
salvage value of the mill is P 50,000 and
interest. end of 20th year, what is the depreciation in
the third year? Use SYD method.
cost of P 1OOM at the end of each 20
years.

u the cost of dismantling is estimated to be P


15,000. Using straight-line depreciation,
A.
B.
P 791.26
P 950.00 A. p 1,000.00 A. P 275.3M
what is the annual depreciation charge C. P 971.12 B. P 857.00 B. P 265.5M
and what is the book value of the machine D. P 845.32 C. P 937.00 C. P 295.3M
at the end of six years? D. P 747.00 D. P 282.1M
984: CE Board November :1995
A. p 74,500 ; p 340,250 A machine costing P 720,000 is estimated 988: ECE Board Aprilt.999 992: CE Board May :1997
B. P 76,500 ; P 341,000 to have a book value of P 40,545.73 when A Telephone company purchased a A corporation uses a type of motor truck
C. P 76,500 ; P 342,500 retired at the end of 10 years. Depreciation microwave radio equipment for P 6 million, which costs P 5,000 with life of 2 years
D. P 77,500 ; P 343,250 cost is computed using a constant freight and installation charges amounted and final salvage value of P 800. How
percentage of the declining book value. to 4% of the purchased price. If the · much could the corporation afford to pay
980: CE Board November :1997 What is the annual rate of depreciation in equipment will be depreciated over a for another type of truck of the same
The cost of equipment is P 500,000 and %? period of 10 years with a salvage value of purpose whose life is 3 years with a final
the cost of installation is P 30,000. If the 8%, determine the depreciation cost during salvage value of P 1,000. Money is worth
salvage value is 10% of the cost of A. 28 the 5th year using SYD. 4%.
equipment at the end of 5 years, B. 25
determine the book value at the end of the C. 16 A. P 626,269.10 A. P 8,450.66
fourth year. Use straight-line method. D. 30 B. P 642,786.07 B. P7,164.37
C. P 638,272.08 C. P 6,398.24
A. P 155,000 985: CE Board May :1996 D. P 627,989.90 D. P 9,034.56
B. P 140,000 A machine costing P45,000 is estimated
c. p 146,000 to have a book value of P 4,350 when 989: ME Boaa·d April :1998 99~: ME Board October :1995
D. P 132,600 retired at the end of 6 years. Depreci~:~tion An asset is purchased for P 9,000.00. Its A company must relocate one of its
cost is computed using a constant estimated life is 10 years after which it will factories in three years. Equipment for the
98:1: ME Board Apriil1998 percentage of the declining book value. be sold for P 1,000.00. Find the book loading dock is being considered for
An asset is purchased for P 500,000.00. What is the annual rate of depreciation in value during the first year if sum-of-years' purchase. The original cost is P 20,000,
The salvage value in 25 years is P %? ·digit (SYD) depreciation is used. the salvage value of the equipment after
100,000.00. What is the total depreciation three years is P 8,000. The company's
in the first three years using straight-line A. 33.25% A. P 8,000.00 rate of return on the money is 10%.
method? B. 32.25% B. P 6,500.00 Determine the capital recovery rate per
C. 35.25% c. p 7,545.00 year.
A. p 48,000 D. 34.25% D. P 6,000.00
B. P 24,000 A. P 5,115
C. P 32,000 986: ECE Board November ll.998. 990: EE Board April :1997 B. P 4,946
D.: p i6,000 ABC Corporation makes it a policy that for The maintenance cost for a sewing C. P 5,625
any new equipment purchased, the annual machine this year is expected to be P 500. D. P 4,805
98:&: ME Board April 1:998 depreciation cost should not exceed 20% · The cost will increase P 50 each year for
A machine has an initial cost of P 50,000 of the first cost at any time with no salvage the subsequent 9 years. The interest is 8 9941 EE Board October 1998
and a salvage value of P1 0,000.00 after value. Determine the length of service life % compounded annually. What is the The annual maintenance cost of a
10 years. What is the book value after 5 necessary if the depreciation used is the approximate present worth of maintenance machine shop is P 69,994. If the cost of
years using straight line depreciation? SYD method. for the machine over the full 10-year making a forging is P 56 per unit and its
period? selling price is P 135 per forged unit, find
A. p 35,000.00 A. 9 years the number of units to be forged to break-
B. P 25,000.00 B. 10 years A. P 4,700 even.
C. P 15,500.00 C. 12 years 13. p 5,300
D. P 30,000 00 D. 19 years C. P 4,300 A. 886 units
D. P 5,500 B. 885 units
566 :1001 Solved Problems in Engineering Mathematics (2nd Edition) by Tiong & Rojas Day 23- Engineering Economics (Annuity, Depreciation, Bond, etc.) 567

C. 688 units 998: ME Board October 1990 10011 ME Board April1998


A company which manufactures electric
,,,,, D. 668 units Compute for the number of locks that an

""'a:a
(/l~
995: CE Board May 1998
A manufacturer produces certain items at
a labor cost of P 115 each, material cost of
ice plant must oe able to sell per month to
break even based on the tbllowing data:
Cost of electricity per block- P 20.00
Tax to be paid per block- P 2.00
motors has a production capacity of 200
motors a month. The variable costs are P
150.00 per motor. The average selling
price of the motors is P 275.00. Fixed
Real Estate Tax- P 3,500.00 per month costs of the company amount to P 20,000
I P 76 each and variable cost of P2.32
each. If the item has a unit price of P 600, Salaries and Wages- P 25,000.00/month per month which includes taxes. The
number of motors that must be sold each
how many number of units must be Others- P 12,000.00 per month
manufactured each month for the Selling price of ice - P 55.00 per block month to break even is closest to:
manufacturer to break even if the monthly
overhead is P 428,000. A. 1228 A. 40
B. 1285 B. 150
c. C. 80
A. 1053 1373
B. 1138 D. 1312 o. 160
C. 946
D. 1232 9991 EE Board October 1997
The annual maintenance cost of a
9961 ME Board April1996 machine is P 70,000. If the cost of making
Steel drum manufacturer incurs a yearly a forging is P 56 and its selling price is P
fixed operating cost of$ 200,000. Each 125 per forged unit. Find the number of
drum manufactured cost $160 to produce units to be forged to break even.
and sells $ 200. What is the
manufacturer's break-even sales volume A. 1015 units
in drums per year? B. 985 units
C. 1100 units
A. 1250 D. 1000 unit!;
B. 2500
C. 5000 1000: ME Board Aprill998
D. 1000 XYZ Corporation manufactures bookcases
that sells for P 65.00 each. It costs XYZ
9971 JRT Industries manufactures Corporation P 35,000 per year to operate
automatic voltage regulators at a labor its'l)lant. This sum includes rent,
cost of P 85.00 per unit and material cost depreciation charges on equipment, and
of P 350.00 per unit. The fixed charges on salary payments. If the cost to produce
the business are P 15,000 per month and one bookcase is P 50.00, how many cases
the variable costs are P 20.00 per unit. If must be sold each year "tor XYZ to avoid
the automatic voltage regulators are sold taking a loss?
to retailers at P 580.00 each, how many
units must be produced and sold per A. 2334
month to breakeven? B. 539
C. 750
A. 104 D. 2333
B. 200
C. 120
D. 150

..~
~'
;I
Day 23- Engineering Economics (Annuity, Depreciation, Bonds, etc.) 569. II
I

4.1
ID F1 =P1(1 + i)"
21
012345 n F1 = 500(1 + 0.07)
1&.1
lllll"'""""""""'' "'""""'""'! F1 =20,702.81
K':ll'
I •rlr·:··'
e '] rnuity
.f!~ ..
) '

;~A
~~ :•\

t_
!.111on
Topics

Ordinary Annuity
Annuity Due
p
AAAAA
~...1. . 1. . L. . L. .L. . . . . . . . . . . . . . ..i
A
F2 =
A((1+i)"-1]
.
I

F. - 4000[(1+0.07) -:-1]
4

----=-.:..-o.=o1--'---=
0 I-
2 -

Deferred Annuity P= A[(1+i)"-1J F2 = H, 759.772


Tue Perpetuity (1 + i)" i
l"'l L_J
--~ \Ned
Capitalized Cost
!! i!._nnual Cost
1 Bond 100,000 =
Alr(1 +0.12.) 12(20)_1]·
12 .
F3 =F3 (1 + i)1
F3 =17,759.772(1+0.07)
Theory
r-·--~ Dc1preciation 0.12)12(20) F3 =19,002.95
[~ ( 1+-- i
LJ -Straight Line Method
A=1,101.08
12
Money left = F1 - F3
Thu


Problems -Sinking Fund Method
r~-·o Money left= 20,702.81-19,002.95
[I] ~
- Declining Balance Method
- Sum-of.. Years Digit Method Money left= 1,699.86"' 1, 700
Solutions Fri

[J (._J I
Break-Even Analysis
Legal Forms of Business
P= A[(1+i)"-1]
(1+i)"i
111 F = A[(1+i)"-1]
1 Organization
Notes Sat i
6
P-= 2.ooo[(1+T) -1J 6000[ (1 + 0.15) 5 -1]
F = ----'=-'-------'-------=!
~---·---·· ·---·----· 6 0.15
1 0.12) .
ANSWER KEY · RATING ( +4 I F = 40,454.29

I 951'
954.A
A
952. J\
I 953. P..
I 955. B
. 956. B
957.A
964.
965.
966.
967.
968.
969.
970. B
D
B
A
B
c
B
977. c
978. B
979. 8
980.C
981. A
9!32. D
983.A
990. A
991. C
992. B
993.C
994.A
995.A
996. c'
t
c:J·43-51 Topnotcher

[~
c:J
0
33-42 Passer

25-32 Conditional

0-25 failed
•t
p = 10,834.38

p1 ............................................................Jo-jF1 ·
• F = A((1+i)"-1]
i
5,000 = A[(1 +0.06)
6
-1]
958.C 971. D 984. B 997. c If FAILED, repeat the test. . 0.06
0 2~


959.A 9l2.C 985. B 998.A 16 17 18 19 20 A·= 717
960. D 973. B 986.A 999.A
96'1. B
962. A
974. D 987. B
975. D 988.A
1000.
1001.D
~~ llll
A A A A
963.C 976. D 989. c P= A[(1+i)"-1]
L.. .L.. .L..~Fl (1+i)"i
l2
L..,..:F3
A((1 + o.;o)2(5)_ 1 J
F= A[(1+i)"-1] 10, 000 = --:-"-';.._..__~-!,...------""
i (1+ o.~or5) (0.05)

A= 1,295.05
570 · 1001 Solved Problems in Engineering Mathematics (2"d Edition) by Tiong & Rojas Day 23- Engineering Economics (Annuity, Depreciation, Bonds, etc.) 571

k.l
(J.l~
'···
a·a
• 012345

i l l l l 1
ER = {1+it
ER={1-+:i)
0.12 = {1 + if -1
i =0.009488
12
2
=1
-1
p1 = p2 (1 + i)"
58,880.69 = p2 (1 + 0.06)
p2 = 43,999.078
5
12,0,00 =A+

A= 2,400
A~1 + 0.08) -1]
(1 + 0.08)5 (0.0.08)
5

~ ~ ~ ~ ~ F Total amount= 100,000+P2


I ~ L....L. .L. .J =12(10) =120 ml
n
P1 .•.• n Total amount= 100,000+43,999.078
. !
Pz ............... •
.J
J
Total amount= 143,999.078
r rrr . . . . . . . . .
0 1 2 3 359 360

1 p1 + p2 = 10,000
p = _A,_(( 1_+_i)_"---"'1
(1+i)"i
120
mJ 8 payments AAA.A AA
A({1 + 0.009488) -1]
~--A··
~., ..L. .L. . L. . . . . . . . . . . L. .!
. . . . . . . . . . lll . . . . . n
A[(1+i)"-1] F 1,000,000 = 120
p
__h..;.__:.~ + - - = 10,000 {1 + 0.009488) (0.009488) ( \
0 1 9101112 1718
(1+i)"i (1+i)" A= 13,994.17


2000((1+0.12) -1]
4
F _ Solving for the interest rate per month:
_ --=-.:.._---,-4_ + < -10,000 1+i = (1+if 2
(1+0.12} (0.12) (1+0.12} A A A A A
F = 6,917.72 P= A((1+i)"-1] P1 r. L. J. . ..t. .............L. . ~ I + 1 = 1.1 0 = ( 1 + if
i = 0.007974
2

(1 + i)" i Pz ~.................. .
Note: From the choices, the nearest
-1]


answer is 6,919.28 - 2000[(1 + 0.09)
10 Total number of payments = 12(30) = 360
P- 10
(1 + 0.09} (0.09) A[ (1 + i) "-1] Borrowed, money = A + P
p1 = ----'=-------"-
p = 12,835.32 (1+i)"i
8 A[(1+0 007974) 359 -1]
I'll 3 years 10 payments
A[(1 + 0.05) -1]
pl = ----'=---~
8
(1 + 0.05) (0.05)
100,000 = A + _ ____e_:.___~__:_---"---
359
(1 + 0.007974) (0.0 007974)

P1 = 6.643A

r. . . . . . . Tn. . . . . n
A= 839
i.= 0.12/12 = 0.01
=
n 12(2) 24 =
F= A((1+i)"-1]
i

=sao[( + ~)
100,000
0 1 5 6 7 8

A A A

~•.L. . L....L.. . . . . :l... J


15 16

A A
P1 = P2 (1 + i)"

6.643A = 187,400(1 + Q.o5l


A= 44,982 04
• !". . . . . . . . . . lll . . . . .
0 1 4 5 6
10 payments

7
n 14 15
ml
2
F 1 M)_ 1] P1
p !
2 ~...................... J
0.12 01234$
500,000 A A
!l l l l l
12 A A A
F = 13,486.70
p __
A,_(( 1_+_i)_"-_,1 J AAAAAA
pl r.L . . .l. ....L. . . . . . l.... J
p2 ...................... :
~.. J. . .l. . . l_J.. . !
Note: From the choices, the nearest 1 ~
- (1 + i)" i p


answer is 13,500
0
_ 8ooo[(1+0.06f -1]
p1- 10 Cash price = A + 1-' _ A[(1 + i)
Pl-----.
"-1]
(1 + 0.06) (0.06)
12,000 =A+ ---'A[~(_+_:_i)_"-...:!.]
1 1 (1+i)"i
Solving for the interest rate per p1 = 58,880.69
month,!:· (1 + i)" i
I.
I

57Z : l 00 l Solved Problems in Engineering Mathematics (2"d Edition) by Tiong & Rojas
Day 23 -Engineering Economics (Annuity, Depreciation, Bonds, etc.) 573

"'
,,,...
(/'Ill
Q.'l
p1-
- 100,000[(1 + 0.14)
(1+0.14)
P1 = 521,611.56
10
10

(0.14)
-1]
p,;, A[(1+i)"-1]
(1+i)"i

100,000[(1+
0 4 24
·~ ) -1]
• (1-+ i)" i
15
- 25,000[{1+0.06} -1]
J
p = _A[h.:.(1_+...:...i)_"---=1
Note:
A = revenue/year- operating co.sVyear

A= 9,000-3,500
A= 5,500
i'l
I

I P=~
2 p= 24
p- 15
(1 + 0.06) (0.06) 11111·11111'1!1

(1+i)"
(1+ 0.~4) (0~4) p = 242,806
Substitute the value of A in Eq. 1: I, IIIII


p - 521..611.56 0
2 4 p = 1,605,836.76 5500[(1 + if -1] 2000
- (1+0.14)
- - - = - - . .10
:---"' +-- 10 = 20' 000
p2 = 308,835.90 350,000 {1+i) i (1+i) '

i

F = _A[=-(1_+_i)_"--=-1] By trial and error, i = 24.8%
Cash price = 500, ooo + P2 i
16 0 2 3
Cash price= 500,000 + 308,835.90 100,000[(1 + 0.035) -1]
Cash price= 808,835.90
F= .
0.035
F = 2,097,102.97
! l-1
A A A
Down payment= 10% of cost of house
and lot
Note: From the choices, the nearest : : :

p~ ... L.....L .....l


answer is 808,811 Lump sum price = P + F 200,000 = 0.10(cost)
Lump sum price= 1,605,836.76 cost= 2,000,000
il
+ 2,097,102.97
p = A[(1+i)"-1] Balance= cost- downpayment
Cash price = Down payment Lump sum price= 3,702,939.73
+ Present worth of the
Annuity
.....,.
liiltiiiill
(1 + i)" i
3
Balance = 2, 000,000- 200,000
Balance= 1,800,000
- 200,000[(1+i) -1]
1 1
10 000 = 2 000 + _A.=.:[('--+_i:_)"--....=. J 350,000- 3
= 0.0125
. . . . . . . . . . . . l' ! ! ! (1 + i) (i) i = 0.15/12
' ' (1+i)"i 0 17 18 19 20 21
15 By trial and error, i = 32.7%


750[ (1 + i) -1]
10,000 = 2,000 + 15
(1 + i) i p = __,A[:.:_(1_+i)_"----=!1]
A A A A

10.667 = (1 + if5 -1
15
(1+i} i
p1 r. .L ...L...l. ...! p2 ~.......................... ......................sv
~

~ 800,000 =
(1+i)"i
42,821.87[(1 +0.0125) 12•-1]
--=~12:;::----'--
p~·······················
By trial and error, i = 4.61% p1
~T . . .1. . .1_. . . . . . . . . r. . l :
12
(1+0.0125) x(0.0125)
2

Ill p __
A['=-(1_+;..;..,i}:--n
--=-1 J A A A A A
(1.0125) -1 = 0.52543(1.0125f x
2
x

1- (1 +I")".I (1.0125f x =2.1072


16 payments 24 payments
2,000[(1 + 0.05) -1]
4
iii ii Taking log on both sides:

-n. . . lll r. . . n
p1 = 4 0 1 2 3 9 10
(1 + 0.05} (0.05} ·················-
!
0 1 2 15 16 17 18 39 40
12xlog1.0125 = log2.1072 il!
P1 = 70,919 I
x = 5 years
20,000
A A A A A A A A
P=__!l_

LJ.._...Lltl. L_.. J.
(1 + i)" p1 + p2 = 20,000
...i p = 70,919
A[(1+i)" - 1 L_~~ n =20,000
~Eq.1
p1 (1+0.05t -·(1+ i)"i (1 + 1)
p = 30,941.72 ·I
:j
574 1001 Solved Problems in Engineermg Mathematics (2"d Edition) by Tiong & Rojas Day 23- Engineering Economics (Annuity, Depreciation, E!onds, etc.) 575

u.l
lll
'Ill'
al
• F= A[(1+i)n-1]
i
12
1}
. . . . . . . . . . . . !!-!!1
0 6

A A A A A
7 8 9 10 Total= P1 + P3 + P5
Total= 19,963.55 + 21,738.97 + 57,899.186
Total= 99,601.71
• d =Co -Cn
n
d = 50,000-10,000

• lll .........n
A[(1 + 0.12) 10
I 80(2,000) = ' 12
0.12
12
-
p2 r. .L. .L. .L. .L..J d = 4,000

p3 ....:................... . Let: Cs = book value at the end of 5 years


A = 12,615.80


0 1 2 3 14 15
A2 [ ( 1 + i)n - 1] C5 = C 0 -d(m)
p2 - --'~----=' C 5 = 50,000- 4,000(5)
- ( 1+ i)" i A A A A A C5 =.30,000

( 1+ N4R

(1+
J
0.~8J =(1+i)2
= (1 + i)2

i = 0.0404
p2 =
8ooo[(1+0.o8t -1]

p2 =31,941.68

p - p2
'
(1 + 0.08) 5 (0.08)
L .....i ...... i........................L. ..

F= A[(1+i)n _ 1]
i
J ..~ F
• d = C0 - (Cr,- cost to dismantle)
n
d = 80,000- (50, 000- 15, 000)
10
50(1,000)= A[(1+0.04t' -1]
3 - (1 + i)5 d = 76,500
0.04
P=~ p3 = 31941.68 A= 2,497
i 5 Let:, Ce = book value at the end of 6 years
p = 15,000
(1 + 0.08)
Let: F1 =value of the given annuity when
. 0.0404 p3 = 21,738.97 C6 =C 0 -d(m)
n = 12 years
p = 371.287.13 C6 = 800,000- 76,500(6)

• 012345

l ! !!1..
A A
. .. . .
A A A
0

p4 •
11

l !!1
A A A A

I I II
oc
F~-

F1-
_ A[(1+i)" -1]

- 2,497[(1+0.04)
.
0.04
.
I
12
-1]

C6 = 341,000

C0 = 500, 000 + 30, 000

....
C0 = 530,000
. .. .
P1...:....: ..... .:. ..... :..... .:. .... : Ps ,.; I F1 = 37,519

1m en= 0.10(500,000)
A d =Co- Cn en= 50,000
A1 [(1+i)n -1] p4 =-:-
p1 - ---"'-~--"' 1 n
- (1+i)"i p - 10,00.Q d = 45,000-2,500
4 d =Co -Cn
5ooo[(1+0.08}
p1 = - - " - - . , .5 - - - -
5
-1] -

p4 = 125,000
0.08
d = 8,500
5
n
d = 530, 000 - 50, 000
(1 + 0.08) (0.08)
p - p4 5
p1 = 19,963.55 5- {1+ifo depreciation rate = _i_ d = 96,000
Co
p - 125,000
5 10 8 500 . Let: C4 = book value at the end of 4 years
- (1+0.08) depreciation rate = · x1 00%
45,000
P5 =57,899.186 depreciation rate = 18.89% C4 =C 0 - d(m)
C4 = 53o,ooo- 96,000(4)
C4 ... 146,ooo
l,.

576 1001 Solved Problems in Engineering Mathematics (2nd Edition) by Tiong & Rojas _______ .!_)_~ 23- Eng!~ng Eco!!2._mic~!!\nnuitL.~nreciati<tl.1..l.!1_1?rt'!~!·tc.) ~!I. 1111

4 .1
u.l
(Jll
a:~
• d =Co -c.
n
d"' 500,000-100,000
k

k
=1-19/40,545.73
=0.25
k=25%
720;000
Using formula for sum of an A.P .

Lyears =.!'_l(n + 1)
-----
2
l~ years "' 3~(20 + 1)
"'"' 2--
L years "' !~+ 1)
'2--
2)ean.; = 10(10 + 1)
,.._..., years= 55
?
2
,I

i
I

I =
d 16,000
25
El1 L, years= 210 Substituting:
I

Let: D = total depreciation after- "m" years lj


k=1-_rs;
vco- Substituting:
d, "'(9,000 -1,000)1(_!Q).
\
D = d(m)
\55
D =16,000(3) Book value at the end of 6 years, C6: 18 \1
- "(10
" ' 000-- 0) ('···-·-)
210 d1 ""t454.54
C1o = 4,350 d3
0=48,000
d3 = 857 Solving for C1:

a Substituting in the formula:

C 1 ~, 9,ooo- 1,454.54
d=Co-Cn
&iii~!
1!'-R / 4
I -.. n_-_---··j
l
C1 '1,545.46


P ,,
n d,, =(Co ... '"'"\L:rears
d = 50,000-10,,000

d = 4,000
10
......
llifiM
k =32.25%
Using formula for sum of an AP.

I years "'' n(~_2--


+ 1)
Using uniforrn gradient formula:

Let: Cs = book value at the end of 5 years Using_SYD~ the largest charge of lll'(.1·-·'"-•] ro(1+1)·" ---1 ·1!
_ '"' IIJ '
depreciation is the first year.
~)ears = 1Oi:!2_:~ 1}
p o' ·-"-"-. ____..__ + G[-'------------- -- ----.11. ------J
C5 =C0 -d(m) (1+ i)" i - f (1+ i)" i(1+ i)"
2
C 5 = 50,000- 4,000(5)
Lyears = 55 - 5oo[(1+o.oat -1l
=(Co -en { L :ears) p == ----~-..._________ _).


C 5 = 30,000 d1 '
(1+fl08 )10 (008).
C0 == 6, 000, 000 + 0. 04( 6, 000, 000)

+50[tf.~"!~i.'rj
d =(Co- Cn) ~
0.20C 0 = Co ( LY:ars) C0 = 6,240,000
' (1+i)"-1
LY!'!ars = n
c, ~' 0.08(6,240,000) ________1~--------
d = .!...(1_...;.0,0_0_0-_5_00-d-).!....(0_.04...!..)
c, = 499,200 .! (0.08){1+0.08t
0
(1+0.04f -1 Using formula for sum of an A. P.
p =4,653.88
d =791.26 Substituting:
ryears =·n(n + 1)
Note: From the choice!i, the nearest
2 d5 = (6,240,000- 499,200)( 5~)
Ill answer is 4,700

k=1-~:
5n = n(n +1)

n
2
=9 years ....
~
d5 = 626,269.10

Capitalized cost = C 0 + Co-- Cn


Book value at the end of 10 years, C1o:
=
C1o 40,545.73 a C1 =Co - d1

d1 = (Co - C~ {~):ars
, )
Capitalized cost = 250 + ---
(1 + i)" -1
100
(1+0.06)20 -1
--

Substituting in the formula:


d3 =(Co- c.) ) years)
n-2 I Capitalized cost = 295.3 million
r
Day 23 -Engineering Economics (Annuity, Depreciation, Bonds, etc.) 579

u.l
u.J
K·~
Let AC 1 = annual cost of the old motor
truck
AC 2 = annual cost of the new motor
truck
To breakeven:
Income = Expenses
135x = 56x + 69,994
79x = 69,994

Let: x = number of units to be forged

Income= Expenses
125x = 56x + 70,000


x = 886 units
I 69x = 70,000
\.
AC ;=(C )i+ (Co1-Cn1)i x = 1,014.49 units
1 01
ir -1

-
(1 + x"' 1,015 units
Let: x =number of units to be
AC ~ (5,000)(0.04) + (5,000- ~00)(0.04) manufactured per month
1 2
(1 + 0.04) -1
Income = Expenses
AC1 = 2,258.82 Let: x = number of cases to be sold each
600x =c 115x + 76x + 2.32x + 428,000
year
406.68x = 428,000
AC = (C )i +(Co2- Cn2)i
2 02 X= 1,052.42 Income = Expenses :'Ill
(1+if-1 65x = 50x + 35,000 1il


x"' 1,053 units
AC = (C )(0.04) +(Co2 -1,000)(0.04)
2 02 3
15x = 35,000 :I
'I.
(1 + 0.04) -1 x = 2, 333.33 cases 'l!lllli,l
1

-
i
Let: x = number of units to be sold out per x "' 2,334 cases I'

Equating: ,11,1111
year 1.11,,11

AC 1 = AC 2
Income = Expenses I
Let: x = number of motors to be sold each !;

200x = 200,000 + 160x month


2,258.82 = (C 02 )(0.04) +(Co2 -1,000)(0.04)
40x = 200, 000
3
(1+0.04) -1
x = 5, 000 units Income = Expenses
2,258.82 = 0.04C 02 + 0.32C 02 - 320.35
275x = 150x + 20,000
C02 = 7,164.36 ml 125x =20,000
:I!

Let: x = number of units to be produced x = 160 motors


per month

Let CR =Capital recovery rate per year Income = Expenses


580x = 85x + 350 + 20x + 15,000
125x = 15,000
CR = Annual Cost


x = 120 units
CH = (Co)i+ (Co -Cn)i
(1 + i)" -1
CR = 20,000(0.10)+ ( 2 0,000-8,000)(0.10) Let: x = number of blocks to be sold per
3
" . (1 + 0.1 0) -1 month
1· .R = 5,625 Income = Expenses
55x = 20x-t2x +3,500+ 25,000+12,000
33x =40,500
x = 1,227.3 blocks
i f~t: X :::: 10er of units to be forged x "' 1,228 blocks
Appendix A - Glossary 583
582 I Op i Solved Problems. in En¢!1eermgMathexnatics~2nd Edition)·by Tiong·& Rojas
Appendix A: + Glossary
' .r ' " " ~ '

.
' • .;, "" ">-' <' . ~ ·~ ~ ' ~' < ' •'

A abstract algebra the part of algebra that deals with


study of groups, semiiJroups, rings, modules,
fields and similar structures.

v •• _;«,
A the number 10 in the hexadecimal number abundant number a natural number where the sum
system of its distinct factors exceeds the number. For
···>! ,., ••.. """"" •0· ,.. "" , , . , ..... .,. •.•
example, the factors of 12 are 1,2,3,4 and 6.
·-~.-· ........ ~.- -~· i'i=-~~,~·-·,,, " < .. ~ < :
abscissa the position of any point on a plane from The sum ol the factors is 16. Since 16 is
the y-axis. Also known as 'the x-coordinate. greater than 12, hence 12 is an abund.ant
·> i The plural form of abscissa is abscissae. number.
< t '• '<," \ ~ " • < .(• ·~, M"' .. .< •0 ,0 ~ ,..,.,_·' *!.....,

-~ ",. "" * , ~.:-.. 'I ,,. ' ~-. " ,• .., fi,<" " ~-" ' • _,; :·' ._, ·+· ~- ,, .. "''* ., * v-axis acceleration the rate of change of velocity per unit
time. Acceleration is a vector quantity. The
"''' 'f~ .... ~ --~ •-< ~--. -~--· ~ ~ ~ >·•?:·' "'"""'""' ~ ... ~.-_... '!>~~-"~-~ .... _,,..~~-- x P(x,y) standard unit is meter per second per second
............. ·-·~

..... ,...·,,..... " : • •• ·< "' .:~ • .. '· \ ,..·.!..«.-'I'S '>~';?, f'~<'< '"- •< r/,;!J'->1.:> (m/s2). The gravitational acceleration on earth
vrdinate is denoted as g and is equal to 32.2 feet per
!:' " ~ ~ ~ "'"' .. ' •• > ~..,•• ,.,..""" ,.~ "-"'""-~-~ ~ .,, l<- ~"'."1 )' :'<"' ~."' .... '"""' "; ....... - ....~ <
!Y second2 or 9.81 meters per second2
: ~ ... " ' ' ~ ~ "j/>-., ·~'
I h ~ 1 ~ N ,. 0
. ... V
.h <!>' h ', , •. ' ·•., ,. <' " •. h" 'f " ... 1" ' ':: ' ··;- ,..,., -~.. 4 -~ ~ "•""" ..... :' ,... • " h ( ' .,. :• t' "' t " .. ' , -axis
,.,,, ~-··"•' -~· ..... ,.,;-.~;. ('•·-"·""'"'~""""...;,.·.,~~ ... ~
On'gin(O,O) I< >-I accuracy a measure of precision of a numerical
abscissa value !!f some quantity.
,·,;.~~~"""""' ··\\"'' -~~ ',(.< ~ .. ~ ~ '

'1)"' ~ 'II: .' 0 ~ ~-~ ' ~ ' "· v ,


., .. ~
acre British unit of area which is equal to 4840
Rectangular coordinate system showing square yards. In metric equivalent, 1 acre =
' i• ..... .• -~ •! ~ ·: -~" * -~ .... ~ '· abscissa and ordinate. The abscissa of 4046.8 square meters or 0.40468 hectares.
~ • N 1 "' 0 • 0 °o"O ~ < -/.:,,> •• <:' ' ~ :·
point Pis x.
. .; ,, ~ '
acute refer to something less than 90° It comes
absolute error the difference between the from Latin 'acus" which means "needle"
approximate value and the exact value:·
''·
...
• ~, . .., "·);':'
acute angle an angle less than a right angle or 90
absolute term (syn. constant term) a term in an degrees.
,.... , ',x . , .• ·.,, expression which does not have a variable .
.. ,
~<90°
, ...., ;
absolute value the number written arithmetically
• ,. -;;.·"' ' .,_ ,· • < ~ ~ 1 ~ -., " ~ '" ·~ •« ;. ,Y ~ : <i' v Y, < <
omitting the sign that prefixed it. The absolute ·
,, .•.. ,.~~,.-,_~·'-··~·~·· .. ~ ' ' value of the number x is denoted as I x I
The angle {1 is an acute angle.
~ ~ ~ 'J· ~ ., 'l, ~ ~ • ·" , .• i ~ , ~ :'. '
.. 4 " v , ••• ' .., -.' ' ' :..,

absolute value of complex number the distance of


... · the complex number from the origin when the acute triangle a triangle having all angles acute
complex number is represented as the point angles.
"' • ~ ... ,. ' ' ' h ••• ' ~ • ' .,_,.. -~ '
with rectangular coordinates (a,b). Absolute
value of a+ bi is Ja 2
+ b2 add to combine numbers of quantities by getting the
toti:il number of units contained in them.
absolute value of a vector the numerical value of addend the number added to a certain number to
the length of a directed line segment produce a sum. Example: In -the equation 5 +
representing the vector. Absolute value of the 2 = 7, the number 2 is the addend whi,e the
vector ai + bj + ck is Ja 2
+ b2 + c2 nu~ber 5 is the augend and 7 is called the
sum.
.584 _1001 Sol~d Pro~el!!!'! in Engi_neering Mathematics (2nd E<,!~tionLey Tiong:_& Rojas. Appendix A - Glossary 585
addition the process of combining numbers or algebra the study of operation and relation among 284, while the numb_er 284 have proper
quantities. The result of addition is called sum. numbers through the use of variables or literal factors of 1, 2, 4, 71, and 142 which sums up
e
""'K{
ld addition formulae formulae which express a
trigonometric functions of the sum or
difference of ·two angles in terms of the
symbols instead of just constants. The Latin
term "algebra" comes from the Arabic "al-jabr"
which means "to set or consolidate'.
to 220.

ampere an Sl unit of electric current


function of the individual angle. The foilowing algebraic curve a curve !hat describes an algebraic
I are the addition formulae: equation
amplitude the maximum displacement between in
either positive or negative direction from a
(a) (b) reference level.
sin (A +B) " sin A cos B +cos A sin 8 algebraic equation an equation of the form f(x) = 0 (}and f3 are alternate angles.
sin {A - 8) :: sin A cos B - cos A sin 8 where f is a polynomiai of dr;.'gree n with amortization as applied to the capitalized cost the
Figure 5b shows equal alternate angles since
cos (A +B) = cosA cos B -sin A sin B coefficients in a given base field, usually distribution of !he initial cost by periodic
the lines are parallel.
cos (A - B) = cos A cos B +sin A sin B rationals, n is the degree· of the algebraic charges to operation as in depreciation or the
tan A ·+-tan B equation. For example, x2 + 3x + 4 = 0 is a alternating series an infinite series whose reduction of a debt by either periodic or
tan(A +B)= ·---------
1-tan A tanS second degree (n = 2) algebraic equation. successive terms are alternately positive and irregular prearranged program.
tanA-tanB negative, such as
lan(A- B)= - - - - - - - algebraic expression any combination of symbols
1+tan A tan B 1 -1/2 + 1/3- 1/4 + 1/5 ..... anallagmatic curve a curve that is invariant
and numbers related to !he fundamental under inversion. This curve includes the
operation of algebra. altitude a line segment drawn from a vertex Cassinian ovals, Lirnacon of Pascal,
additive identity an identity clement of an additive
operation. The additive identity is the integer perpendicular to the opposite side (called Strophoid, Cardioid, etc.
zero. algebraic number a real number that is a root of base).
a polynomial equation with integer Analytic Geometry the branch of Mathematics
additive inverse the negative of the numbHr. The coefficient altitude of a trapezoid a perpendicular distance which deals with the properties, behaviors and
additive inverse of 5 is -5. between the bases or parallel sides of a solutions of points, lines, curves, angles,
algorithm a step by step procedure by which an trapezoid. surfaces and solids by means of algebraic
ad infinitum (latin term) continuing with no end. operation can be carried out. method in relation lo the coordinate system.
altitude of a triangle the perpendicular segment This was invented by Rene Descartes.
adjacent lying next to. An adjacent angle (side) is aliquot part refers to any divisor of a number from a vertex of the triangle to the line •
an angle (side) that lies next to anothei angie that is not equal to the number itself. This is containing the opposite side. It also refers to angle the basic figure formed by two line segments
(side). also known as proper divisor. the perpendicular distance from the apex of a or rays with a common end point. The symbol
triangle to the base. for angle is".!". Angles are measured in
adjacent angles two angles that have !he same Ai·jabr wa'l muqabalah Rough Translations, different units, namely degrees, radians, grads
point or vertex and a common side between Balancing Equations, an Arab texbook written ambiguous case the case of a solution of a plane and mils.
them. in the early 800s by AI-Khowzrizmi (from triangle where the given data. lead to two 1 revolution = 360 degrees
whose name the word algorithm was coined). solution:;. c = 2n radian

)p
A~B
This is where !he name algebra came from = 400 grads
and from this text Europe came out of the dark = 6400 mils

Q{o_ ages and learned algebra.

alphanumeric (syn. alphameric character)


combination of alphabet, numerical symbols,
punctuation marks and other symbols used in
8 - .............:..... · .
angle, of depression or elevation the angle
between the horizontal and the line of sight to
the observed point. If the observed point is
8, and f3 are adjacent angles below the horizontal from the observer, it is an
computer works. Figure shows that if sides a, band angle angle of depression. It is an angle of elevation
A are given, each of the two triangles if the observed point is above the observer.
agonic another !enn for skew as in agonic line or
alternate angles either pair of angles contained ABC and AB'C satisfies the given
skew line.(see skew)
between two given lines and a transversal conditions.
and lying on opposite sides of l.he transversal. Object
aleph the first letter in the Hebrew alphabet, ~. amicable numbers two integers such that each is ,,,....,. honzontal
These angles are equal if the given lines are
Lineof~-x
~\f~"x
This letter was first used in mathematics by equal to the surn of the ·distinct proper factors
parallel.
Georg Cantor to denote the various orders, of the other. An example of amicable numbers
or sizes, of infinity are the numbers 220 and 284. The number horizontal f Obiect
Line of sight __
220 has a proper factors of 1, 2, 4, 5, 10, 11,
20, 22, 44, 55 and 110 which have the sum of Angle of elevation Angle of depression
~
IIIII

586 100·1 Solved Problems in Engineering Mathematics (2"d Edition) .by Tiong & Rojas Appendix A ~Ql()f!l!~ 587

angle of inclination the smallest positive angle that apotome a number that has the form of ·are a larger unit of area in the metric system. It is astroid (syn. ~tar curve) a hypocycloid with four
w. the straight line makes with the positive x-axis.
)(JA- JB). This is a number
equivalent to 100 square meters. cusps and with parametric equation of x =
cos3 t , y = sin3 t. The rolling circle of this , I

~I
angular relative to or in terms of angles such as categorized by Euclid as one of the area a numerical measure of a two dimensional hypocyloid has a diameter one-fourth that of
angular acceleration, angular velocity etc. irrational numbers geometric figure enclosed within a specific the fixed circle.
boundary
annulus (syn. circu/auing) a plane figure which appreciation (ant. depreciation) increase in !tie
contains an area of a ring-shaped region lying value of an asset. Argand diagram (syn. Gaussian Plane) the
between two concentric circles. Another term rectangular coordinate system used for the
for annulus is ring. Area of annulus is 1t(R2 - approximation a number which is a close estimate representation of a complex number. The x-
r2). of another number. The symbol of axis and the y-axis are known as the axis of
approximation is "'· reals and axis of imaginaries, respectively.
Named after the Parisian bookkeeper, Jean Astroid
arabic numerals the symbols 0,1 ,2,3,4,5,6, ... that RobertArgand (1768 -1822):
represent the counting system in the decimal astronomical unit (AU) the mean distance between
number system (base 10). (see numerals) argument (1) the angle between OZ, where Z is the earth and the sun. It is about 1.495 x 108
the point representing a complex numbE;r km.
Annulus or Circular ring arbitrary constant a non-numerical symbol· holding on an Argand diagram and 0 is the origin,
a place for an unspecified constant. For and the real axis (2) theinput of a fUnction asymmetric not symmetric
annuity a series of equal payments occurring at example, in the general linear equation y = mx (3) an informal mathematical proof
equal interval of time. Type of annuity are, + b, m and b are arbitrary constants while x
asymptote a straight line that approaches the curve
ordinary annuity, annuity due, deferred annuity and y are variables. arithmetic the branch of mathematics which deals more and more closely but never really
and perpetuity. with calculation of integers using the touches it except as a limiting position at
arc a part of a circle between two points on the fundamental operations of addition, infinity. The word asymptote was coined by
annuity due a type of annuity where the payment is circle. A portion of a curve between two of its subtraction, multiplication, division and the Thomas Hobbes (1588- 1679), using various
made at the beginning of each period starting points A and B. extraction of roots. latin stems meaning roughly "to fall together
from the first period. but not touch".
Archimedes Principle Any body immersed in a arithmetic mean (syn. mean, average) the sum of
antecedent the first term of the ratio. In the ratio fluid is subjected to a buoyant force which is 'n" numbers divided by n. For example, the atmospheric pressure the pressure caused by the
2:3, 2 is the antecedent and 3 is the equal to the Weight of the fluid displaced. This mean of 2,9 and 7 is 6. weight of air at a given point. Standard value
consequent. (See consequent). The was discovered by Greek Mathematician, is 14.7 pounds per square inch or 760 mm of
antecedent is equal to the product of the ratio Archimedes( 287- 212 B.C). arithmetic progression is a sequence in which the mercury. It is also equal to the weight of a
and the consequence. difference between any two successive terms column of water about 30 feet high and in II,
is a constant and is called the common metric system, it is equal to 100 kPa. II,
Archimedean solid a solid made from regular I
antilogarithm the inverse function of a logarithm.
polygonal· sides of two or more types that difference.
Aubel's theorem "Given a quadrilateral and a
!I
meet in a uniform pattern around each
antiprism a semi-regular polyhedron from two n- arithmetic series the sum of the terms of an square drawn on each side of it, the two
corner. It is a convex semi-regular
sided polygon and 2n triangle. arithmetic progression. lines connecting the centers of the squares
polyhedron.
on opposite sides are perpendicular and of,
apex the highest point of a figure with respect to the arm one of the lines forming the a:1gle. equal length."
Archimedes' spiral (syn. Archimedean spiral) spiral
base or plane of the base.
with the polar equation r = ae, with its graph
as shown in the figure. The radius vector, array an arrangement of numbers in rows and augend the number or quantity to which the addend
0 columns (see matrix) is,added. Example: 5 + 2 = 7. The number 5 is
increases with polar angle, e, a is the constant

E~C
the augend while 2 is the addend.
of proportionality.
assets refers to everything a company or
y corporation owns and has a money value. automorphic number a number n whose square
Assets are as current assets (cash, bank ends in n. Example: 5 is an automorphic '"!!
A 8
account and other items that can be converted number because square of 5 is 25 and 25
Dis the apex of the polygon ----1--t~------X
into cash), trade investment (investment in ends with the number 5.
'
associated companies), fixed assets (land,
apothem the radi~s of the circle inscribed in a Archimedes' spiral building, etc) and intangible assets (goodwill, auxiliary ~ircle a circle with radius equal to half the
regular polygon .. A line from the center of a patent, etc.) major (transverse) axis and its center is at the
polygon perpendicular to one of its sides.
center of the ellipse (hyperbola).
588 l 001 Solved Problems in Engineering Mathematics (2nd Editi<m) by Tiong & Rojas ------------------------=AP:.cP:.:::e::.n:::d=ix A- Glossary 589

Ellipse possible outcomes of an event (i.e. success and bundle a family of lines or planes which all passes
w. Bang's theorem 'If all the faces of a tetrahedron
have the same perimeter, then the faces are
failure) and the possibilities if the outcomes are
independent and constant
through a single point.

l&J all congruent triangles."


butterfly theorem Let M be !he midpoint of a chord

K~
binomial expansion expansion of a binomial in the PQ of a circle, through which two other chords
bar graph a chart or diagram consisting of form of (x + y)" in accordance with the AB and CD are drawn. If AD intersects PQ at
horizontal or vertical rectangles or bars, each binomial theorem. X and CB intersects PQ at Y, then M is also ,I!! I
I auxiliary_____fi
of which represents an interval of values of a
variable and has height proportional to the binomial theorem the theorem that gives the form
the midpoint of XY. The resulting figure of this
arde theorem forms a butterfly.
quantities. of the expansion of any positive integral power ,rll
of a binomial (x + y)". Its general equation is byte a sequence of bits; a unit of information !
Auxiliary circle barycenter (see center of gravity) equivalent to a single character; a unit
(x+y)"=x"+nxn·1y +n(n-:!.)_xn-2Y2+ ... capacity of a computer.
Avagadro's constant the number of atoms of n base a 'side of a polygon wh1cn 1s at the bottom of
2!
grams in an element with atomic weight n. The the orientation. ·
value of this constant is 6.02214199 x 1023
This was named after the Italian phys1cist
Amedio Avagadro (1776 -1856)
Bertrand's conjecture 'If n is an integer greater
than 3, then there is at least one prime
number between n and 2n - 2." This postulate
... +n x yn-1 +yn

bisect to divide a geometric figure into two equal


parts.
c
average the usual term used for arithmetic mean. is named after the French mathematician C the number 12 in the hexadecimal number
Joseph Bertrand (1822- 1900) bisector (syn.bisectrix) a line or plane that bisects a system. The number 100 in Roman numerals
average acceleration the change in velocity of a given angle or line or any geometric figure.
body divided by the time interval during which bicorn any of a collection of quartic curves that has calorie the amount of heat required to raise the
a rectangular equation of y2( a2 - x2) = (x2 + bit (abbreviation of binary digit) the digits 0 and 1 in temperature of one gram of water 1°C.
the change occurs.
the binary number system.
2ay- a2)2. This is also known as "cocked-hat".
average velocity the displacement of a body calculate to determine the value of a given
divided by the time interval during which the bicuspid curve a quartic curve with an equation (x2 book value the recorded current value of an asset. mathematical procedure; to compute.
- a2)(x- a)2 + (y2- a2)2 = 0 The value of an asset that is recorded in the
displacement occurs.
book of records of the corporation. Calculus the branch of mathematics created in the
axiom a statement of truth which is admitted without bilateral having two sides .or relating to the right seventeenth century by Isaac Newton(1642-
Boolean Algebra an algebra which deals with the 1727) and German mathematician, Gottfreid
proof. side and left side of an object
operation of complementation, union and Wilhelm von Leibniz (1646-17'16) which rest
intersection. It is devised by the British on the basic principles of limits. Calculus is
axiom of induction 'Any property that belongs to billion refers to one million million (1012) in the
mathematician George Boole (1815 - 1864) divided into two subjects, namely, Differential
zero, and also to the immediate successor of United Kingdom and Germany and one
who is best known for his innovatory work in Calculus and Integral Calculus.
any natural number to which it belongs, thousand mi~ion,(109) in the United States and
formal logic.
belongs to all natural numbers.' France. In US and France, 1 million =
1,000,000. canonical form a form of any given polyhedron
Boyle's Law At constant temperature, the pressure distorted so that every edge is tangent to the
axis the fixed reference line used in a coordinate
is inversely proportional to the volume. unit sphere and the center of gravity of· the
system. binary relating to the binary notation or binary code.
tangent points is the origin
Brianchon's theorem 'Given a conic section, if we
axis of symmetry a line around which a geometric binary logarithm logarithm of a number to the base
2
circumscribe a hexagon about it, then the cap the symbol n, which is used to denote the
figure is symmetrical.
major diagonal of the hexagon are union between two sets
concurrent.'
axis of the conic the line through the foc;us and binary number system (syn. Dyadic number
perpendicular to the directrix. system) a system of notation for real numbers cardinal numbers numbers used to count objects.
Briggsian logarithm (see common logarithm) Example: one, two, three, ... twenty...
that uses the place value method with 2 as the
base. Only two digits are considered, 0 and 1

B sometimes called as 'bit' (abbreviation of


binary digits).
l:lritish thermal unit (BTU) the amount of heat
required to raise the. temperature of one
pound of water 1" F. 1 BTU = 252 calories •
cardioid a heart-shaped curve generated by a fixed
point on a circle as it rolls roung another circle
r
of equal radius. Its equation is = a(1 - cos<j>).
binol1Jjal a mathematical expression of two terms.
B the number 11 · in the hexadecimal number bulk modulus the ratio of the volume stress to the II
Example: 5x + 4y
system. volume stress.
binomial distribution (syn. binomial probability) the
di:;trillrllion of p10hahilil1P:. wlmrn lhnm are two il
II
"!'
.1

590 · 100 1 Sblved Problems in Engineering Mathematics (2nd Edition) by Tiong & Rojas Am>enciix A- Glossary 591

Carmichael number a number n that is a Fermat ceiling function the function of a number x that is center. Its equation is (x - h)2 + (y - k)2 = r2 coefficient of kinetic friction the ratio of the
pseudoprime to any base, that is, it divides (a" the smallest integer that is not smaller than x. where center IS at (h, k) and radiU$ is r. The frictional force to the perpendicular force. It is
- a) by any a. This number is also known as parametric equation is x = r cos 8 or y = r sin denoted as mu (J.!).
"absolute pseudoprime' center of gravity (syn. centroid, center of mass, 8. The word circle comes from Latin "circus"
barycenter ) the point through which the which means 'large round" coefficient of restitution the ratio of the total
Cartesian coordinates •(syn. rectangular resuliant of. the distributed gravity force momentum after collision to that of before
coordinates) a method of locating a point by passes regardless of the orientation of the circular cone a cone whose base is a circle. collision. It is denoted as 'e'.
pair of numbers denoting the distances from body in soace.
two fixed reference intersecting lines. The first circular cylinder a cylinder with a circular right coefficient of static friction the ratio of the limiting
number is called abscissa which is the center of mass (see center of gravity) section. frictional force (maximum) to the normal force.
distance from the y-axis while the second is The coefficient of static friction force. The
called ordinate, Which is the distance from the centesimal degree (see gon or grad) circular prime a prime number that remains prime coefficient of the static friction is always
x-axis. The two intersecting lines are called on any cyclic rotation of its digits. greater than the coefficient of kinetic friction.
coordinates axes. centillion a number 10 raised to a power of 600 or
10600 . circular ring (syn. annulus) see annulus coefficient of volume expansion (see bulk
r.ash flow the flowback of profit plus depreciation modulus)
from a given project. central angle an angle whose vertex is at the circumcenter the point of concurrency of the
center of a given circle. perpendicular bisectors of the sides of a cofunction (syn. complementary function) In
Catalan solid a polyhedron that is a dual of an triangle. trigonometry, the function of a pair have equal
Archimedean solid. These solids are named central conic a conic with a center. Examples are values for complementary angles.
after the Belgian mathematician Eugene ellipse and hyperbola. The only non-central . circumcircle a circle that circumscribed a given Example: Sin 30° = Cos (90° - 30°) or Cos
Catalan (1814- 1894) conic is the parabola. polygon. (60°). Hence, sine and cosine are
complementary functions.
· catenary a plane curve described by a heavy central tendency a central value between the circumference the boundary of geometric figure,
uniform, flexible cable ha'lging freely between upper and lower limits of a distribution around especially a circle coincident having all points in common.
two points. The term 'catenary" comes from which the scores are distributed.
the latin word which means chain. Its equation circumscribe to draw a geometric figure around collinear points points that lie on the same line.
is y =a cos h (x/a). centrifugal force a force acting outward on a body another geometric figure in such a way that
as it traverses a curvilinear path · they are in contact but does not intersect. cologarithm the logarithm of the reciprocal of a
catenoid the surface generated when a catenary is number.
rotated about its directrix. centroid (see center of gravity) cissoid a curve with a rectangular equation of
x3 columns the numbers in order which appears
cathetus a line that is perpendicular to another line. century a period of 100 years. The word century y2 = - - - vertically in a matrix.
This usually refers to one of the lines in a right comes from Latin 'centuria' whfch means 'one (2a- x)
triangle that is not the hypotenuse hundred" combination an arrangement of a set of objects in
clelia the locus of a point P that moves on the no specific order.
caustic the envelope of rays of light reflected or .chain rule a rule of differentiating a function of a surface of a sphere in such a way that <j>/8 is
refracted by a given curve from a given point function , f [u(x)]:dj/dx = (dj/du) · (du/dx) constant, where 4> and 8 are the longitude and The combination of 'n' different things taken
source of light colatitude, respectively 'r' at a time is given as
characteristic the integer part of the logarithm. n!
Cavalieri's Principle Given two solids and a plane. cochleoid a spiral curve that has the rectangular C(n,r)=~.
\n- rI'11
r,
Suppose that every plane parallel to the given
plane, intersecting one of the two solids, also
intersects the other and gives a cross-sections
chord a segment whose end points lie on the circle.

chord of contrast the chord joining the points of


2 2
equation of ( x + y ) tan-
1
U) = ay and a
If taken all, C1n.n1 = 1
with the same area, then the two solids have tangency of the two tangent lines from a point . asine
polar equat1on of r = - - . . combinatorics the study of the ways of choosing
the same volume. Named after Bonaventura P outside the circle. · e and arranging objects from given collections
Cavalieri (1598- 1647).
cipher an old name for zero. lt is derived from and the study of other kinds of problems
coefficient In algebra, it refers to the numerical relating to counting the number of ways to do
Cayley's sextic a sinusoidal spiral curve with a Ara.bic "sifr", Latin "cephirum" and Italian factor of a term. For example: In the term 5x, 5
rectangular equation of 4(x 2 + y2 -ax)3 = 'zevero". something.
is the coefficient of the term.
27a2(x2 + y2)2. This curve is named after
circle a close plane figure every point of which is commensurable refers to a ratio of two values that
Arthur Cayley
equidistant from a fixed point called the results to a rational number. Example: If two
distances have a ratio which is a rational
59Z 100 l Solved Problems in Engineering Mathematics (Z"d Edition) by Tiong & R~ Appendix A- Cl?ssary 59~

number, then the two distances are compound interest the interest charges under !he conjugate angles (see explemenlary angles) conclusion and the conclusion becomes the

"-' commensurable. condition that interest is charged on any


conjugate arcs two circular arcs together make a
hypothesis

~I
previous interest earned in any period of time,
common denominator an integer or polynomial as well as the principal. Fomula for future full circle convex polygon a polygon with no side extended
·that is exactly divisible by each denol111nator. amount of a principle in compound interest is will pass through the cenler of ihe polygon.
F =P(1 + i)", with n as the number of periods conic (see conic section) Also it contains no interior angle gr<Jater !han
common difference the difference of two and i is the interest per period. HJ0°. Regular polygons are said to be convex.
successive terms of an arithmetic progression conic section the locus of a point which moves
or sequence. compound number a quantity expressed in such that its distance from a fixed point (focus) coordinates the abscissa and ordinate together,
different but related units. Example. 5 hours is in constant ratio, e (eccentricity) to its Also known as rectangular coordinates or
common factor (syn. common divisor) Of two or and 32 minutes. distance to a fixed straight line (directrix) Cartesian coordinates.
more integers or ·polynomials, an integer of
polynomial which is a factor of each. For computable number a real number for which there conical surface a surface generated by a moving coplanar r)oints set of points that lie on the same
example: 5 is a common factor of 20 and 25. is an alqorilhm !hi, given n, calculates the nth straight line (generator) which always plane
digit intersects a fixed plane curve (directrix) and
common logarithm (syn. Briggs logarithm) which always passes through a fixed point coprime refers lo two or more numbers thai have
logarithm to the base 10. This was created by concentric circles circles having the same center (vertex) not in the plane of the curve no factors in common other than 1
the geometry professor of Gresham College in with unequal radii.
Lo11don, Henry BrirJgs (1561-1630) as an conoid a surface or solid formed by rotating a conic corollary a statement of truth which follows with
improvement of the natural logarithm. conchoid a shell-shaped curve that has a section about one of its axes. no
little of proof trorn the theorem
rectangular equation . o{
common multiple an in!eger or polynomial that is consequent the SflCOnd term of a ratio. See corporation (type of business mganiza!ion) a
a(x- a)(x 2 + y2 ) = k2 x2
multiple of each in a given set. For example: antecedent distinc;t legal entity separate from the
90 is a multiple of 15 and 18. indi~iduals who own it and which can engage
conclusion a oart of the theorem which is to be constant of integration an arbitrary constant term in any business transaction which a real
proved. · in the expression of indefinite integral of a person can do.
common tangent a line that is tangent to two or
more curves. function.
concrete number a number that counts a physical cosine law a law which relates the sides and angle
quantity. For example~5 apples, 8 cats constant term a term in a polynomial thai does not of an oblique triangle. I! is used in solving the
commutative law law stating that the sum or
product is unaffected by the order of the contain a variable parts of an oblique triangle.
concurrent having a common point.
terms. In addition, a + b = b + a. In
multiplication, a times b =b times a. continued fraction a fraction in the form of: cotermina! angles angles of rotation which the
conditional equation an equation which is satisfied 1 same initial side and terminal side. The
by some, but not all, of the values of the x = ao +- 1 coterminal angle of 75• is 435 and -285•. The
complementary angles two angles whose sum is
variables .for which the members of the a1 + - -,----1-- difference of coterminal angles is always
90" or right angle
equation are defined. a2 +--· --1 360".
completing the square the process of modifying a a+--
cone a solid bounded by a conical surface (lateral 3 a4 ...
counting numbers (see natural numbers)
quadratic polynomial to obtain a perfect
surface) whose directrix is a clbsed curve and
square (trinomial).
a plane (base) which .cuts all the elements. continuity a property in mathematics that refers to couple a pair of forces equal in magnitude, opposite
complex fraction any fraction which contains one smoothness of a function or curve in direction and not in !he same line
confocal conics two conics which share the same .
or more fractions in either numerator · or
focus. A confocal . ellipse and hyperbola continuum any set that can be bought into one-to- Cramer's rule a method of solving linear equations
denominator.
intersect at right angle. one correspondence with the set of real of several unknowns simultaneously using
complex number a number of the form a + bi with a numbers determinants or matrices. II is named after the
congruent geometric figures having identical shape
and b real constants and i =the square root of S•,yiss mathematician and physicist Gabriel
~. .
and size. convergent A convergent sequence has a finite Cramer (1704 -1752).
limit. A convergent series has a finite sum
conjecture a mathematical statement which has Opposite of convergent is divergent. critical point (syn. Stationary point) a point at which
composite numbers an integer which is the
neither been proved nor dehied by a function has a first deril,talive of infinity, thus
product of two integers, both different from 1
counterexamples. Some of the famous converse of a theorem* another theorem wherein havipg a slope which is vertical.
and -1. The integer 15 is a composite number
conjectures are Fermat's Last Theorem and the hypothesis and conclusion'of the first are
since 15 "' 5 limes 3. Goldbach Conjecture. reversed; that is, the hypothesis becomes the cross product (see vector product)

J
!I!~
594 1001 Solved Problems in Engineering Mathematics (2nd Edition) by Tiong & Rojas. Appendix A- Glossary 595
II
,,

crunode a point where a curve intersects itself so cyclic number a number of n digits, which, whem de Moivre's' theorem A theorem that links complex Example of degenerate conics are the point-
that two branches of the curve ,have distinct multiplied by 1, 2, 3, 4, ... , n, produces the numbers and trigonometry. It states that for ellipse, two coincident lines and two

~
tangeo1t lines same digits in a different order. · any real number x and any integer n, intersecting lines
(cosx+isinx)" =cos(nx)+isin(nx)
cube a polyhedron whose six faces are all squares. cyclic polygon a polygon whose vertices lie on a degree 1. A measure of an angle which is equal to
A cube is a regular hexahedron. circle. 1/360 of a revolution.1degrce = 60 minutes
decagon a polygon of ten sides and 1 minute = 60 seconds 2. The highest
I cubic curve an algebraic curve described by a
polynomical equation of the general form ax3 +
cycloid the plane curve traces out by a fixed point P
on the circle as the circle rolls along a line, the decahedron a polygon with ten faces. There is no
exponent or sum of exponents in any given
term of a polynomial. The polynomial 7x5y2 +
bx2y + cxy2 +dy3 + ex2 + fxy + gy2 + hx + iy + j such thing as a regular decahedron. 8x4ys:2x2y is 9th degrees polynomial.
base of the cycloid
=0
decimal fraction number that consists of an integer
cylinder a solid bounded by a .closed cylindrical delta curve a curve that can be turned inside an
part (which may be zero) and a decimal part equilateral triangle while continuously making
cubic equation a polynomial equation of the third surface and two parallel planes (see circular
(less the one) that follows the decimal marker, contact with all three sides.
degree. cylinder)
which may be a point or a comma.
cubit a measure of length which is approximately cylindrical surface a surface generated by a deltahedron a polyhedron whose faces are all
decimal number system a system of notation for equilateral triangles that are all of the same
equal to the length of a person's forearm (from moving straight line (generator) which is
real number that uses the place value method sizes
the elbow to the fingers). A Roman cubit is always parallel to a fixed line, and · which
with 10 as the base.
equal to 17.4 inches while the Egyptian cubit always intersects a fixed plane curve
is equal to 20.64 inches (directrix) not in the plane with the fixed line deltoid a non-convex quadrilateral with two pairs of
decimal point a dot place between the integral and adjacent equal sides. , I~
fractional parts of a number.
cuboctahedron a polygon obtained by cutting the cylindroid a cylinder with elliptical cross-section. . il
•ri'
corners oft a cube or an octahedron. A denominator the number written below in a given '!
declining balance method a method of computing fraction. The nurnber written above is called
cuboctahedron has eight faces. that are
equilateral triangles and six faces that are
squares .. D depreciation in which the annual charge is a
fixed percentage of the depreciated book
value at the beginning of the year to which the
the numerator. For example, 3/4 has a
denominator of 4. '
depreciation applies. This method is also density (syn. specific weight) the mass per unit
cuboid hexahedron will all faces rectangles l·l·i
0 the number 13 in the hexadecimal number known as percent on diminishing value. volume of a substance. I
system; the number 500 in the roman
Cunningham chain a sequence of prime numbers
numerals .. decrement the negative.of an increment.
in which each rnember is twice the previous depletion the lessening of the value of an asset due
one plus one. to the decrease in the quantity available. This
Oandelin spheres the two spheres that just fit defective equation any equation which, because refers to the natural resources such as coal,
inside the cone, one on each side of the plane of some mathematical process, has fewer oil and timber in the forest lj
cup the symbol v, which is used to denote the and both tangent to it and touching the cone, roots than its original I
union of two sets when the cone is 'sliced through by a plane depreciated book value t~e first cost of the
defective number (see deficient number) capitalized asset minus the accumulation of
curvature a measure of the amount by· which a 0' Alemberts principle When a body is subjected
curve deviates frorn a straight line annual depreciation cost charges
to an acceleration, it is reacted upon be a deferred annuity a type of annuity where the
reverse effective force opposite in the payment of the first amount .is deferred a depreciation the decline in the value of an asset
curve (see locus) direction of the acceleration to balance. The certain number of periods after the first. due to the passage of time and constant use.
value of the reverse effective force is equal to
cusp a point on the curve where two branches
coming frorn different directions, meet and
the mass time acceleration. Named after the deflection angle (syn. exterior angle) determinant a number which is expressed as a !,II
French mathematician and physicist, Jean Le square matrix (with the number of rows equals
have a common tangent. The word cusp Rond d' Alember (1717 - 1783). deficient number (syn. defective number) any
comes from Greek "kera" which means "horn" the number of columns)
natura! number the sum of its proper divisors.
and from Latin "cuspis" which means "sharp" de Malves' theorem "Given a tetrahedron in which All prime numbers are deficient numbers. deviation in statistics, it refers to the differenGP.
the edges meeting at one vertex, X, form three between any one of the sequence of observed
cute number a number such that a square can be right angles (i.e. the tetrahedron is the result definite integral an expression of integrating an
cut into n squares of, at most. two different value of a variable to some value such as the
of chopping off the corner of a cuboid), the integrand between two limits of integration. mean.
sizes. square of the face opposite X is equal to the Integrals with limits are definite integrals.
sum of the squares of the other three faces."
cyclic quadrilateral a quadrilateral with all vertices degenerate conic conic obtained when the cutting
lie on a circle plane passes through the vertex of the cone.
J1 .~!Ii
596 '1 001 Solved Problems in Engineering Mathematics (2nd Edition) by Tiong & Rojas
Appendix ~~-~Qlossary 597
I

,' li
devil's curve a curve with a rectangular equation of Diophantine approximation the approximation of a
division the process of obtaining the quotient; the eccentricity the ratio of the distance from point to I

'ID" y4 - a2 y2 = x4 - b2 x2 and the polar equation


r2 ( sin 2 0- cos 2 0) = a2 sin 2 0- b2 cos 2 0

diagonal a segment joining two non consecutive


real number by a rational number

Diophantine equations If there exist more


unknowns than the number of equation but
still can be solve bec(luse the values of the
inverse of multiplication

divisor (see dividend) the number that divides the


dividend
the focus (focal distance) to the distance from
the point to the directrix. Eccentricity of
parabola, ellipse and hyperbola are 1.0, <1.0
and >1.0, respectively.
'
I

vertices. It may be calculated using the unknown are integers, these equations are
I formula, n/2 (n-3), where n is the number of called Diophantine equations (named after
dodecagon a polygon with 12 sides ecenter (see. excenter) II
sides of the polygon ~ Diophantus of Alexandria, a Greek
dodecahedron a polyhedron of 12 faces. Each ecircle (see escribed circle)
mathematician" in the 3rd century BC).
face of a regular dodecahedron is a regular
digit any specific symbol use to denote a number Diophantus in his book 'Arithmetica' carried
pentagon. economic return the profit derived froni a project or hi
whether singly or in combination. out his extensive study to the solution to
indeterminate equations. business enterprise without consideration of
domain the set of all first elements of a relation obligations to financial contributors and claims
diameter a chord containing the center of the circle
directrix a fixed line opposite the focus of a conic of others based on profit
dot product (see scalar product)
difference the result of subtraction. section which the eccentricity of the conic
section is defined. economical number a number that has no more
dozen a term use to denote the number 12. This is digits than there are digits in its prime
differential calculus a branch of calculus that deals derived from French "douze" which means
Dirichlet's theorem 'For any two positive coprime factorization
with the evaluation and use of derivatives and twelve. 1 dozen = 12 items.
differernials. integers, a and b, there are infinitely many
prime numbers of the form an + b, where n > effective interest the true value of interest rate
0.' duodecimal number system a number system computed by the equations for composed
· differential eq<~ation an equation to be solved for
using 12 as a place value. This number interest for a 1 year period.
an unknown function which involves the first or system still survive today as in 1 year = 12
discontinuity a point at which a function is not Effective interest can be computed using the
higher derivatives of the function ; an iquation months, a clock dia! has 12 hours, 1 foot= 12
continuous. This is also kn?wn as 'jump" formula,
that contains one or more terms involving inches, 1 dozen = 12 items and 1 gross = 12 ER = (I + i)m- 1
derivatives of one variable with respect to dozens.
another variable. discriminant' the quanti!¥ that discriminates among where: m = number of interest period per year
· the pvssibilities of a quadratic equation. lt is i = interest per period
expressed as b2 - 4ac. The discriminant dyadic number system (see binary number II!
Ordinary differential equation - one that
• determines the nature of the roots. system) Egyptian fraction a fraction in which the numerator
involves function of a Gingle variable and
some of its derivatives. If b2- 4ac =0, the roots are real and equal. is 1. This is also known as "unit fraction"
dynar:nic (syn. kinetic) branch of mechanics that
If b2- 4ac > 0, ttie roots are real and unequal
if b2...: 4ac < 0, the roots are imaginary and unequal deals with the forces that produced a motion. Egyptian triangle a right triangle with sides 3,4 and
Partial differential equation - one that
5 units I
involves functions to two or more variables II
and some of !heir partial derivatives displacement a vector quantity which represent dyne a unit of force in the cgs metric system and is
equivalent to 1 gram-cm/s2. 1 dynes= 10·5 N. eigenvalue a complex number, A, the satisfies the
ill
the charge in position of a point. It is equal to j,
Order ot a differential equation - refers to the product of the velocity and lime. equation Ax = AX, where A is an n x n matrix Iii
and x is some vector. ,,'
the order of the highest derivaiives that is
present in the equation. distance the length of the shortest line segment
between two points.
E eight curve a curve with an appearance of a figure
of eight and has a rectangular equation of
,'I
i'
I'' '
Degree of a. differential equation - the
highestyo.wer of the highest-order derivatives. distance formula the formula used lo measure the E the number 14 in the hexadecimal notation. 4 2
x = a (x 2
-
2
y ) • This curve is also known '~II' '

distance between two points. This formula


a Lemniscate of Gerono
~
I ,
differentiation the operation of finding the was derived by the. use of the Pythagorean e il transcendental number which is approximately ':
derivative theorem. equal to 2.71828 ... 1t is commonly called as II
element 1.member of the set 2. part of a geometric I
d=~Vc2 -x~J +(y2 -y~J
the Euler Number because it was introduced
by Leonhard Euler(1707 - 1783) in the 18th figure such as point, line or plane). ,I
dihedral angle the angle between two planes
intercepted by a plane perpendicular to the century. It has the equation il,
divergent not having a finite limit eliminant (see resultant)
common edge
eccentric geometric figures not l:aving the same
dividend in the expression a I b, a is iha dividend center. ellipse a locus of a point which moves such that
dimensions of the matrix the number of rows a'nd
and b is the divisor the sum of distance from two. fixed points (foci)
columns of a matrix
is constant and is equal to the length of the
major axis (2a).
598 100 l Solved Problems in Engineering Mathematics (2"" Edition) by Tiong & Rojas Appendix A - Glossary 599

Eccentricity of ellipse is always le$S than 1. error the difference between an approximate value exterior angle (syn deflectign angle) is the aAgle
Standard equation of the ellipse is x =(a +b)cos(t) -ccos[(~ -1}] and the true value which it approximates. formed by the prolongation of one side and
the adjacent side of the polygon. The sum of
(x-11)2 + {y-k)2 = 1
a2 b2 y =(a+ b)sin(t) -csin[ ( + ~ 1}] escribed circle (syn. excircle, ecircle) a circle
tangent to one.side and to the extension of the
all exterior angles in any polygon is always
equal to 360°.
'!I
two other sides of a triangle. I
ellipsoid (syn. spheroid) solid of revolution of an extract to find the value of a root.
ellipse when rotated about one of its axes. equal having the same value. The symbol of equal Euler line a line that connects the centroid aml the
When rotated about its longer axis, it is called (=)was introduced in 1557 by Robert Recorda circumcenter of a triangle extrapolate to estimate the value of a quantitr or
as prolate ellipsoid. When rotated about its in his algebra textbook 'The Whetstone of '"'"""''11.1
measurement beyond the values which are
shorter axis, it is called oblate ellipsoid. The Witte'. I:
Euler number the number e which is equal to already known. Opposite of extrapolatiOft is '!:·1:
general equation of an eiHpsoid· in rectangular 2,71828 ... To obtain this number, get the anti- interpolation.
xz yz zz equally likely a term used to describe natural logarithm of 1. Or In e = 1. The formula
coordinates is 2 + 2 + 2 = 1 equiprobable events. fore is, · extremes the first and the fourth terms in lle
a b c
proportion of the four quantities
equation a mat[lematical statement showing that
elliptic curve t[le type of cubic equations whose
solutions takes the form of
two expressions have the same value. e =Lim
n~a:
(1+ .!)n
n
y2 +axy+by= x3 +cx 2 +dx+e equiangular having all angles equal. An equilateral
triangle is also equiangular. The symbol 'e' was introduced by the Swiss
Mathematician, Leonard Euler (1707-1783).
F
emirp a prime number that becomes a different
prime number when its digits are reversed. equichordal point a point inside a closed convex F the number 15 in hexadecimal notation.
The word 'emirp' is 'prime' spelled curve in the ~lane, all the chords through even function a function f(x) such that f(x) =f(-x)
which have the same length for all x face a plane surface of a geometric figure.
backwards. Example is the number 13 which
will become 31 when the digits are reversed
equidistant being of equal distance to any given even number numbers which are exactly divisible factor (syn. divisor) each of two or more numbers
but still a prime number
points or lines. by two. All even numbers has for its last digit which is multiplied together to form a product.
an even number.
empirical based only on observations and
experimental evidences equilateral hyperbola (see hyperbola) factorial the product of the n consecutive positive
event in probability, it refers to the possible outcome integers from 1 to n. Example: n! = 1x 2 x 3 x
equilateral triangle a triangle having all sides equal of a trial. ... x (n- 1) x n.· By definition, 0! = 1. The
empty set a set the contains no element at all
in length. An equilateral triangle is also symbol factorial ( ! ) was introduce by
equiangular. Each interior angle of an evolution the operation of root extraction. The Christian Kramp (1760 -1826) in 1808.
Engineering Economy the application of
equilateral triangle is equal to 60°. opposite of involution.
engineering or mathematical analysis and
synthesis to economic decisions Recursion formula states that:
equilibrium the condition when a body is acted on exact·accurate, precise
ennea • (syn. nona) a prefix which means nine. A by no forces of several forces such that their (n!)(n + 1) = (n + 1)!
vector sum (resultant) is zero. Forces are said excenter (syn. ecircle) the center of the ascribed Substituting 0 for n, results to 0! = 1
polygon of nine sides is called a nonagon or
to be in equilibrium if they are-concurrent circle.
enneagon.
(having a common point for the forces' line of factorion a natural number that equals to the sums
excircle (see ascribed circle) of the factorials of its digits in a ~iven base
envelope an envelope of a family of plane curves is action).
a curve that is tangent to every member of the
equiprobable having the same probability. explementary angles (syn. conjugate angles) two factoring the process of converting expressions as
family·
angle whose sum is 360" sums into equivalent expressions as products.
epicycloid the plane curve traced by a fixed point equivalent having some properties (geometric) in
common. exponent (syn. index) number (usually written fallacy a contradictory or false proposition; an
on a circle as it rolls along the outside of a superscript to another number) that is used to
fixed circle. invalid argument of form of argument.
equivalent numbers a number such that the sums simplify repeated product. Example xJ = X·X·X
epitrochoid a curve traced out by a point that is a oftheir aliquot parts (proper ciivisors) are the family ( syn. family of curves) a collection of related
same ·exradius (syn .. eradius) the radius of the ascribed geometric configurations; a set of related
distance c from the center of a circle of radius
circle. curves or surfaces whose equations vary only
b, where c < b, that is rolling around the
outside of another circle of radius a. This eradius (see exradiusj in the values of the constants.
curve is described by the followin!J parametric
III ~II ill :.
equntion~
II~
~

\~i:!il il l!l
600 1001 Solved Problems in Engineering Mathematics (2nd Edition) by Tiong & Rojas Appendix A- Glossary 601 I
I
Fermat's Last Theorem ( syn. Fermats' Great Oblong numbers are numbers which can be flat angle (syn. straight angle) an angle whose fr3e body diagram the diagram of an isolated body
Theorem) a famous conjecture of drawn as dots and arranged in rectangular measure is 180". with the representation of all external forces
1.1'
mathema\ics which stales that X'' + yn = zn, figure. acting on it ' I

.where n is 3 or greater x, y and z are all Example: 2, 6, 12, 20, ... flow chart a sequence of logical computations often i
positive integers. This is regarded as the the represented with rectangles, parallelograms frequency the number of times an event occurs ,II
least proved theorem in Mathematics. • Pentagonal numbers are numbers which can and arrows. within a given period.
Fiere de Fermat(1601,1665) claimed !hat he be drawn as dots and arranged in pentagonal
founded a marvellous root to this theorem, 'figure. focal measurement from the focus to a certain frequency polygon a graph on which the
but the margin is too narrow to contain it. This Example: 1, 5, 12, 22, 35, ... point. frequencies of classes are plotted at the class
is also known as the hardest problem in
Mathematics or the "Mount Everest of Gnomon numbers are numbers which can be focal chord a line segment joining two of its points
mark and the class marks are connected by
straig hi lines
i I

Mathematics'. drawn as dots on equally long legs of a right and passing through a focus of a conic
angle. Example: 1, 3, 5, 7, 9, ... friction the limited amount of resistance to sliding
Fermat number a number defined by the formula focal radius is a line SE!!lment from a focus to one between the surfaces of two bodies in contact
F.:=22"+1 cu'bic numbers are numbers which can be point of the conic.
drawn as dots and arranged as a cube. frustum a part of cone or pyramid lying between the
Example: 1, 8, 27, 64, ... focus a fixed point on the concave side of a conic base and a plane parallel to the base.
Fermat's little theorem "If P is a prime number section.
!hen for ooy number a, (aP - a) must be Tetrahedral numbers are numbers which can frustum of a regular pyramid the portion of a
divisible by P." ,II
be ·drawn as dots and arranged as a Folium of Descartes a plane curve which forms a regular pyramid included between the base
tetrahedron. · loop on one side and intersect itself at a node. and a section parallel to the base
Fibonacci numbers ( syn. Fibonacci sequence) Example: 1, 4, 10, 20, ... Its standard equation is
the unending sequence of integers formed xJ + yJ = 3axy where x + y + a = 0 is the frustum of a right circular cone the portion of a I
according to ihe rule that each integers is the Square pyramidal numbers are numbers equation of the line. right circular cone include between the base il
sum of the preceding two. The Fibonacci which can be drawn as dots and arranged as and a section parallel to the base.
sequence is 1,1 ,2,3,5,8, 13,34,55,89 ....... a pyramid with square base. Example: 1, 5, formula a symbolic statement of mathematical ,,li
Name after italian merchant and 14, 30, ... expression which is syntactically correct. function a relation in which every ordered pair (x, y)
mathematician, Leonardo Fibonacci (1170 -c. has one and only one value of y 1:'1
1250). finite can completely be counted off from 1 to the Fortune's conjecture 'If q is the smallest prime corresponding to the value of x
last whole number. greater than P + 1, where P is the product of
figure 1. any arrangement of points, lines, curves the first n primes, then q - P is prime.· fundamental operations of arithmetic referring .to
forming a geometric shape 2. The symbol for first cost the initial cost of a capitalized property, · the four operations - addition, subtraction,
an integer, such as '8' or '69'. including transportation, installation, fourth proportion In the proportion a:b =c:d, d is multiplication and division. In algebra,
preparation for service, taxes, and other the fourth proportion. fundamental operation follows the sequence
figurate numbers numbers that are represented by related initial !)xpenditure in order to make the of 'My Dear Aunt Sally' which means that
arrangements of dots as geometric figures. In property functional fractal a geometric shape that can be subdivided at Multiplication and Division must be calculated
plane figures, the following are examples of any scale into parts· that are, " at least · first before Addition and Subtraction.
I:
figurate numbers: Triangular numbers, Square first derivative the derivative of a function. approximately, reduced-sized copies of the
numbers, Oblong numbers, Pentagonal Normally the first derivative of the function is whole. The term 'fractal' comes from Latin fundamental theorem of algebra •The result that
numbers, Gnomon numbers. In solid tigues the slope of the function. "tractus' which means a broken surface any polynomial with real or complex coefficient
are the Cubic numbers, Tetrahedral numbers has a root in the complex plane.'
and Square Pyramidal numbers. First Proposition of Pappus the area of a surface fraction a ratio of two integers such as a/b , with a
Triangular numbers are numbers which can
generated by rotating.any plane curve about a not a multiple of b and b is not zero or one. fundamental theorem of arithmetic "Every
non intersecting axis in its plane is equal to The value 'a' is called the numerator and the positive integer greater than 1 is a prime
be drawn as dots and arranged in triangular
figure. Example: 1, 3, 6, 10, 15, 20, ...

Square numbers are numbers which can be


the product of the length L of the curve and
the distance traveled by its centroid. Or
. expressed as
value 'b' is called the denominator. When the
numerator is less than the denominator, it is a .
common or vulgar or proper fraction otherwise
number or can be expressed as a unique
product of primes and powers of primes.' .,
it will be called improper fraction. An improper future worth the equivalent value at a designated
drawn as dots and arranged in square figure. Area = length of arc x circumference fraction is a:ways written as a mixed number, future based on time value of money.
Example: 1, 4, 9, 16, 25, ..'. described by the centroid of arc that is an integer and a proper fraction such as
3Y,.
This was formulated by Pappus (c.300-{;.350)
of Alexandria. frame a structure with at least one of its individual
member is a multiforce member.
',!
I
I

602 · 100 1 Solved Problems in· Engineering Mathematics (2nd Edition) by Tiong & RojilS Appendix A- Glossary 603

~
centesimal minutes or centigon and 1 Hankel matrix a matrix in which all the elements hexadecimal number system a number system
G centesimal minute is divided into 100
centesimal seconds or milligan.
are the same along any diagonal that slopes
from northeast to southeast
which uses a place value of 16. This number
system uses 16 symbols (10 basic numbers
and the letters A, 8, C, D, E. and F).
g notation for gravitational constant. g =32.2 feet googol the number represented by 1 followed by a harmonic analysis the method of expressing
per second per second or 9.81 meters per hundred zeros or 10100. The name was coined periodic functions as sums of sines and hexagon a polygon of six sides. I

second per second.• by a 9 year old nephew of the American cosines


mathematician Edward Kasner in 1930s. hexahedron a polyhedron with six faces. A cube is
G notation for gravitational constant. harmonic division the division of line segment by a regular hexahedron. i
googolplex the number represented by 1 followed two points such that it is divided externally and I
G =6.67 x 10-11 N·m2/kg2.
by a google of zeros. 1 googolplex = 10 google = internally in the same ratio hippo pede a quartic curve which has a rectangular
il 1
' 11

1010'00 equation of
Gabriel's hom a surface of revolution of y = ..! harmonic mean a term in between two harmonic 2
(x +y
2
t +4b(b-a)(x 2 2
+y }-4b 2 x2 =0
X terms of a harmonic progression. It is the
gr'ad (syn. gon or centesimal degree) unit of angle
reciprocal of arithmetic mean. The harmonic where a and b are positive constants
generat"r (syn. generatrix) a point. line or plane measurement with one revolution equivalent
to 400 grads. . 2ab
whose motion forms a geometric figure. mean of a an db 1s - -
a+b • histogram a vertical bar graph that shows the
gradient slope of line or the ratio of vertical change frequencies of scores or classes of scores by
geometric mean (syn. geometric average) The the height of the bars
geometric mean of n numbers is the nth root of 1o horizontal change. harmonic progression a sequence of numbers
the product of the numbers. For example, the whose reciprocals form an arithmetic
graph a. plot of x values against y values for a given progression. holism the idea that the whole is greater than the
geometric mean of a and b is M . The function sum of its parts
geometric mean is always less than the Harshad number a number that is divisible by the
arithmetic mean except if all numbers are -Gravitational . acceleration the constant sum of its own digits. This is also known as homogeneous For polynomials, it is the one having
equal. acceleration of a free falling body. Denoted as Niven number. all terms the same degree. For example, x3 +
3x2y + y3.
'g' and is equal to 9.81 meter per second per
geometric progression a sequence of numbers second or 32.2 feet per second per second on height the measure of an altitude of any polygon.
such that the ratio of any term· to the Hooke's Law Within the elastic limit, the stress is II
earth's surface. On moon, g =1.62 m/s2 and
preceding term is constant. on sun, g = 274 m/s2. helix a curve in space which lies on a cylinder and proportional to strain. The constant of [I'll,,
crosses its elements at a constant angle. proportionality is called the modulus of I•

geometric series a series whose terms form a gravity the force of attracting between two bodies elasticity, E or sometimes known as the li:
1.,',

geometric progression. due to their mass. hemisphere a portion of sphere cut off by a plane Young's modulus. Named after the English
through its center..A hemisphere is one half of mathematician and scientist, Robert Hooke ll':,j.
Geometry the branch of mathematics which deals great circle the intersection of the sphere and the a sphere. (1635-1703). :1,
II'
with the properties and relations of plane through the center of the sphere. lr1
constructible plane and solid figures. heptagon a polygon with seven sides Hydraulics the branch of Physics that deals with 1 1
1·1

II!
1

greatest common divisor the largesi integer that the properties and characteristics of fluids.
gnomon a geometric figure which is formed by divides each. of a sequence of integers Heron's formula the formula for the area qf a ,·r1ii
cutting a parallelogram from one corner of triangle with all sides given. This was named hydrodynamic~ is the branch of hydraulics th!lt
exactly. This is also known as 'greatest 11!11

another but larger parallelogram. common factor" · after the first century Greek mathematician deals with l'fuids in motion.
Heron (c.A.D.75) of Alexandria. The formula is
gnomon numbers (see figurate numbers) hydrostatics is the branch of hydraulics that deals.
greatest lower bound the largest real number that A= ~s(s-aXs-bXs-c)
is smaller than each of the numbers in a set of with the fluids at rest. 1:

Goldbach conjecture the conjecture that every real numbers l'i'


even number (except 2) equals the sum of two where a,b and c are the length of the sides . hyperbola the locus of a point which moves such il l

· prime numbers. This was named after the gross twelye dozen. This is equivalent to 144 items. and s= (P.+b+c) that the difference of the distances from two 1

II1!111h.
1

Prussian-born number theorist and analyst, 2 fixed points (foci) is constant and is equal to ,,I il

Christian Goldbach {1690-1764). the length of the trilnsverse axis (2a). I

gon {syn. centesimal degree, grad) 1/400 of the full


angle. 1 revolution = 400 gons and 1 right
H Heronlan triangle a triangle with integer sides and
integer area
Eccentricity of hyperbola . is always greater
than 1.

angle= 100 gons. 1 gon is divided into 100 Hyperbolic spiral a curve with a polar equation of
Half-angle formulas formulas that express n re =a
trigonometric function of hnlf nn anglo.
Appendix A- Glossary 605
604 ·100 1 Solved Problems in Engineering Mathematics (2nd Edition) by Tiong & Rojas
inertia property of a body resisting any change in its Definite i'ntegral an integral defined by the
hyperbolic functions functions of angle expressed icosahedron a polyhedron with twenty faces. A state of rest or of uniform motion in a straight limit values of the independent variable.
in exponential functions such as regular icosahedron has faces equilateral line.
triangles. Double integral an integral in which the
sinhx = (ex- e-x) inferential statistics the methods used to describe integrand is integrated twice.
identity referring to the property of being the same. a population (universe) by studying a random
2
sample of that population Triple integral an integral in which the
idempotent the element x in some algebraic integrand is integrated thrice.
coshx = (ex+ e-x) functions such that x times x =x. infinite not finite, having value which are extremely
2 large amount. Integral Calculus the branch of calculus which
tanhx = ex -e-x imaginary number number in the form of ~ deals with evaluation of integrals and their
ex+e-x and denoted as I infinitesimal a number that is greater than zero yet applications.
smaller than nay positive real number
hyperboloid a surface of revolution produced by improper fraction a ratio of positive integer in integral part the biggest integer not greater than
which the value of the numerator is greater infinity the concept of being beyond the finite value. the given number. In the number 5.12, the
rotating a hyperbola in space about one of its
axes, called its axis of revolution than.that of the denominator. The symbol of infinity (oo) was introduced by int!'Qral part is 5.. In the number -5.12, the
John Wallis (1616-1703) in his Arithmetia in integral part is -6.
hypocycloid the plane curve tr;:~ced by a fixed point impulse the product of the force and the time during fini torum in 1655.
on a circle as it roll along the inside of a lixed which it acts integration the operation of transforming a function
circle. The parametric equations are: inflection a point on a plane curve where the curve to its definite or indefinite integrals.
incenter the point of concurrency of the angle has a stationary tangent, at which the tangent
x =(a -b)cos(t)+ bcos[ (~ -1}] bisectors of a triangle is changing from rotating in one direction to
rotating in the opposite direction
intercept refers to the intersection of two geometric
figures. X and y intercepts are the intersection
incircle the center of the circle inscribed in a of the curve with the x and y axes
y =(a- b)sin(t) -.bsin[( ~-1}] triangle. inscribe to draw a geometric figure inside another respectively.
geometric figure in such a way that the two
inclination (see angle of inclination) figures havillJ common but not intersecting interest the periodic compensation for the use of
hypotenuse the longest side of a right triangle or points. money. Sometimes referred to as the time
the side opposite to the right angle. Incomputable number a real number with an value of money.
infinite decimal expansion that cannot be inscribed angle an angle whose vertex is a point
hypothesis the part of the theorem which is enumerated by any universal computer on the circle and whose sides are chords interest rate the ratio of the interest payment to the
assumed to be true. principal for a given unit of time and is usually
increment small change in the value of the variable. instantaneous power the limit of the rate of work expressed as a percentage of the principal.
hypotrochoid a curve formed by the path of a point The increment of x from x = a to x = b is the done as time approaches zero.
attached to a point c, which is not on the difference, .b - a. International System (SI) the metric system,orunit
circumference of a circle of radius b that rolls instantaneous velocity the limit of average defined by the General Conference of Weights
around the inside of a larger circle or radius a. indeterminate an expression with no direct velocity of the body as the . time interval and Measurements in 1960. Sl stands for the
The parametric equations are: meaning \IS a number. Example are 0/0, oo/oo, approaches zero French equivalent, Systeme lntemational

(~ -1}]
0/oo, 00...
x = (a-b)cos(t)+ccos[ Integers are counting numbers (natural numbers). interpolation the procedure for estimating
index (syn. exponent) refers to the number n of a and th~ negative of the counting numbers and intermediate value that are not listed in a table
~
y =(a- b)sin(t)- csin[ ( -1}] railical rr the number 0. of numerical values. The . simplest form of
interpolation is linear interpolation, which
Integrand the function to be integrated has for its variation, which has for its variation
induction a method of reasoning by which one of the functional described by a straight line. If
infers a generalization from a series of the function does not satisfy the condition of
I instances

inelastic not elastic; having elasticity tess than one.


Integral the result of an integration. The integral
sign J by Leibneiz. The word integral linearity of variation, graphical interpolation
is used.
eomes from a Latin origin which means
i notation for imagi[lary number, ~· 'making up a whole". intersection a point where the. curve crosses the
inequality a statement that one mathematical · coordinate axis. This also refers to the set
expression is 1greater than or less than another Indefinite integral (syn. Primitive integral or consisting of the elements that are common to
I roman numeral for 1.
antidiverivative) an integral with no restrictions the original set.
imposed on its .independent variable.

j
I' IIIII

606 · 100 1 Solved Problems in Engineering Mathematics (2"ct Edition) by Tiong & Rojas Appendix A- Glossary 607

intersection of two sets the set of all objects kilowatt a unit of power equivalent to 1000 Watts; Law of Universal Gravitation "Every particle in
common to both sets

invariant a value lhat is not changed when a


J . 1 kN-m
eqUivalent to - - -
s
the universe attracts every other particle with
a force that is directly proportional to the
masses of the two particles and inversely
II

particular function is applied kilowatt-hour (kWh) the usual commercial unit of proportional to the square of the distance .. , I!
notation for imaginary number , ~ for between the centers of masses".
electrical energy. Kilowatt-hour is a unit of
involute of a circ)e a curve which is the path of engineering and physics applications. work or energy, not power. 1 kWh= 3.6 MJ .II
the end of a taut string as it is unwound from leading coefficient the coefficient of the term of
a circle. Johnson's theorem 'If three congruent circles all kinematics the study of motion without reference to highest degree in a polynomial of one
intersect in a single point, then the other three the forces which causes the motion variable. In the polynomial, 4x3 + 3x2 - 8x - 10
involution the operation of raising to an integral points of intersection will lie on another circle , 4 is the leading coefficient.
power, X". This is opposite to evolution. of the same radius. kinematic viscosity the ratio of viscosity to the
density of the body. leg Of a right triangle, any of the two sides other
irrational equation an equation in which a variable Jordan matrix a matrix whose diagonal elements than the hypotenuse.
appears under the radical sign are all equal and non-zero and whose kinetic another name for dynamic.
elements above the principal diagonal are lemma a proved proposition which is used mainly
irrational number any number which cannot be equal to 1 but all other elem~nts are 0 kinetic energy In Physics, the quantity equivalent to as a preliminary to the proof of a theorem
expressed as a quotient of two integers: 1/2 mv2, where m is mass and v is the
joule (pronounced 'jewel') a unit of work in the Sl lemniscate (syn. Lemniscate of Bernoulli) a curve
Examples are n, e, ../2 , etc. velocity.
units, named in honor of English physicist, whose equation is r2 = a2 cos 28, where (r,8)
James Prescott Joule (1835 - 1889). 1 joule = kite a quadrilateral with two pairs of congruent are polar coordinates. This was conceived by
isochrone a set of points with the property that a (1 Newton)(1meter). adjacent sides and forms like that toy kite. Jakob Bernoulli (1654- 1705) in 1694.
given process or trajectory will take the same
length of time to complete starting from ay of
length measure of a line segment.
the points

isodiametric having all diameters of equal length.


K L least common multiple the smallest integer that is
an exact multiple of every number in a set of
kappa curve a curve that resembles the Greek integers
.isogon an equilateral polygon.
L the number 50 in the roman numerals.
letter, K and has a rectangular equation of
least upper bound the smallest number that is
isomorphism a transformation in geometry tht does y2 (x2 +y2) = a2x2 Lame curve any family of curves related· to the . larger than every member of a set of numbers
not change the lengths of sides and the
ellipse with a general equation of
measure of angles of the figure involved
Kepler's Laws the laws which described the motion Leibniz harmonic triangle a triangle of fraction
1
isoperimetric figures are figures that have the of stars, planets and comets, formulated by
Johannes Kepler (1571 - 1630).
1;1" +ltl" = which is closely related to the famous Pascal's
triangle and takes the form of
same perimeters
Kepler's three laws of planetary motion:
1. All the planets · of the solar system lamina a thin sheet of uniform thickness and 1/1
isosceles having two sides of equal length. An density
isosceles triangle has two sides and two describe elliptical orbits, having the sun 1/2 1/2
angles equal. The term 'isosceles" comes as one of the foci. 1/3 1/6 1/3
2. A radius vector joining any planet to the lateral area area of the surface exclusive of bases. 1/4 1/12 1/12 114
from Greek 'iso' (same or equal) and 'skelos"
(legs). sun sweeps out equal areas in equal
periods oftime. lateral edge the intersection of the lateral faces.
isosceles trapezoid a trapezoid having non 3. The squares of the periods of revolution Lemniscate of Bernoulli a curve which looks like a
of the planets about the sun are directly lateral face any side of the polyhedron other than bow of a ribbon and has a rectangular
parallel sides equal in length the base.
proportional to the cubes of their mean 2
equation of ( x2 + y2 ) = a2 ( x2 - y2 )
isosceles triangle a triangle having two side equal distances from the sun (the major semi-
axes of the elliptical orbits). lateral surface the union of the lateral faces of a
in length prism L'Hospital Rule (pronounced as Lopital's Rule) a
kilogram (kg) mass of a particular cylinder of method in calculus in evaluating indeterminate
platinum-iridium alloy which is now kept at the latus rectum a line through the focus, parallel to quantities such as 0/0 and oo/oo. Name in
International Bureau of Weights and Measures the directrix and intersecting the curve honor of Johann Bernoull's pupil, Guillaume
at Sevres, near Paris. 1kg = 1000 grams. Francois de L'Hospital, (1661-1704).
608 . 100 l S0lved Problems in Engineering Mathematics (2nd Edition) by Tiong & Roj~s
Appendix A - Glossary _209
life the period of after which a machi#le or facility long radius the distance between a· center and a matrix a rectangular arrays of numbers forming m median 1. a line drawn from the vertex of a
should be discarded or replaced because of vertex of a regular polygon. rows and n columns. triangle to the midpoint of the base 2. (in
its excessive costs or reduced profitability. Types of matrices: stalistic) a point in the distribution of scores at
Also refers to the period of time after which a lozenge a rhombus with a 6Qo angle 1. Square matrix -· a matrix where the which 50 percent of the scores falls below and
·machine or facility can no longer be repaired number f columns equals the number of tile 50 percent of the scores fall above
in ord'er to perform its design function Ludolph's number a name used in Germany for rows.
properly. the number pi (rr) 2 Row matrix - a matrix which only one median of a trapezoid a line joining the midpoints
row. of the non parallel sides
Limacon of Pascal a snail-shaped curve with a June the portion of a sphere lying between two 3. Column matrix - a matrix which has only
rectangular equation of semi-circles of great circles. one column. bisecto~
( x2 + y2 - 2rx t 2
=k (x
2
+ y2 ) , where r is 4. Lower triangular matrix - a matrix where
all entries above the main diagonal are
mediator the perpendicular
segment.
of a line

the radius of the rolling circle or the rolled


circle and k is a constant M zero
5. Upper triangular matrix - a matrix where
all entries below the main diaqonal are
Mersenne number a positive integer of "the form
2" - 1, where n is a prime number. This was
named after a French mathematician and
limping triangle a right triangle with two shorter zero.
M the number 1000 in the roman numerals. Franciscan Priest. Marin Mersenne (1558-
sides differ in length by one unit 6. Scalar matrix - a diagonal matrix where 1648).
Macclaurin trisectrix an anallagmatic curve that a11 = an =a33 =.. . = k. where .k is a
line the shortest distance between any two points constant.
intersects itself at the origin, It has a meter defined in 1960 as 1,650,763,73 wavelength
rectangular equation of 7. Unit matrix or identify matrix- a scalar of !he orange,red light emi!IE)d by atoms of
line segment a, portion of a line bounded by two matrix where k =1 krypton 86(BSY·) in a glow discharge tube.
points. y2 (a + x) = x2 (3a- x)
8. Null matrix - a matrix in which all entries Redefined in 1983 as the distance !hal light
are zero travels (in vacuum) in 1/299,792,458 second.
linear equation an equation in which the variable or magnitude the absolute value of a vector quantity. 9. Complex m&trix - a matrix with at least The latest definition is more accurate than the
unknown appears only the first power and only one of the entries a complex number. first.
in the numerator of any fractions mantissa the decimal part of a logarithm . In the
expression log 25 = 1.39794.. the value mean (syn. average) 1. The arithmetic average of method of exhaustion a method of finding an area
literal equation an equation in which some or all of 0.39794 is called the mantissa while the value all the scores in the distribution 2. the average by approximating it by the areas of a
the known quantities are represented by 1 is the characteristic of two quantities. sequence of polygons
letters.
mass a measure of inertia of a body, which is its mean proportional the second and the third term~
lituus a plane curve with equation r2 = ~ with (r,e) resistance to a change in ve~ocity.
mil a unit of angle measure with one revolution
of a proportion with the second term equals equivalent to 6400 mils
as polar coordinates; It resembles a trumpet the third term.
mathematical expectation another term for mile a unit of length.
shaped spiral and originated with the English
expected value. means the secood and the third terms in the Statue mile = 5280 feet
mathematician, Roger Cortes (1682- 1716).
The word lituus, is of Latin origin which means proportional ,of four quantities Nautical mile " 1 minute angle on a great
mathematics the group of subjects (Algebra, circle
'a crooked staff. Plural form of lituus is litui.
Trigonometry, Calculus, Geometry, etc.) used measure a way of determining how large something = 6280 feet.
in investigation of numbers, space and the in terms of weight, volume, mass, length, etc. The nautical mile is 800 feet longer than the
locus the path of a point which moves according to
many generalization of these concepts. It is
a given law or equation. Plural form is loci. statue mile. The word mile comes from Latin
also defined as the science of patterns, real or measure theory the part of mathematics which 'mille passes" which means "one thousand
' imaginary. The word mathematics comes from investigates the conditions under ·which
logarithm an exponent when a number, N is paces"
Greet 'mathema' which means 'knowledge' integration can be carried out.
represented as a power of a fixed number
called based. milliard In United Kingdom, a thousand million.
Matheson Formula the formula used for Declining mechanics a branch of physical science which Billion (instead of milliard) is used ir1 the
logarithmic spiral a type of spiral with a polar deals with state of rest or motion of bodies United States of America.
_{CO
equation of r = ab6
Balance Method depreciation, k =
vcn under the action of forces
million a thousand thousand. The word comes from
where Co and Cn are the firSt and last cost.
logic the branch of mathematics which concern with respectively. With this method d computing medial triangle the triangle .whose vertices are Latin "miile" which means one thousand and
how one statement can imply others, or how depreciation, the last cost, Cn should not be midpoints of the sides of a given triangle the suffix 'ion' means great; so million iiterally
set of statements can be connected by chains equal to zero. means a great thousand
of implications
minimal prime a prime number that is a substring
of another prime when written in ba~ ~ 10
610 1001 Solved Problems in EngineeriJ;Ig Mathematics (2nd Edition) by Tiong & Rojas _____
A~dix A- Glossary 61.1
-------------------------------·
minuend In an expression x- y, x is the minuend monotonic property of a function that is always Rule 2 (Co-op rule) The sine of any middle null equal to zero; empty
strictly decreasing or strictly increasing but pari is equal to the product of the cosine of the
minute a measure of both time and angle never both ppposite paris. null hypothesis a hypothesis that is being tested
for rejection
Time: 1 minute = _!_ hour = 60 seconds motion any change in the position of a body. nappe either of the two parts into which a cone is
. 60
divided by the vertex. number theory the study of the whole numbers and
Angle: 1 minute = _!_ degree = 60 seconds multinomial the algebraic expression consisting of their properties and relationships
60 a sum of any number of terms. narcisssictic number an n-digi! number equal to
the sum of its digits raised to the nth power. numeral · symbol or combination of symbols
Mirifici Logarithmorum Canonis Description a multiple any number of polynomial that is a product Another term for this is "Plus perfect number" representing a number such as. Arabic and
book published by John Napier in 1614 which of a given number or polynomial and a or 'Armstrong number". Roman numerals. Arabic numerals (e.g. 0, 1,
means • A Description of the Marvelous Rule multiplier. For example; 15 is a multiple of 3. 2, 3, 4, .. ) are the modification of the Hindu-
of Logarithm'. natural logarithm (see Napierian Logarithm) Arabic Numbers. The Roman numerals are
multiplicand the number being multiplied by certain letters of the Latin alphabet.
mixed number a number formed of an integer and another. In an expression 5 x 2 = 10, 5 is the natural number are numbers considered as Roman numeral Arabic equivalent
a proper fraction. Example 51h multiplicand and 2 is the multiplier. Both 5 and counting numbers. Example: 1, 2, 3, ... : Zero I 1
2 are factors of 10. and negative numbers are not considered as v 5
Mobius strip a twisted surface in space formed by natural numbers. X 10
turning one side of a rectangle through 180° multiplicative inverse the reciprocal of the L 50
(relative to the opposite side) and join it to the number. The multiplicative inverse of 5 is 1/5. negative less than zero. c 100
opposite side. This was named after a D 500
German mathematician Agustus Mobius multiplier (see multiplicand) negative angle angle measured in M 1000
(1790-1868). . counterclockwise rotation.
mutually exclusive a condition where two events
mode the most frequently appearing score or group newton the amount of force thai gives an numerator In the fraction ~ , x is the numerator, y
cannot happen at the same time, or when one y
of scores appearing in the distribution occur, the other one will not occur and vice acceleration of one meter per second squared
to a body with mass of one kilogram. Denoted is the denominator.
versa.
model theory The study of mathematical structures as N. 1 N =1 kg-m/s2
that satisfy a particular set of axioms, numerical integration (syn. approximate
myriad very large number. II comes from Greek integration) the process of finding an
especially in the field of logic 'murios' meaning "uncountable' Newton's First Law Every body continues in its
state of rest. or of uniform motion in a straight approximate value of a defmite integral without
modulus of common logarithm in converting line, unless it is compelled to change that carrying out the process of evaluating the
indefinite integral.
natural logarithms into common logarithms,
the following formula is applied; log N =
0.43429 in N. The number 0.43429 is the
'N state by forces impre~sed on it. This is also
knowh as the law of inertia.

modulus of common logarithm


nadir the point on the celestial sphere directly below
the observer.
Newton's Second Law The rate of change of linear
momentum of a particle is C;Jqual to the total
applied force.
0
modulus of elasticity the ratio of the tensile stress
to tensile strain. This is sometimes called oblate spheroid (syn. oblate ellipsoid) prOdUGed by
Youg's Modulus, in honor of Thomas Young. Nagel point a point in a triangle where the lines Newton's Third Law For every action, there is rotating an ellipse through a complete
from the vertices to the points of contact of the always an equal and opposite reaction. revolution about its minor axis.
moment of force another name for torque. opposite sides with the excirc!es to those
sides meet · nominal interest the number employed loosely to oblique angle consist of all angles except right and·
moment of inertia the quantity equivalent to the described the annual interest rate straight angles
area times the square of the distance from the Napierian logarithm logarithm to the base e =
centroid to the axis considered. It has a unit to 2.718281828 ... This is also known as Natural nonagon a polygon of 9 sides oblique cal'tesian coordinate system a cartesian
the 4th power. logarithm. Denoted as In or log•. COOrdinate system in Which the X and y axes
normal another name of perpendicular or are not perpendicular
momentum the product of !he rnass and velocity of Napier's rules rules used in solving spherical right orthogonal.
the body triangles. oblique circular cone a circular cone whose axis is
normal number a number in which digits not IJerpendicular to the base of the cone.
monomial an algebraic expression of only one Rule 1 (Taro-ad rule) The sine o/uny middle sequences of the same length occur with the
term part h; equal to the product of llln[lenl of the same frequency oblique triangle a triangle with no right angle.
adjacent paris.
612 1001 S~lved Problems in Engineering Mathematics (2"d Edition) by Tiong & Rojas Appendix A - Glossary 613
I
oblong another term for rectangle. This comes from
Latin "ob' meaning excessive and "longus'
meaning long

oblong numbers (see figurate numbers)

obtuse angle an angle that is greater thc'm 90


orthogonal normal or perpendicular. Its symbol is

p $ Paraboloid
1
4
1

3
2

6
3
4
The number can be found by adding the two
parallel line or plane that will never intersect no numbers above it. For example, 6 in the
degrees but less than 180 degrees. palindronic number a number such as 1,234,321 matter how far they are extended. triangle was obtained by adding 3 an 3, the
that reads the same forward and backward numbers above it. Named (;lfter French
obtuse triangle a triangle having one· obtuse parallelepiped a prism whose base is a mathematician, philosopher and physicist,
angle. pandigital number a number that contains eacli of parallelogram region. Blaised Pascal (1623 -1662).
the digits from zero to nine exactly once and
octagon a polygon of eight sides whose leading digit is non-zero. Example pearls of Sluze curves that are generated by the

B
1,432,576,098 rectangular equation y" = k(a- x)P xm where
octahedron a polyhedron with eight faces
Pappus theorems theorems that determine tha! n,m and p are integers
octal number system a number notation which surface area and volume of a figure
uses base 8 as a place value. It uses the generated about an axis. This is named after pedal curve the locus of the feet of the
digits 0, 1, 2, 3, 4, 5, 6, and 7. Pappus of Alexandria. perpendiculars from a given point to the
Rectangular parallelepiped
tangents to a given figure
octants referring to the eight compartments 9f the First proposition of Pappus: parallelogram a quadrilateral in which both pairs of
rectangular coordinate systems in space "If an arc is rotated about an axis, it generates pedal triangle a triangle inscribed in a given
opposite sides are parallel.
as surface are which is equal to the length of triangle whose vertices are the feet of the
odd not divisible by two. arc times the circumference described by the three perpendiculars to the sides from some
centroid of the arc.' point inside the given triangle.
odd function a function f(x) with the property that
f(-x) = -f(x) for any value of X. Second proposition of Pappus: Pelf equation a equation of the form y2 = ax 2 + 1
"If an area is rotated about an axis, it where a is any positive whole number except
ogive a graph of cumulative frequency distribution generates a solid of revolution, whose volume a square number
plotted at the class marks and connected by is equal to the area times the circumference Parallelogram
straight lines described by the centroid of the area'. percent on diminishing value (see declining
parameter generally an arbitrary constant. balance method)
ordinal numbers numbers which state their position "parabola a locus of a point which moves so that it is
in a sequence. Example: First, second, always equidistant from a fixed point (focus) partial fraction the parts of an algebraic expression pencil a collection of lines that passes th~ough a
third, ... and to a fixed line (directrix). Ecr..entricity of which contain a polynomial in a single variable fixed point or a given point.
parabola = 1. in the denominator, or in the denominator and
ordinary annuity an annuity where the payments
are made the end of each period starting from numerator, when split pentagon a polygon. of five sides
parabolic segment the plane region bounded by a
the first period. chord a parabola perpendicular to the al{is and partnership (type of business organization) an pentagonal numbers (see figurate numbers)
the arc of the parabola cut off by the chord. association of two or more persons for the
ordinate the position of any point on a plane from
The area of the parabolic segment is 213 of purpose of engaging in a business for profit. pentagram (syn. pentangle, pentacle) a star-
the x-axis. Also known as the y-coordinate.
the circumscribing rectangle. shaped figure formed by extending the sides
origin the intersection of the x and y axes of the A= ~bh pascal (Pa) a ~nit of pressure. 1 pascal = 1 newton of a regular pertagon and meet at vertices.
cartesian or rectangular coordinate systems. 3 per square meter.
The origin has coordinates of (0,0). pentedecagon a polygon of fifteen sides
paraboloid a solid of a revolution of a parabola. Pascal's Law "If any external pressure is applied to
orthic triangle the triangle whose vertices are lhe Volume of the paraboloid is always equal to a confined fluid, the pressure will be increased per cent a word of latin origin which means every
feet of the altitudes of a given triangle one-half of the volume of the circumscribing at every point in the fluid by the amount of the hundred. Its symbol is %.
cylinder. external pressure.'
orthocenter the point of intersection of all the percentage a ratio by which the denominator is
altitudes of a triangle. Pascal's Triangle a triangular array of numbers 100.
which is made up of the binomial coefficient of
the binomial expansions:
,'1
614 1001 Sblved Proble~s in Engineering Mathematics~d Edition) by Tiong & Rojas Appendix A- Glossary 615

perfect number a number the sum ·of whose polygonal region is the plane figure formed by precision the accuracy in which a calculation is
factors including one but excluding itself is Mathematically, A= i+(%)-1, where i is fitting together a finite number of triangular pertormed.
exactly equal to the number. Example: The regions
factors of 6 are 3, 2 and 1. Adding the factors the number of interior lattice points and b is present worth the equivalent value at the present ,
.will yield the number itself such as the number of boundary lattice points polyhedron a solid bounded by planes base on time value of money

1 + 2 + 3 =6. Hence, 6 is a perfect number. pie chart a circular diagram divided into sectors of polyiamond a shape made from identical pressure the force per unit area. It has a unit of
which the area are in proportion to the equilateral triangles that have been joined at pascals in the metric system. Since pascal is a
perfed power an integer of the form m" where m magnitude of the represented values. their edges small unit, the unit bar or MPa are used
and n are integers and n > 1 instead. 1 pascal = 1 newton per square
place-value system a number system in which the polynomial an expression of several terms. It may meter. 1 bar = 105 Pascals.
perfect square a number that is the product of two value of a number symbols depends not only include any number of terms.
equal whole numbers. Example 4 =2 x 2. on the symbols itself but also on the position The gauge pressure can be calculated using
where it occurs population (syn. universe) in statistics, it refers to the formula: p = roh, where ro = density
perigon an angle equal to one revolution (360°) all the members of a particular group of items (specific weight) of the fluid and h is the
Planck time the shortest meaning period of time in or individuals. pressure head.
perimeter the sum of the sides of a polygon. This is quantum m.echanics
known as circumference for a circle. positive having values greater than zero. The absolute pressure is the sum of the gauge
plane a surtace such that a straight line joining any pressure and the atmospheric pressure.
permutable prime a prime number with at least two two points in it lies wholly in the surface postulate in Geometry, the construction or drawing
distinct digits, which remains prime on every of lines and figures the possibility of which is Standard atmospheric pressure = 14.7 psi =
rearrangement of the digits. planimetry the measurement of plane areas. admitted without proof. 1.01 x 105 Pa = 760 mm of Hg

permutation an arrangement of a set of objects in a point a dimensionless geometric object having no pound a unit of force in the English (British) system. primitive integral (see integral)
particular order. The permutation of 'n' properties except location and place. tf is equivalent to 1 slug-ft/s2. Also 1 pound
different things taken 'r' at a time is given as =4.448221615260 N. prime number an integer which has no other
point of inflection !he point on the graph where the factors except 1 and itself
n!
P(n,r)=~ curve changes from concave up to concave
1[1- r I'1
power 1. the rate at which work is done or energy is
down and vice versa transferred 2. (syn.. exponent) the number of principal In economics, it is the amount invested.
times the number is multiplied by itself.
perpendicular (syn. normal, orthogonal) forming a point of tangency is the point of contact of the prism a polyhedron of which two faces are equal
right angle. tangent and the curve power law a type of mathematical pattern in which polygons in parallel planes, and the order
the frequency of an occurrence of a given size faces are parallelograms
perpetuity an annuity where the payment periods Poisson ratio The ratio of the unit deformations or is inversely proportional to some power of its
extends forever or in which the periodic strains in a transverse directions is constant size. prismatoid a polyhedron having bases two
payments continue indefinitely. · for stresses within the proportional limit. polygons in parallel planes and for lateral
power series an infinite series in which successive faces triangles or trapezoids with one side
Philosoph late Naturalis Principia Mathematica terms are of the form of constants times lying one base and the opposite vertex or side
('Mathematical Principle of Natural Science')a polar angle (syn. vectorial angle, the argument, the successive integral power of the variable. It lying on the other base of !h9 polyhedron.
book publish by Sir Isaac Newton in 1686. amplitude of the azimuth of the point) the takes the form of ao+a1x+a2x2 +a:JX3 x ..... .
This book clearly slates the fundamental law~ angle the vector makes with the polar axis. prismoid a prismatoid in which the two bases are
of nature which is now referred to as the power set the set of all subsets of. a given set, polygons of equal number of sides and the
Newton's Law, the cornerstone of mechanics. polar coordinates coordinates in the form of (r,6) containing the original set as well as the lateral faces are quadrilaterals.
used to locate a point .in the rectangular empty set.
pi an irrational number which the ratio of ttie coordinates system. prismoidal formula formula used in finding the
circumference to the diameter of a circle. Its To convert polar to rectangular, use the following powerful number a positive whole number n such volume of a prismatoid such as follows,
value is 3.14159 ... and has for its symbol 'Jt. r'!lations: X ::: r COS 6, y = r sin 6 that for every prime number p dividing n, p2
This symbol for pi {n) was introduced in 1706 also divides n. · V=~(A1 +4Am +A2).
by William Jones (1675 -1749). pole the origin of the polar coordinates system where, L is the distance betw.een end areas,
practical number a number n such that every A, and A2 are end areas and Am is the area at
Pick's theorem 'The area of a polygon can be polygon a closed figure bounded by line segments. positive integer less than n is either a divisor the mid-section.
found simply by cotlnting the lattice points on or a sum of distinct divisors of n
the interior and hoiJndary of the polygon.'
!
616 1001 Solved Problems in Engineering Mathematics (2"d Edition) by Tiong & Rojas
1 ll
App~!lciixA- Glossary 617
probability the ratio of the successful outcome of rational expression any algebraic expression that
an event to the total possible outcome of an
even!. The value of the probability is always
less than 1.
Q R is a quotient . of two other algebraic
expressions

QED latin word quod erat demonstrandum, which radian the angle between two radii with an rational number any number which can be
product the result of multiplication. expressed as a .quotient of two integers
.means that a proposition has been proven. intercepted arc equal to the radius of the
The Greek equivalent was used by Euclid in (division by zero excluded).
progression a sequence of numbers in which one circle. 1 revolution is equal to 27t radians.
the 3rct century B.C.
is· designated as first. another as second , rationalizing the denominator the process of
another as third and so on. Types of radical the symbol . [ This , symbol was removing the radicals (or fractional. exponents)
quadrantal spherical triangle a spherical triangle
progression are Arithmetic progression, introduced by Christoff Rudolff in 1525. from the denominator
with at least one side a quarter of a great
Geometric progression, Harmonic
circle.
progression, Infinite Geometric progression, radical axis the locus of points of equal power with ray a straight path of points that begins at one point
etc. respect to two circles and continues in one direction
quadrants referring to the four divisions of the
rectangular coordinates system.
projectile initial velocity of a body and then follows radicand the quantity inside the radical (square real number a non imaginary number. It includes
a path determined entirely by the effects of root sign) the .rational numbers as well as the imaginary
quadratic equation· an equation in which the
gravitational acceleration and air resistance. numbers all integers and natural numbers.
maximum power of the unknown or variable is
2. Standard quadratic equation is in the form radius a segment from the center to a point of the
prolate spheroid {syn. prolate ellipsoid) an circle reciprocal multiplicative inverse of a number. For
ax2 + bx + c = 0.
ellipsoid produced by rotating an ellipse example, the reciprocal of 5 is 115.
through a coMplete revolution about its major radius of gyration the distance from a given axis
Quadratrix of Hippias the first curve recorded in .
axis that a particle of the same mass as a rigid rectangle a parallelogram all of wthose angles are
history that is not a part of a line or a circle.
This curve has a rectangular equation of body must be placed in order to have the right angle
proper fraction a ratio of positive integers in which same moment of inertia.
the value of the numerator is less than that of rectangular hyperbola a hyperbola with length of
y = xcot(;:)
the denominator radius vector the distance of any point P from the semi-transverse axis, "a' equals the length of
origin in the polar coordinate system the semi-conjugate axis, 'b'. Eccentricity of
.Ptolemy's Theorem In cyclic qua9rilateral, the sum quadrature formulas- refers to the formulas used this hyperbola is square root of 2.
of the product of two opposite Sides is equal to in numerical intregration. radix the base of a number system. For example, 2
the product of the diagonals. Named after is the radix of a binary number system and 10 rectangular parallelepiped a polyhedron whose six
Ptolemy of Alexandria or Claudius Ptolemaeus quad~ilateral a polygon offcur sides is the radix of the. decimal number system faces are all rectangles
(c.100- c. 168).
quantity something with a magnitude or numerical random variable a numeric quantity which can be rectilinear pertaining to straight line.
puro quadratic a quadratic equation of the form ax2 value. measured in a random experiment.
+ c =0, that is , the coefficient of the first reduction the process of converting a fraction into a
degree term, b is equal to zero quarternary having four variables. range the set of all second elements of a relation decimal form.

pyramid a polyhedron of which one face, called the quartic a polynomial or polynomial equation that rate of return the interest rate at which the present redundant equation any equation which, because
base, is a polygon of any riumber of sides and contains the fourth power of the variable, but work of the cash flow on a project is zero, or of some maihematical process, has acquired
the other faces are. triangles which have a no higher power the interest earned by an investment an extra root
common vertex
quinary number system which pertains to place ratio the quotient of two numerical measure of two reentrant angle an inward-pointing angle of a
pyramidal numbers (see figurate numbers) value notation of base 5. magnitudes of the same kind. The word ratio concave polygon
comes from the Latin verb 'ratus' which
Pythagorean theorem The sum of the squares of quintic a polynomial ~r polynomial equation that means 'to estimate'. reflex angle any angle greater than 180 degrees
the sides of a right triangle is equal to the contains the..fifth power of the variable, but no but less than 360 degrees
square of the hypotenuse; in equation, a2 + b2 higher power ratio of similitude the common ratio of the ·
= c2 with a and b are legs while c is the corresponding sides of two similar polygons regular polygon a polygon with all sides equal and
hypotenuse. This is named after the Greek· quotient the result of division. all angles equal. A regular polygon is
philosopher and mathematician, Pythagoras rational equation an equation which is satisfied by equiangular and equilateral. Also, a regular
'
(c. 580- c. 500 B.C.) of Samos.
I all value of the variables for which the
members of the equation are defined
polygon is convex.
r

618 I 00 I Solved Problems in Engineering Mathematics (2nd Edition) by Tiong & Rojas A~endi:~tl\_~_Giossary 619

regular pyramid a pyramid whose base is a regular rounding (of numbers) replacing it with another score another term used for the number twenty (20) shear stress a stress that is caused by forces
polygon and whose center coincides with the number to produce fewer significant decimal acting along or parallel to the area resisting
root of the perpendicular dropped from the digit. For an integer, fewer value carrying scrap value the value of an equipment if disposed the force
vertex to the base (non-zero) digit. Example: 3.14159 is rounded as junk. This is sometimes referred to as junk ,Ill
off to four decimal places as 3.1416. For value. short radius the shortest distance from the center •I

relation is any set of ordered pairs (x,y) numbers ... 5, the rule states that it should be of a regular polygon to any of its sides. !i:
rounded off to the .nearest even rounding secant a line which intersects the circle in two
relative ~ensity (see specific grav1ty) boundary to minimize the systematic rounding points. The reciprocal of the trigonometric
error. function, tangent. 111:

relative error a measure of the difference between


a number and an estimate. rows the numbers in order which appears second originally defined in 1889 as the fraction
horizontally in a matrix 1/86400 of the mean solar day, and redefined
Relativity theory a theory formulated by Albert in 1967 as the duration of 9,192,631,770

s
Einstein the revolutionized of man's periods of the radiation of a certain state of the
understanding about time, space and gravity. cesium-133 atom Figure shows R, the short radius
Mathematically, it is expressed as E = mc2,
where E, m and c are energy, mass and Second proposition of Pappus The volume of any Siegel's paradox "If a fixed fraction x of a given
speed of light, respectively solid of revolution is equal to the generating amount of money P is lost, then the same
sample any subset of a population
area times the circumference of the circle fraction x of the remaining ~ nount is gained,
remainder the amount left when a quantity cannot described by the centroid of the area. the result is less than t!:w v11ytnal and equal to
sample space the set of all possible outcomes of
be divided exactly by a divisor. the final amount if a fraction x is first gained,
an experiment.
section of a solid the plane figure cut from the solid then lost.'
resultant the single vetor quantity which is the sum rv passing aplane through it
salient angle an outward-pointing angle of a
of two or more vector quantities. Sierpinski number a positive, odd integer k such
polygon
sector a part of a circle bounded by the radii and an that k times 2n + 1 is never a prime number for
rhombus (syn. diamond, lozenge) a parallelogram arc. any value of n
salient point the point where two branches of a 1

all of whose sides are congruent


1

'1'111'11:: 11 ·1'
.,:11,,
curve meet and stop and have different
segment part of a circle bounded by an arc and a significant figures fdigits the meaningful digits in a 1
tangents
right angle angle equivalent to 90 degrees chord. number. A number is considered significant
unless it is used to the place a decimal point.
.salvage value the cost recovered or which could be
right triangle a triangle having one right angle septagon a polygon of seven sides
recovered form a used property when
similar having the same shape but not necessarily
removed, sold or scrapped. It is sometimes
rigid body a body which will not be· affected or sequence (syn. progression) a succession of . the same size
referred to as second hand value
deformed when an extremely large or numbers in which one number id designed as
extremely small load or temperature is first, another a~ second, another as third and simple interest the interest charges under the
scalar quantity a physical quantity that is described
applied. soon condition that interest in any time is only
by a single number only, the magnitude. It
charged on the principal
does not have a direction in space. ·
Rolle's theorem "Suppose a continuous function series sum of a finite of infinite sequence.
crosses the x-axis at two points a and b and is sine curve (syn. sine wave) a curve with equation y
scalar product of A and B is denoted as AB.
differentiable at all points between a and b; serpentine a curve which nas a rectangular =sin x.
that is, it has a tangent at all points on the Because of this notation, scalar product is also
called as dot product . ( ) abx
curve between a and b. Then there is at least equat1on y x = -(- ) sinking fund method a method of depreciation
x2 -a2
one point between a and b where the where a fixed sum of money is regularly
scalene triangle a triangle having all sides of
derivative is 0 and the tangent is parallel to the deposited at compound interest in a real or
unequal lengths set (syn. class) a collection of objects.
x-axis imaginary fund in order to accumulate an
amount equal to the total depreciation of an
scattergram the relation between two variables is sexagesimal pertaining to the number 60.
Roman numerals (see numerals) asset at the end of the asset's estimated life.
shown by a series of dots plotted on a graph
root value that satisfy a given equation. sexagesimal number system a number system skew lines two lines that are not coplanar. This is
scientific notation a number ·represent using .using a place value of 60. This was used by also known as "crossing lineS"
powers-of-1 0 notations used to described a the Babylonians or Mesopotamians and is
rose curve a curve that has the shape of a flower
very large small numbers. considered be the oldest number system
with petals had has a polar equation of slant height (syn. element) the length of a
r asin(rrO) which dates back to 2 millenium B.C. generator of a circular cone.
"

620 · 1001 Solved Problems in Engineering Mathematics (2nd Edition) by Tiong & Rojas Appendix A- Glossary 621

slope ratio of the vertical distance to horizontal spherical excess the sum of the angles of a statically indeterminate the condition exists in supplementary chords two chords which join a
distance. It also refers to the tangent function polygon over (n-2) 1so· with n as the number. structures where the reactive forces or the point on a circle to the end points of a
of the angle of inclination. Also refers to rise of sides of the polygon internal resisting forces over the estimated life diameter.
over run. In Analytic of the asset in terms of the periods or units of
spherical polygon the portion of a spherical output
Geometry, slope of line is denoted as m. surface bounded by three or more great circle
m = Y2 -Y1 arcs stationary point a point on the graph of a function
x2 -x1 where the tangent to the graph is parallel to
spherical pyramid the portion of a sphere bounded the x-axis or, equivalently, where the
by a spherical polygon and the plane of the derivative of the function is zero.
Smith number a composite number, the sum of sides
whose digits equal to the sum of the digits of
statistics the study of ways that lots of data can be
its prime factors. spherical sector a solid generated by rotating a represented using a few numbers and the Angle 0 is the supplemental angle
sector of a circle about an axis which passes study how such numbers can be chosen and
snowball prime a prime number whose digits can
through the center of the circle but which used to draw reasonable conclusion about the supplementary angle two angle whose sum is
be chopped off, one by one, from the right-
contains no point inside the sector data equal to 180°
hand side, yet still leave a prime number.
spherical segment asolid bounded by a zone and stellation the process of constructing new surd an irrational number which is a root of a
Soddy's formula a formula used for four circles of
the planes of the zone's bases polyhedron by extending the face planes of a positive integers or fraction or it is a radical
radii, r1, r2, rJ and r4 drawn not overlapping but given polyhedron past their edges expressing an irrational number.
each touches the other three. The following is
Types of surds: Example
the Soddy's formula:
spherical triangle a triangle extracted from the steradian ( sr ) a unit of measure of solid angle.
2
(b1 +b2 +b3 +b4) = 2(b/ +b/+bl+b/) surface of a sphere. Then maximum value for a solid angle is a full
Quadratic J2
where b1 = 11r1 and so on ... sphere which is equal to 4n steradians. Cubic ~
spherical wedge the portion bounded by a lune and
Quartic
(2
solid is any limited portion of space, bounded by the planes of two great circles strain the change of relative positions of points due The type of surd is named after the index of
surfaces to stress. Unit strain is equal to the ratio of the the radical.
spheroid another term for ellipsoid. deformation to the total length.
solid angle an angle formed by three or more Pure surd is a surd that contains no rational
planes intersecting at a common point. Solid spherometer an instrument for measuring the stress a force per unit area number (i.e. all its factors or terms are surds).
angles are measured in steradians curvature of a surface.
strobogrammatic prime a prime number that Example: J2 , J3 + J2
solidus the slant line in a fraction such as alb square a rectangular all of whose sides are remain the same when rotated through 180
dividing the numerator from the denominator congruent degrees. Example is 619,' when rotated 180° .•Mixed surd is a surd that contains at least
remain the same- 619 one rational term. Example: 2 + ..J 3
space a set of all points square free an integer that is not divisible by a
perfect square, n2, for n > 1 subset a set that contains some of the elements of symmedian reflection of a median of a triangle
specific gravity the ratio of the density of the a given set about the corresponding angle bisector
substance to the density of water. Specific square matrix (syn. determinant) a matrix with the
gravity of water at densed condition (4.C) = same number of rows columns subtrahend the number to be subtracted. Example:
1.0

sphere a solid bounded by a closed surface every


square numbers (see figurate numbers)
7 - 5 =2, 5 is the subtrahend.

sum the result of addition.


T
point of which is equidistant from a fixed point square pyramidal numbers (see figurate numbers)
sum-of-years' digits method (syn. SYD method ) a table c!Wlpilation of values such as trigonometric
called the center table, logarithmic table, etc.
standard deviation a quantitative measure defining method of computing depreciation in which the
spherical angle the opening between two great the extent to which scores are dispersed amount for any year is based on !he ratio:
throughout in relation to the arithmetic mean. (years of remaining life/(1 + 2 + 3 + .. + n), with tangent a line (in the same plane) which intersect
circle arcs. A spherical angle is measured by the curve in one and only one point. In
the plane angle formed by the tangents to the This is also equal to the square root of the n being the total anticipated life of the
variance. equipment trigonometry, it is the ratio of the side opposite
arcs at their point of intersection to side adjacent in a right triangle.

tangent plane of a sphere a plane which intersects


the sphere in exactly one point
~
~

622· 1001 Solved Problems in En'gineering Mathematics (2nd Edition) by Tiong & Rojas Appendix A- Glossary 623

tension force in longitudinal direction. truncated value the value of number when written universal set the set that contains all elements

~
with the further digits have been suppressed
terminal speed the final speed Vt attained by the and replaced with three dots. For example, the untouchable number a number that is not the sum
·falling body. The principle is that when a. body number 1t = 3.141592653... can also be of the aliquot part of any other number.
first. start to move, v =0, the resisting force is written by truncation as 3.14159 ... The
zero and the initial acceleration is a = g. As it
speed increases, the resisting force also
increase until finally it equals the weight in
magnitude. AI this time, the acceleration transverse axis the axis of the hyperbola which
truncated value is not a rounded off value and
therefore always smaller than the exact value.

truss a framework composed of members joined at


v
becomes zero and there is no more increase passes through the foci, vertices and center their ends to form a rigid structure. variable an expression than is assigned a certain
in its speed.
set of values.
trapezium (syn. trapezoid) commonly used term in Tschirnhaus's cubic a curve that has a rectangular
ternary a number system using a place value United Kingdom rather than trapezoid. In 2
equatior. of 3ay 2 = x (x- a) variance a measure of the dispersion of scort:ti tn a
noiation with 3 as the base. United States of America, the tem1 trapezoid distribution away from the arithmetic mean.
is used.
twin primes prime numbers that appear in pair and The mean of the squared deviations about the
tetrahedral numbers (see figurate numbers)
differ by 2. Examples are 3 and 5, 11 and mean.
trapezoid a quadrilateral in which one and only one
theorem a statement of truth of which must be pair of opposite sides are parallel. The parallel 13, ...
Varignon's Theorem The moment of the resultant
established by proof sides of the trapezoids are called bases. or two concurrent forces with respect to the
time value of money the cumulative effect of
elapsed time on the money value of an event,
based on the earning power of equivalent
tree a graph which the property that there is a
unique path from any vertex to any other
vertex traveling along the edges
u center in their planes is equal to the algebraic
sum of the moments of the components with
respect to the same center. Named after the
French mathematician, Pierre Varignon (1654-
invested funds capital should or will earn undecagon polygon of eleven sides. 1722).
trefoil curve a plane curve that has a rectangular
ton a mass of 1000 kilograms.
equation of x4 + x2 y2 + y4 = x( x2 - y2 ) undulating number an integer whose digits are vector quantity a physical quantity the described a
alternate. Example 343,434 magnitude ('row much' or "how big") and the
torque (syn. moment of force) a force times a
moment ami. triangular numbers (see figurate numbers) direction in space
unimodal sequence a sequence that first increases
and then decreases vector product of A and B is denoted as A x B.
torr a unit of pressure which is equivalent to mm of triangular region is the union of a triangle and its
interior Because of this notation, vector product is
mercury (Hg). 1 torr = 1 mm of Hg. 1 torr is unimodular matrix a square matrix whose also called the cross product.
equivalent to 133 Pascals. determinant is 1
trident of Newton 'a curve that has a rectangular
velocity rate of change of displacement.
torus (syn. anchor ring or doughnut) a solid formed equation of xy = cx 3 + dx 2 +ex+ f union a set consisting of all elements that appear at
by revolving a circle about a line not least once in the original set. Venn diagram a pictorial description of the
intersecting it trigonometry branch of mathematics which deals probability concepts of independent and
with triangles and trigonometric functions. union of two sets a set of all objects that belong to dependent events. This was named after
trace the sum of the terms along the main diagonal one or both sets
of a matrix English logician, John Venn (1834 -1923).
trillion a million million or 1012
unit circle a circle of radius one unit and is used to vertex point of intersection of two sides of a
trajectory the path followed by a projectile. It is a trinomial having three terms. determine the sign of all trigonometric polygon.
graph of a parabola. functions in all quadrants
trivial considering the values of all the variables as vertex figure the polygon. that appears if a
translation a parallel displacement of the original zero. unit fraction a fraction whose numerator is 1 polyhedron is truncated at a vertex
system along one or more of its axes.
Truncatable prime a prime number, n that remains unit vector a vector having a magnitude of unity, vertical angles angles that are opposite to each
transpose to transfer to the other side of the a prime when digits are deleted from it one at with no units and is used only to described a other and formed by two intersecting straight
equation. When a term is transpose, the sign a time. direction in space lines. Vertical angles are equal.
must be changed.
truncated prism ihe portion of a prism included unity referring to one.
transversal the intersecting line of two parallel or between the base and a plane not parallel to
non parallel lines the base cutting all the edges or elements universe (statistics, see population)
:Jl
,f
l:\
~

624· 1001 Solved Problems in Engineering Mathematics (2"d Edition) by Tiong & Rojas Appendix 13 - Conv~sion_ .§25 ·
Weight =mass x gravitational acceleration

whole number another term for natural number


AppendixB: + Units
Wilson's theorem 'Any number p is a prime

Vertical angles (a = 8)
number if, and only if, (p - 1)! + 1 is divisible
byp.'
The English System of~eigh~l A. !.!near Measure,.t!:J!nm.!:!l
and Measures
1 inch = 1,000 mils .
.vigesimal pertaining to the number 20. word problems (worded problems) real problems
Definitions: 1 foot= 12 inches
that are usually given orally or written in words
vigesimal number system a number system using 1 yard "' 3 feet
the base 20 work the force time a distance.
Inch - the length of three barley grains
placed end to end 1 rod= 5.5_yards
vinculum the bar that is placed over repeatinp 1 chain = 4 rods
decimal .fractions to indicate the portion of the
pattern that repeats. X Digit- the breadth of a finger, about·
0.75 inch 1 furlong "' 10 chains
1 furlong :::: 40 rods
Vlnogradov's theorem 'Every sufficiently large odd Palm ... the breadth of a hand, about 4
x-axis the horizontal · axis of the ~ rectangular inches 1 mile = 8 furlongs
number can be expressed as the sum of three
coordinate system.
prime numbers."
Hand - the length from the wrist to the 1 mile = 5280 feet
x-intercept the value of the abscissa of the point end of the middle finger, about 8
Viviani's curve a space curve the marks the 1 mile= 1,760 yards
where the curve crosses the x-axis inches
intersection of the cylinder (x - a)2 + yz =a2
and the sphere xz + y2 + z2 = a2. The
1 league = 3 miles
parametric equations of Viviani's curve are:
x=a(1+cost) y Cubit- the length of the forearm from
the point of the elbow to the end
of the middle finger, about 18 B.Naut~
y =asint inches
1 fathom :::: 6 feet
z=2asin(4J yard a unit of distance equivalent to 3 feet
Foot- the length of man's foot, about 1 cable = 120 fathoms
y-axis the vertical axis of the rectangular 12 inches_
·coordinates system. 1 nautical mile = 6,080 feet
volume space occupied by a solid. Volume is
expressed in cubic units. Fathom -the length of rope when 1 nautical mile per hour = 1 knot
pulled between a man's two

w
outstretch~d arms, about 6 feet
y-intercept the value of the ordinate of the point C. Surve~~!l Measure
where the curve crosses the y-axis Furlong - the length of a short race,
about 1/8 of a mile 1 link= 7.92 inches
Young's modulus (see modulus of elasticity)
Waring's conjecture 'For every number k, there is Mile - the distance of a thousand 1 chain = 100 links
another number s such that every natural
number can be represented as the sum of s
kth powers.' ·

watt the Sl unit of power, This is equivalent to joules


z paces,· as marked off by a length
5 feet between lifts of the same
foot. The mile is 5280 feet.

league -the distance a person can


1 chain = 66 feet
1 furlong = 10 chains
1 mile == 80 chains
per second. zenith a point in the celestial sphere directly above see across a flat field, about 3
the observer. · miles p.' Square Measure (Area}
weak inequality an inequality that permits the
equalitt:Case. zero (syn. cipher) void, emptiness or nothing. Zero
is derived from Hindu word ·sunya" 1 square foot= 144 square inches
weight the force · of the earth's gravitational 1 square yard == 9 square feet
attraction for the body. It is .. 1ownward force zone a portion of the surface of a sphere included
between two parallel planes 1 square rod = 30.25 square yards
acting at the centroid or center of gravity of the
Wdy. 1 acre = ·t60 square rods
626 · IOO,f Solved Problems in Engineering Mathematics (2nd Edition) by Tiong & Rojas Appendix B - Conversion 62!

Second.,.. the duration of 9,192,631,770


1 acre = 4,840 square yards I, Weifiht !Avoirdupois) M. Angular or Circular Measure periods of radiation
·1 square mile = 640 acres corresponding to the specifiC
The avoirdupois system is used for 1 minute = 60 seconds transition of the cesium-133
general weighing 1 degree = 60 minutes · atom
E. Cubic Measure (Volume) 1 zodiac sign = 30 degrees
1 dram= 27.3438 grains Ampere - the constant current that,
1 cubic foot= 1,728 cubic inches 1 radian= 57.2958 degrees flowing in two parallel
1 ounce = 16 drams
conductors 1 meter apart in a
1 cubic yarc.t= 27 cubic feet 1 pound = 16 ounces 1 quadrant I right angle = 90 degrees vacuum, will produce a force\ of·
1 gallon (US)= 231 cubic inches 1 circle :::: 360 degrees 2 x 10"7 newtons per meter of
1 stone= 14 pounds
length.
1 gallon (UK)::: 277.27 cubic inches 1 long hundredweight (UK)
1 bushel (US) = 2,150.42 cubic inches N. Hardness Kelvin - base on the triple point of
= 4 quarters water, which is the point at
1 bushel (UK)= 2,219.36 cubic inches 1 short hundredweight (US) The scale for hardness of precious which water can exist in three
metals or gems runs from 10 to 1, with states: liquid, vapor and ice. It is
= 100 pounds
10 being the hardest. defined as 1/273.16 of the
F. Liquid Measure (Capacity) 1 short ton (US) = 2,000 pounds temperatl)re of the triple point of
Diamond= 10 water.
1 tablespoon = 3 teaspoon 1 long ton (UK) = 2,240 pounds
Corundum= 9 Candela - intensity of the black-body
1 fluid ounce = 2 tablespoons
Topaz= 8 radiation from a surface of
1 gill = 4 fluid ounces J. Weight (Troy)
1/600,000 square meters at the
Quartz= 7 temperature of freezing platinum
1 cup = 8 fluid ounces The troy system is used for weighing
Labradorite = 6 and at a pressure of 101,325
1 pint = 2 cups precious metals or gems
pascals.
Smithsonite = 5
1 pint= 4 gills 1 carat= 3.086 grains
Fluorite= 4 Mole - an amount of substance in a
1 quart = 2 pints 1 pennyweight = 24 grains system that contains as many
Calcite= 3 elementary entities as there are
1 gallon = 4 quarts 1 ounce = 20 pennyweights
Alabaster = 2 atoms in 0.012 kilogram of
1 pound= 12 ounces carbon 12.
G. Dry Measure (Capacity)
The Metric. System of Weights The following are supplementary units:
1 quart =2 pints K. Weight (Apothecaries) and Measures
Radian - a unit of angular measure
1 peck = 8 quarts The apothecaries system was formerly equal to the central angle whose
Definitions:
1 bushel = 4 pecks used by pharmacists sides are two radii of a circle
that cut off an arc whose length
us dry quarts The following are the basic units:
1 UK dry quart = 1.032 1 scruple =20 grains is equal to the radius of the
Meter - the distance traveled by light in circle.
1 dram = 3 scruples
H. Dry Measure (Cooking) vacuum in 1/299.792.458
1 ounce = 8 drams second Steradlans - a unit of measure equal to
the solid angle whos.e vertex is
1 pinch = 1/8 teaspoon 1 pound= 12 ounces
Kilogram ..;. the unit of mass equal to in the center of a sphere, which
1 dash= 1/16 teaspoon the mass of the platinum-iridium cuts off an area equal to the
cylinder kept by the International rad.ius squared on the surface of
1 sprinkle= 1/32 teaspoon L. Wood Measure
Bureau of Weights and the sphere.
1 board foot = 144 cubic feet Measures in France.
1 cord foot= 16 cubic feet
1 cord = 8 cord feet
Appendix B - Conversion 629
628 1001. Solved Problems in Engineering Mathematics (2"ct Edition) by Tiong & Rojas
Standard time- Introduced in 1883 by Reference point oc oF
Poundal - the force needed to international agreement, Absolute zero -273.15 -459.67
The following are derived units:
accelerate a 1-pound mass 1 Standard Time divided the Earth Frozen yogurt -11 12
Newton - the unit of force equal to the foot per second squared. into 24 time zones. Calculated A snowy day -5 23
force needed to accelerate 1 on solar time, the base of the Water freezes 0 32
kilogram by 1 meter per second 1 poundaf = 13,825.5 dynes system is the zero meridian that Water at densed 4 39.2
squared. passes through the Royal condition
1 poundal = 0. 138255 newtons
Greenwich Observatory at Cold water 7 45
Joule- the unit of energy and work Greenwich, England. Time is A cool fall day 10 50
equal to the work done when the Newton - the force needed to measured east or west of this
point of application 'of a force on accelerate 1-kilogram mass 1
Room temperature on 20 68
Prime Meridian according to the winter day
1 newton moves 1 meter in the meter per second squared time zones.
direction of the force.
A hot day 35 95
5
1 newton = 10 dynes Normal body 37 98.6
Daylight Saving Time -In the US,
Hertz -·the unit of frequency in the field temperature
1 newton= 7.23300 pouridals standard time plus one hour.
of electromagnetism defined as Daylight Saving Time seems tc:>
A warm bath 38 101
1 cycle per second make the day last longer by ari A high fever 40 104
Pascal -.unit of pressure which is hour. A hot bath 45 113
Watt- the unit of power defined as the equal to 1 newton per square Highest air
power of 1 joule per second meter. Equivalent values: temperature recorded 58 137 I

on Earth
Volt - the u!'lit of voltage defined as the 1 pascal= 0.020855 pounditf 1 nanosecond = 1 billionth of a second Broiled steak 60 140
difference of electrical potential Hot faucet water 66 151
between two points of a Atmosphere- unit of pressure at sea 1 microsecond = 1 millionth of a second Hot soup 80 176
conductor carrying a constant level. 1 millisecond = 1 thousandth of a second Water boils 100 212
current of 1 ampere when the Hot oven 190 374
·power used between them 2 1 minute = 60 seconds
1 atm = 14.6952 pounds/in Lead melts 327 621
equals 1 watt, 1 hour= 60 minutes
1 atm = 2,166.102 poiJnds/W
Ohm - the unit of electrical .resistance 1 atm = 1.0332 kg/cm2 1 day = 24 hours
equal to a resistance that 2 1 week = 7 days [Y~rthq~~k~-l~t~~sit!J
1 atm = 101,325 newtons/m
passes a current of 1 ampere
when there is an electrical 1 month = 4 weeks
c--~-----J
potential difference of 1 volt
across it.
~ Measure of Time _ 1 calendar month = 30 days The Richter Scale, developed by
seismologist Charles Richter, is used to
1 year = 52 weeks express the amount of energy released at
Sidereal time -Also known as 1 year = 12 months the focus of an earthquake. The scale is
astronomical time. This method
[ nne~$~-;e-oTFo~cea~dP~essu;e] of measurement uses the. 1 common year = 365 days
logarithmic and based on a numerical
system of exponents. For example, the
movement of stars to calculate 1 leap year = 366 days difference between 6.0 and 7.0 on the
time. An average sidereal day is Richter scale is not one but a factor of ten ..
Force and pressure are closely related. 23 hours, 56 minutes and 4.09 1 decade = 10 years Thus, an earthquake that measures 6.0 on
Force is anything that changes the motion .seconds long. 1 century = 100 years the Richter scale is a hundred times more
or state of rest in a body while pressure is powerful than an earthquake that
a force acting on a surface per unit area .. Solar time ·- A measurement of time in 1 millennium = 1000 years measures 4.0. An earthquake of 8.0 is 10
which noon occurs when the million times greater than a 1.0
Dyne - the force needed to
accelerate a 1-gram mass 1
centimeter per second squared.
sun is at is highest point over a
given location. L Temp-erature 1
earthquake.

The great San Frar1cisco earthquake in


Atomic time -A measurement of time 1906 has a magnitude of 8 ..3 which
1 dyne=: 0.0000723 poundal using atomic clocks. Atomic The two most common scales used for destroyed the city. The earthquake that
clocks calculate time with measunng temperature are the Celsius causes the sinking of the island of
1 dyne = 1o- newtons
5
extreme accuracy based on !he and Fahrenheit scales. Krakatoa in .1883 has a magnitude of 9.9.
natural resonance frequency of
!he cesium 133 atom
630 100 l Solved Problems in Engineering Mathematics (2nd Edition) by Tiong & Rojas Appendix B - Conversion . 631

The following is a table for conversion of units from English System to Metric
Richter Comment Guidelines for Commercial Paper System and from Metric System to English System.

~
Number Weights:
9 . Near total devastation
8 A disaster, few buildings left
standina Onionskin paper = 9 pounds
7 Many buildinas.destroyed Standard cc;>pier paper = 20 pounds·
6 Buildings shakes; roads and walls
I crack Typewriting paper = 20 pounds
5 Strong rumbli11g; china and dishes Letterhead paper = 24 pounds
break .
Weak; much like a passing truck For printinv on both sides = 60 pounds
4
3 Very weak; less than 3.5 Typical business card = 65 pounds
2 Detectable only bv seismograph
Poster .paper = 120 pounds
1

I Metric Prefixes ··--] .


1· Computer M;~~ry I

Metric prefix Symbol Value·


· Computer memory is based on bits and yotta y 1024
bytes: zetta z 102 .
exa E 10'"
8 bits = 1 character = 1 byte peta p 1015
12
tera T 10
1 kilobyte = 1,024J>ytes = 210 giga G 10~

1 megabyte = 1,048,576 bytes or2 20 mega M 1()6


kilo k 10J
1 gigabyte= 1,073,741,832 bytes or 230 hecto h 102
deka dk 10
I Paper Weight~-n-] deci
centi
d
c
10'
10'2
milli m 10'~

micro ll
10'6
Standard Amounts:
nano n 10'"
pico p 10·12
1 quire = 24 sheets to_,,
femto · f
1 printer's quire = 25 sheets alto a 10·1ll
zepto z 1o:z·
1 ream = 20 quires
_ yoct~_ y 10·24 -
1 printer's ream= 21.5 quires
1 bundle= 2 reams.
1 case = 4 bundles
1 printer's bundle = 4 reams
1 bale = 10 reams
1 short ream = 480 sheets
1 long ream = 500 sheets
632 1001 Solved Problems in Eng_ineering Mathematics (2nd Edition) by 'I'iong & Rojas Appendix C- Physical Constants 633

Appendix C: Physical Constants


+!!!!!!!!!!!!!!!!!!!!!!!!!!!!!!!!!!!!!!!!!!!!!!!!!!!!!!!!!!!!!!!!!!!!!!!!
634 IOO 1 So~ved Problems in Engineering Mathematics (2nd Edition) by Tiong & Rojas Appel\dix F- Greek Alphabets 635

Appendix D: + Numeration
Appendix F:- · Greek Alphabets
!!!!!!!!!!!!!!!!!!!!!!!!!!!!!!!!!!!!!!!!!!!!!!!!!!!!!!!!!!!!!!!!!!! + . .

Appendix E: + n!! ! !
!!!!!!!!!!!!!!!M!!!!!!.!!!!!!at!!!!!!h!!!!!!N!!!!!!o!!!!!!t!!!!!!a!!!!!!ti!!!!!!o

•i
636 · 100 1 Solved Problems in Engineering Mathematics (2nd Edition) by Tiong & Rojas

Appendix G:
!!!!!!!!!!!!!!!!!!!!!!!!!!!!!!!!!!!!!!!!!!!!!!!!!!!!!!!!!!!!!!!!!!!!!!!!!!!!!!!!! +Divisibility Rules
!!!!!!!!!!!!!!!!!!!!!!!!!!!!!!!!!!!!!!!!!!!!!!!!!!!!!!!!!!!!!!!!!!!!!!!!!!!!!!!!!!!
ei
& MARKETrNG ~
ft
~~
"
PHIUPI?lNE'
EXCELLENCE('!;
AWARDS ~
~~-~
An integer is evenly divisible by ~~
~-~-~-~.~(~SIQ

2 - if it is even or it ends with an even number

3 - if the sum of its digits is divisible by 3

4 - if the number formed by the last two digits is divisible by 4 THE PHILIPPINE MARKETlNG
5 - if it ends with either 0 or 5 EXCELLENCE AWARD
6 - if it is divisible by 2 and 3
is given to
7 - if the number formed after cancellation of the units digit and
subtraction of twice the value of the units digit is divisible by 7 EXCEL Review ·center
8 - if the number formed by the last three digits is divisible by 8

9 - if the sum of its digit is divisible by 9


Most Outstanding Engineering Review Center
10 - if it ends with zero
In recognition of its outstanding marketing performance leading
11 - if the difference between the cross sums of alternate digits is
divisible by 11 towards market dominance, goodwill, high-level customer
confidence, and market acceptability.
12 - if it divisible by 3 and 4

Awarded by the Philippine Marketing Excellence Awards Institute,


Sales and Marketing Magazine and the Asian Institute of Marketing
and Entrep}:eneurship this 7th day of July, 2005 at the Westin
· Philippine Plaza, City of Pasay, Philippines.

~
FELIX ~LAO, JR., Ph.D.
Chairman, Awards Committee
National President, Philippine Marketing
Association (1987)

You might also like